You are on page 1of 375

Maths Extension 2

Zndedltion
by
S.K. Patel
-------
J

L~, ~~-et\1'\'c~
...
-.- --~-- -,
X\"- ··~,..,

;..,
')
Copyright © S.K. Patel and Pascal Press
First Edition published by Pascal Press April 1989
Revised Edition published October 1989
Second Edition published January 1991
Reprinted 200 I, 2003, 2004

ISBN I 87531 226 9

Pascal Press
PO Box 250
Glebe NSW 2037
(02) 8585 4044 ' ., ' .
www.pascalpress.com.~u

Printed in Singapore by SNP SPrint Pte Ltd

COPYING OF THIS BOOK BY EDUCATIONAL INSTITUTIONS


A purchasing educational institution may only photocopy pages within this book in accordance with
the Australian Copyright Act 1968 (the Act) and provided the educational institution (or body that
administers it) has given a remuneration notice to the Copyright Agency Limited (CAL) under the Act.

It is mandatory that ALL photocopies are recorded by the institution for CAL survey purposes.

For details of the CAL licence for educational institutions, contact:


Copyright Agency Limited
Level 19, 157 Liverpool St
Sydney, NSW, 2000
Telephone: (02) 9394 7600
Email: info@copyright.com.au

COPYING BY INDIVIDUALS OR NON-EDUCATIONAL INSTITUTIONS


Except as permitted under the Act (for example for fair dealing for the purposes of study, research,
criticism or review) no part of this book may be reproduced, stored in a retrieval system, or transmit-
ted in any form by any means, without the prior written approval of the publisher. All enquiries should
be made to the publisher at the address above.
PREFACE
As always, my aim in this book has been to impart the knowledge of mathematics
to students who will need it in their later academic and professional work.

I would like to thank and acknowledge the help and contribution of the following
individuals:
- Roger Myers of the Banks town Technical College for his many thoughtful
suggestions
- Judy Faulkner who typed the manuscript in the required time
- all my colleagues in the Technical Colleges and many other school teachers
for their useful suggestions.

I sincerely hope that this new edition will be found to be even more useful than the
previous one.

Suresh Patel

- ./
CONTENTS

CHAPTER 1 CURVE SKETCHING


1.1 Basic Curves 1
1.2 Method for Sketching the Curves 2
1.3 Graphs of Basic Functions 3
1.4 Curves with Vertical and Horizontal Asymptotes 8
1.5 Trigonometric Graphs 11
1.6 Exponential Curves 14
1.7 Drawing Graphs by Composition of Ordinates 15
1.8 Implicit Differentiation and Sketching Curves 24
1.9 Applications 26
1.10 Miscellaneous Curves Sketching 30
Exercise: l:A 40
Exercise: l:B 43

CHAPTER 2 INTEGRATION
2.1 Standard Integrals 49
2.2 Change of Variable: Substitution 51
Exercise 2A 53
2.3 Integration by Part 54
Exercise 2B 57
2.4 Trigonometric Integrals 58
2.5 Use of t = tan (X/2) 61
Exercise 2C 63
2.6 Reduction Formulas 64
Exercise 20 66
2.7 Trigonometric Substitution 67
Exercise 2E 68
2.8 Integration of Rational Functions 68
Exercise 2F 72
2.9 Method of Partial Fractions 73
Exercise 2G 77
2.10 Completing the Square (Integration) 78
Exercise 2H 80
2.11 Integration: Special Properties 81
Exercise 2I 85
Exercise 2J 86
CHAPTER 3 VOLUMES
3.1 Formulas for Volumes 88
Exercise 3A 97
3.2 Volumes: Shell Method 99
3.3 Volumes: Washer Method 103
Exercise 3B 104
3.4 Worked Examples (Miscellaneous) 105
Exercise 3C 109

CHAPTER 4 COMPLEX NUMBERS


4.1 Introduction 112
4.2 Operations with Complex Numbers 113
Exercise 4A 116
4.3 Complex Plane (Argand Diagram) 118
Exercise 4B 121
4.4 Multiplication and Division of Complex Numbers
Using Trigonometry 122
Exercise 4C 124
4.5 Powers of Complex Numbers: De Moivre's Theorem 125
Exercise 4D 126
4.6 Roots of Complex Numbers 127
Exercise 4E 130
4.7 De Moivre's Theorem and its Applications 131
Exercise 4F 134
4.8 · Square Roots of a Complex Number 135
Exercise 4G 136
4.9 Properties of Conjugate Complex Numbers 137
Exercise 4H 138
4.10 The Complex Roots of Unity 139
Exercise 41 141
4.11 Miscellaneous: Factorisation over the Complex Field 143
Exercise 4J 145
4.12 Geometric Representation of Complex Numbers 146
Exercise 4K 149
4.13 Product and Quotient: Rotation 150
·.Exercise 4L 152
4.14 Locus Problems with the Complex Variable z 153
Exercise 4M 154
4.15 Miscellaneous Locus Problems (Including Inequations) 155
Exercise 4N 159
Exercise 40 160
CHAPTER 5 POLYNOMIALS
5.1 Introduction 164
Exercise SA 167
5.2 Zeros of a Polynomial/Multiple Roots 168
Exercise SB 172
5.3 Polynomial over the Complex Field 173
Exercise SC 177
5.4 Relation between Roots and Coefficients of P(x) = 0 178
Exercise 50 183
5.5 Miscellaneous (Worked Examples) 185
Exercise SE 187

CHAPTER 6 CONIC SECTIONS


6.1 Introduction 191
6.2 Ellipse (e < 1): (Focus and Directrix Definition) 192
Exercise 6A 195
6.3 The Hyperbola 196
Exercise 6B 198
6.4 Shape of the Conics 199
6.5 ParaiJ~.etric Equations of the Circle and the Ellipse 201
6.6 Equations of chord, Tangent and Normal to the Ellipse 203
6. 7 Parametric Equations of the Hyperbola 205
6.8 A Special (Rectangular) Hyperbola xy = c2 206
Exercise 6C 208
6.9 Miscellaneous Problems on Conics 209
Exercise 60 214
6.10 Tangents and the Chord of Contact 222
6.11 Geometric properties of the Ellipse 224
6.12 Geometric Properties of the Hyperbola 227
6.13 Properties of the Rectangular Hyperbola 228
6.14 Geometric Properties of Rectangular Hyperbola xy = c2 229
Exercise 6E 232

CHAPTER 7 ELEMENTARY PARTICLE DYNAMICS


7.1 Introduction 234
7.2 Laws of Motion - Force 234
Exercise 7A 239
CHAPTER 8 MOTION PROBLEMS IN TWO DIMENSIONS
8.1 Introduction 240
8.2 Simple Harmonic Motion (Revision) 241
Exercise 8A 244
8.3 Motion of a Projectile (Revision) 246
Exercise 8B 249
8.4 Resisted Motion: Other Laws of Motion 252
Exercise 8C 259

CHAPTER 9 CIRCULAR MOTION


9.1 Introduction 265
9.2 Angular Velocity: Period 266
9.3 Circular Motion: Tangential Velocity 267
Exercise 9A 269
9.4 Acceleration of a Particle Rotating in a Circle 270
9.5 Uniform Circular Motion 271
Exercise 9B 272
9.6 Conical Pendulum 273
Exercise 9C 275
9.7 Banked Tracks 276
Exercise 9D 278
9.8 Components of Acceleration (Variable Angular Velocity 279
Exercise 9E 281
9.9 Miscellaneous (Worked Examples on Circular Motion) 282
Exercise 9F 285

CHAPTER 10 HARDER 3 UNIT TOPICS


10.1 Harder Trigonometry 288
Exercise lOA 291
10.2 3 Unit Co-ordinate Geometry: Circles (Harder Problems) 292
Exercise lOB 2%
10.3 Plane Geometry: Circles (Harder Problems) 297
Exercise lOC 300
10.4 Inequalities 305
Exercise lOD 307
10.5 Method of Mathematical Induction 308
Exercise lOE 310
10.6 Properties of the Integrals 312
Exercise lOP 315
Appendix 1 316
Appendix 2 317
Answers 318
Chapter 1: Curve Sketching

INTRODUCTION

A structure, no matter how complicated, is composed of basic building materials. In


mathematics, the study of complicated functions such as f(x) = -1 (ex + e-x), log x -
2 e x
sin-l ex, etc. can be facilitated by sketching the graphs of these functions. But how
can we best accomplish this? Firstly,by identifying them by the basic curves (listed
below) and then using a number of systematic steps to produce reasonably good
graphs. The objective is to draw a quick and neat sketch of the curve showing all
the essential features (see section 1.2). Any thought of drawing the graph by brute
force i.e. plotting scores of points must be abandoned, as it would most probably miss
the essential features, such as asymptotes and the critical points.

1.1 Basic Curves

The following basic curves are well known to students. (See Chapter 1 for the graphs
of the basic functions.)

Function

1. Linear: ax + by + c =0 A straight line


·y = mx + b

2. Quadratic: y = ax 2 + bx + c A parabola
3 2
3. Cubic: y = ax + bx + ex + d A cubic curve
4 3 2
4. Quartic: y = ax .. bx + ex + dx + e A quartic curve
k
5. Y=x, xiO A rectangular hyperbola
2 2
6. x + y + 2gx + 2fy + c =0 or
2 2 2
(x - a) + (y - b) = r
7. Exponential: y = ax , ex An exponential curve
8. Logarithmic: y = loga x , loge x A Iogarithmrc curve
9. Trigonometric: y = sinx, cosx, tanx A sine curve, cosine
curve, tangent curve
. .-1 -1 -1
10. Inverse Tng: y = sm x, cos x, tan x An inverse sine curve etc.

11. Type: y = x 1/n , n = 2, 3


2

1.2 Method for Sketching the Curves

A sketch of the curve shou1d convey the general shape of the curve, showing the
following information.

1. Intercepts on the axes


x =0 gives the y-intercept, y =0 gives the x-intercept.

2. Symmetry
The curve is symmetric about the
(a) y-axis if f(x) = f(-x) (b) origin 0 if f(-x) = -f(x)
3. Asymptotes
If y = f(x) is a rational function of the form f(x) = ~~x~ , where g(x) and h(x) are
_ x polynomials, then the:
(a) vertical asymptotes are g1ven by h(x) = 0

(b) horizontal asymptotes are given by ;i: I co I f(x) if it exists, say y = c.


2
Example: The vertical asymptotes of the curve y = 2X + 1 are given by
2
2 X - 1
x - 1 = 0 , i.e. x = .! 1 and the horizontal asymptotes are given by
2
Lim 2/ + 1 = 2 and y = Lim 2x + 1 _ . _
Y =X +co. 2 x~-oo 2 - 2 'I.e. Y- 2•
X -· 1 X - I

4. Critical points
(a) Relative maxima: f' (x) =0 , f"(x) <0
(b) Relative minima: f' (x) = 0 , f"(x) >0
(c) Vertical tangent at x =a if f' (a) is undefined
(d) Point of inflection at x = a, if f"(a) = 0 and f"(x) changes sign as x
increases through x = a.

5. Rising and falling curves


(a) f' (x) >0 for the increasing function (rising curve)
(b) f' (x) <0 for the decreasing function (falling curve)

6. Concavity
(a) f"(x) <0 indicates where the grapl:l is concave down
(b) f"(x) >0 indicates where the graph is concave up

7. Zones (regions) of exclusion:


The domain in which the graph of y = f (x) does not exist (i.e. y is undefmed) is
an important consideration, because we don't have to worry about the graph in
this interval. ~
Example: y = f(x) = V 9 - /

The graph of this function is real if -3 ~ x ~ 3. It does not exist for lx I > 3.
The curve IS the upper half of the circle x 2 + / = '3.

j.
3

1.3 Graphs of Basic Functions

1. The graph of a linear function y = mx + b


jsa straight line, with gradient m and
y-intercept b.
x = b represents a vertical line. tanS
X
y =c represents a horizontal line.

2. The graph of a quadratic function

y = ax 2 + bx + c
. a para bo Ia.
1s
Its vertex lies on the axis of symmetry
b
x = - 2a •
b
If a > 0, y has a minimum at x =-2a

. b
If a < 0, y h as a max1mum at x = - 2a
Discriminant /J. = b 2 - 4ac .
If A < 0 , the curve does not intersect
the x-axis,

If /J. 0, the curve touches the x-axis.

If IJ. > O, the curve intersects the x-axis


b /:;
at x =- 2a + 2a

Examples: Sketch the curves:


(a) y = x 2 - 4x + 3 (b) y = -x 2 + 2x
(c) y = x2 + x + 1
4

Solution:
2
(a) y = X - ~X + 3
a > 0, A = 16 - 12 > 0
The intersection with x-axis is given by y = 0
:. (x - 3) (x - 1) = 0
A(l, 0), B(3, 0)
When x = 0 , y =; · 3 v (2, -1)
Vertex at x = 2 , y = -1 y

(b) 2 V(l,l)
y = -x + 2x
a< 0
y = 0 gives x (x - 2) = 0
0 (0, 0) and A (2, 0)
Vertex (1, 1)

(c) y = X2 + X + 1

a> 0, A=l-~< 0
No intersection with x-axis

Vertex (- l 3
2,4 )
Some points on the graph
A(l, 3), B(O, 1), C(-1, 1) X

2
The equation of the form x = ay + by + c
represents a parabola with its axis horizontal.

b
Vertex on ax1s of symmetry dt y = - 2a
For a > 0 and a < 0, the shapes
are as shown.

If A < 0, the graph does not intersect


X
y-axis.

Example: y
Sketch the curve X = y2 + 2y
Solution:
a = l > 0.
b
Vertex: y = -2a -22 = -1, V(-1, -1) -1 X

x = 0 gives y (y + 2) = 0 i.e.
v
(0, 0) and (0, -2)
5

GRAPHS OF POLYNOMIAL FUNCTIONS

3. For sketching the graph of the polynomial function we need reasonably


sufficient information, such as:

a. The function y = f(x) is an even function if f(x) = f{-x).


The graph of an even function is symmetric about Y-:axis.

b. The function y = f(x) is odd if f{-x) = -f(-x) and the graph is


symmetric about the origin.

c. The intersection of the curve y = f(x) with


- the x-axis is given by y = 0 • This is not always simple, but worth
trying.
- the y-axis is given by x = 0.

d. The behaviour of the curve as x - + co

e. The nature of the turning points, i.e. a maximum or a


minimum

f. Concavity and points of inflexion i.e. f" (x) =0 •

Example: (I)
3 3
Sketch the curves y = x and y = -x
3
y=x
Solution:

f (x) = x 3 and f (-x) = -x 3


3
The graph of y = x is symmetric about the origin X

0 (0, 0) is a point on the curve


As x - co, y - + co
X _,.-oo, y - - co

~ 3x 2
dx
But x = 0 is not a turning point, since ~ does not change its sign in passing
through 0
Some points: A (1, 1) , B (2, 8) • This is reasonably sufficient information to sketch
the curve.
6

Example: (2) Sketch the curve:


y = (x -t 2) (x - 1) (x_- 3)

y = 0 gives A(-2, 0), B(l, 0), C(3, 0)


l'= 0 gives D(O, 6) ---,....-1:----~-~lc-----:::---:Jf:::-~x
y -+ ~ as x-+ ..,
y--a.as x-+-~
Other points x = 2, y = -4
X = -1 1 y :: 8

Example: (3) Sketch the curve:


2
y = (x + 2) (x - 2)
y = 0 gives A(-2, 0), B(2, 0)
The double zero x = 2 indicates
that the x-axis touches the
curve at x = 2
~2 X
Y-i'.:!:.a.asx~;t""

x "' o gives y = 8
Other points;
C(-1, 9)1 D(l, 3), E(J, 5)

Example: (4) Sketch the curve:


y = (x -t 2) (x + 1) (x - 2) (x - 3)
The sketching is similar to that
of Example 2, noting thdt y .... oo

dS X~- oo

X=O,y~l2

(I, I 2) is another point


X
7

4. Graphs of y = x 1/2 dnd y =x 1/3


EXAMPLE: (1) Draw a sketch of y = .JX by analysing the
behaviour of the function near x =0 •
SOLUTION:
The function f(x) = ,jx exists and is continuous for x ~ 0 •
Also f(x) ~ 0 for x ~ 0 and f'(x) = -~ for x >0 •
2 V X
The point x = 0 is a critical point as f' (0) does not exist. 0 X
The curve y = {x has a vertical tangent at x = 0 •
Since f' (x) > 0 for x > 0, f(x) is an increasing function Graph of y = ,fX
and f(O) = 0 is the absolute minimum, but f(x) has no
absolute maximum.
I -3/2
It may be noted that since f" (x) = - ii x < 0 for x > 0,
the curve is concave down.
This is sufficient information for drawing a reasonable graph of y = .(X.
Note: y = vx is the upper half of the parabola / = x.

EXAMPLE: (2)
Draw a sketch of y = x 1/3 •

SOLUTION:

Y =x
1/3 ~
' dx
I -2/3
=3 x
fr
' dx2 =

f (x) = x l/ 3 exists for all real x


1/3 . 1/3
f(-x) = (-x) =-x = - f(x), so the curve has point symmetry about the origin.
f' (x) does not exist for x = 0, so the curve has a vertical y
tangent, at x = 0.
Also f' (x) > 0 for all x, x I 0, so it is an increasing
curve. x = 0 is a critical point.
f (x) ~ oo as x -t oo

f (x) -t - oo as x __,. - oo

f"(x) > 0 for x > 0 and f"(x) < 0 for


x < 0, so the origin is an inflection point.
Also f"(x) > 0 for x > 0 -J
tells us that the curve is concave down,
while f"(x) < 0 tells us that the curve
is concave up for x < 0. A few simple
points are 0, A(l,l), B(-I,-1).
8

1.4 Curves with Vertical and Horizontal Asymptotes

Example: (1)
2
Sketch the curve y =
X

y has same sign as x, hence curve in first and


third quadrant 2
As X -IX>' y - a+ (from above)
As x - -co, y -+ 0- (from below)
The x-axis and y-axis are the asymptotes of the 2

curve.
The asymptotes are very useful in sketching the
curves. This curve is a rectangular hyperbola -
rectangular because the asymptotes are perpendicular.

Example: (2)
I
Sketch the curve y = x + x
1
y=x+x' xl- 0
1
Near x = 0 , the term
x.
dominates, i.e. x is negligible
I X
compared to x

:. The curve y = x + .! behaves


X

like hyperbola y = .! near x = 0


X

As x-+ ""• y - x i.e. the


curve approaches the line y = x
The graph is symmetric about
the origin as f( -x) = -f{x)
The turning points are:
~ -
dx -
I
---:z
1
X
0 * p (1, 2)
Q(-1,-2)

The curve does not intersect either axis


Other points: (2, 2.5) , (-2, -2.5)

This is reasonable enough information to sketch this curve


9

Example: (3) y

-
X- 2
Sketch the curve y = --2-
x
Solution: -------"-0-+--i'i 4 X

2 X-
y = -2- CRITICAL
x
The function is undefined at x = 0, i.e. y-axis is \ I POINTS

the asymptote
X '"' 2 ' y = 0 • A (2, 0)
y
When x > 2, y > 0 , x < 2, y < 0
Near x = 0 , the graph behaves more like
As x - oo , y -+ 0 + from above
x -+ -oo, y -+ 0 from below

·~ = 0 gives x =4
dx

1
P (4,
2> is a maximum turning point.
The first sketch shows the important features, the
second shows a complete graph
5
Other points: (:-1, -3), (-2, -1}, (-3, -9), (1, -0

y
Example: (4)
2
X
Y = (x - 2) (x + 2)

Solution:
The curve is undefined at x = + 2
x = 0, y = o, 0(0, 0) is a point on
the curve.
The Jines x = 2 and x = -2
are the asymptotes.
As x-+ ""• y - I+ from above
X
x - - oo , y - I+ from above
This is so because the numerator
is greater than the denominator
when x is large.

For - 2 <x <2 , y < 0 and !h.E'


curve is asymptotic to x = ±2
10

It would be instructive for you to study the sketches of the following curves and
reasoning out the shape and critical behaviour of each of them.

Example: (5) y
2
X
y = (x + 2) (x - n
What are the asymptotes?
Why is the RIGHT (TOP) branch
approaching y = 1 from
__ I _ - -
I I
---~. .:-
below? -2' -1 0

Why is the LEFT (TOP)


I 1 2 3 4 X

branch - y = 1 from
above?

Useful points are at


X = 0, X = 2, X = 4, X = -3
X = -1.

/! j~
Example: (6)
(x + 1) (x- 1) 2
Y = (x + 2) (x - ~)

What are the asymptotes? - - - -- -- - ,_ - - ,_-- ----- y =I


1
As x - ~ oo , to which line
does the curve approach? -2 I 3 4 X

The useful points:

X = 0, I, 1.5, -1, -1.5

Example: (7)
x (x + 2)
y = 2
x - I
Questions:
a Asymptotes?
b After x = I, as x - oo -1 I
what does y approach?
I
c Before x = -1, as x - -oo,
how does y approach 1?
d. As x - + 1, what does
y- ? -
11

1.5 Trigonometric Graphs


1. y = a sin bx y
The amplitude =a
The period, p = b2n
f (x) = sin x
f(-x) = -sinx X
:. f(x) = sinx is an odd
function.

2. y = Acos bx
Amplitude = A
n . d 2n
reno p =b X

f (x) = cos x is an even


function.

3•. y = tan x y
y - oo as x - (2n - I) ¥
The asymptotes are at

x = (2n - I) ~ ,
X
n = 0 , .:!:_1, .:!:_2,, ••••
f(x) = tanx is an odd
function.

If. (a) y = cosec x , period 2n


cosec x is an odd function
cosecx- "" at x = nn (n =Q.,.:!: 1,.:!: 2, ••• )
(b) y = secx , period 2n
sec x is an even function
lT
sec x - oo at x = (2n + 1)
2, (n=0,_:!:1, ••• )
(c) y = cot x is an odd function, period n, with asymptotes at
x =nn, (n = 0, .:!:. 1 , ••• )
The sketches are on the next page.
12

y I y I

u
----+-o---------4-n---------+l-2n~_.
1\
I
I
I :X

z1
!\
I
I
I X
-n 0
r
-1

\ n: I
I
\!
I

I
I
-1 :nn/2
I
I
I
I
I
I
I
I
I
I
I
I
I
I

y =cosecx y = secx

y =cot x

y
y lx I "'
lx I "' I
IYI


X

. -n/2 0

. -1 -1
y = sm x Y =COS X
13

Example: (1) Sketch the foHowing curves:


(a) sin(x + y) = (b) y = Icosx I, Ix I ~ 2u
2 ( . -1 x )
(d ) y=cossm
(c) y = sin x
y
Solution:

(a) sin(x + y) =

:. x + y = nu + (-l)n ~ is the

general solution. [n = O, .:!:_1, .:!:. 2, ••• ]


Now x + y = K represents a series of
paraHel lines.

(b) y = lcosx I , -2u.(: x" 2u


The easiest way to draw the sketch
of any absolute function is to draw
the curve of ordinary function and X

then draw the REFLECTION of the


graph below the x-axis

2 . 2
k) y = sin x y = sm x
y
y ~ 0 for all real x
The greatest value of y is
Some points:

(0, 0) ' ( *' 0.5) ' ( ~ ' l) X


( ~u , 0.5) , ( u, 0) etc.

(d) . -J x )
y = cos (sm y
Here lxl ~ 1
We simplify the function by replacing
. -J -1 r----7_
sm x by cos { 1 - x'
. -1 x ) B A
y =cos (sm
(-1, 0) 0 (1, 0)
cos~os- 1 M)
X
=

y=~
Graph of y = cos (sin- 1 x) is a
semi-circle, above x-axis, radius 1,
centre 0.
'~
117
14

1.6 Exponential Curves


Sketch the following curves:

y
(a) Y =e X
y > 0 for all x
y =e X
As X~ co, y - co

x--, y-.. 0 /
/
Some points:

(0, 1), (1, e = 2.7), (2, 7.4), (-1, 0.3)

) 4 X

2 y
(b) y =e-x
1. y > 0 for all x • A (0, 1)
2. f(x) =f(-x), even function. The graph
is symmetric about the x-axis
3. As x-co, y - 0

~ = -2x e-x gives a maximum at


2
4.
X : 0, y =1 -I X

d2 2 2
5. ~= -2[1- 2x ]e-x = 0 gives inflexion
dx
at x = 0.7 and x =-0.7, y = 0.6
(c) y = x e-x
1. y >0 for x >0
y <0 for x <0
2. As x- ""• y - 0
As X- -co, y _...., X

3. ~ = e-x(l- x) =0 gives
-1
x = 1, y = e = 0.4 (Maximum)
4. Some points
(0, 0), (1, 0.4), (2, 0.26), (-1, -2.7)
15

1.7 Drawing Graphs by Composition of Ordinates

(a) Addition or Subtraction of Ordinates

EXAMPLE: (l) Sketch the curve y =1 + 2 sin (2x), - 11 ~ x ,< 11.


SOLUTION: We separately sketch y
1
=1 and y = 2 sin 2x.
2
y I
I
I
y =1 + 2sin 2x I
I
- - - - - - - -1 - .
I

X
-111 11 ' 11
I(
4 I I
\ 1
l y = 2sin 2x
' . _..,"-..:.,.::~
\
' _ .,
\
/
-2

Algebraic addition of ordinates gives us the equation


y = y 1 + y 2 = 1 + 2 sin 2x • • • • (1)
By graphical addition we obtain the graph of y =1 + 2 sin 2x ' - 11 ~ X~ 11 ' shown
by the heavy line.
Note that the amplitude of y = 2 sin 2x is 2 and its per~od is ¥ =n •
Addition of ordinates must be done at all key points, such as end-points, at the
intersections with the axes and the critical points (Maxima, Minima) of both graphs.
It is useful to note that the graph of y = 1 + 2 sin 2x is obtained by translating the
graph of y = 2 sin 2x one unit upwards, thus preserving all the characteristics of
y = 2 sin 2x.

EXAMPLE: (2)
Sketch the graph of the function f(x) = 2sinx + x, 0~ x~ 211.

SOLUTION: = 2sinx and y = x


Let y
1 2
The sine graph has an amplitude of 2 units and a period of 211. y = xis a straight
2
line through the origin. We separately sketch these graphs and by graphical addition
at key points x = O, ~ , 11 , ~11 , n , obtain the composite graph of y = x + 2 sin x,
as shown by the solid line.
(cont.)
16

y
/ "
,/ y: X

6 ,"•B
/ I
/ I
/ I
// I
4 / y =x + 2sin x
/

·I
y = 2sin x
....._, I

X 0 n/2 n 3n/2 2n

Y2 0 1.57 3.14 4.71 6.28

y1 0 2 0 -2 0

y1+y2 0 3.57 3.14 2.71 6.28

Note that the points of intersection of the required curve y = x + 2 sin x and
y =x are where the curve y = 2sinx cuts the x-axis. So, 0, A(n,n), (B(2n, 2n)
are the points on the graph.
Cr itica1 Points:
y = x + 2 sin x The stationary points are given by l + 2 cos x =0
l
i.e. cos x =-2
2n 4n
1 + 2cos x x =T and x T are stationary points.

=- 2 sin x f"(2n) <0 and


3
2n 2n
So C (
3 , 3 + {3) is a maximum point. c (2.1, 3.8)
4n 4n
and 0(3 , T- fj) is a minimum point. D (4.2, 2.5)
We have sufficient information to sketch the graph.
2
Note:
dx
!!.f
= 0 gives the point of inflection, i.e. sin x = 0 and sin x changes sign
as x increases through n •
:. x =n is the point of inflection A ( n, n)
17

(b) Drawing Graphs by Reflecting Functions in Co-ordinate Axes

A function y = - f(x) can be graphed by reflecting the graph of y = f(x) in the


x-axis. The graph of y =c - f(x) may then be obtained by a suitable translation.

EXAMPLE: Sketch the following curves y


(a) y = -sin x (b) y =1 - sin x
y = 1
SOLUTION:
We draw the graph of y = sinx (dotted line), then
reflect it in the x-axis to obtain the graph of ---.;:;.:::.,---
y = - sinx and finaUy shift it vertically upwards by 1/ \
1
1 unit. , 1 I
The graph of y = l - sinx is shown by the heavy ,• . 1
I y -sinx \ 1
\
=
·.I '
- -4,-,-.....::::.W:::-......:.f...---..&---.-;,.~.o• x
11 311
.. 2
line.
The key points to consider for plotting are
0
\
11
2 1
1 · 2
11
n 3n \ I
X = 0,
2, 1T ,
2 , 2lf \ I

-1 ''•" / ·......··' y = sinx

(c) Graph of y = IHx) I from the graph of y = f(x)


Sii)Ce I f(x) I ~·o for all x, all we have to do is to draw the reflection of any part of
the curve y = f (x) below the x-axis, in the x-axis, leaving unchanged those parts
above the x-axis.

EXAMPLE: Sketch the curves: (a) y =X2 - 4 (b) y = I x 2 - 41


2
(c) y = 2 + Ix - 41
SOLUTION:

(a) y =y 1 = / - 4 is a (complete)
parabola. (The part below the x-axis
is shown by a dotted line)

(b) Reflecting only the part of the


parabola below the x-axis, we obtain
the graph of y = y 2 = Ix 2 - 4 I

(c) Shifting Y 2 , 2 units vertically


upwards, we sketch the graph of

y =y
3
=2 + I x 2 - 41 -2\ 0
I
I 2 X

'\ -2 , I

'' ,, yl
'' 4
18

(d) Graph of y = f(,.. x) by initially graphing y = f(x)

y = f(-x) is the reflection of the graph of


y = f (x) in the y-axis.

EXAMPLE: (a) Sketch y = Fx


(b) Sketch y = log (-x)
e
(c) Sketch y = log
e
I xl
Note: The graph of y = loge lx I is the graph
0 X
y : log X , X ) 0
e y = log (-x)
= log (-x) , x <0 e

i.e. both branches make the graph of


Y =loge I xl.

y = log lxl
e

(e) Graph o.f y = f(x - a) by graphing y = f(x)


The graph of y =f(x - a) is obtained by shifting the graph of y = f{x) to the right
by a units, if a >0 and to the left by Ia I units if a < 0.

EXAMPLE:
Sketch the curves (a) y = cos(x - 1) (b) y = loge (x + 1)

y =cos(x - 1)

0 \ X
\
\
y = cosx '
19

(f) Graphing the functions by multiplication of ordinates


EXAMPLE: Sketch y = x e-~
We separately sketch the curves y =y 1 =x \
y
/
,,
-x -x \ ," y =X
and y = y
2
=e y =e '\ /
We derive the properties of the graph
-x . '' /
/
"
y = y • y =xe from F1g. 1. ,/
1 2
When x < 0, y = x e-x < 0
X ) 0, X eX) 0
As X~ Q)' xex +0;
-1 2 X
As x"'"-""• xe-x-+-oo
2 x = 2 is the point of
dv -x ) d y -x
~ = e (1 - x , - - = -e (2 - x) , the inflection
2
dx
stationary point x = 1 is a maximum.
Some useful points: (0,0), (1,0.37), (2,0.3),
(-1, -2.7).

The graph can now be sketched (solid line).

(g) Graphing the functions by division of /


y
ordinates "
~y=x

_... -- ---
f(x)
Gr~phing y = grx) by division of ordinates. /
/
/

logex 2 / " ....


EXAMPLE: Sketch y = - x - by
/ " ,. ,.., .... ~y =loge x
separately sketching /
/ / P(e, 0.37)
y = logex and y = x • / /

SOLUTION: E)
~
2 3 4 5 X
~

log x / -I
y = --f- exists only for x >0 •
A few points:

X 0.5 2 e 5
0 0.7 1.6 Asymptote at x = 0.
0 0.35 0.37 0.32 As x + oo , y _. 0.

Critical Points: ~~ _!_ (x • !


2 x
- loge x)
X

~ddx = 0 ~ log e x = 1 ~ x = e is the critical point.


Using the graph of y = log x , we find:
e
Since x > 0, ~~ has the same sign as the numerator
2
N =I - logex •
For x < e,
1 - log x
_ e >0 and for x > e, 1 - log x
e < 0, so the maximum point is at
1
x = e .., P (e, e )
1ogex
The sketch of y = - x - is shown by the solid line.
20

(h) Graph of [f(x)]n , n > 1 by graphing f(x)


1
Let y = g(x) =[f(x)f, then g'(x) = n. f'(x). [f(x)f- , so all the stationary points
and the x-intercepts of y = f(x) are the stationary points of g(x). The following
properties of [f (x) ]n may be useful.

1. If lf(x)l > 1, then Ig(x) I > lf(x) I, e.g. x > 2 ~ x 2 > 4 ~ x 2 > x
then 0 < lg(x) I < I f(x) I, e.g. 0 < x < ~ ~ x < x
2
2. If 0 < lf(x) I < 1,
3. [f(x)]n ~ 0 for all x, if n is even, e.g. (x 2 - 4)
2
>0 for all x
3
4. If n is odd: (a) [f(x)]n >0 for f(x) > 0, e.g. (x - 2) >x - 2 for x >2
(b) [f(x)]n <0 for f(x) < 0, e.g. (x - 2)3 <x - 2 for x <2
5. Points of Intersection are useful and obtained by solving, g (x) = f (x).

2 2
EXAMPLE: (1) Sketch y = (x - 1)
2 2 2
Le y = g(x) = (x - 1) , f(x) = x - 1 y
(1) f' (x) =0 and f(x) =0 give
2
2x = 0 and x - 1=0
So, x = O, !: 1 are the stationary points. '
.\ ............. .
2
g'(x) = 4x(x - 1), g"(x) = 4(3i- 1)
g"(O) < O, g"(l) > O, g"(-1) > 0

(2)
So, the maximum occurs at x = 0 and
minimum occurs at x = !:
g(x) = g(-x), so g(x) is symmetric about
1
''
' ,_ "/ /
,
X

the y-axis.
(3) g (x) ~ 0 for all x, g(x)-. co as x ... !: co
(4) .
Pomts o f mtersect10n:
. . (x 2 - 1)2 = x 2 - 1 gives X=.!. 1, + l/'[ =.!. 1.4

(5) Additional points: (0, 1), (.!.1,0), (!: 2, 1), (.!. 2, 9)

2 y
EXAMPLE: ( 2) Sketch y = sin x
2
The period of g(x) = sin x = .!. (1 - cos 2x) is n.
2
Let f(x) = sinx.
g(x) ~ 0 for all x. g'(x) = 2sinxcosx
f(x) = 0, f'(x) = 0 give sinx = O, cosx = 0 1T n 2n X
. . - n 3n 2
So stationary pomt5 are at x = 0, n,
..:n, 2, 2• ···
2
Observe that 5in x < Isin xI except at the stationary points.
The graph of g(x) can now be sketched with the points (Fig. 1).
(0,0), <-?, 1), (n,O), (~ 1), (2n,O).
21

(i) Graphs of the Reciprocal Functions


By using the properties of the reciprocal function y = r/xr , in relation to the known graph
of y = f (x) , we can draw the sketch of y = f ~x) .

To sketch y = f ~x) , consider the following:

a. If y >0 , then !y > 0 and if y <0, then !y < o. As y increases !y decreases.

b. Solve the equation y = !y i.e. y = + I


-
for the common points of intersection •

c. The asymptotes of !y are given by y = 0

d. Behaviour of the function as x -+ + ""

e. Simple points at x = 0 , ~ I , etc.

Example: (I)
y
I
Sketch y = loge x and y = log x
e
x >0 for y = logex I
y = logex is a known graph Y = logx

y - - -

I.
- logex
1

as follows.
is shown by solid line and can be sketched

(base, e, omitted for convenience)


The asymptote for y = - - -
1ogx
I
is at x = I /

2 e 3
--
y = logx

4 X
2. The points of intersections are given by solving:

i.e. x =e ~ 2.7
and x = e-l = 0.37

3. As y = log x increases from 1


1
y = - - - decreases. (both positive)
1ogx

4. As x - I

X-+ 1-, y -
+
, y =--
I
1ogx
-oo.
-"" and

5. Some useful points (0.1, -0.4) , (0.9, -9.5) ,


(1.1, 10.4).
22

y
Example: (2) Sketch the curve 8
1
y = 2""--:-
x - 4 6
I
Solution: \ 4 I
I
y = x2 - 4 is a parabola as shown in the .1'
figure. (broken line) 2

The graphs of both functions


-1 o. X
y = - 2- I.- 2
and y = x - 4
X - 4
are both positive or both negative.
These two meet at points given by
2
X -4 = - 2-1-
X - 4
2
x - 4 = +1

X=~ 15' ~,fj


The graph of y = - - 1 - has vertical asymptotes at x = ~ 2 and horizontal asymptotes as
2
X - 4
the x-axis. Also x = 0 , y = - k. This is reasonably sufficient information to sketch the
graph. (SOLID LINE)

Example: (3) Sketch the curves


y
I
3 I 3
y = x + I and y = - 3 - - y : X +
x + 1 I
Solution: I
3 I
1. Draw the graph: y = x +
I I
1 y =-3--
2. Asymptote of y = - - - at x = -1 X + 1
3
X + 1
~/11 0 2 X
3. x =0 , y =1 is a common point.
4.

.5.
As x -

As y
1.
y = -3--
x + 1
""• y -

>0
0 for

and increases, > 0 I


I
I '\
I I
I
I
I
Solve: y

X
2

=0
+ =+ 1
, -1.2
y
and decreases. I
6. As y <0 and increases, !y < 0 I
and decreases.
7. Two graphs intersect at x = 0 and
X = -1.2
23

(j) Graph of VtW by graphing f(x)

By notmg the followmg, the sketch of y = g(x) = v'fW can be developed.

I. g(x) = Jf(x) exists only for f(x) ~ 0. 2


e.g. Jx - 4 :::> x
2
~4 ~ x >, 2 or x ~ -2

2. g (x) ~ 0 for all x in the domain of g (x).

3. (a) {Hx) < f (x) If f (x) > 1, e.g. .JX < x if x >I
(b) fiW = f(x) if f(x) =1
(c) J'iW > f (x) if 0 < f(x) < I, e.g. {X> X if 0 <x <I

4. ~ flit;)• so f'(x) =0 gives the location of stationary points and

f (x) =0 gives the positions of the vertical tangents.

EXAMPLE: y I
(a) Sketch the curve
3
3
y = f(x) = x - 4x
and hence sketch
(b) y = ./f (x)

1.2 0 2 X
I
\
I
\ I
\ I
\
_.,
I
Solution
-3
' /

3
(a) The graph of y = f(x) = x - 4x = x(x - 2) (x + 2) is as shown in the diagram
(broken line).
(b) Let g (x) = Mx> .
The domain of g(x) is given by f(x) ~ 0. From the graph of y = f(x), this
is -2 ~ x ~ 0 or x ? 2. Remembering that:
(i) g (x) ~ 0 (i;) g (x) = {ff;) ~ f (x) for f (x) ? I
(iii) g(x) >f(x) for 0 < f(x) < I ,
We take the square root of the ordinates of y = f (x) in the domain of g (x).
The.graph of y = g(x) is shown by the solid line. Verify that the maximum

turning point of f(x) and hence g(x) occurs at x = A = 1.15.


24

1.8 Implicit Differentiation and Sketching Curves


2 2 2
Consider the function y =x and the relation x + y = 4 •

For y =x2 ~
dx
2x.

But to find ~ from x 2 + i = 4, where y is not defined explicitly in terms of x,

we have two ways of finding ~ •

2 2 2 2
(a) X + y = 4 (b) We differentiate x + y = 4
Solve for y as it is, term by term.

y = {;7 or y = - j 4-x 2 2x + 2y ~ = 0
-x or ~ = --,-.c;..x=~
~--­ ~ X
dx-~ dx~ dx y
X X
=-- =--
y y

It is not always possible to solve explicitly for y, as in the example x


3 +
3
y - 3axy = 0 •
For these types of functions, called the implicit functions, we find the derivative by

differentiating each term and then solving for ~ •

Example:

(a)
3 3
x + y - 3xy = 0, find ~ (b) x siny = 2, find ~
Differentiate with respect to x Differentiate with respect to x

3x
2
+ 3/ ~- 3 (x ~ + y) = 0
.
• • X CO<; y •
~
dx +
.
Silly : 0

~ 2 2 ~ =-~
dx (3y - 3x) = 3y - 3x
dx x cosy
2
dy ~ _ tan y
dx = 2 X
y - X
25

2
Graphs of y = l(x)

EXAMPLE: Sketch / = x(x - 1) (x - 2)

We have y = .:!: Vx (x - 1) (x - 2) ••• (1)

1. The graph of g(x) = x (x - J) (x - 2) shows


that g(x) >/ 0 for 0 ~ x ~ 1 or x ~ 2 • X

This 1s the domam of f(x) = Vx (x - 1) (x - 2)

2. Relation (1) shows that the required curve is


symmetric dbout the x-ax1s. Hence there is a
loop m the intervdl 0 ,< x ,< 1. As x + oo,
y ... .:!: co.

2 3 2
3. y = x - 3x + 2x
y
dv 2
2y d,c = 3x - 6x + 2 gives the turning point
at x = 0.42.

4. ~ is undefined dt x = 0, 1, 2, so x = 0, x = 1,
X

x =2 are the vertical tangents.

5: IntersectiOn with the axes:


X = 0, 1, 2

y
EXAMPLE: Sketch y
2 = (x - J) (x - 3) 2

We have y = .! (x - 3) v'x'=J ••• (1)

1. The curve 1s symmetric about the x-axis.

2. Intersection with the x-ax1s: x = I, x = 3 0 X

3. The domain 1s x ~ 1. There is a loop between


x = 1 and x = 3, due to symmetry.

4. As x -+ co, y ..,. .! co

5. 2Y ~ = 2 (x - I) (x - 3) + (x - 3) 2
= (x - 3) (3x - 5)
5
The possible cnucal points are x = 3 or 3 .

x = I is the vertical tangent.


26

1.9 Applications

By using the graphs of functions, we can:

(a) Solve an inequality such as cos 2x ~~ or 21 xI - I x - 21 ~I


(b) Find the number of solutions of an equation, such as 2 sin x = x or x = e-x
f (x ) .
1
and by application of Newton's formula x 2 = x 1 -l'{i(}> , fmd a particular root
to any required degree of accuracy.

(c) Solve physical and engineering problems involving equations which are either
impossible or extremely difficult to solve.

EXAMPLE: (1)

211 X

The sketch shows the graph of y = sin 2x and y =~ • The intersections P, Q, R, S


1
are obtained by solving sin 2x = -
2
=> 2x = ~6 , 511 , 1311 , ..!.Z2rgiving x _ ..2!. 5n 1311 1711
6 6 6 - 12' 12 ' I 2 ' 12

Hence the solution of the inequality sin 2x >, ~ is given by ;


2
~ x ~ :; or \ ~11 ~ x ~ \7 11 •
2
EXAMPLE: (2) Solve for x: 2lx I - Ix - 21 ~ 2
To solve this, we draw the graph cf y = 2lxl-lx- 21 as follows:
(i) For x ~ 2, y
1
= 2x - (x - 2) =x + 2
(ii) For 0 < x < 2, y
2
= 2x -- (x - 2) = 3x - 2
(iii) For x~ O, y = -2x -- (x-2) = -x-2
3
Now draw the line y = 2.
This line intersects the graph of
y = 2lx I - Ix - 21 at P and Q.
At P, y3 = 2 =9 -X -2 = 2 ==> X = -4 -4 3 X

At Q, y 2 = 2 =9 3x- 2 = 2 ==> x = ~ = 1~ -2 A
Hence the solution of 2lx I - I x - 21 ~ 2 is
given by x ~ - 4 or x ~ 1~
Note that the sharp corners are located where lx I = 0 and Ix - 21 = 0.
27

EXAMPLE: (3) Find the stationary points of the function y = f (x) = (x + l)(x + 4 > •
X

Sketch the graph and find the domain and range of f(x). Using the graph:

(a) Solve the inequality (x + 1)x (x + 4) ~ 10

(b) Shade the region R between the line y = 10 and the curve
(x + 1) {x + 4)
y = x • Find the area of this region R.

(c) Find the volume of revolution when the region R is rotated about the x-axis.

SOLUTION:
2
f(x)=(x+0(x+4) x +5x+4 y
X X
4
f(x) = x + 5 + - ••• (l)
X

4
f' (x) = 1 -
2X ... (2)

f"(x) = 8 3 ••. (3)


X

f' (x) = 0 ~ x
2
- 4 = O, the
stationary points are where x = .!2.
f"(i) >0 and f"(-2) <0
••• A (2, 9) is the minimum and
B (-2, l) is the maximum point on
the curve.
(i) Vertical asymptote at x = 0.
(ii) Intercepts: y = 0 gives the x-intercepts x = -I, -4.
(iii) As I xI ~co, using (l) f(x) .... x + 5 (oblique asymptote).
The graph can now be sketched. From the graph, we find that the domain is all real
X and the range is y ~ I or y 9 9.

(a) To solve the inequality (x +O (x + 4 ) >,.. 10, draw the line y = 10. The line
X
cuts the graph at P and Q. Since x > 0 for P and Q, (x + 1) (x + 4) >,..lOx
:. x - 5x + 4 9 0 (x - I) (x - 4) > 0 ~ x 9 4 or 0 < x ~ 1 •
2

(b) Area of the shaded region between the line y = 10 and the curve
1
y = f (x) = x + 5 + '!. is given by
2
b X f4 2 4
1 f
A= a (y -y )dx=
2
1
4
[IO-(x+5+x)dx = [5x-y-4logex]
1
A= 5(4- I)- 1 06-1)- 4(log4 -Jogl) = 15- 7.5- 81oge2 = 7.5- 81oge2 sq.u.
2

(cont.)
28

(c) Volume = n
f ab (y 2 - Y 2) dx
1 2
=n f
1
4
x) ]
[ 100 - (x + 5 + 4 2 dx

4
v =n
f
1
2 16 40
(67 - x - lOx - - - - ) dx

3
l X

=n [67x - Tx - 5x 2 + x16
- 40 logex] 41
=n [67(4- 1)- 31 (64- 1)- 5(16- 1) + <-;;--
16
16)- 40loge4]

= (93 - 40 loge 4) n c.u.

EXAMPLE: (4) Determine the number of roots of the equation 2sinx - x = 0 and

find the positive root to 2 decimal places by using Newton's formula


f(x )
1
x2 = xl - 'fi'<xJ

y
SOLUTION:

The graphs of y = 2sinx and y =x 2


intersect at three points where
2sinx =x
i.e. 2sinx - x = 0
X
At 0, X =0
At P, x ~ 1.9 and at Q, x ~ -1.9
y = 2sinx
So, the equation 2 sinx - x =0 has
3 roots.

Let f(x) = 2sinx - x and x


1
= 1.9,
then f'(x) =2cosx - 1.

Substituting f(l.9) = 0.18141, f'(l.9) = 1.8323 in the formula


f(~ 1)
x =x 1 - ~ = 1•9 °·
+ _1•8323
18141
= 1•80 to 2 d ec1ma
. I paces.
I
29

EXAMPLE: (5)
A rod of length L is hung from a pivot A, attached o~r----r------~
to the rim of a horizontal circular disc of radius r.
The system rotates with a constant angular velocity w
about the vertical axis OD. It can be shown that
2
(r + L sin 9 ) w = g tan 9 , where g is the acceleration
due to gravity.
Show by a graphical method that, if 0 ~ 9 ,< ; , then
there is just one value of 9 which satisfies this D
relation.
y

SOLUTION:

We draw the graphs of


Yi = w2 (r + Lsin9) and y 2
= gtan9.
A + Bsin 9
y
1
is of the form y
1
=A + B sin 9
I g tan 9
where A = r w2 > 0 , B = L w2 > 0 B
.I· •.JI B . 9
sm
y
1
is obtained by drawing B sin 9 and then
shifting it vertically up by A units.
A .
~/
1
I
It is seen that these graphs intersect at only one I
I
,point P, where 9 = 9 1 , so there is just one value I
of e-. ~hich satisfies the given relation. 9J 'IT
2
X
30

1.1 0 Miscellaneous Curve Sketching

In this section we sketch the functions involving combinations of trigonometric,


exponential, rational and irrational functions. Several or all of the techniques of
section I. 2 may be required. A sketch of a function should show the following
features:
1. Intercepts 2. Symmetry 3. Asymptotes
4. Critical points 5. Concavity 6. Rise and fall
Do not attempt a scale drawing, unless one is stipulated.

x (x + 1)
EXAMPLE: (1) Sketch y
X - I
SOLUTION:
(i) Intercepts: y
..
,'
y :: 0 gives the x-intercepts B
/
,,
x = 0 and x = -1. ·I /
: I /
x = 0 gives y = 0 :I /
(ii) Symmetry: . I "
:: I/ "
f(-x) I f(x), - f(x) /I
No symmetry. 2 ": \ 2
/ \ y ---
(iii) Asymptotes: '../"' 2 - X - I
Vertical asymptote: x
2
=1
.....
---- ----
We write y = x + 2 + - - (by long
X - 1
division), as x ~ co,
y+x+2 2 2.4 X
•·. y = x + 2 is the
oblique asymptote.
We work out the rough
shape of the curve by
separately sketching
2
y 1 = x + 2 and y 2 =x-:-y
(hyperbola) and then usin
y=yl+y2.

(iv) Critical points:


2
f' (x) = x - 2x 21 ' x I 1
(x - I)
The critical points are given by f• (x) = 0.
x - 2x - 1 - 0 gives x = I .:!: ¥2,·~ -0.4, 2.4
2
f"(x) is too complicated, so we use the signs of f' (x). Now (x - I) 2 > 0 for all x,
x I I, so by drawing a parabola given by the numerator N(x) = x 2 - 2x - I, we find:
For x < -0.4 or x > 2.4, f' (x) > 0 i.e. f (x) is increasing
For -0.4 < x < I, I < x < 2.4, f' (x) < 0 i.e. f (x) is decreasing
Also the points A(-0.4, 0.2), B(2.4, 5.8) are respectively, the maximum and the
miminum points. This is sufficient information for the sketch (solid line).
31

EXAMPLE: (2) Sketch the curve y = 2cosx + cos 2x, 0 ~ x {. 2n

SOLUTION:

The period of f(x) = 2cosx is 2n and the period of g(x) = cos2x is n, so the
function y = 2cosx + cos 2x is periodic with period 2n. A few key points are
necessary to guide us correctly along the curve. These are:
3
(0, 3), <-¥, -1), (n, -1), ( ; , -1}, (2n, 3)

Both f(x) and g (x) are continuous curves for all x.

To find the critical points, we have:

~ = - 2 sin x - 2 sin 2x = - 2 sin x (1 + 2 cos x)

The critical points are given by sin x = 0 or cos x = - ~


2n 3 4n 3
These are (0, 3), (n, -1}, (2n, 3) and (3,- z>• (3,- z>
It is unnecessary to check the nature of these points as we have sufficient points to
sketch the curve.

.. I

··'
. .
I • I •

n
I •

.. ,' I

-1

-2

We can also sketch this curve by the addition of ordinates of the curves
f (x) = 2 cos x and g(x) = cos 2x at the selected key points x = 0, ; , ~ 3
, : , .... '
7n
4 , 2n • This is shown in the figure.
32

EXAMPLE: (3) Sketch the curve Y = 1 + sin x

SOLUTION:

(i) Intercepts:
No x-intercept.
The y-intercept IS (0, I)
-t----
(ii) f(-x) -i f(x), -f(x), No symmetry 1

-n n 0 1T 1T 31T 21T X
(iii) Asymptotes: -2 2 2
Vertical asymptotes are where sin x = -I

0 + n 3n 7n l ln
x = 2n n +
2 , n = , _1
31T , -+ 2 , .•• => (x
,-2·2•2•2•···>
-cosx
(iv) Critical Points: f' (x) = sin x I - I
. )2
(l + smx '
3n iT 5n
f' (x) = 0 when COS X = 0 , giving X
'- 2' 2 ' 2' ..• where we exclude
those points given by sin x = - 1.
f"(x) = sinx. (l + sinx) + 2~osx (l + sinx) cosx
2
v)
(l + sin x)
2
x -_ sinx(l + sinx) + 2cos x , now usmg
f "() . cos 2x = (l - sin
· x ) (l · x)
+ sm
(l + sin x) 3
.
an d cance 11 Ing . x , f"(x) __ .· 2- sinx
1 + Sin
· (l + sin x) 2
,'. f" (x) >0 for all real x, and the curve is concave up.
(vi) f(~) = ~ (All critical points on the line y = ~)
(vii) y >0 for all x (sin x -i - J) and the curve IS periodic with a period of 2n.
The pnncipal branch is in the interval - ~ < x < 32n. The curve is as shown in
the figure.
(viii) A few simple points (0, 1) and (n, J) are useful in sketching.

EXAMPLE: (4) Sketch the curve y = ..,....--.....:....,-


+ sin x y
by imtially sketching y = 1 + sin x. I
I
SOLUTION: I
I
I
After sketching y = sin x, we shift the curve I
I
1 unit vertically up. I
I
The asymptotes are where l + sin x = 0.
These points are easily seen to be
n 3n
x= ... ,-2 ' 2 ' ' "
The maxima of y = 1 + sin x are the minima
-Tf\ -!! / 0 n 'Tl·. :3
:2_.: 2n
,. X
of the reciprocal curve y = +l sin x These \ 2 .... 2
··....._.,· ·.. : .··y =sin x
. ·-·- - .... -- ·..; ·.. - ..... .
3n n 5n
are at x = -
2 , 2 , 2 , ...
I
The graph of y =1 + Sin X
. can now be sketched. (Heavy line)
A few simple points such as (0, 1) , (n, 1) are useful.
--~-----------------------------------------------

33

EXAMPLE: (5) y
2 2
Sketch / =x (1 - x )

SOLUTION:

(i) Intercepts:
y = 0 gives the x-intercepts
-1
X = 0,
! I
x = 0 gives y = 0
(ii) Symmetry:
+ -~
y =- X "1 - X- , -J ~ X~ 1
So the curve is symmetric about
the x-axis.
Let y = f (x) = x
shall eventually combine the
/1 - l
We .
graph of y = f (x) with the
reflection in the x-axis, i.e.
y =- x ~ to obtain the
the required graph of
/ = x2 (1 - x2) •
Now f(-x) = - f(x), so y = f(x) is point symmetric about 0. Putting these
facts together, the curve is symmetric about both axes as well as the origin.

(i-ii) No asymptotes.
(iv) f'(x) = ~- x2 (1 - x2rl/2 = 1 - 2x2 X :/: 1, -1
/1- x2
2
The critical points are given by 1 - 2x =0 ~ x =! 1

Since
r---7
VI - x- > O, the sign of f' (x) depends on the sign of the numerator
12
2
N (x) = 1 - 2x (A parabola) This graph tells us that:

(a) f(
1
) is the minimum, f(- .!.:.. ) - ~, ( -~ - 0.7)
~2 v2 v2
(b) f( ~ ) is the maximum, f (-1-) = -21 •
v 2 V'i
(c) The curve is rising for - Jz < x <~ as f' (x) >0 and falling for

1 1
-I ~ X ~ V"[ and VL ~ X ~ 1

Also, f' (x) does not exist at x =! 1, so there are vertical tangents at
X = _:!: 1.
This is sufficient information to sketch the curve which is a double loop, as shown
in the diagram.
34

EXAMPLE: (6) Sketch / = (x + 1) (x + 2)


X
SOLUTION:

We have: y .:!: J<x + l)x (x + 2)

The curve is symmetric about the x-axis. /


/
(i) Intercepts:
x J 0, so, no y~intercept "c
y = 0 gives x = - 1, x = - 2
X
(ii) Symmetry: I

L et f ( X)
_ ,Ax + 1) (x + 2)
- t X
.I
I

f(x) J f(- x)
No symmetry about the y-axis.
(iii) Asymptote at x = 0.
We work out the rough shape by initially
f\
x + 2 =x + 3 + ~
2
.
sketchmg g (x) = x + 3x x
(shown by the dashed line).
y = ~g(x) is easily sketched by noting that
vgw ~ g(x) for g(x) ~ 1, Vi[X> > g(x) for g(x) < 1.
Hence the graph of y = .:!: JgW can be sketched.
. . (x + 1) ( x + 2)
(iv) Domam: y ex1sts for ~ 0 , x I 0.
X

Multiply by l,
then x (x + I) (x + 2) > 0
The domain is x > 0 or - 2 ~ x .$. - 1.
Since the curve is symmetric about the x-axis,
X
there is a loop in the interval - 2 .$. x .$. -I.
(v) Critical Points:
2
x + 3x + 2
w.r.t. x:
Differentiating /
X
2 2
2 ~ x (2x + 3) - (x + 3x + 2) x - 2
Y dx 2 -2-
X x
2
-~-~
• • dx - 2 x J 0, y I 0 i.e. x I I, - 2.
2x y
Vertical tangents at x = 0, x = - 1, x = - 2
~ = 0 gives x = .:!: .fi as the critical points, i.e. A (1.4, 2.4), B (1.4, -2.4),
C (-1.4, 0.4), D (-1.4, -0.4).
The curve being symmetric about the x-axis, we have altogether 4 critical
points. For obvious reasons the maxima and minima occur at both point'
x = fi and x = - .fi.
There are vertical tangents at x = 0, -1, -2. This is sufficient information to
sketch the curve.
35

1/x
EXAMPLE: (7) Sketch y = e

SOLUTION:
1/x
y = f (x) = e
y >0 for all real x, x i 0. ----------------
(i) f (- x) i f(x), - f (x)
No symmetry

(ii) X =I 0 , y ~1 0 X

The graph does not intersect


the axes.
0
(iii) As x + .:!: oo, y + e =I
The horizontal asymptote: y = 1.

(iv) Critical Points:

~
X
= - +
X
el/x < 0 for all x, x i 0. So the function is strictly decreasing

for either x >0 or x < 0.


(v) Behaviour near x = 0

As x + o+, y • e"' -+ oo, so the y-axis is a vertical asymptote for the right
branch of the curve (x > 0).
As x + 0-, y + e-"' ~ 0 for the left branch (x < 0)
el/x (2x + l)
(vi) f"(x) =~ el/x + el/x • - ; = ~. el/x. (2 + ~) 4
X X X X

f"(x) >0 for x >- ~ , x :/. 0

f"(x) < 0 for x < - 21


So x = - ~ is a point of inflection with the curve concave down for x < - 21 and
1
concave up for x > -
2 but x i 0.
This is sufficient information for a sketch of y = el/x.
36

y
EXAMPLE: (8)

Sketch y = ex cos X for 0~ X,< 2n

y = e-x cos x

3n
n 2 Q
2'1T
X
p
SOLUTION:

(i) Intersections:
X = 01 y = 1
y = 0 gtves
. cos x =0 ~ x =2n , 23 n
(ii) Symmetry:
f(x) /. f(-x), -f(x) y

No symmetry about the y-axis or 0.

(iii) No asymptotes.

(iv) Critical points:


dv -x . X
~ = - e-X cos x - e sm x

= -e-x(cosx + sinx)
dv 3n 7n
~ = 0, when cosx + sinx =0 ~ tanx = -1 ~ x = 4, 4
2
d
~ = e-X (cosx .
+ smx + smx - cosx)
.
= 2e-X smx
·

dx
2
When x =43n , -=-t
d v
> 0, soy has a minimum at x = 43n , P(
3n )
4 , -0.07.
dx
2
~
When x = 47n , 2
< 0, so y has a maximum at x = 47n , Q (7n
4 , 0.003).
dx
d2
Also ~ =0 ~ sinx =0 ~ The points of inflection at x = O, n, 2n.
dx

(v) It is useful to draw the basic graphs y = e-x and y = cosx.


From these two we find that e-x cosx >0 for 0 < X< ; and ex cosx <0
TT 3TT
for
2 < x <2 .
The simple points are (0, 1), ( ;, 0), ( 11, -e-n ) i.e. (n, -0.04), ( 3; , 0) and

( 2n, e- 2n ) = (2n, 0.002)


The sketch is as shown in the diagram.
37

EXAMPLE: (9) I
y '
I
Sketch y = log (1 + cosx) ,
2 --I--- - - ~0~~- t_
e I
I I
- 2n ~ x .$. 2n I I
I I
SOLUTION: I I
I I
(i) Intercepts: I I
I I
y =0 when cosx = 0, i.e. I I
I
+ n + 3n -2n -3n
X=-2•-2 0 n
2 2
Also x = 0 ~ y = loge 4.

(ii) f (x) = f (-x), so the curve is


symmetric about the y-axis.
(iii) Asymptotes:
y ~ -oo when 1 + cos x + 0
The asymptotes are x = ~ n
2 (-sin x)
(iv) ~ the critical points are given by sin x =0
dx 1 +COS X '

x = O, ~ 2n (:. cos x f. -1, we exclude x = ~ n)

_ ( 1 + cos x) cos x + sin x (sin x) _ (1 +cos x) - 2


2 2
+ cosx
2 2
(1 +cos x) (1 +cos x)
2
For x = 0, ~ 2n , 4
dx
< 0, so the critical points are maximum points.

These are (0, loge4), (2n, log 4), (-2n, loge4).


2
.
Ot h er pmnts are ( -+ 2n , O), ( -+ 2
3n , 0)

This is sufficient information to draw the sketch.


38

EXAMPLE: (IO)
(x - 1)2
Sketch the curve y = 3
(x + I)
SOLUTION:

(i) Intercepts:
x = 0, y = I and y = 0 gives x =

(ii) f(x) J f-x), -f(x), so, no symmetry

(iii) Asymptotes:

As x ~ - I+, y ~ oo + and as
x~-1-, y~oo-

x = - I is the asymptote.
As x ~ + oo , y + 0 + -1
I
As x ~ -oo, y -+ 0- Horizontal I

asymptote y = 0.

(iv) Critical Points:


3 2 2
f'(x) = 2(x-l}(x+l) - ~(X+I) (x-1)
(x + 1)
2
2 (x - 1) (x + I) - 3 (x - I )
(x + 1) 4

••• f, (x) -_ (x - I) ( 5 - x) , so t h e N
4
(x + I)
critical points are (1, 0), (5,
2~)
4
Since (x + 1) is always > 0, the
signs of f' (x) depend on the
numerator N = (x - 1) (5 - x).
The graph of N (x) is a parabola as shown in the figure.
This graph indicates that f(J) is the minimum and f(5) is the maximum point.
The sketch can now be drawn with this information.
39

y
EXAMPLE: (ll)
2
(logex)
Sketch y = X

SOLUTION:

(i) Intersections
x J 0, so, no y-intersection X
y =0 => logex =0 => x =I
(logex)2
(ii) f (x) = X
exists for x >0
f(x) ? 0 for all x

(iii) f(x) -j: f(-x), -f(x). No symmetry.


(iv) Asymptotes:
Vertical asymptote x =0
As x + .,, y-+ 0
y =0 is the horizontal asymptote. Verify this by taking x =e 10 , whence

f(e10) = 1~g ~ 0
e

(v) Critical Points:

_ x (21ogx) 1/x - (logx) 2 log x (2 - log x)


~ 21ogx - (logx) 2 e e
dx - 2 2 2
X X X

~ddx = 0 => log x


e
=0 or log x
e
=2
2
:. x =I and x =e =7.I+ are the critical points

For 0 < x < 1, ~ < 0 , so y is decreasing

for 1 <x <e2 , ~


dx > O, y is increasing

x = 1 is the minimum point


2 . h . .
x =e 1s t e maximum pomt.

This is sufficient information to sketch the function.


40

Exercise 1A

1. Sketch the functions y :: f (x) and y = g (x), hence by the addition of ordinates,
sketch the graph of y =f(x) + g(x):
(a) f(x) =x , g(x) = Vx (b) f(x) =sinx, g(x) =cosx
(c) f (x) =x , g (x) =cos x (d) f (x) =x , g (x) =~
2. Sketch separately the functions y = f (x) and y :: g (x), hence by the subtraction
of ordinates, sketch the graph of y = f(x) - g(x):
(a) f(x) = x , g(x) = Vx (b) f (x) = sin x , g (x) = cos x
1
(c) f (x) =x , g (x) =cos x (d) f (x) = x , g (x) = -X
3. Sketch the function y = f (x), hence by reflection in the y-axis, sketch y = f (-x):
1
(a) f(x) = cosx (b) f(x) = siri x (c) f(x) = ex

4. Sketch the function y = f(x), hence by translation, sketch y = f(x + a):


(a) f(x) = cosx , sketch f(x - 1) (b) f(x) =ex , sketch f(x + I)
5. Sketch y = f (x) and y =g (x), hence sketch y = f (x) • g (x) (any other
informaation may be used):
(a) f (x) =x , g (x) =cos x (b) f(x) =x , g(x) = loge x
f(x)
6. Sketch y = f (x) and y = g (x), hence sketch y = grxJ (any other information
may be used):
(a) f (x) =x - 2 , g (x) =x + 2 (b) f(x) =sinx , g(x) =x
2
7. Sketch y =[f(x) ] , given:

(a) f (x) =i -4 (b) f (x) = logex

8. Sketch y = f (x) and hence y = VtW :


(a) f (x) = x3 (b) f (x) = sin x (c) f(x) :: X
2 (4 - X
2)

9. Sketch the following functions by the most appropriate techniques, noting


intercepts, asymptotes, critical points, behaviour near x = 0, when x -+.:!:. oo, etc:

(a) f (x) =e-x sin x (b) f (x) =H- sin (x - l) I (c) f(x) =x 2 e-x
4
(d) f(x)::x~x--21) (e) f(x) = ;x~x- -/) (f) f(x) = ~
X - 1
2 x 2 (2 + x) 3
(g) y :: 2- X (h) f(x) = (x - 2) + 1
41

10. (a) Show that x _ 2 - x ~3 + 3 = (x ~x:) (/ + 3)


(b) Find the vertical and horizontal asymptotes of the curve y = f (x) = (x _ ) (x + )
3i- 7
2 3
(c) Find the turning points and determine their nature.
(d) Sketch the curve.
2
7
(e) Using the sketch, solve the inequality 0 < (x ?x ) (x + 3)
2
< 3

(f) Find the area between the curve and the x-axis between their points of
intersection.

11. Solve the following inequalities by graphical methods:


(a) Sketch separately the $raphs of y = 2lx - II and y =I x + 21, hence solve
the inequality 2lx - J I ~ lx + 21 •
(b) Sketch the function f(x) = cos2x , 0~ x ~ 2n, hence solve the inequality
1
cos2x ~ 2 .

2
12. Sketch the curve / = x(x - 2)
(a) Find the area of the loop in the exact form.
(b) Find the volume of the solid when the region inside the loop is rotated
about the x-axis.

13. The chord AB subtends an angle a at the centre


0 of the circle, where 0 < a < 1T

(a) Show that the perimeter p of the minor


segment APB is given by p = 2r sin (~) + r a
(b) If p is half of the circumference, prove that
2 sin ( ~ ) =n - a p
(c) Find a, in this case, by separately sketching the graphs of y = 2sin (;)
and y = 1T - a ' 0 < a <1T.
Give your answer to 1 decimal place.

(d) Using one step of Newton's method, find the value of 9 to 2 decimal
places. (Hint: f(9) = 2sin (;)+a-n)

14. By sketching the graphs of y =l + 2 and y = ! on the same axes, find the
X
approximate root of the equation f(x) = x3 + 2x - l = 0. Hence, using Newton's
method, find to 2 decimal places a better root of f(x) = 0.
42

3x + 2
15. (a) Sketch the function y : f(x) : ( 2x _ 1) (x + J) , showing the asymptotes
and the points of intersection with the axes.

(b) Express f( x ) m
. part1a
. 1 f ract1ons,
. .
1.e. ( 2x 3x
_ I)+ (x
2 + 3) A
= 2X":-r B
+ X"+3

(c) Shade the region whose area is given by the integral J: t (x) dx. Find
this area.

16. By sketching the graphs of y : e-x and y =x on the same axes, find to one
decimal place the approximate root of f(x) : x - e-x = 0. Hence by using
Newton's method, find to 2 decimal places a better root of f(x) : 0.
43

Exercise: 1B

Sketch the following curves, showing the CRITICAL points and labelling the axes
carefully. All logarithms are to the base e.

1. (a) y IX - 21 (b) y \2x- 31


(c) y x- lx I (d) y IX I - X

(e) y lx + 1 I + \x - II

2. (a) I Yl = X- (b) lxl - IYI = 2


(c) IXI + IYI (d) IYI = lx + II
(e) IY - 1 I X (f) IY I IX I + 1
(g) IX + YI 2 (h) y = IX + 21 + X

3. For all real values of x, the symbol [x] denotes 'the greatest integer not
exceeding x'.
Sketch the graph of y = [x], lxl ~ 3

/.j.. Given that 0 < a < b, draw the sketch of y =I x - aj for lx I ~ b. Hence

evaluate rI
-b
X- a! dx

5. y = max (x, 1 - x) where max (a, b) denotes the greater of the two numbers
a and b, i.e.
max (a, b) a if a >b
b if a <b

6. (a) y x + [x] , -2 ..$, X ..$, 2

(b) y x - [x] , where [x] has the meaning given in Exercise (3)

2
7. (a) y = /.j.x - (b) y
2
(c) IY I = l.j. X (d) y
44

8. (a) y =x 3 - 2x
2
- 4x + 4 (b) y=x(x-1)
3

(c) y=(x
2
-O(x
2
+1) (d) y =41 (x - 2)
2
(x
2
- 1)

(e) y =(x 2 - 1)
2

X 2x - 4
9. (a) y - --
- X - 2
(b) y ="""'i("':i:'"2
2 X
(c) y = {d) y = 2
(x + 1)2 (x - 2)
2
X
(f) 1
(e) y =x:~ y =-2--
X + 1
2
1 X
(g)
Y =(x - 2) (x + 3)
(h) y =- 2 --
X + 2
(x + 2) (x - 3)
(i) Y =(x + 1) (x - 2)

2
x (x + 3) X - I
10. (a) Y =(x + 2) (x - 2)
(b) y=-3-
X

2
X - 4
(c) y =--x:;:--3

2 =x3 2 2
11. (a) y (b) y =9x (x - 4)

2
2 X + 1 (d) 2 X
(c) y =x (x - l)
y =x:-1
2
2 2 -~
(e) y =4x (x - 2) (f) X - 2
4 - y
45

2 2
12. (a) y = sin (2x) , (-11 ~ x~ 11) (b) y = cos (2x) , (-11 ~ x ~ 11 )

(c) sin (x + y) = 0 (d) cos(x + y) = 0

(e) y = lsinxf lXI ~ 211 (f) y = sinJx J, Jxl ~ 211


'
sin x . ( . -1 x )
(g) y =-- (h) y=smsm
X

(i) . -1 ( smx
y =Sin . ) (j) . (cos-1 x )
y = sm

(k) . -1 (COSX)
y = Sin
2
(1) y = M. X
(m) y = /cos x

2
X X
13. (a) y=e (b) y = xe

X -X
e + e X- 1
(c) y = (d) y=--
2 X
e

-X
(e) y = [1-x]
- - ex (f) y = e COS X
1 + X

(a) 2
14. y =log Jxl (b) y = loge (x - 1)

(c) (d) X
y = xlogx Y = logx

(e) y rx = logx (f) y = llo~x I


2 2 2
15. (a) X + 2y - I = 2(x + 2y) (b) X (y - 1) = y

(c) (x + 3) (y - 2) = 1 (d) (x + 2) (y + I) =1
46

MISCELLANEOUS

2
16. Determine the domain and the range of the function y = loge (x - x ) •
(a) Find the nature of the turning points and
2
(b) Hence sketch the curve y = loge (x - x )

x -x
17. A function is defined by f (x) = e - e .
x -x
e + e

(a) Find (i) lim f (x)


x-+oo
(ii) lim f (x)
X- -oo

(b) State the domain and the range of the function.

(c) Determine the stationary points and their nature and hence sketch the curve
y = f (x) •
1 1
(d) Determine the inverse function y = r (x) and draw the sketch of y = r (x).

2
18. A function is defined by f (x) = xsin-l (x )
(a) State the domain and the range of f (x)
(b) Show that f (-x) = - f (x)
(c) Find f' (x)
(d) Describe the behaviour of ((x) near x =0 and x =1
(e) Hence sketch the curve y = f (x)

19. Sketch each of the following:


(a) y = (x - 1) (x - 2) (b) y = I(x - 1) (x - 2) I
(c) Y = Ix - 1 I (x - 2) (d) y = (x - 1) lx - 21

(e)
Y = (x - 1) (x - 2)
47

20. A function f (x) = logex X ) 0 •


X

(a) Show that the curve y .= f (x) has a maximum at x = e and y = .!.e
(b) Discuss the behaviour of f (x) near x =0 and when x -+- co

(c) Sketch the curve y = f (x), by considering the points at X = 1, 2, 3, e'1 10

2x - 8
21. Sketch the graph of f ( x ) = (x _ 3) (x + 1) , showing the points of intersection

with the axes, the turning points and asymptotes.

2
22. Sketch the graph of the function y =x - x + 1
2
. . .
, showmg the pomts of mtersectton
.
(x - 2)
with axes, the turning points and asymptotes.

2x
23. Sketch the graph of y = 2 - - ·
X - 1

24. Prove that the curve y = x 2 ex has a minimum turning point at (0, 0) and a
4
maximum turning point at (2, > • Hence sketch the curve.
2e

x2
25. Sketch the curve y = - - - , showing the vertical and horizontal asymptotes,
2
X - 1
turning points and inflexions.

2 2
26. Sketch the curves y = 10 x e-x 12 and y 10 ex '
2
on the same axes, showing
the turning and inflexion points.

2x
27. Sketch the curve y = - - - , showing that it has a minimum at A (-1 ,-1) and
2
+ X
a maximum at B (1, 1) and inflexion at 0 .
48

1
28. Sketch the curve / = (x + ( 2 _ x) , showing the vertical and horizontal
0
asymptotes as well as the turning points.

29. Sketch the following curve, showing the critical features:

y = x2- lx I.

2
Sketch the curve y
2 x
= - - - , showing the vertical and horizontal asymptotes.
30. 2
X - 4
(Hint: x = o, y = 0 is a point, a lonely point detached from the rest of the
graph.)

2
31. The following curves are of the type y = f (x) • Sketch them.
2 2 y2 = (x - 2)3
(a) y = (x - 1) (x - 2) (b)

(c) / = (x - I) (x - 2) (x - 3) (d) / = / (x + 2)

(e) y
2 =X- X
3

32. Using the properties of the reciprocal functions, sketch the graphs of the following
pairs of functions on the same set of axes.

(a) I
(b)
2 I
y = X - 2 and y = i("':"2' y = X - 4 and y =-2--
X - 4

2 1 3 1
(c) y = 9 - X and y = - - 2
9- X
(d) y =X -1 and y = -3--
X - I

1 1
(e) Y=lxt-2 and y=~ (f) y = (x - 1) (x - 2) and y = (x _ 1) (x _ 2)

33. Use CALCULUS to find the turning points, the sign of ~ etc. to sketch the
following curves:
2
2 3 X
(c)
2 3
(a) y =x + I (b) y = -2-- y = X
x + 4

·
34. Fmd the turmng .
pomts o f t he curve y = I x- 2I - 3I an dh ence s ketc h.1t.
X X

4
35. Find the turning points of the curve y = 8x - and hence sketch it.
2
X
49

CHAPTER 2 INTEGRATION

Almost any function y == I (x) can be differentiated, but unfortunately, the reverse
process of integration is not only difficult, there is no systematic method for integration.
Even worse there are functions which can never be integrated, for example:
2
e-x
sin x
-2-
x
So first we have a standard Jist of integrals and then Jearn a few more tricks.
NOTE: All logarithms in this chapter are to the base e.

2.1 Standard Integrals

J.
d rxn + I] =
dx l""'j)""';T
n
X
- f xn dx
X
=n-:;:1
n+l
+ c' 1\ 1- -1.

2. dx

d
11
d [<ax + b) + ]
a (n + I)

pin(a~
1
:: (ax + b)n
-J ( J (ax + b)n dx ==
(ax + b)n + 1
a (n + I)
+ C, n 1- -1.

3. + b)] == cos(ax +b) -+ cos ax + b) d x ==


sin (axa + b) + C
dx

4.
d [cos (a~ + b)]
dx
== -sin (ax + b) -+ J sin (ax + b) dx ==
cos (ax + b)
a
+ c

5.
d
dx
[tan(a~ + b)] = sec2(ax +b)
- J sec 2 (ax + b) dx ==
tan (ax + b)
a +C

6.
d [ax+bl
dx
e
a
:: e ax + b -+ f eax + b dx == e
ax + b
a
+C

7. d
dx
[ e f (x)] = f'(x)ef(x)
- Jf'( xe) f(x) dx=e f(x) + C
50

d 1
8. -d (log x)
X e = X
loge x + C

9. -+- J t;{:/ dx = loge f(x) + C

J --
1
- dx =.!.log (ax +b)+ C
ax + b a e

u.
- . -J
=Sill
X
a +C

12.
a
2
a
+ x
2 -J a
2
I
+ x
2 dx = aI tan-1 a
X
+ c

13.
x2 -a 2
2a
Jx 2 -a
I 2 dx = _I_ log
2a [ ~lc
+ cij X

for x > a, a > 0

14. d 1oge [a+x]


dx a-:lZ 2
a - x
2a
2 - Ja
2 1 2 dx = _1_ log [a + x] + C
- x
2a e a - x
for x < a, a > C

15. d I (
dx oge x + vxL:!:. a" -~
12:2) _
x
I

_a
-
16.
d
-d (a ) = a
x
X X
• log a
e -J ax dx = lo:x a
e
+ C ' a> 0

d 2
17. dx(cotx) -cosec x -4- J cosec 2 x dx -cot x + C

d
18. dx (sec x) sec x tan x ~ I sec x tan x dx sec x + C

d
19. dx (cosec x) -cosec x cot x 4 I cosec x cot x dx -cosec x + c

The following two formulas should be noted:

20. Jsecxdx log (sec x + tan x) + C


e

21. Jcosecx dx - log (cosec x + cot x) + C


e
51

2.2 Change of Variable: Substitution

We are easily tempted to apply a known method to new situations.


2
=;
3 . 2
= -sin3-
3
x
J
x dx + C , so why not
Jsm x dx + C ?

Now
d [sin~x] =
. 2
Slll X COSX
. 2
not sm x.
dx '
2 2
Consider y = e
X
then ~ 2x e
X
dx

But given J2x ex 2 dx , how do we proceed to find such integrals?


The method of substitution, leading to a change of variable, is used to solve such problems.
The symbol I will be used to represent the integral in each question.

WORKED EXAMPLES

I. f 2x ex
2
dx

Solution: Solution:

Let U = X2 Let u = x2 - 4

du du
2x 2x
dx dx

2
J2x ex dx Substitute for
2
2x and x - 4 in terms of u

du
feu dx
dx
f)u du
dx
dx

Jeu du I u
-1/2
du

e
u
+ c 2 ru + c

~+ c
2
X
e + c 2

Thus f f(x)dx Jg(u)du by using either u = r$(x) or x 9(u)

In either case f g(u)du is easy to integrate.


52

Integrate the following:

(3)
.
-1
~ l d
J~ dx (4) (5)
J X loge X X

Solution:

(3)
f. Sln-1 X
dx Let u =
. -1
Sin X
j 1 - x2
du 1
Ju. dudx • dx
dx
~
= Judu
2
u
2 + c
21 (sin-1 x )2 + C

(4) Jexdx Let u = e


X
e2x + 1
du X
dx e
du
J dx2. dx
u + 1

J- 2du- -
u + 1

-1
tan u + c
-1 X
tan (e ) + c
(5) Jxlog
dx
x
du
Let u = 1ogx , dx = xl
e

I lo!x
.!.X dx +- Watch this: -+ dx
X
= du

J~ . du

loge u + c
log (log x) +
e e
c
53

Exercise 2A (change of variable)

Find the indefinite integrals:

I. (a) J 2x ( / + 1)
3 dx (b) f 2x (I - x2) 4 dx
J 4~
3 3
(c) dx (d) J 4x dx
X + I (x 4 + 1)2

(e) Jx v'i-7 dx (f) J2x d\


I + X

(g) J x dx (h) J dx
~ IX ./I - rx

2. (a) Jcos 2 x sinx dx (b) Jsin 3 x cosx dx

(c) Jtan 4 x • sec 2 x dx (d) fcot 4 x cosec 2 x dx


(e) J -si~x dx (f)
Jo + cosx)
4
sinx dx
COS X

2 2
(g) Jsec 5 x tanx dx (h)
Jo + tanx) sec x dx

3
3. (a) fx2 e X dx (b) J e-X dx
I + e2x

. -1
(c)
J e sm x dx
(d) J ex dx
~ ~

4. (a) J~ logx dx (b) x


Jl (logx) 2 dx

(c) J dx (d) J dx
x (iogx) 3 x (I +logx) 3
54

5. (a) I. SlnXCOSX ecos 2x dx (b) I -sinx ~x


I +cos X

(c) I 2 sinx cosx dx (d)

6. Evaluate:

I~
Tl/4 3
(a) 2x (x
2
- 3)
3 dx (b)
I 0
sin x cosx dx

(c) I 3
x dx
(d)
f
e dx
xlog x
0~ 2 e

(e)
I~
eX dx (f) Irr/ 2
cosx dx
e
X
+ I 0 (I + sinx) 2

2.3 Integration by Parts

d dv du
The reverse process of the product rule dx (uv) u dx + v dx is the method of
integration by parts.
From the above,
dv d du
u-
dx dx (l•v) - v-
dx

I dv • dx
u • dx uv - I v du • dx
dx
.•. (I)

or, I u dv uv - I v du ... (2)

WORKED EXAMPLES

dv x
Let u = x, dx = e
du
• . dx =I , v = Je X
dx = eX
Using the formula I uv' dx = uv - I v u' dx

I x ex dx x ex - Iex • I • dx

x ex - ex + C
55

2. Jx cosx dx 3. Jlogx dx
U =X ,
dv
dx = COS X Write flogx dx as f (logx) • 1 dx
u' =1 , v Jcosx dx and let u = log x dv
, dx =I
sinx • du
• • dx = x' V =X
Jx cosx dx :. Jlogx dx
uv - Jvu' dx = uv - Jv u' dx
x sinx - Jsinx dx (log x) x - Jx • ~ dx

x sinx + cosx + C x logx - x + C

Warning: A bad selection of u or v can lead to a disastrous situation.

For example in f x logx dx , if you choose u = x , ~~ f


= logx , then v = logx dx is
2
dv
= 2x
not easy to integrate. Again in
Jx . cosx dx, if u = cosx , dx =x , then v

~ Jx2 sinx dx ,
2
Jx cosx dx = uv - Ju'v dx = ; cosx + a situation worse than

Jx cosx dx •
4. Jexcosx dx Let u =ex dv
, dx = cosx
u' =ex , v = sinx
Jex cosx dx
uv - Jvu' dx
X.
e smx - JX·
e smx d x ••• ()
1

We integrate Jex sinx dx by parts again

ex (-cosx) - Jex (-cosx) dx


-ex cosx + I where I
Then from ( 1)
X .
e smx

2I ex (sinx + cosx) , divide by 2


x cos x d x = 1 e x(. C
Je 2 sm x + cos x) +
56

Note: It is important to note that the given integral may occur while integrating it by
parts, but actually this occurrence helps to find the solution, as seen above.

3
Integrals with l or x require repeated use of the 'integration by parts' method. For
example to find:

(5) Il cosx dx u
2 dv
x , dx = cosx

2
x sinx - I 2x sinx dx u 1 = 2x , v = sin x

Again I x sinx dx = x (-cosx) - I I • (-cosx) dx u =x , v1 = sin x


-x cosx + sin x u 1 = I , v = -cosx
2
x sinx- 2(-xcosx + sinx) + C
2
x sinx + 2xcosx- 2sinx + C

-1
Find
(6)
I sin x d)(

Write = I 1
(sin x) • 1 • dx Let u =
. -1
Sin
dv
x , dx =

= uv - Iv u 1 dx
I I
U=T2,V=X

X Sin
. -1
X - I x dx .•. (I)
v 1-x~
~
Now for I x dx 2 du
Use u = I - x , dx = - 2x
~
du
+- or xdx =- T

-~I~ [It's easier to substitute


for x dx , than x itself]
-~ Iu-
112
du

-Vu
- vr:7
. -1
X Sin X + + c [from (I) ]

So it seems there is no end to the number of tricks you may be required to play in
Integration. But that is what makes it so fascinating!
57

Exercise 28

Integrate the following:

1. (a) I X C-x dX (b) I x 2 e X dx (c) I ex sinx dx

2. (a) I x cos 2x dx (b) I 2


x sjnx dx (c) I 2
x cosx dx

3. (a) Jx 2
sec x dx (b) I 2
x sin x dx (c) J x cosec 2 x dx

4. (a) J x 2 logx dx (b) f rx logex dx (c) J (logx)


2
dx

5. (a) f cos-1 x dx (b) I tan-1 x dx (c) I x tan-l x dx

6. (a) I log(x
2
- 1) dx (b) I e-2x cos3x dx (c) I x (loge x)
2
dx

2 2 2
7. (a) Jx tan x dx (Hint: tan x sec x-1)

(b) I 2
x cos x dx (Hint:
2
2cos x = 1 + cos 2x)

8. (a) J~dx (Hint: Write I<~l). 1 dx)

(b) J~dx
9. Evaluate the following definite integrals:

(a) I 1

0
x ex dx (b) r ' 2 x cosx dx
0
(c) f e xlog x dx
1 e

(d) ( ' 2 sin-1 2x dx (e) r'2 excosx dx (f)


I n/2 2
x sin x dx
0 0 0

10. (a) Use the substitution y = J2 . 9 to find


Sin
r'3 ~dy
0

. -1
(b) Find 1: Sin
v'J:Y"
:i d
y
58

11. (a) Use integration by parts to show that

I cos-1 (X
6 ) dx = X COS
-1 (X)
6 -

11
6n. 1
+ c

(b) Show that ( tan- 1 u du 4 - 2 loge2


0

12. Evaluate:

(a) r'30
x sec 2 x dx (b) I; log 10 x dx (c) I
0
11
x cosx dx

4
r r
2
(d) Ie I
log x dx
e
(e) x logex dx (f)
e
dx
xlogex

(g) r'40
x sin 2x dx (h)
r - 4- logex

X
dx (i)
Io
1
t e- 2 t dt

I
I~
log2 -t 1
-1 . -1
(j) u tan u du (k) te dt (I) Sin t dt
0 Io
11/4 2
(m)
I 0
x cos x dx
2 1
(Hint: cos x = (I
2 + cos 2x) )

2.4 Trigonometric Integrals Powers of sinx, cosx, tanx

. 2
Using sm x = 2I (I - cos 2x) and
2
cos x = 2I (I + cos 2x) , we find:

~ I (I - cos 2x) dx !
2
(x _ sin 2x )
2
+ C

~I (I + cos 2x) dx I ( sin 2x)


2x+-2-
+ C

I. Sin
2X • Sin X
. dX
u cosx

I 2
(I - cos x) sinx dx
du
dx
-sinx

I -(1 - u ) du
3
2

u
-u + T + C
1 3
-cosx +
3 cos x + C
59

4. I 3
cos x dx I
(I - sin x) cosx dx
2
sinx = u

du
dx = cosx

.
smx- 1 sm
. 3x + c
3

~ I (I - 2cos 2x + cos 2x) dx


2

~ I[l- 2cos2x +~(I+ cos4x)] dx


~ [~ x - sin 2x + ~ sin 4x] + C

6•
ICOS
. 2Xd X
5 X Sin
u = sinx

I 4
COS X • Sin
. 2X • COS X
dX du
dx = COSX

I u2 (I - 2 u2 +

3 5
u 2u u7
3--5-+T + c where u = sin x

7. Itanx dx
I sinx
cosx dx = -log cosx + C

= log secx + C
8. I 2
tan x tanx dx

f (sec 2 x - I) tanx dx u = tanx

I 2
tanx sec x dx - Itanx dx

I u du + log cos x + C

.!. tan2x + log cosx + C


2
60

I 2 2
tan x.tan xdx

2 2
I (sec x - l) tan x dx

J 2 2
sec x tan x dx - I 2
tan x dx

2 2
Iu du- I<sec x-l)dx u =tanx for the

first integral
31 tan 3 x - tan x + x + C

10. Isecx dx
I secx (secx + tanx) dx
secx + tanx

2
I sec x + secx tanx dx
tanx + secx

f'(x) dx f(x) =tanx + sec x


J fTxl
log (tanx + secx) + C
e
2 2
Jsec x (l + tan x) dx u = tanx

2
J (l + u ) du
3
u
u + T + C , where u = tan x
3
tanx + .!. tan x + C

All the even powers of sec x and cosec x are integrated as above (odd powers by
integration by parts).

_ J _ (cosec x + cot x) cosec x dx f' (x)


12. J cosec x dx cosec x + cot x
- loge (cosec x + cot x) + C
f TIXf dx = log f(x)

Jsecx • sec 2 x dx Integrate by parts

secx tanx Itanx secx • tanx dx

secx tanx I 2
secx (sec x - I) dx

secx tanx J
3
sec x dx + log (secx + tanx)
Solve for f 3
sec x dx , then

f 3
sec dx 2
I
sec x tan x +
1
log (sec x + tanx) + C
2
61

2.5 Use oft = tan(x/2)

Rational Express1on 111 sinx and cosx

Expression of the form Jacosxd:bsinx +C can only bt> integratt>d by special

. X
SUbStltUtlOil t = tan
2
dt I 2 X 1 2 = 2dt 2
We have dx = 2 sec 2 2 (I + t ), so dx
I +t

2t
Also sinx cosx
I +t 2 '

WORKED EXAMPLES

(1) Find: J3 1
+ 2cosx dx

2 dt
-.--::7"

J_2.5!!.._i
5 +

2
./5 tan-1 [ {5
t + C J
2
tan
-1 tan-1 -
2
X]
v0 [ + c
./5
dx
(2) Find:
J sinx + cosx
2 dt
1:7

2 dt
J1+2t-i
J 2 dt 2
2 - (1 - t)

(cont. next page)


62

J-2 d~ y = 1 - t
2- y

2 J-;Ly - 2

2
~ log n: ~I + c

71
1
log ,1-t-/21
1-t+Vi'
+ c

1 1- .f2- tan~
log +C
1 + /i.- tan~
.f2 e

(3) J2 sinS
- cos9
d9

Jd~ U = 2 - cos9

dU .
loge U + c d9 = sm 9

loge (2 - cos9) + C

(4 ) J3 COSX
+ 2cosx
d
x

I 3
Arrange numerator as
2 (3 + 2cosx) - 2
I 3
(3 + 2cosx) -
2 2
J 3 + 2cosx dx

J[ 4 - 3 + icosx] dx

3 -2
I x-
- - t an-I l-tan ~2 ) + C using example (I)
2 2".f5 15

X
2 -./5 tan
3 -J
[tan~]
7'5 + c
63

Exercise 2C

Integrate the following:

TT /4
4
l. 2.
J0 cos x dx

TT/4
4 .2 2 d
3. J
0
tan x dx 4.
Jsm xcos x x
5 . 2 X dX
6•
J COS X Sin

7. J sin
512
x cosx dx 8. Jsin52x dx
COS X

9.
3
J cot x dx 10. Jcosec 3 x dx
4
11. J cosec x dx 12. Jcos 3 2x dx
3
13. J tan 2x dx 14. f 3
sin 2x dx

15. J cos
213 5
x sin x dx 16. .4 3 dX
J Sin X COS X
2 .3
17. Jc~s4 x
Sin X
dx 18. f sm /
COS X
dx

19
• J co~x dx 20. f 1- ~in x dx

n/3 TT /2 d
21. f 0 1 + sinx dx 22.
IO
-,.2---"'x:.:..__-
+ cosx

dx 24. J 2 sinxcosx dx
23.
J sinx - cosx
+

TT/2
25 •
J0 3 + 5cosx dx
64

2.6 Reduction Formulas

A reduction formula is one that reduces the index (usually an mteger) of the integrand.
By repeated applications the index is reduced to either one or zero. By using the method
of integration by parts, we can establish the following reduction formulas:

I.
Isin
n
X dx = - nI Sln. n-1 X • COS X +
(n - I)
--n-
I.
sm
n-2
x dx

2. Jcos" x dx -n cos
n-1 . n - 1
x. sm x + - -
n
Icos n-2 x dx

3. f tan" x dx tan
n-1
n - 1
X I tan n-2 x dx

Solution:

Let In I sin" x dx , so that In_ 2

Using integration by parts:



sin
n-2
x dx

I. sm n-1 x. smx
. dx

I sin n-1 x • d
dx (-cosx) dx
. n-1
[ 1et u = sm dv . ]
x , dx = sm x

sin n-1 x • (- cos x ) - I( ) .n - 1 sm n-2· x cos x • (- cos x ) dx

- cosx • sinn-l x + (n - 1) I 2 2
sinn- (I - sin x) dx

- cosx • sinn-l x + (n - I) In_ - (n - 1) In


2

Solve for In , then:

I
n n
• sinn-l x cosx + !!....:....!
n
I
n-2' where I
11
Isin" x dx

You can see why these formulae are called the 'REDUCTION formulas'. The index n is
reduced to n - 2 or n - 1. For the example above, starting with, say, n = 10, we shall have

n = 8, 6, 4, 2, 0, which means you only have to evaluate I sin° x dx = fdx • If n is originally

odd, then you have to evaluate only I sin x dx, etc. Proof of I cos" x dx is similar.
2 2 2 2
For tanxdx, we write In= Jtan"xdx = Itan"- x(sec x-I)dx = Jtan"- x sec xdx- In_
2
n-1
tan x
--n:l - In-2
65

WORKED EXAMPLES

1. Jsin6 x dx 5
=-t sin x cosx + i 14

- ~ 3
sin x cosx + ~ 12

- 21 smx
. cosx +
2
1 I
0

J dx = x

6
Jsin xdx - 61.5
Sln X COSX +
5(1.3
6 - ij Sin X COSX + ij
31)
2

- 61.5
Sln X COSX -
24
5Sln
.3X COSX + 851 2

- 61Sln
. 5 X COSX -
24
5Sln
.3 X COSX + g5 (
-1 .
2 SlnX COSX + 211)
O

6 5 3
Jsin xdx = -tsin xcosx- 2!sin xcosx- ·1!·sinxcosx + 1!x

k n/2
Since [sinm x cos x} ... 0, we have
0
n/2 n/2
J0 6
sin x dx = 1 [x} ! 0
5n
32

So, if the limits are 0 and i , the easier method is:


n/2
J0 6
sin x dx [-
1 5
6 sin x cosx
1 Tf '
2
+ 65 14 = 5614
0
3 1 3 1 3 1 Tf
14 = ii 2 ii 2 1o ii 2 2
n/2
J0 sin 6 x dx
5
6 ii
3 1
2
Tf
2
5
3i . Tf

n/2
2.
J0 sin 8 x dx
7
8 16
7
8 . 65 3
ii 2
1 Tf
2 =
35n
256
n/2
You must show that [sinm x cosk x} = 0. Also remember you can not use this easier
0
n Jn/3 6
method for limits other than 0 and
2 . For 0 sin x, we must use the long
method.
66

Exercise 20

Find a formula finding In in terms of In-l : (for examples 1-4)

1. I
n J
n x
x e dx
f X
n e-2x dX

3. I 4. I
n n

Find a formula reducing In to In_ : (for examples 5-9)


2

5. I
n
Jcot" x dx , hence find Jcot 6 x dx
6. I Jcos" x dx , 6
hence find (a) Jcos xdx (b)
Jn/2 cos8 x dx
n
0

7. I
n Jsin" x dx , hence find (a) r ' 2 sin 8 x dx (b) r'4 sin 4 x dx
0 0

8. I
n Jtan" x dx , hence find (a) Jtan 5 dx (b) r ' 4 tan 8 x dx

r
0

9. I
n
J(sin-1 x) n dx , hence find (sin- 1 x) 3 dx (Hint: x = sinU)
0

o xn 1/2
10. Show that Un

evaluate u3 •
I-1
(1 + x) dx , hence

mx n
11. If In =
Je tan x dx , show that

I emx tann-1 x m
n - -
n- 1
-n:l

12. Find Jx4 ex dx , using the reduction formula developed in Exercise 1.

13. If I
n Jsec" x dx , prove that

n-2
sec x tanx n- 2
I + --·I
n n - 1 n - 1 n-2

Hence, evaluate r ' 4 sec 6 dx

14. Prove: I n -x
0
x e dx = -e-x . x n + n I x n-1 • e-x dx

Hence, find: Jx3 e-x dx


67

2.7 Trigonometric Substitution

Integrals involving expressions like


/"=>2
ll"aL+XL J etc. are simplified by

substitution. x=atane, x = a sinS , x = a cosa etc.

WORKED EXAMPLES

1. Find J /: 2 1 2 dx
a + x
dx
X a tanS ,
d9

2 a
a sec a d9
f a seeS
Fig. I
JseeS d9
loge (sec a + tan 9) + c
log ( 1:2? + ~) + C

log ( .;;:z-;-;!
a
+ x ) + C

log (x + .;;;z-:-;;z ) - log a + C


e
log (x + ~) + K

2. Find J ~ 2dx 2 , x >a >0


x -a
Let x = asec9 a sec9tan9

I = I v~/
a seeS tanS d9
2
(sec a - 1)
2
1 + tan 9

Jsec a d9

log (seeS + tanS) + K

x jx2
log ( -
aa2
- I)+ - + K

log (x
e
/x27)+ + C

[Note: log a is a constant]


68

Exercise 2E

Use x =a sin 9 for Ia2 - x2 , x =a tan9 for /a2 + x2 and x =a sec9 for

F-2 x a , to find:

1. J dx 2.
dx 3. I 2
x dx
~ J
X 2 9-
£ Xi ~
4. J?dx
5.
2
J x dx
2
6. J/xQ dx
/25- x

J~
2
2
7. J /a -l dx 8. J x dx 9. dx
;:G
10. I /25- x
2
dx 11. I {4 +
dx
x2)3/2
12.
I dx

x./:7-

13. fi J 2 - dx
x 4
14. f ~X
X d 15. J9dx

16.
I{.6)3
dx 17.
J J)2_ 4 dx 18. JJT7.
2 dx
X

19. f dx 20. f~dx 21. f.[;.2:9


x2 + 3 dx
X~ X

2.8 Integration of Rational Functions

TYPE 1: We shall consider the rational function a~{:) b where P{x) is a polynomial.
We divide P{x) by ax + b by long division and write

P{x) = Q{x) + ~b where Q{x) is a polynomial and R is a constant.


ax +
69

EXAMPLES: Integrate the following:

(a) B= ~ ~ dx (b) J xx~ 2 dx (c)


J
3
x - 1
'"'X""'7T dx (d)

SOLUTION:
1
(a) Divide 2x - 1 by 2x + I 2x + 1 f2i(':"l
2x + 1
~
~dx
J 2x + 1 Jo-2x:ldx
= x - loge (2x + I) + C

When the denominator is linear, we can divide by inspection. For example,


2x - 1 2x + 1 - 2 _ __2_
1
2x + I = 2x + 1 2x + 1

(b)
J ~dx
X - 2 X- 2
x
)x3
2
+ 2x + 4

J[x 2x
2
+ + 4 + x _2
8
J dx 3
x - 2x
2
2
3 2 2x
x + x + 4x + 8 log (x - 2) + C 2
3 2x - 4x
4x
3 4x - 8
(c) Observe that x - 1 can be factorised as (x - I) (x 2 + x + I)
-'-&
J
L.:.l_
X - I
dx = J<i + x + l)dx =

3
Of course, we can divide x - 1 by x - 1 and get the same result.

(d) To make division easier, let U = 2x + 1

3 :. ~~ = 2 _,. dx = ~
I (U - I) dU
16 u when x=O, U=l
X : 1, U : 3
3 3 2
1 u - 3U
U
+ 3U- 1
dU u- 1
16 J 1 Also x=~

3
1 2 1
16 J1 (U - 3U + 3 --) dU
u

-I
16
[u 3
--r - -3U
_, 2
2
+ 3U - log e U]
3

1
1
- 16
- [! - J
3 loge 3
70

P(x)
TYPE D: Integral of the form
f ax
2
+ bx + c
dx •

2
If the degree of P(x) ~ 2 we divide P(x) by ax + bx + c and express

ax
2
P(x)
+ bx + c
in the form Q(x) +
ax
:x + D
+ bx + c
2
We shall only consider the cases where ax + bx + c is irreducible over the real field,
2
i.e. ax + bx + c cannot be factorised. Method of completing the square and the
following integrals are required.

dx 1 -1 X
c
1.
f X
2
+ a
2
-tan
a
-
a
+

2.
f 2
dx
2 2a loge [~]
x +a
+ c, lxl > a
X -a

[~]+
dx c,
3.
f 2 2 2a loge a-x lx I < a
a - X

EXAMPLES:

(I) f 2
dx The quadratic x
2
- 4x + 5 is irreducible
x - 4x + 5
over the real field, so we complete the

fu 2
dU
+ 1
square.
2
x - 4x + 5 = (x - 2) 2 + 1 = u2 + 1
1
tan u + c where U = x- 2
-1 dU
tan (x - 2) + C dx = 1 ~ dx = dU

(2) f 2
2x- 3 dx The derivative of / - 4x + 5 is 2x - 4
x - 4x + 5
We write 2x - 3 = 2x - 4 + 1
f 2
2x- 4 dx + f 2
dx
x - 4x + 5 x - 4x + 5
2 -1
log (x - 4x + 5) + tan (x - 2) + C, using example (l ).
71

(3) I x
2
dx
+ 6x + 4
The quadratic x
2
+ 6x + 4 is irreducible
2
I dU
2
u - 5
:. x + 6x + 4 (x + 3)
2
- 5

where
2
1
V5 log
[u-v'5J
U + v5 + c

- -1- 1og [x+3-V5J + C


-2./5 X+3+/5

(4) I 2x
2
dx
+ 7x + 13
2x
2
+ 7x + 13 cannot be factorised.
2
G 2
~ x + 1i]
= 21 I dU
2x + 7x + 13 = 2 +

u2 55
+16 =2 rx + ~)2 + ~~ J
4
2 v'55
tan
-1 ( u )
\/55/ 4 + c = 2 [u
2
+ ~~ J
where U = x + 47
-2- t an-1 (4x
- +- 7) + c
\/55 1/55
72

Exercise 2F

Find the integrals:

I. J 6x
3x + 2 dx 2. J1 ~4x dx
3. J__£
X+ J
dx 4. J2x1 - - 2x3 dx
5. J ax + b dx
ex+ d
6. J3:~ 1 dx
7. J 2
dx 8. J 2
dx
x + x + 1 x - x + 1

9. J2 dx 10. J 2
3dx
x + 2ax + b 9x - 6x + 2

11. J 2
4x + 3 dx 12. J 2 x dx
x + 2x + 3 x - x + 1

13. J 2 .x dx 14. JX~ + X + J dx


x + 2x - 2 X -X+ J

Evaluate the following in the exact form:

15.
r
2
2
dx
X - 6x + 10
16.
r 2
J X + 2x- 1
dx

J 7.
I: (x + 1) dx
2
X +X+ 1
18.
r 2
2x + J
-1 X + 6x + 10
dx

19.
I: 2
3 dx
X +X- 2
20.
r
a
3
_x_ dx
J - X
(1 < a < b)
73

2.9 Method of Partial Fractions

It is easy to add two fractions.

a c ad+ be
1. b + d bd

x-2-x+3
2. (x - 3) (x - 2) (x - 3) (x - 2)

1
The reverse process of separating a fraction such as (x _ ) (x _ ) is not so simple.
3 2
We shall now study a method of splitting the general rational function ~~~) into its
partial fractions.

If the degree of P(x) ~ A(x), we first divide P(x) by A(x), and express ~~~~
as the sum of a polynomial and a rational function ~ ~x~ where the degree of
R (x) < the degree of A (x) • x
The method of decomposing a rational function ~~~~ into partial fractions can be
stated as follows:

I. Factorise the denominator.


I I
For example: - - - =
3 2
x - I (x - I) (x + x + I)

2. A theorem on polynomials ensures that every polynomial R (x) with real coefficients
can be factorised into the product of powers of either linear terms of the form (x- a)n or
2
irreducible quadratics of the form (x + bx + c)m .

We are only required to study cases where n = I and m i.e. the factors are not
repeated.

3. In each partial fraction, the degree of the numerator < the degree of the
denominator.
74

Example: (1)

2x - 1 dx
Find
J (x - 2) (x - 3)
by partial fractions.

Solution:

2x - 1 B
Let + - - , where A and B are constants
(x - 2) (x - 3) X- 3

Multiply both sides by (x - 2) (x - 3)

2x - 1 = A (x - 3) + B (x - 2) = x (A + B) - 3A - 2B

This is an identity, so by comparing coefficients of similar terms:

A + B = 2 and 3A + 2B = 1

Solve these: A =-3 , B =5


2x- 1
J(x - 2) (x - 3) dx I[ __::_L
X - 2
+ x -53] dx

- 3log (x - 2) + -5log (x - 3) + C

5
log (x - J) + C
e (x - 2)3

Example: (2)

Find f (x
2
2x +
+ 2) (x - 2)
3 dx

Solution:
2
Here we have an irreducible quadratic factor x + 2 , so we write
2x + 3 A Bx + C
(x - 2) <l + 2) x - 2 + x2 + 2

Multiply both sides by (x - 2) (x


2 + 2)

2
Then 2x + 3 A(x + 2) + Bx (x - 2) + C (x - 2)

2
x (A + B) + x (- 2B + C) + (2A - 2C)

- 2B + C =2 and 2A- 2C =3
(cont. next page)
75

7 7
Solve: A = B -6'
6'

J(x -
2x + 3
2) (x
2
+ 2)
d
X ?.6 J- X-
1
- 2 dx + .!. J _ 7x
6
- 2 dx
X2 + 2

1 1
?.6 -- dx -
Jx-2 ?.6 J-2 x - dx - .!.3 • J 2-- dx
X + 2 X + 2

7 ( ) 7 ( 2 2) - -1- • tan-1 - X
6 log x - 2 - T2 log x + + C
JVi v'2

Example: (J)

f (x - 2) :x + 1)2 dx

Solution:

We have a repeated factor (x + 1) , 2 times. We write:

A B C
-- +~ +
2 x - 2 \X + 1, (x + l)2
(x - 2) (x + 1)

1 = A (x + 1)2 + B (x + 1) (x - 2) + C (x - 2)

Here it is easier to use suitable values of x •

1
Put X = -1 , 1 = -3C , or c =- 3
1
X = 2, 1 = 9A, A= 9
1
X = 0 , 1 = A- 2B- 2C, B=-9

~
1 1
1 dx - J [.!.9 1
~ - 9 rx+n -
1 2Jdx
J (x - 2) (x + 02 - (x + 1)
1 1 1 1
9 log (x - 2) -
9 log (x + 1) +3·rx+n + K
1 x-2 1 1
9 log i(""""+'"1 + 3 lx+n + K
76

Example: (4)

I x3 ~1 dx

Solution:
3 - 2
We have x - 1 = (x - 1) (x + x + 1)
1 Bx + C
-3-- +
2
x - 1 x + x + 1
2
Multiply by (x - 1) (x + x + 1)

2
Then A(x + x + 1) + Bx (x - 1) + C (x - 1)
1
Put X 1, 3A = 1 or A=3

2
X o, A - C = 1 or c = -3

X = -1, or
1
A +2B- 2C = B = -3

Hence
f 1
- x_3
1
dx= 1
3 •
f 1 dx-.!J
x - 1 3 2
x+2 dx
X +X+
1 1
3 log (x - 1) - I , where
3

X+ 2 d J21 (2; + 1) + 2
3
dx
f2 X + X + 1
X
X + X + 1

.!f2x+l
2 2
X + X +
+ ~ • J(x + 2)
/ 2 + 43 dx

2
1
Iog(x
2
+X+ 1) + If tan-lf(x + ~)]
t VJ/2
1 1 1 2 .fj -1 2x + 1
J ~ dx =
3
log (x - 1) -
6 log (x + x + 1) -
3 . tan
v'3
+ c
A (x) d
SUMMARY: To integrate
J P\xr x

1. If degree of A (x) ) P (x) , then divide A (x) by P (x)


2. If deg. A (x) < deg. P (x) , then factorise P (x) completely.
A
(a) For each linear factor (x - a) , write the corresponding term
x - a
(b) For (x - a)0 , write ~ + _B__ + ••• + C
\X - a, (x - a) 2 (x - a) 0
2 Ax+ B
3. If ax + bx + c occurs, then write
2
ax + bx + c
Warning: Do not use A or B twice; all constants must be different.
77

Exercise 2G

Use the method of PARTIAL FRACTIONS to find:

1. f (x - 1) lx - 2) dx
2. f (x - I)
x + 2
(x - 2)
dx

3.
f
i
+ 2
(x - 1) (x - 2) dx 4. f x
2 2x + 3
- 7x + 12
dx

5. I 3
x 2 + 5 dx
X +X
6• I (x - 2) (x \
2
2) (x + 3) dx

7. f (x + 1) ~x2 + 4) dx
8. f x (x
3
x 2+ 2
+ 1)
dx

9. f x (x ~ 1)2 dx
10. f (x - 2)
2x :x
(x + 1)

3
11.
f
x dx
4 12. J -f- dx
X - 1 X - 1

13. I (x + 2)
~X
(x
2
+ 4)
14. I (x
2
+ 1) (x
dx 2
+ 2)

~Hint: let x
2
= y only for decomposing ]
15. I (x - 2) (x
x
2
i 1
+ x + 2)
dx
~y + I)l(y + 2 ) into partial fractions

Evaluate the following:


4 dx 2 dx
16. 17.
f3 x
2
- 3x + 2 f1 x
2
(x + 2)

1 n/4
18.
I 0 ( x
2x dx
2
+ 2) (x + 4)
19. f 0
cosS dS
2 + sinS
ll/2 (Hint: 2 + sinS = y)
• cosS dS
20
fo 2
sin s + 5sinS + 6
21. I (x2 ~ 1)2 dx
(Hint: y = sinS)

22. I 1 +
1
rx
dx (Let U = v'x>
2 2
(Hint: (x 2 - 1) 2 = (x + 1) (x - 1) )
78

2.1 0 Completing the Square (Integration)

2
Integrand of the type involving the irreducible quadratic ax + bx + c over the real
field is easily found by completing the square and then using one of the following:

1. f x2 :x a2 == i . tan-l(i) 2. h
f ::log (x + J. x2 :_ a2)
3. f dx ==

./.T7
Example: (1)

Find f x
2
1
- 4x + 5
dx

Solution:

2 2
x - 4x + 5 = (x - 2) + 1 = U2 + 1
2
, U = X - 2

1
:. I = = tan-! U + c = tan (x - 2) + c

Example: (2)

f dx
~x-i
Solution:
2 2
4x- x - (x - 4x + 4 - 4)
2
4 - (x - 2)

4 - u2 , u = x - 2

f dU
~
Sin
. -1 u + c
2
. -1 (x - 2)
Sin -2- + c
79

Example: (3)
I 2
2x + 3 dx
/ x + 2x + 3
Solution:
2
x
2
+ 2x + 3 = (x + 1) + 2 = u2 + 2
d 2
Also dx (x + 2x + 3) 2x + 2

Rearranging:

I = J2x + 2 + I
2
Jx + 2x + 3

I (2x + 2) dx + f 1 dx
Vx 2
+ 2x + 3 v'1x + 1) 2
+ 2

2 dz
For 1 , let z X + 2x + 3 2x + 2
1 dx
For 1 , let
2
u X +

I =
f .tz I ~ -
dz
+ dU

2 Vz + log (U + vlJZ':2i + c
2 /x
2
+ 2x + 3 + log (x + 1 + /x 2 + 2x + 3) + c
2
Example: (4)
I x
2
X

- 4x + 6
dx

Divide out

+ 2 4x - 6
x - 4x + 6
J dX d 2
dx (x - 4x + 6) = 2x - 4

2(2x - 4) 2
+ U =X- 2
2 2
x - 4x + 6 (x - 2) + 2

= x + 2log (x 2 - 4x + 6) + 2 I - dU
2
--
U + 2

= x + 2log (x 2 - 4x + 6) + ../i tan 1 (x - 2) + c


V2
80

Exercise 2H

Find:
2x
1. dx 2. dx
J x2 ;x + + 4 J X2 + 2x + 4

3. f 2 x2 + 1 dx 4. J dx
x + 2x + 4 /x 2 + 2x + 4

5. J x dx
2
/x + 2x + 4

dx
6. J dx 7.
J 2
~ 2x + x + 5

8. f X + 2 dx 9. J x dx
lx 2
+ X + 1 / 6x- x 2

10. f dx 11.
J
x dx

/2- x- x
2 Jx 4 - 3x 2 + 1

12. f x dx
2
13. J (1 - 2x) dx
2
/1 - 2x - x / x + 2x + 3

14. Show that:

r lx
0
dx
2 + 2x + 2
= loge If : vA I
15. Evaluate:

dx
( 0
X
2 + 4x + 5

16. IV) X + 12
-2-- dx
0 X + 9

17.
r hx7
0
dx
81

18. f X + 1

19.
I rr-:;_
{~
dx
[ Hint: I
=
J 1
- x
./(I - x) (I + x)
dx

=J ]
;-;-:;
1 - x dx

20. f/ x : 2 dx

2.11 Integration: Special Properties

Use of the following formulas simplifies the work and saves time.

1. I
0
a f(x) dx = J0 a f (a - x) dx

2. f
-a
a f (x) dx = 2 I0
a f(x) dx , if f(x) = f(-x) , i.e. f(x) · is an even function.

a
3.
f
-a
f(x) dx = 0 , if f(x) = -f(-x), i.e. f(x) is an odd function.

Proof:

1. Put a-x= U , when x =0 , U =a


x=a, U=O

RHS = J
0
a f(a- x) dx = J
a
0
f(U) (-dU) = - J
0

a
f(U) du = JQ
a f(U) du

LHS = JOa f(x) dx RHS.


82

2. The graph of an even function is symmetrical y

r
about the y-axis, i.e.

-a
f(x) dx Area PABQ
f(x) = f (-x) •
M

2 r
2 x Area MOBQ

f(x)dx

Thus for an even function J-a


a f (x) dx =2 fa f (x) dx
0
A
-a -x 0 X
B
a X

Fig. 2

3. The graph of an odd function has a point y c


symmetry about 0.
Area OBA = - Area ODC

r-a
f(x)dx 0 (as an integral)
a
X

Thus for an odd function

fa f(x) dx = 0
-a
Fig. 3

Note:

Though as a pure integral f a f (x) dx = 0, if f (x) is odd, we need to be careful if the


-a
area bounded by the curve y = f (x) and the x-axis is required. Then we use:

A = A
1
+ A
2
= I f
-a
0
f(x) dx I + I f0
a f (x) dx I = 2 I f a f (x) dx I
0
83

WORKED EXAMPLES

Evaluate:
2
J.
J-2
2
x dx 2. fn sinx dx
-1T

Solution:

. an even f unct1on
. 2 2
1. f(x) : x 2 1s as f (-x) : (-x) : x

16
::; 3

2. f(x) : sinx and f(-x) : sin(-x) -sinx


y
f (x) is an odd function.

I1T
-1T
sin x dx : 0

Note: The area »etween the curve and

x-axis from x : -n to x =n is X

given by

A = 2 J: sinx dx =
:
2[-cosx]~
2 X 2
= 4 Fig. 4

3. Use the result J: f(x) dx = J: f(a - x) dx to evaluate

n/2 .
(a) . sm x dx
J0 smx + cosx (b)

Solution:

(a) =
Jn/2 sinx
dx ••• ( 1) ' Replace x by 2 - x
n
sinx + cosx
0

sin (n/2 - x) cosx dx ••• (2)


r'2
= sin ( n/2 - x) + cos (n/2 - x) cosx + sinx
0

(cont. next page)


84

Add (1) and (2): 21 = Jn/2 sinx



+ cosx d
X
SlnX + COSX
0

n
:: 2

n
ii
2
J 2 (2 -
(b)
10
x 2 (2 - x) 1/2 dx =
0
x) 2 [2 - (2 - x) ] 1/2 dx

2
2 1/2
=J (4 - 4x + x ) • x dx
0
r~ ~ ~ x7/2]
2
x3/2 - x5/2 +
l3 5 7 0

128/2
105

1+. Using the property of odd and even functions, evaluate:

(a)
t: 5
tan x dx (b)
x3
-1+-- dx
X + 1

Solution:
3
(a) f(x)
5 (b) f(x)
X
= tan X
4
X + 1
f(-x) [tan (-x) ] 5
3 3
(-x} -X
[-tanx]
5 f(-x) = 4 -1+--
(-x) + X + 1
5
- tan x .. f(-x) = - f (x)
-f(x)
.. f(x) is an odd function
f(x)

I1
-1
is an odd function

5
tan x dx 0
... r-2 X
X

+ 1
3
- 4 - - dx 0
85

Exercise 21

1. Using the properties of the odd and the even functions evaluate:

2 ll/4
(a)
f-2
x
3 dx
(b)
f-n/4 COSX dx

1
2
(c) (d) x tanx dx
f-1
1 5
(e)
f-1
(x
2 2
+ x sinx) dx (f)
X
- 4- -
X + J
dx

2 5 n/2 3 4
(g) (x cos x - 100 x + 2) dx (h)
f-n/2 sin x cos x dx

2. Prove that J
0
a f (x) dx = Ja
0
f (a - x) dx and evaluate

(a) fon/2 ~ dx (b) J,(01 3


x (1 - x)
6
dx
~ + lcosx

TI/2 Tl ) 2 ll/2 2
(c)
JO
X ( 2- X COS X (d)
f0
cos x dx

1
99
(e)
f0
10 100 x (I - x) dx

3. Using the properties of odd/even functions, evaluate:

(a)
r -1
x
2
+ x
} + X
3
2
.
+ smx d
X (b) J1f sinx dx
2
0 a+ b cos x
a
b
>0
>0

4. (a) Show that if f (x) is even and f' (x) e;{ists, then f' (x) is odd.

(b) If f (x) is odd and continuous for all x , and b >a >0 , use a sketch to

explain why f_: f (x) dx Ja


b
f (x) dx
86

Exercise 2J (REVISION)

Use any suitable method. Some integrals can be found in more than one way.
Integrate the following:

I. I-x
.;x:-'2
dx 2.
I: X
2
~dx

3.
rI e
e
X
2x

- I
dx , using u = e
X
- I 4.
I1
2
X
4 5
sin(x ) dx

sin 2x
5.
Ja 2 +
2 2
b sin x
dx 6.
f 2
a co/x +
2 2
b sin x
dx

2 2 2 2
(Hint: t = a + b sin x) (Hint: Divide by cos x)

7. I 2(1
dx
+ x) /X
8. Jtf-X + I
dx
2
(Hint: U = x )

9. J dx
10. J I + e; dx (Hint: t = ex)
I +ex

r
I - e

II.
2
x 2 sinx dx 12. Jx 2 1ogx dx
dx
13.
f x v'f'"+i( dx 14.
f Vt-:7
X
2

X
dx
15.
f X~ 16.
f _e_ _ dx
I + e2x

17.
f X
2
+X +
dx
5
18.
I xsin- 1x dx

dx dx
19.
J 4x
2
+ 4x + 5
20.
f X
2
+ 4x - 5
dx dx
21.
f + 3x - x
2
22.
f y{x
2
+ 4x + 5

23.
f 4x
2
3x + 2
+ 4x + 5
dx 24.
f /4) + 4x
3x + 2
+ 5
dx

25.
f dx
5 + 4sinx 26. I 3sinx + 2cosx
3cosx + 2sinx
dx

[Hint: 3sinx + 2cosx


A Ocosx + 2sinx) + B(-3sinx + 2cosx)]
87

27. I/§ a-x dx


(Hint: multiply
by .ja"+X)
28.
I /x2- X+ 1 dx

29.
I dx
2 - 3cos2x
2
2
(cos2x = 1 - 2sin x,
divide by cos x)
30.
I dx
(3cosx + 2sinx)
2
(Hint: divide
2
by cos x)

31.
I cosx
5- 3cosx dx 32.
I cos

3
v'X dx
2
(Hint: X = t )

X
33.
J 2
2x + 5
X - X- 2
dx 34.
f 2
X - 3x + 2
dx

2x dx 2 dx 3 = x(l
35.
f (x 2 + 3) (x 2 + 1) , (x = t) 36.
J -X -- 3
X
[x - x - x)(l + x)]

37.
f + 3ex + 2e 2x
dx (ex = t) 38.
I O+X+X
dx
2 + x3)
(Factorise)

39.
cosx dx
f (I + sin x) (2 + sin x) 40.
I dx
sinx + sin 2x
(Multiply by sinx, then t = cosx)
n/ 2 n/2 n-2
41. Show that
f 0
n n - I
sin a da = - -
n I 0
sin a da , and hence show that

n/ 2 . 4
I0
sin ada =
.
3n
16

42. Apply successively the result of exercise 41 to show that


n/2
f0
sinna da
(n - 1) (n - 3) ••• 4.2
n (n - 2) ... 3.1
(n - 1) (n - 3) ••• 3.1 n
if n is odd

and if n is even.
n (n - 2) ... 4.2 2
43. Establish the reduction formula (using integration by parts)

f xnsin bx dx = - x: cosbx + 5 f x
n-1
cos bx dx ,

n/4
hence evaluate:
I 0
2
x sin 2x dx

44. Establish the formula (using integration by parts)

f 1 xn+ 1 . -1
xn sin- x dx = n + Sin x -
1
n+T
1 f xn + 1 dx + C
~
Hence find
0
f
x sin-l x dx
1

45. Establish the reduction formula

Jcot n-2 x dx
I cot
n
dx = -
cotn-l x
n - 1
-

[Hint: cotnx = cotn- 2 x (cosec2x - 1)]


88

CHAPTER 3 VOLUMES

3.1 Formulas for Volumes

y
In elementary calculus, we have the
following two formulas for calculating
the volumes of revolution.

1. The volume of revolution


generated by the region bounded
0 a l:!.x b X
by the curve y = f (x) , above
the x-axis, between x = a and Fig. I
x = b is given by:
y
b 2
V =
f
d
ny dx (Fig. 1) b

2. The volume of revolution generated l:!.y X f(y)


by the region bounded by the
curve x = f (y), between y =a a ---------
and y = b is given by:

b 2
v
f
a nx dy (Fig. 2) 0
Fig. 2
X

In many cases, it is necessary to calculate volumes whose boundaries are not surf.1ces of
revolution and hence the two formulas stated above can not be used. For example we
cannot find the volume of a pyramid or a doughnut shaped solid by these two formulas.

Fig. 3: Pyramid Fig. 4-: Doughnut


89

In what follows, the formulas for volumes would be derived intuitively by means of
simple examples.

The volume of a solid of uniform


cross-section A and height h is
h
given by

V =A. h
Fig. 5
If h is very smaJJ, we have an
element of volume given by

ll V = A • llz

where h = llz
h

Let us calculate the volume of the


solid (pyramid) shown in the
figure.

We slice the \l.ho!t' pyramid by


n planes para1Jel to the base of
pyramid.

Let the distance between two


successive planes be ll z.

We find the area A(z) of one of


the cross-sections at a distance z Fig. 7
from the vertex V. Quite clearly
A (z) is a function of z.

We express A(z) in terms of the sides a and b of the base. From the two similar
triangles shown in the diagram, we have:

X z i.e. x
a h ahz]
•••• (l)
Similarly bz
y
h
90

A (z) area of the rectangular cross-section at P

X • y
za zb [from relation (I) ]
h h
2
abz
7
The volume of the pyramid is

V = f. !:J. V = f. A (z) !:J.z

As n -+ ""• f. 6. V -+- V and f. A (z)!:J.z = I


0
h
A(z)dz

v I
0
h A (z)dz y

v •••• (I)

Now from solid geometry, we know that the volume of the pyramid is given by
I
V: B.h
3
where B = area of the base = ab

The formula V ~ Jb A(z)dz used above is quite general and the following general
a
statt·ment can be mJ.ck: (see diagram above)
The volume of a solid whose cross-sectional ~rea is a continuous function A (z), is
given by:

V Ib
a
A (z)dz

where z is the distance of the cross-section from the pre-determined point (or a
plane). The limits of integration are chosen to include the entire volume. To evaluate
V, we must express A (z) and dz in terms of a single variable.
91

We note that the general formula for

the volume V = Jab A (z) dz includes


y
the volumes of revolution given by
b 2
V = f
a
ny dx •

In the diagram:

Z :: X

A(z) A(x)

= area of the disc at a distance


x from the origin

n/
Hence v t
d
A(z)dz
a
f
b 2
ny dx.
Fig. 8

Note that the volume of revolution given by the last formula is also known as a DISC
method.

An Important Method

Finding lengths from the given diagram is of particular interest in calculating the
required volume. An elegant method is presented below and you will be asked to
derive the same result by using similar triangles.

Example: (I) 4
Find the length y in terms of h, from
the diagram which shows an isosceles
r--- ~~-I~ ~--~y~----~
5
trapezium;

Solution: L----~~ 12
We observe that:
Fig. 9
y 4 when h 0
y 12 when h 5
4
Since y is always proportional to h, ----rT
y is a linear function of h
y = mh+b
T
5 f - - - - ' - y - - - \ - - -,'
I h
j_

l
I
Substituting the given values I
12
8
b =4 and m = 5
Fig. 10
92

Example: (2)
Using the intercept properties of parallel lines, prove the above result. (Fig. I 0)

Examp!e: (3)
Find the volume of the
block shown in Fig. 3.

Solution:

We consider the block to be made up


of horizontal laminas of length 1,
width w and thickness dh •

Hence, V = l .5
wldh
12
Fig. 11
.5 7h 8h
= 1 <-r
0 + 3) <-r + 4) dh
w = 7h ...

r:
3
T
= 2~ (14h
2
... 6.5h ... 7.5) dh =
8h
T + 4

3 2
= 4 [14h
2.5 ---:r 6.5h
+ -2- ... 7.5h
].5
0

= 283 to 3 significant figures

The students familiar with the prismoidal formula

h
v = 6 (A • B • 4M) can verify the result.

A = 3 X 4 = 12

B = 10 X 12 = 120

( 10 ... 3) (12•4)
M = 2 2 = 52

... v = i (12 + 120 ... 208 X 4) = 1133.33 ••


93

Example: (If)

The base of a certain solid is the

ClrC1eX 2
0
+ y2 = 4•
Each plane section of this solid cut
out by a plane perpendicular to the
y-axis is an equilateral. triangle with
one side in the base of the solid.
Find the volume.

Solution:

A typical slice is an equilateral


triangle PBC.
Fig. 12
BC 2x
A = Area of !1PCB

= ~ PC • PB sin 60"
1 ,fJ
= 2 .2x.2x.T
2
= ,flx

v = I b A(x) dy
a

=I
2 2 2
13x dy X +y :If

I2 ~(If- y 2)dy The limits are the ends


-2
of the diameter.

= 2 Io2 Vl(lf - y2) dy (Owing to symmetry)

= 2/i [4y- ~f
3 0

= ¥
94

Example: (.5)

Find the volumes generated when the areas bounded by the given curves and lines are
rotated about the x-axis.

(a) y X + I ' X 2' y 0

(b) y cosx, ' X


0' X n
2
(c) y 2x- x , y 0

Solutions:
y
Since the rotation is about the x-axi~, we use
b 2
the formula V =
Ja n y dx
3 p (2, 3)
(a) y = X+

:. The limits are a = - I, b = 2

••• V = n I2 y 2 dx = n 12 (x 2 + 2x + I) dx
-1 I -1 0 2 X

11
rt\3 2 ]2
+X +X -J
Fig. 13

9n

2 9 3
nr h 11 X X
9n
Verify: V volume of a cone = - - 3
3

y
(b) y cosx , a = 0 , b = n
2 I
? cos x = (I + cos2x)
2

v ~n J; (I + cos2x) dx

11
n [x + I sin 2x] X
2 2 0

2
n
2
95

y
2
(c) y = 2x - x , y = 0

Solve these: x = 0 or x =2

2
2 3 4
11
J0 (4x - 4x + x ) dx 0 X

11[ 4x3 - x4 + x5] 2


3 5 0 F1g. l 5
1611
TI

Example: (6)
2
Find the volume generated when the area bounded by the curve y = x , y = 2 and
x =0 is rotated
(a) about the y-axis (b) about the line y = 2

Solution:

(a) A typical strip is shown and


it will generate a volume y

2
ll.V = 11X fl.y

v = JO2 11X 2 dy , 2
X =y

= 11 c y dy

11 [~1: Fig. 16
v 211
96

(b) The typical strip has a radius


2
r = y1 - y
2
=2 - x

The volume generated when the


y
region OABO is rotated about the

line y =2 , is given by:

V =1r J
!22r dx
0

= liJ./202 (2 - x 2 ) 2 dx

=1r J
li (4 - 4x
2 4
+ x ) dx
0 X

0 yl =2
= 1T r4x - 4x3 +
l' 3 5
x5]fi0 y2 =X
2
Fig. 17

=
32Jin
15

Note: The rotation is not about the x-axis, hence we can not use

b 2 2
V =
f a
n (y
1
- y ) dx
2
97

Exercise 3A

For Exercises 1-10 find the volumes of revolution when the areas bounded by the given
curves and lines are rotated about the x-axis.

1. y = rx, X =0 , X =2
2
2. y=x +1, X = 0, X =2
2
J. y=x -x, y =0
4. y = sinx , X= 0, x =n
5. y = cosx •
n
x=2

n
6. y=tanx, X = 0, x=;;

2
7. y=x -4, y =0
8. y = sinx cosx, y = 0.

9. y=logx,
e X =1 , X =2

lO. y = 21 (e x + e
-x
) , X = 0, X =1

For Exercises 11-15 find the volumes of revolution when the areas bounded by the given
curves and Jines are rotated about the y-axis.

11. x+y=4, y = 0. y = 2

12. y = rx. X = 0, X= 4

2
JJ. X = 4- y
• X= 0

14. y = Jogex, X= J X = 2 (Hint X = eY)



. -1 (Hint x = siny)
15. y = sm X ' X = 0, X = 1
98

2 2
16. The base of a certain solid is the circle x + y = 9. If each plane section of
the solid cut by a plane perpendicular to the x-axis is

(a) a square with one edge in the base of the solid, find the volume of the
solid.

(b) an equilateral triangle with one side in the base of the solid, find the
volume.

(c) a semi-circle with its diameter in the base of the solid, find the volume.

(d) an isosceles right-angled triangle with the shorter side in the base, find
the volume.

(e) an isosceles right-angled triangle with its hypotenuse in the base, find the
volume.

2 2
17. Find the volume of revolution when the region inside the ellipse x + ~ =
2
a b
is rotated around the x-axis.

18. A rugby ball has a volume that is the same as the volume generated by
2 2
rotating the region inside the ellipse ~
5 + ~ = l about the x-axis.

Find the volume of the ball.

19. A solid has a base in the shape of an ellipse whose major axis is 12 units and
minor axis 8 units. If each section perpendicular to the major axis is an
isosceles triangle with altitude 12 units, show that the volume of the solid is
14411.

2
20. The base of a solid is the circle x + / ·= 8x and every plane section perpen-
dicular to the x-axis is a rectangle whose height is one third of the distance
of the plane of the section from the origin. Show that the volume of the solid

is 6411 .
3
99

3.2 Volumes: Shell Method

In Section 3.1, the volume of revolution was found by rotating the rectangular
strips perpendicular to the axis of revolution. The elementary volume was a disc.

If the rectangular strip is parallel to the axis of revolution, a cylindrical shell is


generated. A shell is a solid contained between two parallel concentric surfaces.

To find the element of volume contained in a shell of inner radius, r = x and outer
radius, R =x + 8x , length y, we have
2 2
AV n (R - r )y
2 2 2
n y (x + 2xAx + Ax - x )
2
= 2 n xy • Ax + n y • Ax
2
As llx is very small, (llx) is negligible.

Hence AV ~ 2nxy . Ax

Fig. 18

To find the volume when a region bounded b


the curve y = f (x) , a ~ x ~ b ,
y
and the x-axis, is rotated about the
y-axis, we divide the volume into thin
cylindrical shells. The cross-section of
one typical shell is shown in the diagram.
The element of volume is given by
X
AV 2nxy Ax y

V = Jb 2nxydx,
a
where limits are chosen to include the 0 a Ax b X

entire volume.

Fig. 19
100

Example: (1)

The region bounded by the parabola

y = x2 , the x-axis and the line x =2


is rotated about the y-axis. Find the y

resulting volume by using the shell


method.
A(2, 4)

Solution:

The volume swept out by the typical


strip (height y, breadth Ax) at a
p
distance x from 0 , is given by
y
AV = 2 nx • y • Ax

The limits of integration are a = 0 to b = 2 0 2 X

The required volume is

v =
r 211 xy dx y =X 2
Fig. 20

= c
0
2nx 3 dx

= 8ll

The method is very general and each example requires a good deal of pre-planning and

modification, so let us make it general.

Element of volume AV = Inner circumference x Height x Thickness

Four usual cases are summarised below.

In each case the region R is rotated about a specified axis.


101

b y
(a) v
y = J a 2 n x • y • dx

gives the volume generated when R

the region R bounded by y


y = f(x) , x =a, X= b 1 y =0
is rotated about the y-axis. X
X
0 a Ax b

Fig. 21

(b) v
X
= t
b
2ny. x. dy
b
y

-----
gives volume when the region Ay
bounded by the curve X = f(y) X
1

lines Y =a' y = b' X= 0 R


is rotated about the x-axis.
a
y

0 X

Fig. 22

(c) The region between two curves y

~
y = f(x) and Y2 = g(x) is
1
y2
rotated about the y-axis. I
The volume is given by I
I

vy = t
b
2nx(y 1 - y )dx
2
I
I
0 a X

Fig. 23

(d) The region R bounded by the y


curve y = f(x) , a~ x ~ b , and

above the x-axis is rotated about


the line x = c , where c >b ,
the volume of revolution is given
b
by: V = Ja 2n (c - x) y dx 0 a Ax c X

Fig. 24
102

Example: (2)
2 2
Find the volume of the solid formed by rotating the circle x + y = 4 about the
line x = 4 •

Solution:
y

x r=4-x
-------

-2 X

H
t:.x
Fig. 25

2 2
X + y 4
Volume of cylindrical shell at P (x, y)
y t:.V = 2nrh t.x
r 4- X (PA)
h 2y (PQ)

Using the shell method for finding the


volume, we have:
2
V = J 2nrh dx
-2

~ dx
2
2 nJ-2 (4 - x) . 2 . -2 X

4TI 1-22 (4 - x) .;::7- dx Fig. 26

~ dx
2
4nJ 41:7 -2
dx- 4Tir -2
x

The second integral is zero, since the function f(x) = x /:'"7 is an odd function.
(See Fig. 26) For the first integral, let x = 2 sin 9
103

n/2 I 2
V 16n
f
-TT/2
1/4- 4sin S (2cosSdS)

f2
64n n/ cos 2 e dS
-TT/2
n/2
32n
f
-n/~
(I + cos2S)dS
n/2
32nJS + 2 sin 29] -n/ 2
32TT

3.3 Volumes: Washer Method

A washer is a small hollow cylinder


(with small height). If the radii of two
concentric circles are R and r, and the
height of the washer !J.y ·' then the
I !J.y
volume of the washer is

!J. V = 11 (R 2 - 2
r ) !J.y
Fig. 27

y
Example: · (I)

Use the washer method of finding the


volume generated when the region
2
bounded by the parabola y = x
and the lines x =2 and y = 0 is
rotated about the y-axis.

Solution: 0 X X

A typical strip at P (x, y) in the


given region will sweep out a washer
2 2
of volume !J.V = TT (R - r ) !J.y
2
=n (4 - x ) !J.y [r=x, R=2)
Fig. 28
4 2
=
:. Required volume V
J0 11 (4 - x ) dy

=( 11 (4- y)dy' y = X2

= 11 [ 4y - ~]:
11 [16 - 8)
811
104

Exercise 38

In Exercises 1-.5, find the volumes generated when the region bounded by the given
curves and lines is rotated about the x-axis.

2
1. y =X + 1 y = .5
'
2
2. y =X + 2' y =X+ 4
2
3. y =1- X y = 1- X
2
'
4. X = 3y- Y X=0
'
.5. y=x+2, y = 4' X= 0

In Exercises 6-10, find the volumes when the region bounded by the given curves and
lines is rotated about the y-axis.

6. y = X' X = 2' y =0

7. 2
y =X X = 2' y =0
'
8. 3
y =X X = 2' y =0
'
2
9. y =X' y = 3x- x
1T
10. y = sinx , y = 0' 0~ X~ 2

(Hint: Jxsinx dx , integrate by parts)


2x
11. The region bounded by y = -1 , y = e , x = 0 and x = 2 is revolved about
the line y = -1 • Find the volume of the resulting solid.

2 2
12. A circle of radius 2 is given by x + y = 4 and is revolved about the line
x = .5. Using the washer method, prove that the resulting volume (called a
TORUS) is 40n 2 •

(Hint: Simplify your work by remembering that

2
J-2 Q dx = area of a semi-circle = 2n )
105

3.4 Worked Examples (Miscellaneous)

EXAMPLE: (1) The triangle with vertices A (2,2), B (2,4) and C (4,4) is rotated
about (i) the x-axis, (ii) the y-axis, (iii) the line x = 8.
Find the volume generated in each case.

SOLUTION
4
I
y
B
p c

~
(i) Equation of AC is y =x •
Equation of BC is y = 4 •
The element of volume, l:i V = 11 (y
y =4 and y =x
2
2 - y 2 ) /:ix
1 2 A
v
2
Substitute in the formula
1
---------
X

R
b 2 2 f4 2
V = 11 (y - y ) dx = 11 (16 - x ) dx 0 2 /:ix 4 X
fa 2 1
2
Fig. 29
v = 11 ~6x _ x: J~ = 4~ 11 y
4 B c
(ii) V =
f 11 (x
2
2 2
- x ) dy ,
1
4
/
= 11
2

r</ _
2
4) dy
/:iy
Qv:p
A :
~ 4J ~ 2
= 11[ - II Y
I

~
I
6
= 11 [ ; - 16 - ( - 8)] I _..
X
3211 0 X
T Fig. 30

(iii) r
2
= PR = 8 - 2 = 6, r 1 = QR = 8 -- x = 8 - y (y = x)
y
4
2 2
••• V =
J2 11 (r 2 - r
1
) dy
4
B c
4 2
p h R

= 11
f2
I 4
11 [36 - (8 - y) ] dy

(-28 + 16y _ l> dy


2 v
2
= 11 f 28y + 8/ -
3 4
~ J 2 0 2 4
La.
8 X

6411
-3- Fig. 31
106

EXAMPLE (2)
2
The area bounded by the curve y = x + 2 and the line y = 2x + 5 is revolved about
the x-axis.
Find the volume.

SOLUTION:
2 ••• (l) y B
y =X + 2
y = 2x + 5 ••• (2)
Solve (l) and (2) for the intersections.

x 2 + 2 = 2x + 5
2
x - 2x - 3 = 0
(x - 3) (x + l) =0
x = 3 or x = -1
The element of volume when the region ACBA is
rotated about the x-axis is given by: X
2 2 (washer)
I!.V = n (PM - QM ) l!.x
2 2 Fig. 32
= n (y 2 - Yl ) l!.x
2
We substitute y =x + 2, y 2 = 2x + 5
1
2 2 2
••• 1!. V = n [ (2x + 5) - (x + 2) ]
2 4 2
= n (4x + 20x + 25 - x - 4x - 4)
4
= n (20x - x + 21)

b 2 2
:. Volume V =
f a
n(y
2
- y ) dx
1

=f
3
4
n (20x - x + 21) dx
-1
2 x5 3
= n [lOx - + 2lx]_
5 1
243 + 63) - (lO + "5-
li 90 - T
= n~ l 21) J
576n
-5-
107

EXAMPLE: (3) The cross-sections of a certain solid by planes perpendicular to the


2
x-axis are circles extending from the curve y = x to the curve
2
y = 2 - x • The solid lies between the points of intersection of
these curves. Find the volume of this solid.

SOLUTION:
y
2 ... (1)
The curves y = 2 - X

2
and y =X ... (2)
intersect at A (-1, 1) and B (I, 1).
The element of volume at distance x from
the origin is given by:
2
ll V = 11 r • /lx (disc, radius r)
I I I
where r 2 PQ = 2 (PM - QM) = 2 (y 2 - y 1 )
X

PM

QM Fig. 33
2
!:N 11 • r • /lx
I 2
11 • 4 (y 2 - YI) /lx

y 2 - Yt = 2- X 2 -X 2 = 2(1 -X )
2

1
11 J( (1 - 2x
2 4
+ x ) dx
-1

211
I
0
I
(1 - 2x
2 4
+ x ) dx (owing to the symmetry)

1611
15
108

2
EXAMPLE: (4) The region bounded by curve y = 2x - x and the line y = x is rotated
about the y-axis. Find the volume by the method of cylindrical shells.

y
SOLUTION:
2
The curve y =2x - x and
the line y =X intersect at 0 (0,0) and A (1, 1)

We use:

v 2n r
a
x(y2- yl) dx

2
••• (1) X
X

PR = 2x - x , y = QR = x Fig. 34
1

:. V 2n f
1
0
x (2x - x
2
- x) dx
.

2n
f
l
0
(x
2 3
- x ) dx

x3 _ x4] 1
2n [ 3 4 0

2n (j- i)
l!
6

2 y
EXAMPLE: (5) The area bounded by the curve y = x
and the line x = 1 is revolved about
the line x = 3. Find the volume
_ _ _ _ _ _ _g_e_ne_r_a_t_ed_,_b_y_t_he_m_e_t_h_o_d_o_f_s_h_e_ll_s_. ----f;•; _ J~: !L __
SOLUTION
3 X
2
J
V = b 2n (c - x) h dx , C = 3, y = X
a h = PQ = 2y = 2yx

= f 1
4n (3 - x) .Jx dx
0

41!
0
r(3xl/2 - x3/2) dx Fig. 35

4 n [ 2 x3/2 _ ! x5/2] 1
5 0
2
41! (2 - 5)
32n
)
109

REVISION VOLUMES Exercise 3C

1. (a) On a number plane, shade the region R representing the inequality

(x - 4)
2
+ / ~ 4 •

(b) Show that the volume of a right cylindrical shell of height H with inner
and outer radii r and r + I:J.r respectively is given by 2n r H • I:J.r •

(c) The region R in (a) above is rotated about the y-axis, generating a solid of
revolution called a torus. By using the shell method, prove that the
2
volume of this torus is 32n •

2. A solid figure has a semi-circular base of radius 4 units. The cross-sections at


right angles to the semi-diameter of this base are semi-ellipses. If the semi-

minor axis of each ellipse is } of the semi-major axis, prove that the volume
4 2
of the solid is given by V =41T f0 (16 - x ) dx and hence find the volume of

the solid •

2 2
3. The area common to the curves y = x and y = x is revolved about the
y-axis. Find the volume generated by using:
(a) the disc method (washer)
(b) the cylindrical shells.

4. The co-ordinates of the vertices of a triangle PQR are (0, 4) , (2, 2) and
(-2, 2) respectively. The region PQR is rotated about the
(a) x-axis (b) y-axis (c) line x = 4
Find the volume of revolution generated in each case.

5. A certain solid has a semi-circular base of radius 4 units. The cross-sections


at right angles to the x-axis are triangles with one leg in the base. If the
2
heights of these triangles are bounded by the arc of the parabola y = 16 - x ,
show that the volume of the solid is given by:

4
V = (16 - x 2) 3/2 dx and hence find V •
J0
110

6. An electronic valve used in a computer is in the shape of a solid that has a


circular base. At a height z above the horizontal base, the horizontal cross-
section is a circle of radius R given by the relation

R = f(z) = I -
ih
2

If the height of· the solid is h, find the volume of this solid.

7. A frustum of a right circular cone


is shown in the diagram.

R- r
(a) Prove that x = r + (-h-)z ·
h
(b) Prove that the volume of the
frustum is given by
nh 2 2
V = T (R + Rr + r )

Fig. 36

8. A very large waste container is shown in the diagram.


The top face of the container is a rectangle of sides 3 m and 4 m
respectively and the bottom is a rectangle of sides 2m and 3m
respectively. If the height of the container is 1.5 m, find the volume of this
container by integration.

I
I
I
).5 I
I
I
I

I]- • - - - ~----~----'

Fig. 37
111

2
9. The area bounded by the parabola y = 4ax and the line x = a is rotated
about the line x = a. Find the volume generated, by using the Shell method.
10. Find the volume of the torus obtained by rotating the area bounded by the
2 2 2
circle x + y = a about the line x = c, (c > a). Use the method of (a)
cylindrical shells and (b) disc (washer method).
2
II The area bounded by the parabola y = 4ax and the line x =a is rotated
about the y-axis. Find the volume by two methods.
I 2. A ring of altitude 2h is generated by revolving
about the y-axis the area of the segment bounded
2 2
by the circle x + / = a and the chord of
X
length 2h that is parallel to the y-axis.
By using the method of shells, show that the
. . 4nh 3 Fig. 38
volume IS g1ven by - -
3
x2 2
13. The area enclosed by the ellipse
25
+ h
= I is rotated about the line x = 8.
By using the method of shells, find the volume generated.
14. The ellipse in exercise (13) is revolved about the vertical line through
the vertex A (5,0). Find the volume.
15. The triangle ABC formed by the points A (a,O), B (-a,O), C (O,a) is revolved
about the line x = 2a. Find the volume generated by disc (washer) method.
16. The cross sections of a certain solid by planes perpendicular to the x-axis are
2
circles with diameters extending from the curve y = x to the curve
2
y = 8 - x • The solid lies between the points of intersection of these two
curves. Find the volume of this solid.
2
[Hint: radius r is given by 2r = (8 - x ) -
2
i =8 - 2x
2
and V = f2

-2
A (x) dx,
where A = ll r ]
2
17. The area bounded by the curve y = x + I and the line y =3 - x is revolved
about the x-axis. Find the volume of revolution. (Hint: use the washer
method)
18. The region bounded by the curves y = 3x - i and y = x is rotated about the
y-axis. Find the volume by the shell method.
19. The triangle with vertices (a, a), (a, 2a), (2a, 2a) is rotated (a) about the x-axis
(b) about the y-axis. Find the volume generated in each case.
20. The area bounded by the curve / = 4x and the line x = I is revolved about
the line x = 2. Find the volume generated.
112

CHAPTER 4 COMPLEX NUMBERS

4.1 Introduction

- Necessity is the mother of invention.

Invented Number Systems: Complex Numbers

Imagination and art of invention were required several times in extending our number
system from the counting numbers.

1. The first invented system J: the set of all integers as developed from the
counting numbers. In this system we can solve equations such as x .:!:. 2 = O,
in general x + b = 0, where b is any integer.

2. The second invented system Q: the set of all rational numbers p/q as
developed from the integers. In this system we can solve equations such as
2x - 3 = 0, in general ax + b = O, where a and b are rational numbers.

3. The th1rd invented system R: the set of all real numbers x as developed
from the rational numbers. In this system, we can solve not only the types
2
x + a = 0, ax + b = 0, but in addition all quadratic equations ax + bx + c = 0,
a ~ 0 and t:. = b - 4ac ~ 0. The roots are not real if 1:!. < 0. The simple
2
2 2
quadratic equation x + I = 0 or x + x + I = 0 is impossible to solve with
the above mentioned three number systems. There is no real number that
2
satisfies the equation x + I = 0, since x = r-f does not exist· in the real
number system R.

A new kind of number has to be invented to handle the roots which are not real. The

symbol [ i = ;-:II
2
is used with the understanding that I i
2
= -I I i is called

the imaginary number. The roots of x = -1 can now be written as


2 2
x = -1 = i -> x =- .:!:. i
Again the roots of x 2 + 2x + 3 = 0 can be given as

X= -
2
.:!:_ f8 -1 .:!:_ N = -1.:!:. Iii
Now we come to the fourth invented system, the set of all complex numbers of the
form We shall use a single pronumeral z to define a complex number
x + iy, i.e.
X+ iy I
113

x is called the real part and


y is called the imaginary part of the complex number z.
It is important to note that the imaginary part of a complex number is not imaginary!
It is i that is called the imaginary number.
The history of imaginary numbers is very fascinating. The earlier mathematicians
thought that such numbers had no practical use (hence the term 'imaginary'), yet
complex numbers are of great importance in fields such as Electronics.
A complex number can also be defined by an ordered pair of numbers (x, y), without
ever mentioning the imaginary number i, but in this book we shall only work with
the binary form z = x + iy. However, the computer requires the form (x, y) for
multiplication of two complex numbers.

4.2 Operations with Complex Numbers

We have the following definitions:

1. Equality: a + bi c + di if and only if a =c and b = d.


2.. Addition: (a + bi) + (c + di) = (a +c) + (b + d) i
3. Multiplication: (a + bi) (c + di) = (ac - bd) + (ad + be) i

By using these definitions, we can verify that the complex numbers satisfy all the
laws of algebra and hence the complex numbers form a field (denoted by C).

EXAMPLE: (l) If x + iy = 3 - 5i, then x = 3, y = -5


EXAMPLE: (2) Find the sum of 2 + 3i and 3 + 2i

SOLUTION: (2 + 3 i) + (3 + 2 i) = (2 + 3) i = 5 + 5i

EXAMPLE: (3) Find the product of 2 + 5i and 3 + 6i

SOLUTION: (2 + 5i) (3 + 6i) = (2x3- 5x6) + (2x6 + 5x3)i =-24 + 27i


If you ignore the definition of multiplication and expand (2 + 5 i) (3 + 6i), i.e.
2 2
6 + 15i + 12i + 30i and put i = -1, then the result is -24 + 27i, and so you
now discover the secret of strange definitions!

Identity elements:

By definition: (a + bi) + (0 + Oi) = (a + 0) + (b + O)i = a + bi


and (a + b i) (1 + 0 i) = (a • 1 - b • 0) + (a • 0 + b • 1) i = a + b i

Thus, the complex number 0 + 0. i, written as z = 0, is the additive identity and


1 + 0. i written as z = 1, is the multiplicative identity for the set C of complex
numbers.
114

Additive inverse:

We define the additive inverse of the complex number a + ib to be a number x + iy


such that:
(a + bi) + (x + iy) = 0 + 0. i
(a + x) + (b + y) i = 0 + 0 • i
By definition of two equal complex numbers, we have:
a + x = 0 and b +y =0 giving x = -a, y = -b
Hence the additive inverse of z = a + bi is -a - bi, i.e. -z. Using this, we can
now find the difference: (subtraction)
(a+ bi)- (c + id) =(a+ bi) + (-c- di) =(a- c)+ (b- d)i.

Multiplicative inverse:

The multiplicative inverse of the complex number a + bi f. 0 is defined to be a


number x + iy such that (a + bi) (x + i y) = l + 0. i. By definition of equality,
ax - by = l
and bx + ay = 0
We solve these equations for x and y, then
a b
x = and y = - - - - , hence the multiplicative inverse of z = a + bi
2 2 2 2
a +b a +b
is the complex number z-1 = [ ~· a ~-b J
• Note that z. z-1 = z I = I, z" _1
·z:
0.
a +b a +b
We can now divide a complex number z by aflother complex number w, but shall
use a method used in ex. (6} .below, i.e. complex conjugates. We can easily verify that
the complex numbers obey all the rules of Algebra and hence they form a complex
field C. Further, if b = 0, then a + bi reduces to the real number a, hence the
set of real numbers R is a subset of the complex numbers C. If a = 0, b f. 0, then
a + bi reduces to bi, and we call bi a purely imaginary number.

Note that a real number can be written in the form a + bi, for example 2 = 2 + 0. i.
Similarly a purely imaginary number such as 2i can also be written as 0 + 2i.
2
Calculations with the complex numbers do not require any special rules; wherever i
occurs we replace it by -1. Further,
3 4 2 2 5 4
i = -i, i = i x i = (-1) (-I} = 1, i = i • i = i etc.

Complex conjugates:

If two complex numbers differ only in the sign of their imaginary parts, each is
called the conjugate of the other. Thus a + b i and a - i b are the conjugate
complex numbers. Notation z
is used for the conjugate of z, i.e. = a - ib. z
Since z + z = (a + bi) + (a - bi) = 2a, the sum of two conjugate complex numbers is
a real number.

EXAMPLE: (4) Find the sum and difference of 3 + 2 i and 3 - 2i

SOLUTION: (3 + 2 i) + (3 - 2 i) = 6 and (3 + 2 i) - (3 - 2 i) = 4 i
115

Again the product zz 2 2


= (a + bi) (a - bi) = a + b , hence the product of two
conjugate complex numbers is a non-negative real number.

EXAMPLE: (5) (3 + 2 i) (3 - 2 i) = 9 + 4 = I 3

QUOTIENT:
a+ bi .
We can simplify the quotient c + di , I.e. divide a + bi by c + di by using

a+ bi c- di (ac + bd) + (be - ad) i


the following procedure: c + di c- di c2 + d2
ac+bd be-ad.
-2--2 + -2--2 • I
c +d c +d

EXAMPLE: (6) Divide 3 + 4i by 2 + i


2
3 + 4i (3 + 4 i) (2 - i) 6 + 8i - 3i - 4i 10 + 5i
SOLUTION: 2+1 = (2 + i) • "(2:1} 4 + I --5-

3 + 4i
2 + i
2+1
EXAMPLE: (7) If (x + iy) (2 + 3i) = 5 + 6i, find x and y.

. . 5 + 6i 5 + 6i 2 - 3i 28 - 3i
SOLUTION: Wnte x + 1Y = 2 + 3i = 2 + 3i • 2- 3i = --13-

. I d . . 28 TI3
Equatmg rea an 1magmary parts, x = TI , y =-

Alternatively, expanding and equating real and imaginary parts, we find:

and
2x - 3y =
3x + 2y = 6 '
5] solving these:
28
x=n'
3
y=-n
4 5
EXAMPLE: (8) Expand (a) (I + i) and (b) (I - i) and simplify in the form
a+ ib.

SOLUTION:
4 2 3 4
(a) (I + i) = + 4i + 6i + 4i + i It is easier to expand by
+ 4i - 6 - 4i + writing z = I + i

= -4 z4 = i. z 2 = (I + i) 2 (I + i)
2
= (2i) (2i) = -4
5 2 3 4 5 . . I 5 2 2
and (b) (I - i) - 5i + !Oi - !Oi + 5i - i S1m11ar y z = z • z • z
- 5i - 10 + !Di + 5 - i =(-2i}(-2i)0-i)
= -4 (I - i) etc.
= -4 + 4i
1
EXAMPLE: (9) Express (2 - 3if in the form a + ib

.)-1 I 2 + 3i 2 + 3i 2 3 .
SOLUTION: <2 - 31 = ~ X 2 + 3i = -13- = T3 + TI I
116

Exercise 4A

Perform the indicated operations and express the answers in the form a + ib:

l. (3 + 2i) + (2 - 3i) 2. (5 - 2i) - (3 - 2i)

3. (-3 - 4i) - (12 - 5i) 4. (3 + 2i) - (3 - 2i)

5. (3 + 4i) (2 + i) 6. (5 - i) (3 - 4i)

7. 3 i (2 - i) 8. (4 - 3i) 2

(l + i )2 2
9. 10. (l 3if

11. i (2 + i) (2 - i) 12. (-4 i) (2 i)


I i 2i
14. 3 +
+
13. r:-i 5 + 2i
3 - 2i + 2i
15. 16.
5i .3
I

Find x and y in each of the following:

17. 3x + 2iy 12 + 5i 18. (2 - 3i) + (x + 2iy) 5- 4i


2
19. (x - iy) 2i 20. (x + i y) ( 3 + 4 i) = 2 5i

21. Expand: (2 + i) 3 and answer in the form a + ib.

22. If x + i y= 5 (cos 60° - i sin 60°), find x and y in the surd form and hence
2 1
express (a) (x + iy > (b) - -.- in the form a + i b
X- IY

23. If z =2 + i, evaluate:
2 (c) 2z - l (d) (z - 1) (z
2 + z + 1)
·(a) 3z + 4 (b) z - 2z + 3
2z + l
24. If z = x + iy, express each of the following in the form a + ib:

(a) -z (b) -z (c) z + l (d)


Z-1
25. Solve the following equations for z; express answers in the form a + ib:

(a) (l + i)z (b) 2z


= 2 - i
T+i + 3- 2i =
(I - i) z

(c)
2 3 (d) z + 3
- l + i + 2 - 3i
z = r:l Z-1
117

26. Solve the following equations for z; express answers in the form a + i b:
2 2
(a) z 2 + z + I = 0 (b) z - 2z + 4 = 0 (c) 2z - 3z + 2 = 0 (d) z + ..!. 2
z

27. Find the quadratic equations with wots given below:


(a) i, -i (b) + i, I - i (c) 2 + 3i, 2- 3i

(d) 3 + i, + 3i (e) 2 + i, 2 + i

28. Solve the following pairs of equations for z and w where z and w are
complex numbers. Express answers in the form a + ib.

(a) z + iw 2 + 3i (b) 2z + w I +i
z- iw 2 - 3i z-w I - i

(c) (2 + i)z + (2 - i)w (d) z +(I - i) w 2i


(2 - i) z + (2 + i) w 2 w + (I - i) z

29. Given z = 2 + i, evaluate the following in the form a+ ib:


(e) z 3 (f) z 4
(a) (b) z 2 (c) I (d) z 2 + 2 I
z 2
z z
3 2 4 2 z2)
(Hint: z = z z and z z

30. What is thq•fallacy in the following:

V-3 · V-i2 = 1<-3> . <-12) 6 ?


What is the correct answer?

31. Prove the associative law for multiplication of complex numbers:


(z . z )z = z (z . z ) (Hint: Let z = x + iy etc.)
1 2 3 1 2 3 1 1 1

32. Prove the commutative law for multiplication: z z =z z


1 2 2 1
118

4.3 Complex Plane (Argand Diagram)

A complex number is an ordered pair (x, y) of real numbers, so if we consider (x, y)


to be the cartesian coordinates of a point in a rectangular coordinate system, every
complex number (x, y) then corresponds to some point in the coordinate plane.
Conversely, every point (x, y) corresponds to some complex number x + iy. Thus
there is a one-to-one correspondence between the set of complex numbers and the set
of points in the coordinate plane. The plane is called the COMPLEX PLANE or the
Argand diagram (J. Argand: 1768-1822).

• y
y
P(x,y) =x + iy =z

2 t - - - - - - - - , P(3,2) = 3 +2i

0 3 X X X

Fig. l Fig. 2

The complex number z = 3 + 2i is represented by the point P(3,2) in the


coordinate plane (Fig. l). The x-axis is called the real axis and the y-axis is called
the imaginary axis. For z = x + i y ,
x = real part of z = Re(z)
y = imaginary part of z = lm(z)
In the above example: Re(z) = 3, lm(z) = 2, z = 3 + 2i.

TRIGONOMETRIC FORM OF COMPLEX NUMBERS

A vector is a directed line-segment. We can represent the complex number


z = x + iy by the vector drawn from the origin to the point P(x,y) (see Fig. 2).
The length of the vect0r OP is given by:

r = OP = ~
r is called the modulus of the complex number z = x + iy.
r is denoted by lz 1 , so modulus of z is given by:

lz I = r (x2:l
The direction of the vector OP is given by the angle 6 which OP makes with the
positive direction of the x-axis. Hence 6 is completely determined by the equations:

x = rcos6 and y = rsin6


119

The angle 9 is called an argument of the complex number z = x + iy. The angle 9
is measured in radians, but degrees may be used for convenience. Its value is,
however, not determined uniquely, since

cos9 = cos(9.!. 2kn) and sin9 = sin(9.!. 2kn)

where k is an integer. In order to determine 9 uniquely, we impose the restriction


that
-n <9~ n

Thus for the complex number z = x + iy, we can write its trigonometric form as:

z = x + iy = r(cos9 + isin9), -n <9 ~ n.

This is called the modulus-argument form of the complex number z. We abbreviate


this to mod-arg(z) for our convenience. Frequently to save the space, we may write:

z r(cos9 + isin9) as rcis9


For example z 5(cos 60° + i sin 60°) = 5cis 60°.

It is strongly recommended that you draw a diagram depicting the complex number
z = x + i y to determine the angle 9. The formula tan 9 = Y should be used with
X
caution as 9 is defined for -n < 9 ~ n.
Note carefully that arg z is not defined for z = 0.
We have:

Complex Cartesian
Mod-arg form of z
number form

X = rcos9, y = rsin9
z = r(cos9 +isin9) - rcis9
z
X +iy modz = lz I = r = fx +i
2
P(x,y)
arg z = 9
X
where -n <9 ~ n
Fig. 3
120

EXAMPLES
(10) Express 3 + 4i in mod-arg form: (Fig. 4) ·
2 2
SOLUTION: r = /3 + 4 = 5
Fig.4
tan a ~ (8 in the first quadrant)
a = 53° (to the nearest degree)
:. z = 3 + 4i = 5cis53°
X
(11). Express 1 + I in mod-arg form: (Fig. 5)
SOLUTION: r = ~ = ff y P(J,I)
tanS I
a = 45° Fig.5
z = 1 + i = v2(cos45° + isin45°)
cis(11/4)
a
( 12) Express -2 + 2 v'!i in mod-arg form: (Fig. 6) 0 I X
2 P(-2,2 /3) . ,
SOLUTION: r = v1-2)z + (2 /3) = 4 y
211 _,..
a = 3 from tan a = - v3 Fig.6
z = -2 + 2 v'3i = 4cis 211/3
2../3
a
(I 3)
Express -2 - 2 .l!i in mod-arg form: (Fig. 7) -2 0 X

SOLUTION: r = ~(-2) + (-2 .f3)


2 2 X
= 4
8 = -120° = -211/3 from tanS = '1/3
z = 4cis (~
11
J -2 13
-9

Fig.7

P(-2,-2 v'J)

(14) Express 10{3- 10i in mod-arg form: (Fig. 8) Fig.8 y


2 2
SOLUTION: r = /( 10 {)) + (-1 o ) = 20
1
9 = -30° from tan9 = - - -
-/T
10 VJ - 10 = 20cis(-30°) or 20cis(-11/6)

P(IO /3,-10)
(I 5)
Express in rr.od-arg form: (Fig. 9)
(a) 2 (b) 2i (c) -2 (d) -2i 2 y
SOLUTION:
(a) 2=2+0.i=2cis0
(b) 2i = 0 + 2i = 2cis(11/2)
(c) -2 = -2 + Oi 2cis11 -2 0 2 X
(d) -2i = 0 - 2i = 2cis (-11 /2)

-2
(16) Express (a) 2cis 150° (b) 5cis 21 oo in Cartesian form:
..rJ .
(a) 2cis 150° 2 (cos 150° + i sin 150°) = 2(--;_- + ~ l = - 13 + i

5 J3 5i
(b) 5cis 210° 5(cos210° + isin210°) = --2--2
121

Exercise 48

Convert the following to the mod-arg form:

I. 5 2. -5 3. 5i

4. -5i 5. 2 + 2i 6. 2 - 2i

7. -2 + 2 i 8. -2 - 2i 9. -4 II + 4 l'ii
I 0. I 0 1J + I 0 i
I ..13 . 5 513.
II . -2 + 2 1 12. -2- -2-1

IG 2
13. 1+1 14 • 4 + 3i 15. (I - i)

Convert the following to the Cartesian form:

16. 5cis n/2 17. IOcisO 18. 4cisll

19. 2cis (-n /2) 20. 5cis -n/3 21. Vi" cis TI/3
22. l2cis TI/4 23. 5 cis (-5 11/6) 24. 10 cis (-n I 3)

25. 4cis 211/3


122

4.4 Multiplication and Division of Complex Numbers Using Trigonometry

The mod-arg form of complex numbers can be used to advantage in finding products
and quotients. The mod-arg form is useful in finding the roots of complex numbers.

MULTIPLICATION: Let z
1
= r 1 (cos9 1 + isin9 ) and z
1 2
= r 2 (cos9 2 + isin9 ) be any two
2
complex numbers, then:

z z r r (cos e + i sinS ) (cos e + i sin9 )


1 2 1 2 1 1 2 2
r r [(cos9 cos9 - sin9 sin9 ) + i(sin9 cos9 + cose sin9 )]
12 1 2 1 2 1 2 1 2
r r [cos(9 + e ) + isin(9 + 9 )] ... (J)
1 2 1 2 1 2
Thus to multiply two complex numbers, multiply their moduli r and r and add
1 2
the arguments, 9 and e .
1 2
We can easily generalise:
z z z ••• zn = r r r ••• cis(9 + e + e + ••• + 9n) ••• (2)
1 2 3 1 2 3 1 2 3
Letting z
1
= z
2
= z
3
= ••• = z = rcis9

rn (cosn9 + isinn9) ••. (3)

DIVISION:
r (cos e + i sin e )
1 1 1
To find the quotient
r kose + isin9 )
2 2 2
Multiply and divide by the conjugate of cos9 + isin9 , i.e. (cos9 - isin9 ), we have:
2 2 2 2
z r (cos9 + isin9 ) (cos9 - isin9 )
1 1 1 1 2 2
z r kos9 + isin9 ) kos9 - isin9 )
2 2 2 2 2 2

Thus the modulus of the quotient of two complex numbers is the quotient of their
respective moduli and the argument of the quotient is the argument of the numerator
minus the argument of the denominator.
From results (I) and (4), we immediately deduce that:
zl I= Iz 1 I
rI
l 2 12 2l = rlr2 = lz 1 llz 2 1 and
l- 2
2
r2
= - - , but it is not always correct
-
I2 2 I
to say that arg(z z ) = e + e , because the sum e + e may be outside the
1 2 1 2 1 2
domain -11 < 9 ~ 11. It may be necessary to add .::.211 to bring the arg(z z ) or
1 2
arg(z/z ) into this domain. Hence argz z = argz + argz 2:. 211 (if necessary)
2 1 2 1 2
and arg(z /z ) = argz - argz 2:. 211 (if necessary).
1 2 1 2
123

EXAMPLE: (17) Find the product: 2(cos60° + isin 60°) · 3 (cos 120° + i sin 120°)

SOLUTION: Product= 6[cos(60° + 120°) + isin(60° + 120°) = 6(cosl80° + isinl80°) = -6

EXAMPLE: (18)
Find the quotient: 1O(cos 105° + i sin 105°) .;- 2 (cos 4-5° + i sin 4-5°)

SOLUTION:

QUOTIENT 5[cos(l05°- 45°) + isin(l05°- 45°)] = 5 (cos60° + isin60°)


Quotient 512 + (5 ljf2)i

EXAMPLE: (19) Find the quotient [-1/2- ( ./312)i] + (1 - i) in mod-arg form.

SOLUTION: Change each number to the mod-arg form, then:


1 (cos - 120° + isin - 120°)
QUOTIENT
\li(cos - 4-5° + isin - 4-5°)
(1/ {i) [cos(-120°- -45°) + isin(-120°- -45°)
(1 I {'1) (cos - 75° + i sin - 7 5°)
or (1 I {1.> (cos 7 5° - i sin 7 5°)
2cis45° + 3cis30°
EXAMPLE: (20) Solve for r and 9 if r(cos9 + isin9)
4cis 60° - 3cis 30°

SOLUTION: Numerator = N and Denominator = D


N 2(cos45° + isin45°) + 3(cos30° + isin30°)
2cos4-5° + 3cos30° + i(2sin45° + 3sin30°)
4.01229 + 2.914-2 i

Mod-arg form of N = 4-.96 cis (35.99°)

D = (4-cos60°- 3cos30°) + i(4sin60°- 3sin30°)


= -0.59808 + i (1.964-10)

Mod-arg form of D = 2.05cis 106.94- 0

Hence r(cos9 + isin9) = 4-.96cis 35.99°/2.05 cisl06.94°


2.4-2 cis (-70.95°)
r = 2.4-2 and 9 =- 70.95°
124

Exercise 4C

Perform the indicated operation and express the result in the form a + i b.

1. 4(cos40° + isin40°). 2(cos50° + isin50°)


2. 5(cos120° + isinl20°). 2(cos60° + isin60°)
3. 4(cosl35° + isin135°). 3(cos90° + isin90°)
4. 5(cos 2n/3 + isin 2n/3). 2(cos n/6 + isin n/6)
2
5. [2(cos n/3 + isin n/3)]
3
6. [3(cos n/6 + isin n/6)]
7. 12 (cos0° + isin0°) -7 [6 (cos 120° + i sin 120°)]
8. 10(cos 2n/3 + isin 2n/3) + [5(cos n/6 + isin n /6]
9. -10(cos 2n/3 + isin 2n/3) + [-5(cos -n/6 + isin -n/6)]
10. 6(cos- 2n/3 + isin- 2n/3) + [2(cos- n/3 + isin- n/3)]
3 3
II. Prove that [r(cos9 + isin9)] = r (cos39 + isin39)
12. Change each of the following complex numbers to mod-arg form and perform
the indicated operation, giving answers in mod-arg form.
( ) .!.._.!..l (b) (-1 + i) (c) (I + i) (l + v'3 i) (d) ( V3 + i) (l - i)
a I - i V3 + i (l + v'3 i) ( y'J - i)
13. If z
1
= 4cisl20° and z
2
= 2cis30°, find

2 10
(c) (z > (d) (z ) and express your answers in
1 2
the form a + ib.

14. Express = I + IJi and z = {J + i in mod-arg form and hence write


z
1 2
down the following in rcis9 form:
5 4 5 zl
(a) (z ) (b) (z ) (c) (z z ) (d) (e)
1 2 1 2 z2
15. Express (a) z + z (b) z - z in mod-arg form if
1 2 1 2
z
1
= I + ./3i ' z = ./3 + i and hence find

~1zl -+ z2z~
zl + z2 2
(c) and (d) in the form A+ iB.
z1 - z2
125

4.5 Powers of Complex Numbers: De Moivre's Theorem

For any real number x (x > 0), we know that xn = x . x ••• x to n factors, if n is
a positive integer.
The corresponding result for any complex number z = a + ib = r(cos9 + isin9) is of
far-reaching importance and leads to a theorem known as De Moivre's theorem.
Let z = a + ib = r(cos9 + isin9), then by the product rule of two complex numbers
2
z 2 = r 2 (cos9 + isin9) (cos9 + isin9) = r (cos29 + isin29)
2 3
Then z 3 = z. / = r(cos9 + isin9). r (cos29 + isin29) = r (cos39 + isin39)
By successive applications of the product rule, we get:
z 4 = r 4 (cos 4 9 + i sin 49) and so on. These results can be summarised in the
following theorem:

Theorem: If n is any positive integer (n > 0), then


[r(cos9 + isin9)]n = rn(cosn9 + isinn9)

For r = I, we obtain De Moivre's theorem:

I (cose + isin 9)n = cosne + isinne

De Moivre's theorem can also be proved by the method of mathematical induction.


(This proof is given in a later section.) It also holds for n = 0 and for a negative
integer.

6
EXAMPLE: (21) Find (I + i)

SOLUTION: Write z = I + i in the mod-arg form.


z = ../2(cos Tl /4 + isin Tr/4)
6
:. z 6 = [ v'2 (cos Tr/4 + isin Tr/4) ] 6 ( Vi.) (cos 6 Tr/4 + isin 6Tr/4)
8 (cos 3 TI/2 + i sin 3TI/2)
-8i
4 7
EXAMPLE: (22) Evaluate in the form a + ib: (I + .fji) + (-1 + /3i)
SOLUTION: We write z = I + vJi = 2(cos60° + isin60°)
w = -1 + ni = 2(cos120° + isin120°)
4
z
4 2 4 (cos60° + isin60°) = 16 (cos240° + isin240°) and
7 7
w / (cosl20° + isin120°) 128(cos840° + isin840°)
128 (cos 120° + i sin 120°)
4
Hence?
z
1
~~ ~os (240° - 120°) + i sin (240° - 120°)]
w
(1 /8) (cos 120° + i sin 120°), i.e. -1/16 + i( '113/16)
126

Exercise 40

Evaluate in the form a + ib, by using De Moivre's theorem:


5
I. (cos 12° + i sin 12°) 2. [2 (cos 15° + isin 15°) ] 6
4
3. (cos nl4 + isin nl4) 4. [2 (cos nl3 + isin nl3) ] 6
5. [cos(-nl4) + isin(-nl4)] 6 6. [2(cos2nl3- isin2nl3)] 4
(Hint: bracketed expression = cis - 21113)
Express the following in mod-arg dnd cartesian form (7 to 14).

7. (I + i)6 8. (3 + 4if 2
9. (I - i)4 10. ( v'j - i)4
II. (-1 - i)IO 12. (2 - 2 {jif 4
13. (I + 11'304 14. (2 J3 + 2i) 5

Simplify the following in the form rcis9 and wherever possible, express the answers
in the form a+ ib (15 to 20).

15. (1 - i)4 (I + i)3 16. (I - VJi)3 (I + i)4

17.
(I + VJi)3 (2 + 2i) 4
18.
(I - i)4 (I - v'Ji)2
4 5
19.
(2cis nl6) (3cis n I 12)
20.
(4cis nl3) 3 [3cis (-n I 36)] 3
127

4.6 Roots of Complex Numbers

Finding a desired root of a non-negative real number is a simple matter. For


example:

The two square roots of 4 are .:!:. 2


113
The cube root of 8 is (8) = 2 and so on.

But finding the desired roots of complex numbers is not so simple, as we must use De
Moivre's theorem to find the:
three cube roots, four 4th roots, five 5th roots and so on of a complex number
a + ib.

EXAMPLE: (23) Find all the cube roots of -1

SOLUTION: We use two methods to find the cube roots.

Method 1: If z is a cube root of -I, then


3 2
z + 1 = 0, hence (z + 1) (z - z + 1) = 0

The three cube roots are -1,


1 .:!:_ J3i
2

Method II: We write -1 cos 11 + i sinTT and make the expression general by
adding 2kTT to the arg(-1), hence
-l = COS (TT + 2kTT) + isin (TT + 2kTT) ••. ( l)

Now let the required cube root be R (cos~ + 1sin~)

W'cos(TT + 2kTT) + isin(TT + 2kTT) = R(cos~ + isin~)


We take the cube of both sides:

R 3 (cos3~+ isin3~) = cos(TT + 2kTT) + isin(TT + 2kTT)


Equating the real and imaginary parts:

R 3 cos3~ = cos(TT + 2kTT) and sin ( 11 + 2kTT)

From these: 1, i.e., R = and 3~ TT + 2kTT

~ TT/3 + 2kTT/3

Hence the cube roots are given by:


z = (cos~+ isin~) = cos(TT/3 + 2kTT/3) + isin(TT/3 + 2kTT/3)
where k = 0, 1, 2.
128

For k = 3, the angle is n /3 + 2n and the corresponding cube-root is the same as


the root for k = 0. In a similar manner, k = 4 produces the same root as k = 1
1 3
and so on. The three cube roots of (-1) / are:
k 0, zl cos n/3 + isin n/3 1/2 + J3/2 . i
k
k
I,
2,
z2
z3
cosn + isinn
cos 5n/3 + isin 5n/3

These three roots are the same as given by Method I.


The modulus of each root is
-I
1/2 - fi/2 • i
r
I, hence these
three distinct cube-roots lie on a circle of
radius I and centre the origin.
They are equally spaced, the angular distance
between any two roots being 360° + 3 = 120"

Definition: If n is a positive integer, then R(cos\6 + isin\6) is an nth root of


r(cosa + isina) if and only if Rn(cos\6 + isin\6)n = r(cosa + isina)
From this, and using De Moivre's theorem:
11
R (cosn16 + isinn\6) = r(cos a+ isina)
rcosa = Rncosn\6 and rsina = Rnsinn\6
By squaring and adding:
Rn = r, hence cosa = cosn\6 gives n\6 = a + 2kn

Hence R = r 1/n , 0= a +n 2k 1T ' where k = 0, 1, 2, •• • (n - I)

We conclude that the n nth roots of the complex number r(cosa + isina) have the
1/n . a + 2kn )
modulus r and arguments are g1ven by --n--' k = 0, I, 2, ••• (n - 1 •

EXAMPLE: (24) Find the 4th roots of - & + & >/3 i


4
SOLUTION: Write -& + &IJi = 16(cos2n/3 + isin2n/3) = 16(cosl20" + isinl20") = z
• [ (2kn + 2n/3) .. (2kn + 2n/3)]
.. The 1!-th roots are z = 2 cos
4
+ 1sm
4
, k = o, 1, 2, 3

The four roots are:

k 0, zl 2 (cos30" + i sin 30") "3 + 1i


k I, z2 2 (cos 120" + isin 120") -1 + ,/3i
X
·k 2, z3 2 (cos210" + isin210") -.13-l.i
v'3 • i Fig. II
k 3, z4 2 (cos300" + isin300") I -

The points in the Argand diagram that correspond to these four


distinct 4th roots are equally spaced (at an angular distance of 360" .;- 4 90")
on a circle of radius 2 and centre the origin.
129

5
EXAMPLE: (25) Find all 5 roots of z - 32 = 0 and show them in an Argand
diagram.
5
SOLUTION: z = 32 = 32(cos0 + isinO) 32[cos2k11 + isin2k11]

The five fifth roots are given by


2k11 . . 2k11]
z = 2 [cos T + I Sin T where k = 0, 1, 2, 3, 4.

To clearly visualise the roots, we replace 11 by 180", then the five roots of
z 5 = 32 are:
k o, zl 2ciso• = 2 (the only real root)
k !, z2 2cis72"
k 2, z3 2cis 144"
k 3, z4 2cis216"
k 4, z5 2cis288"

Note that the argument of successive root increases by 360° "'" 5 i.e. 72". This way
you can quickly write down all the roots.
Observing that:
lz 11 = lz 2 1 ••• = lz 51 = 2, we can show
these roots on a circle of radius 2, centre 0.

From the diagram: A(z )


1
-
z5 z2 X

z4 z3
Fig. 12

5
EXAMPLE: (26) Solve z 16 .f2 + 16 J2i

SOLUTION: Write 1+ i l'i.cis( 11/4) = v'2cis(2k11 + 11 /4)


5
z 32cis (2k11 + 11/4)
The five 5th roots are given by
z = 2cis(rr/20 + 2krr/5)
The five roots are: k = o, 1, 2, 3, 4.
zl 2cis9"
z2 2cis (9" + 72") 2cis81"
z3 2cis(9° + 144") = 2cisl53"
z4 2cis(9" + 216") = 2cis225"
z5 2cis(9" + 288") = 2cis297"
130

Exercise 4E

1. Find the two square roots of:


(a) -16i (b) 2 + 213i (c) -2/3- 2i (d) 4 + 3i

2. Find all the cube roots of:


(a) (b) -8 (c) 27(~ + ~ 0 (d) -8i

3. Find the four 4th roots of:


(a) -1 (b) -16 (c) -8 - 8 J3i (d) 1 V3.1
2-2
4. Find the five roots of:
.fj 1.
(a) 32 (b) -32 (c) -2 + 2 I (d) 32i

5. Find the solution set for each of the following equations. Express your answers
in the form a+ bi.
3 2
(a) X - i 2 (b) X + (3 - i)x - 3i = 0

(c) 4 (d) 4
X + 1 0 X + 16i = 0

(e) 2 (f) 6
X + i = 0 X 64 = 0
4 1 13. 2
2 -
(g) X 21 (h) X + i = 0
131

4. 7 De Moivre's Theorem and Its Applications

We have used De Moivre's theorem in finding the powers and the roots of complex
numbers. We now prove it by the method of induction and consider some further
applications.
De Moivre's theorem is:

For any integer n, (cos9 + isin9)n = cosn9 + isinn9 ••• (I)

0
Proof: For n = O, (cos9 + isin9) = I = cosO + isinO
For n = I, cos9 + isin9 = cos9 + isin9
So the theorem is true for n =0 and n = I. We assume it true for n =k, a positive integer, so:
(cos9 + isin9)k = cosk9 + isink9 ••• (2)
Multiply both sides of (2) by cos9 + isin9, then
(cos9 + isin9)k+l = (cosk9 + isink9) (cos9 + isin9)
= (cosk9cos9- sink9sin9)+ i(sink9 cose + icosk9sin9)
= cos(k + I) 9 + isin(k + I) 9
This proves that the theorem is true for n = k + I, if it is true for n = k. Since it
is true for n = I, then it is true for n = 2 and so on for all positive integers n.
The theorem is also true when n is a negative integer.
Let n = -m, where m is a positive integer.
(cos e + i sine )n= (cos e + i sine fm = - - - - - - -
(cose + isin9)m cosm9 + isinm9

by using the theorem for positive integer m.


I (cosm9- isinm9) cosme- isinm9
Now
cosme + isinm6 = (cosm9 + isinme) kosm9- isinm9) 1
Finally cosm9 - isinm9 = cos(-n9)- isin(-n9) = cosn9 + isinn9 •
Hence the theorem is true for negative integers.

DERIVATION OF TRIGONOMETRIC FORMULA

By application of both De Moivre's theorem and binomial expansion, we can find:

(a) cos n e and sin n 9 in terms of powers of sine and cos 9 •


11
(b) cosn9 and sin 9 in terms of multiple angles (29, 39, etc.)

For (a) and (b) we shall need the following:


132

Let z = cos9 + isine , then:


n -n
z cosne + isin ne and z cosne- isinne.
-n n
Adding, +z z= 2cos ne ••• (1)
n -n •.• (2)
Subtracting, z - z 2isin ne
1 1
For n = I, Z+- 2cos e z 2isine
z ' z
2 1 2 1
n = 2, z +2 2cos2e , z - 2= 2 isin 2e etc.
z z '

EXAMPLE: (27) Express (a) cos4e in terms of cose


(b) sin4e in terms of cose and sine
(c) tan4e in terms of tan e.

SOLUTION: We use c = cose and s= sine to simplify our work.


By De Moivre's theorem:
4 4
cos49 + isin49 = (cos9 + isin9) = (c + is)

C
4 + 4c3.IS + 6c2.2 2 4c.3
1 S +
3 .4 4
1 S + 1 S

c4- 6c2s2 + s4 + i(4c3s- 4cs3)

( Note t hat 1.2 = - 1, 1.3 = -1,


. 1.4 = 1, etc. )

Equating real and imaginary parts:


4 2 2 4 4 2
(a) cos49 = c - 6c s + s = c - 6c (1 - c 2 ) +(I - c 2) 2 ,
cos e = 1 - sm. 2e
. 2
usmg
4 2
:. cos49 = 8cos e- 8cos e + 1
3 3
(b) sin49 = 4c\- 4cs = 4cos 9sine- 4sin 3 ecose
We note that sin4e cannot be expressed in terms of sine alone.

(c) t e _ sin4e 4c\ - 4cs3


an 4 - cos4e c4 _ 6c2 5 2 + 5 4

4
Divide both the numerator and the denominator by c remembering
sin9/cose = tan9,
3
we have: tan49 4tane - 4tan e
2 4
1 - 6tan e + tan e
133

EXAMPLE: (28)

(a) If z " cos 9 + i sin 9, show that:

(i) zn + _!_ " 2cos n 9 (ii) zn - 2isin n9


n n
z z

(b) Express the foUowing in terms of cos n 9 or sin n 9


4 3
(i) cos 4 9 (ii) sin 9 (iii) sin 9

SOLUTION:

(a) z cos9 + isin9

- £ 1 = cos9 - isin9
z
By De Moivre's theorem:

zn " cosn9 + isin n9,


n
z-n cos n9 - isin n9
z
n
Adding: z + -n 2cos n9 ••• (i)
z
n 2isin n9 •.. (ii)
Subtracting: z n
z

(b) (i) (2cos9) 4 (z + ~) 4 = (z


4
+ -{) + 4(z
2
+ ~) + 6
z z
4 [using results (a)(i) n = 4, 2]
16cos 9 2cos49 + 8cos29 + 6
4 1
cos 9
8 (cos49 + 4cos29 + 3)

4 1 4 4 1 2 1
(ii) (2 isin 9 ) (z - -) = (z + z4 ) - 4 (z + /) + 6
z
4 2cos49 - 8cos29 6 [in (a)(i) n = 4, 2]
16sin 9 +

4
sin 9 81 (cos49 - 4cos29 + 3)

1 3
(iii) (2 i sin 9 )3 (z - -) expand and rearrange.
z '
3 1 1 = 3,
(z -
3z ) - 3(z - -z> [in (a) (ii) n 1]

2isin 39 6isin9
. 39
Sill i (3sin9 - sin39)
134

Exercise 4F

I. Express (a) cos 3 e (b) sin 3 e in terms of cos e and sine, hence express tan 3 e
in terms of tan e

2. Express (a) cos 5 e (b) sin 5 e in terms of cos e and sine, hence express tan 5 e
in terms of tan e

3. Express (a) cos6e (b) sin6e in terms of cose and sine and hence express
tan 6 e in terms of tan e
3
4. Express (a) cos 3 e (b) sin e in terms of multiples of e
5
5. Express (a) cos 5 e (b) sin e in terms of multiples of e and hence integrate
5 5
(c) fcos ede (d) fsin ede

6
6. Express (a) cos 6 e (b) sin e in terms of multiples of e, and hence
TI /2 Tl /2 6
integrate (c)
f
0
6
cos ede (d)
0 fsin ede

7. Find the constants p, q, r and s if: (a) co/ e = pcos?e + qcos5e + rcos3e + s
Tl /2 7
and hence evaluate: (b)
0 f cos e de

7
8. Find the constants p, q, r, s if: (a) sin e = psin?e + qsin5e + rsin3e + ssine,
rr/2
7
and hence evaluate: (b)
0 f sin e de

9. Show that 1'1 + cose + isine)n = 2ncos n( 2


e) ~cos n 2e + 1· sm
· n eu
2 , where n
is a positive integer

10. Prove the following:


2 4 4 2
- 6 tan e + tan e cot 9- 6cot e + 1
(a) cot4e
3 3
4tane - 4tan e 4c'ot 9 - 4cot 9
2 4 5 3
I- 10tan 9 + 5tan 9 cot e - I Ocot e + 5cot e
(b) cot5e
3 5 4
5tan9- 10tan e + tan e 5cot e- IOcoie +·I
135

4.8 Square Roots of a Complex Number

We usually use De Moivre's theorem to find the roots of a complex number; but there
is also a special requirement to obtain the square roots in the form a + ib.
Two methods of obtaining the square roots are:
2
I. We convert z = a + ib to mod-arg form and use De Moivre's theorem to solve
l =a+ ib.
2. We assume the solution z = x + iy and hence solve two simultaneous equations
involving x and y.

METHOD 1: Using De Moivre's theorem.

EXAMPLE: (29) Find the two square roots of 2 + 2/Ji

Let z 2 = 2 + 2 _,i = 4(cos n/3 + isin n/3) = 4[cos(2kn + n/3) + isin(2kn + n/3)]
By De Moivre's theorem:

z = 2 ~os ( 2kn ; n/3) + isin ( 2kn ; n/3) J, k = 0, I.

The two roots are:

zl 2(cos n/6 + isin n/6) = tf3 + i


z2 2(cos7n/6 + isin7n/6) = -13-
Verify that, [!. ( J3 + i) ] 2 = 2 + 2/3i

METHOD U: EXAMPLE: (30)

Let z = x + iy, then Find the square roots of 7 + 61/ii


2 2
z =a + ib SOLUTION: Let (x + iy) = 7 + 6 /2i
2
X2 - y 2 + 2"IXY = a + 1"b :. x - / = 7 and 2xy = 6 v'f
Equating real and imaginary parts.
2
Now (x +
2 2
(x - />
2 2
+ 4x / />
x 2 - y 2 = a and 2xy = b, then: 49 + 72
(x2 +/)2 = (x2 -/)2 + 4x2/ = a2 + b2 121
2 2
.22~
•• x
22
+ y = '1/a- + b. as x + y >0 X +y =11 ••• (1)
2 2 ••• (2)
and x - y =7
Finally we solve
Solving (1) and (2):
i -/ = a and i + / = /l + b2 X=!.3, y=!.fi
for x and y. We check xy = 6{2 for the proper
combination of x and y.
The required roots are:
136

Exercise 4G

1. Find the square roots of the following in the form x + iy.


(a) 3+4i (b) 3-4i (c) 5+2J6i (d) 7-6/ii
(e) 5 + 12i (f) 8 + 6i (g) (h) -8i

2. Solve the following equations, expressing the answers in the form x + iy.
2 2 2
(a) z = -15 + 8i (b) z = I + {:Ji (c) z = 2i

2
3. Use the formula z = (-b :!:. lb - 4ac)/2a to express the roots of the following
equations in x + iy form:
2
(a) z - (I - 4 i) z - (5 - i) = 0 (Hint: J5:l2i 3 - 2i)
2
(b) z + (2 + 4i) z - 11 - 2i 0 (Hint: .Js + 6i 3+i)
2
(c) z + ( 4 + 2 i) z + (3 + 2 i) = 0
2
(d) z + (4 - 2i) z + 6 =0
4. Write each of the following in the form a+ ib.
(Take ,fZ to mean the square root whose real part > 0)
(a) 1 +i
./8+6i
(b) /5 -
- l2i
5+12i
(c) ~
I + z + z 2 where z = v& + 6i

5. Prove the following:

(a) ~ + ~ h (./a
2
+b
2
+a)

(b) ..ra+l>1 va:bT = i h< E:;}_ a)


2 + b2
6. If X+ iy = ~
c + !d '
prove that (X2 + y 2)2 =
a
2
c + d2

7. Simplify:
..t5:lii + ~
v57iTi {5 - 12i
137

4.9 Properties of Conjugate Complex Numbers


y
z= x - iy is the conjugate of z = x + iy. P(z)
If the point P(x,y) represents z = x + iy in the
y
complex plane, then Q(x,-y) is the reflection of
I
P in the x-axis and hence z is the reflection of X I X
.. : -y
z in the x-axis.
conjugate of x - iy.
Note that x + iy is the
We have: Fig.l3
'' .-~ Q<z>
I
I

I. 2. arg -z = -argz 3. - = x 2 + y 2 = lzl2 -- l-zl2


zz
4. z + z = 2x (a real number) 5. z - z = 2iy (a purely imaginary number)
Further, for two complex numbers z and z we prove the following properties:
1 2
6. 7. 8.

z
9. 10. z-I 2
lzl

Proof: Let z = a + ib and z = c + id, then


1 2
(a) z + z = a + ib + c + id (b) z - z = a + ib - c - id
1 2 1 2
=(a+ c)+ i(b +d) =(a- c)+ i(b- d)
~
2 = (a + c) - i (b + d) z
1
- z
2
= (a - c) - i (b - d)
= (a - ib) + (c - id) = (a - ib) - (c - id)
= zl + z2 =zl - z2
= (a + ib) (c + id) (d) We have
= (ac - bd) + i(be + ad)
zI (ac + bd) - i (ad - be) (see Section 4.2,
z • z = (ac - bd) - i (be + ad) example 5)
1 2 z2 c2 + d2
= (a - ib) (c - id)
zI (a - ib) (a - ib) (c + id)
= zI • z-2 Again, - =(c _ id) = (c _ id) (c + id)
z2
I z z (ac + bd) + i(ad - be) h
, ence
= z = zz = Izl2 ' c
2
+ d
2

using property (3)


( :~)
We easily generalise, but shall not prove that:
II. z I + z 2 + z 3 + ••• + zn = z I + z 2 + ••. + zn

12. ZIWJ + Z2W2+ ••• +ZnWn = zl.wl +Z2 W2+ ••• +Zn.wn
In particular, if w is real, then using w = w, we have
13. z 1w"l + z 2w 2 + ••• + znwn w 1z 1 + w 2z 2 + ••• + wnzn

We shall use the properties (9), (10) and (11) in the next chapter on Polynomials.
138

EXAMPLE: (31): If z =~ + 3i, express the following in a + ib form:


(a) z (b) z + z (c) z - z (d) zz (e) £1

SOLUTION:

(a) z =~- 3i (b) z + z =~ + 3i + ~- 3i =8 (c) z- z =~ + 3i- (~ - 3i) =6i


(d) zz =(~ + 3i)(~ - 3i) (e) z
-1
=z1 =zzz =Izlz2 =2
~ - 3i ~
5 = 25
3. . ( )
- 25 1, usmg d •
= 16 + 9
-1 -1
= 25 Note that z
.1-
~ z in general, even though arg z =arg z .

EXAMPLE: (32) If x + 1 y
. = Vr;;-:;Tb
~ , prove t h at ( x 2 + y 2)2

SOLUTION: We square both sides and write


. )( . )
w = (X + IY X + IY = ca + ib
+ id ••• (I)

Using ,
-z
1 • z2 = z1z and
(z1)
Z2
2
. )( . ) a - ib
w = (X- IY X- IY = c - id ••• (2)

Multiplying (I) and (2) and using (x + iy) (x- iy) = x2 +/,etc.
22
2 2 2 2
(x + y ) (x + y ) = a2 +b
2
, hence the required result.
c +d

Exercise 4H

I. Given (a) z = 1 + 2i (b) z =


3 - i, find and sketch the following:
1
o> z (ii) z + z Oii) z- -z ov> z • -z <v> lzl <vi> 1zl <vii) £
2
2. If x + iy = (a + ib) , then without finding x and y, prove that
x2 + / = (a2 + b2)2

(x + i)2 2 + b2 (x2 + 1)2


3. If a+ ib prove that a
2x - i ' ~x 2 + I
~- If X + iy ~
1 ' I
show that X
2
+y
2
=

5. If a =cosa + isina, b =cos B + isinB, c =cosy + isiny and a + b +c =0,


t hen prove t hat: 1 + b 1 + c.!. = 0
a <H.mt: -a1 = lal2
a: etc. >

6. If (a + ib)l/J = x + iy, prove that: ~(x 2 - /> = ~X + ~y


[Hint: (a + ib) = (x 3
+ iy) , equate real and imaginary parts, substitute in~+~]
X y
. 3 + 2isin9
7. Find 9, 0~9~2n, If l- 2 isine ispurelyimaginary.

-1 1 - 2icot(9/2)
8. Prove that (I - cos9 + 2isin9) = 5 + 3cos9
139

4.1 0 The Complex Roots of Unity

The equations of the form zn = a + ib give rise to n roots which are equally
spaced on a circle of radius r, where r is the modulus of any root. In the special
case where zn = I, then r = I, i.e. all of the roots lie on a circle of radius 1,
2
centre the origin (x + / = 1).
In the chapter on Polynomials, we shall learn that the complex roots of zn - 1 = 0
(if any) must occur in conjugate pairs.
We shall also show that if w is a complex root of zn - 1 = 0 with the smallest
positive argument, then the n roots of xn - I = 0 are:
2
1, w, w , ... , wn-J and consequently, the sum of the roots is given by
1 + w + w2 + ... + wn-l = 0 •
3
EXAMPLE: (33) Solve z - 1 = 0
3
SOLUTION: Factorise z - I = O, then
(z - I) ( / + z + 1) = 0. The three roots are:
-1 ,f'J. I {3.
2
1 = 1• 2 2 = 2 + 2 1' 2 3 = - 2- 2 1
Let w = z 1 .f'J . h
2
= -
2 + 2 1, t en
2
w = i-
2
2 . ~. '? i - ~ =- ~ -
i = z3
3
? X

Thus 1, w and w are the three roots of z - 1 = 0


2
. d _ coefficient of z _ Fig. 14
We f m z 1 + z2 + z3 - - . . 3 - 0
coefficient of z
. I 1 2 1 1 ./3. 1 {j_ 0
Alternative y + w + w = z 1 + z 2 + z 3 = - 2 + 21 - 2- 21 =
3 2
The roots of z - I = 0 lie on the circle x + / = 1. The roots are equally spaced
with angular separation between any two roots equal to 360 f 3 = 120°. We observe
that z3 = z2 •

EXAMPLE: (34)
4
Solve z - 1 = 0
SOLUTION: (z - 1) (z + 1) ( / + 1) = 0.
The four roots are: 1' - 1' i, - i
Let w = i, then
i =-I
3 .3 .
W = I = -1 Fig.l5
2 3
Again, 1 + w + w + w = +i-1-i=O

The roots lie on the circle x 2 + y2 = 1, the angular separation between any two
consecutive roots being 360 -;. 4 = 90°.
We also observe that z = z 2•
4
140

7
EXAMPLE: (3}) If w is a complex root of the equation =0, then show that
x - 1
2 3 6 4 .5
the other complex roots are w , w , w , w and w • Also prove
2 3 4 .5 6
that 1 + w + w + w + w + w + w = 0.
7
SOLUTION: z = l = cos 2kn + isin2kn ••• (1)
Let z = r (cos9 + isin9) be a root of / - I = 0, then by De Moivre's theorem:
/ = / (cos79 + isin79) •.• (2)
From (I) and (2), equating the real and the imaginary parts:
r 7 cos79 =cos(2kn) and / sin79 = sin2kn
14
By squaring and adding, r = 1, hence r = 1 since r > 0
• cos79 =cos2kn, whence 7 9 = 2kn
2kn
9 = T , where k = 0, 1, 2, ••• , 6
The seven roots of unity are given by:
z
1
=cosO + isinO = 1 z
2
=cos2n/7 + isin2n/7
z =cos4n/7
3
+ isin4n/7 z4 =cos6n/7 + i sin 6n /7
z =cos8n/7 + isin8n/7 z
6
= coslOn/7 + isin lOn/7
5
z = cos12n/7 + isinl2n/7
7
Now we apply De Moivre's theorem, in reverse order, i.e.
cosn9 + isinn9 = (cos9 + isin9) 7 to each of the above complex roots.
Let z
2
= w =cos2n/7 + isin2n/7
2 2
z = cos4n/7 + isin4n/7 = (cos2n/7 + isin2n/7)
= w
3 3 3
z =cos6n/7 + isin6n/7 = (cos2n/7 + isin2n/7) = w and so on,
4 4 .5 6
hence, z = w , z 6 = w , z 7 = <tJ •
5 2 6
To prove that 1 + w + w + ••• + w = 0, we note that the sum of the roots of
6
7 -coef. of z
z - 1 = 0 is zero, because: z 1 + z + ••• + z 7 = hence
2 7
coef. of z
2 6
substituting for zl' z , etc. we have: 1 + w + w + ••• + w =0.
2

Observations: The roots of zn - 1 =0 are given by:


(1) z = cos ~
n + isin ~,
n where k = 0, 1, 2, ••• , (n - 1).
(2) If n is even, two roots of zn - 1 = 0 are real, i.e. ~ 1.
(3) If n is odd, only one root of zn - 1 = 0, i.e. z =1 is real.
(4) If w is the complex root with the smallest positive root, then the
2 n-1
entire set of roots is given by: 1, w, w , ••• , w •
(.5) 2
1 + w + w + ••• + wn-1 0 . h b .
= , e1t er y usmg the sum of the roots of
zn - 1 =0 or summing up by observing that this sum is a geometric series,
. 1 - wn 1- 1
w1th sum =~ = 1 _w = 0, since wn = 1, w ~ 1.
141

Exercise 41
3 2
1. If w is a complex cube root of unity (i.e. a root of z = 1), prove that w is
also a complex cube root of unity. Further prove that:
2 1 1 3 25 2
(a) 1 + w + w =0 (b) - - + - - = 1 (c) (1 + w) = -1 (d) (1 + w ) = -w
1 +w 1 + w2
3
2. w is a complex root of the equation z - 1 = 0. Form a quadratic equation
whose roots are given by a = 2 + w and B= 2 + d.
3. If w is the complex cube root of unity, show that:
2
2 3 2 3 a + bw + cw
(a) (1 + w- w ) - (1 - w + w ) = 0 (b)
2
c + aw + bw
2
(c) a + b w + c w = til
2
b+cw+aw
2 2 2
4. If x = a + b, y = aw + b w , z = aw + bw, where 1, w, w are the cube
roots of unity, prove that:
2 '2 2 2 2
(a)x+y+z=O (b)(a+bw+cw)(a+bw +cw)=a +b +C -ab-bc-ca
2
5. If 1, w, w are the three cube roots of unity, prove that
2 3 3 3
(a + b +c) (a + bw + ci>
(a + bw + cw) = a + b + c - 3abc.
5 2 3 4
6. If w is a complex root of z - 1 = 0, show that w , w and w are the
other complex roots.
2 3 4
(a) Prove that 1 + w + w + w + w = 0
4
(b) Find the quadratic equations whose roots are a = w + w and
2
B = w + w 3 5 6
(Hint: use w = 1 to reduce w and w )
7

(c) Show the roots of z 5 - 1 = 0 in an Argand diagram.


(d) Find the area of the pentagon formed by the roots.
7. If w is a complex root of z 6 - 1 = 0 with the smallest positive argument, then
2
show that the other roots are w , w , w , w • Prove that:
3 4 5
(a)
2
1 + w + w + w + w + w
3 4 5
= 0.
(b) Find all the roots in the form a + ib and indicate these roots in an
Argand diagram. Find the area of the hexagon formed by the roots.
(c) Find the two quadratic equations whose roots are
4
(i) w and J (ii) and w d
(d) Using part (c), show that
5 2 4
(i) z 6 - 1 = (z- 1) (z + 1) [(z- w) (z- w )] [(z- w ) (z- w )]
2 2
= (z 2 - 1) (z + z + p (z - z + 1)

4 2 2 4 5
(ii) The roots of z + z + 1 =0 are w, w , w and w
142

8. Show that if. w is one complex root of the equation zn - I = 0, then


(a) zn - I = (z - I) (z - w) (z - i> ... (z - wn-l)
(b) Deduce from part (a) that:
zn-l + zn- 2 + ••• + z + I = (z - w) (z - w2) ••• (z - wn-l)
(c) (1 - w) ( I - w2 ) ••• (1 - wn-l) = n
9. Prove by mathematical induction that for any real 9,
cos ne + isin ne = (cose + isin9)n
(a) Find the 6 sixth roots of I, expressing each in the form a + ib.
4
(b) Using part (a), find the four roots of z + z 2 + I = 0 and show their
positions in an Argand diagram.
143

4.11 Miscellaneous: Factorisation over the Complex Field


3 3 2
Consider the factorisation of z - 1 = 0 over C. z - 1 = (z - 1) (z + z + 1)
2
Now z + z + 1 has no real linear factors, but over C we can write
z 2 + z + 1 = (z - a ) (z - 8 ) where a = -1 +2 t/3i , 8 = - 1 - 2/3i

Thus it appears that we can factorise expressions of the form zn - 1, zn + 1 and


(by extension) zn-l + zn- 2 + ••• + 1, into either: (a) real quadratic factors or
(b) complex linear factors.
EXAMPLE: (36)
6
Factorise z - 1 into real quadratic and real linear factors, hence
4 2
factorise z + z + 1.
6
SOLUTION: We solve the corresponding equation z - 1 = 0
z 6 = 1 = cos(2kn) + isin(2kn), hence the six sixth roots are given by
z = cos(2kn/6) + isin(2kn/6), k = 0, 1, 2, 3, 4, 5. i.e.
z = cos 0 + i sin 0 = 1 z = -1
1 4
z
2
=cos n/3 + isinn /3 z5 =cos4n/3 + isin4nj3 =cos2n/3- isin21T/3
z = cos2n/3 + isin2n/3 , z = cos5n/3 + isin5n/3 = cosn/3- isinn/3
3 6
We find that z
6
=z
2
and z 5 = z 3 , hence:
z2 + z6 = z2 + z2 = 2cosn/3 and z3 + z5 = z3 + z3 = 2cos2n/3
Also z z = z z = 1 , z z =z z = 1
2 6 2 2 3 5 3 3
6
Now z - 1 = (z- z ) (z- z ) (z- z ) (z- z ) (z- z ) (z- z )
1 2 3 4 5 6
= (z- 1) (z + 1) [(z- z ) (z- z )] [(z- z ) (z- z )]
2 6 3 5
= (z -l)(z + l)[z 2 - (z 2 + z6 )z + z 2z6 ][z 2 - (z 3 + z5)z + z z ]
3 5
2
(z - 1) (z + 1) [i -
2cos(n /3) z + 1] [z - 2cos(2n/3) z + 1]
2 4 2
Since z 6 - (z - 1) (z + z + 1), we at once have:
6
z
4
+ z
2
+ 1 ~ = [z
2
- 2cos(n/3) z + 1] [z
2
- 2cos(2n/3)z + 1]
z - 1
6
EXAMPLE: (37) Solve z+ 1 = 0. Express the roots in the form a + ib. Show these
6
roots in an Argand diagram. Factorise z + 1 into real quadratic factors.
6
SOLUTION: z = -1 = cos(n + 2kn) + isin(n + 2kn), hence the six sixth roots are given
. 1T + 2k1T
by z = c1s - -- , where k = 0, 1, 2, 3, 4, 5. z2
6
z = cisTT/6 = 13/2 + i/2 z = cis7n/4 =- 13/2- i/2
1 4
z = cisn/2 = 0 + i
2
z 5
= cis3n/2 = 0- i
z = cis 5n/6 = - /3/2 + i/2 z = cis llTT/2 = .fJ/2 - i/2
3 6
We have z = z l , hence Z l + z 6 = Z l + Zl = V3 , Z l z 6 = x
6
and z5 = z2' z2 + z5 = z2 + z2 = 0 z2z5 = 1
and z4 = z3 ' z3 + z4 = z3 + z3 = - VJ' z3z4 = 1

Fig.l6
144

6
The factors of z + 1 are:
z 6 + 1 = [(z- z ) (z- z )) [(z- z ) (z- z )] [(z- z ) (z- z )]
1 6 2 5 3 4
2 2 2
= [z - (z + z )z + z z ] [z - (z + z )z + z z ] [z - (z + z )z + z z ]
1 6 1 6 2 5 2 5 3 4 3 4
2 2
= (z - .f3 z + 1) (z + 1) <i + 13 z + 1)
2 2 2
= (z + 1) (z - {3 z + 1) (z + .f3 z + 1)
We could have easily arrived at this result by writing:
2 4 2 2 2 2 2 2
z 6 +1 = (z +1Hz -z +1) = (z +l)[(i+0 -3z ] = (z +l)(z -13z+l)(z 2 +13z+l).
So why waste time? This is not so, as the algebraic identities that we have just
established can be used to derive numerous trigonometrical relations. See the next
example.
5
EXAMPLE: (38) Factorise z + 1 into real linear and quadratic factors and hence
deduce that 4sin to cos J = 1
5 71 ; 2k 71 , k
SOLUTION: The roots of z = -1 are given by z = cis = 0, 1, 21 3, 4.
The five fifth roots are:
z
1
= cos71/5 + isin71/5 z
4
= cos771/5 + isin771/5 = cos(371/5)- isin(371/5) z
2
z
2
= cos371/5 + isin371/5 ; z 5 = cos971/5 + isin971/5 = cos71/5- isin71/5 z1
z3 =- 1 zl + z5 = zl
+ zl = 2cos71/5' zl z5 = zl zl
z + z + z = 2cos371/5 ,
= z z z =z z hence:
2 4 2 2 2 4 2 2
z5 + (z - z ) (z - z ) (z - z ) (z - z ) (z - z )
3 5 4
1 2
2 2
(z + 1) [z - (zl + zl) z + zlzl] [z - (z2 + z2) z + z2z2]
5 2 2
z + 1 = (z + 1) [z - 2cos(71/5)z + 1] [z - 2cos(371/5) z + 1] ••• (1)
5 4 3 2
Now z + 1 = (z + 1) (z - z + z - z + 1) ••• (2) 1
2 2 4 3 2
hence: (z - 2cos(n/5)z + 1] [z - 2cos071/5)z + 1] = z - z + z - z + 1 ••• (3)
2
Compare the coefficient of z on both sides of identity (3), then
4cos71/5 cos371/5 + 1 + 1 = 1
4cos(71/5) cos(3n/5) = -1 1 now cos 371/5 = sin [; - ~n:l = -sin fa, hence:
4sin(71/10) cos(71/5) = 1

SOLUTION TO THE EQUATIONS


(1) zn-l + zn- 2 + •.• + I = 0 and (2) zn-l - zn- 2 + zn-J- ... + ••• - z + I " 0

We can change these into the form zn + I = 0, as follows


n-1 n-2 n-3 zn - I n-1 n-2 n-3
z +z +z + ••• + z + I = ----z-:T and z - z + z - •. . - z +

We solve the equations zn - I = 0 and zn + I = 0, ar.d remove


the real root I from the roots of zn - I = 0 , and
the real root -1 from the roots of zn + I = 0 , (n is odd)
to obtain the roots of equations (l) and (2) respectively.
145

EXAMPLE: (39)
4 3 2 (b) z
4 -z 3 +Z 2 -z+1=0
(a) Solve z +z + z + z + l
SOLUTION:
(a) We change z + z + z
4 3 2+z + l =0 (b) We change z - z + z - z + 1 = 0
4 3 2

z5 - l z5 + 1
to --z:-r ::
0, z -/. l to """"Z+T = 0, z -/. -1
5
Hence z - 1 = 0 whose roots The roots of the given equation are
are given by: given by:
z = cis(2k11/5), k = 0, 1, 2, 3, 4, 5.
11 2 11
z = cis [ \ k J
We omit the root z = 1, then the where k = 0, 1, 3, 4, sihce we must
4 3 2 remove the root z = -1 given by k = 2
roots of z + z + z + z + 1 = 0
are given by:
z = cis(2k1T/5), k = 1, 2, 3, 4.

Exercise 4J
5 Hence prove that
1. Resolve z - 1 into the real linear and quadratic factors.
211 411 1
cos 5 + cos5 -2.
5 Hence prove that
2. Resolve z + 1 into the real linear and quadratic factors.
11 3lT 1
cos 5 +cos 5 = 2.

3. Find the roots of z 4 + 1 = 0 and show them in an Argand diagram. Resolve


z 4 + 1 into real real quadratic factors and deduce that:
cos29 = 2(cos9 - cosn/4) (cos9 - cos3n/4)
6
4. Find the roots of z 6 + l = 0, and hence resolve z + 1 into real quadratic
factors; deduce that cos39 = 4(cos9 - cosn/6) (cos9 - cos11/2) (cos9 - cos5n/6)
4
5. Show that the roots of (z - 1) 4 + (z + 1) = 0 are.:!:. icot(n/8) and .:!:.icotOn/8)
6 6 are
6. Show that the roots of (a) (z - 1) + (z + 1) = 0
.:!:. i cotn/ 12, .:!:. i cot 5n/12, .:!:. i
7. Solve the following equations:
3 2 5 4 3 2
(a) z + z + z + 1 =0 (b) z + z + z + z + z + 1 = 0
4 2 4 2
(c) z - z + 1 = 0 (d) z + z + 1 = 0
2 4 2
2 4 2
[Hint: z 6 +1 = (z +1) (z -z +1)] [Hint: z 6 - 1 = (z - 1) (z + z + 1)]
146

4.12 Geometric Representation of Complex Numbers

We have remarked that the complex number z = x + iy may be considered an ordered


pair (x, y) of real numbers and further that (x, y) can be regarded as the coordinates
of a point in a coordinate plane. Conversely a point in the coordinate plane
corresponds to a complex number x + iy. y
We call such a plane an Argand diagram
or the complex plane. 2
Three complex numbers
z =3+2i, z =-2+1i and z =0-2i -2 0 3 X
1 2 3 -1
are represented in the plane by points
P(3,2), Q(-2,1) and R(0,-2) respectively. -2
Fig.l7
Any relation between points in a plane can be regarded as a relation between
corresponding complex numbers and vice versa. For example, the locus of a set of
. . f ymg
. 2 2 2
pomts sat1s t he equation
. x + y = a (a c1rc
. Ie, ra d"1us a, centre at t he ongm
. . )
2
can be stated as either zz = a or Iz I = a where z = x + iy. Many more locus
problems can be solved by the use of complex variable z = x + iy. Here we are
killing two birds with one stone! Instead of two variables x and y we work with
only one variable z. Not only that, but by equating real and imaginary parts of the
equation, we obtain a complete set of solutions.

Addition:
Let the complex numbers z =x+ iy and
1 1 1
z = x + iy be represented by the points A
2 2 2
and B respectively on the Argand plane.
Complete the parallelogram OACB. Then the
mid-points of AB and OC are the same. But
rx
n-=- ,
I + X? y1 + y 21
the mid-point of AB is P 2 j 0 F--------.._X
so the coordinates of C are (x·1 + x 2, Y + Y2>· Fig.I8
1
Thus the point C corr.esponds to the sum of the complex numbers z and z ,
i.e. z + z = (x + x ) + i(y + y ). 1 2
1 2 1 2 1 2
Subtraction:
We first represent -z by D, so that AD is
1
bisected at 0. Complete the parallelogram
OBCD, then the point C represents the A(z I)
complex number z + (-z ) i.e. z - z
2 1 2 1 X
Since OA = OD = BC and OA II BC,
OABC is a parallelogram, and hence OC is D(-z 1)
parallel to AB. The coordinates of
Fig.I9
C are (x - x , y - y ) •
2 1 2 1
We also note that the length of the segment AB is given by
AB = lz 2 -z 1 1 = /<x 2 - x / +(y - y /
2
It is advisable to draw a diagram and then calculate arg (z - z )
2 1
147
y

EXAMPLE: (40) c I
6
The complex numbers z
1
= 4 + 2i and z2
5
/ /}
z
2
= 1 + 4i are represented by points A
and B respectively in the plane. 4
B/
/
I
zl + z2. = 4 + 2i + 1 + 4i
The end-point c
= 5 + 6i
of the diagonal OC of 3
I
the parallelogram OACB represents
the sum z = zl + z2 = 5 + 6i
2
I ,/
A
//
c is the point (5,6)
II/ /
~ X
O I 2 3 4 5 Fi g.20

y B(z
EXAMPLE: (41)
3
A(z ) represents z = 2 + i
1 1
2
I
B(z ) represents z = 1 + 3i
2 2
D(-z ) represents -z
2 2
-1 - 3i = A(z

z = z - z = (2 - 1) + (1 - 3) i = - 2i
1 2
I I~ /;
Let C(i,-2) represent z = 1 - 2i -I I 2 3 X

From the scaled diagram we lind that c


is the end-point of the diagonal of
-I
I
parallelogram OACD where
by -z2
D is given
v
D( z )3
2
/ C(z - z2

Fi g.21

y
EXAMPLE: (42)
z I = -2 and z
2
= 3 + 2i
Find the following:

~l~
(a) z = z - z (b) mod(z -z ) (c) arg(z -z )
2 1 2 1 2 1
SOLUTION:

A~
Let A (-2,0) and B(3,2) represent the
X
complex numbers zl and z2
-2 0 I 2 3 4 5
respectively; then:
Fig. 22
(a) z = z2 - z
1
= 3 + 2 i - (-2) =5 + 2i
C (5, 2) represents the difference z
2
- z
1
where OC II and = to AB
= ~ = ./29
~2
(b) mod(z - z ) lz - z 1
2 1 2 1
2
(c) arg (z 2 - z )
1
9 = LCOX = LBAX where tan 9 ) Fig. 23
148

EXAMPLE: (43)

Prove that the diagonals of a rhombus intersect at right angles.

SOLUTION:

For simplicity we represent the vertices of


the rhombus by the. complex numbers 0, z , z + z
1 1 2
D(z -z )
and z 2 where lz 1 1 = lz 2 1 1- 0 2 1
C represents z + z and D represents z - z
1 2 2 1
' ' ',
such that OD is II and = to AB. Our ''
geometrical proposition is equivalent to saying that ''
z2 - z 1 71 • z2 - z 1 ''
arg - - - = ~ - , 1.e. w = - - - is '
z +z 2 z +z 0 A(z )
2 1 2 1 1
purely imaginary. This would be so if w + w 0. Fig. 24
z2 - zl
z2 - zl 2(z2z2- zl zl)
We find that w + w = --- + ---
(z2 + z 1) (z2 + z 1)

TRIANGULAR INEQUALITY
y
We prove the inequality
lz 1 + z 2 1 ~ lz 1 1 + l.z 2 1

Proof: In 60AC, using the fact that


the sum of two sides of a triangle ).
the third side, we have:
OC ~ OA + AC 0 X

lz + z 2 1 ~ lz 11 + lz 2 1, Fig. 25
1
because AC = OB = I z 1 and OA
2 C(z +z )
The only time inequality holds is when 1 2
argz = argz = 9 (Fig. 26)
1 2 (z2)

A(z )
1
9
0 X

Fig. 26
149

Exercise 4K

Perform the indicated operations graphically. Use graph paper.

I. (2 + 3 i) + (2 + 2 i) 2. (3 - i) + (5 + 2i)
3. (3) + (-2 - 3i) 4. (-1 + 3i) + (2i)
5. (3- 2i) - O+i) 6. (6 + 4 i) - (3 - 2 i)
7. (-1} - (-3 + 5i) 8. (2i) - (-3 + i)
9. Given that z = x + iy, ZJ = X] + iy 1> P i~ the point P(x,y) and A is the point
(x 1 ,y 1), draw the sketch of the directed line segment AP in the Argand
diagram, if z - z 1 is equal to:
(a) z- 3 (b) z + 3 (c) z + 2i (d) z- I + 2i (e) z + 2 + 3i

10. Find the modulus and argument of each of the complex numbers z and w,
1+ i .n
z = y-:-r and w = T-=-1

Plot the points representing z, w and z + w on an Argand diagram. Deduce


from the diagram that tan ~TT = .f2 + I •

11. The points P and Q are represented by the complex numbers z = 1 - 3i and
w = -3 + 4i respectively. Find a point R on the real axis such that PRQ is
a right-angled triangle.

12. The points A, B, C and D in the Argand diagram represent the numbers
1 + 2i, 3, 5 + 2i, 3 + 4i respectively. Prove that ABCD is a square. Find
the complex number representing the intersection of the diagonals.

13. If Izl = lw I,
prove that ~ is purely imaginary. By drawing a suitable
z- w
diagram, give a geometrical interpretation of the result.

14. Prove that for any two complex numbers z and z


1 2
lz +z 1 ~ lz 1 lz 1, assuming lz 1 > lz 2 1. When does the equality
1 2 1 2 1
sign hold?
2 2 2 2
15. Provethat lz -z 2 1 + lz 1 +z 2 1 = 2lz 11 + 2lz 2 1
1
Give a geometrical interpretation of the result.
150

4.13 Product and Quotient: Rotation


p
We now give the graphical representation
of multiplication and division of complex
numbers.
Let B and C be the points representing c
the complex numbers. Let A be the point (1, 0)
z = r cisS and z = r cisS respectively.
1 1 1 2 2 2
Construct [PO C equal to 8 l •
Construct LO CP equal to LOAB
Triangles OAB and OCP are similar, X
hence their sides are proportional. Fig.27
OP OC
OB = OA and this gives OP = r 1 r 2 . Hence point P represents the

complex number r r cis(S +


1 2 1
e2) which is the product of r cisS and r cisS •
1 1 2 2
Although similar graphical representation can be given to division, it is not very
elegant and of no practical use. Let us now turn our attention to a very important
application of i as an operator that rotates a directed line segment.

(a) ROTATION OF A POINT A(z) THROUGH +90°


In the product z z
1 2
= r 1 r 2 [cos(S 1 + e2)+ isin(S 1 + e2)}, let z
1
= i, z
2
= z, then

e1 =; and we write e2 = 8, then iz = r[cos(S + ~) + isin(B + ; ) ]

The above results mean that when we B(iz)


multiply z by i, we rotate the point
A(z) through a positive angle of 90° about 0.
If z = x + iy, after rotation through 90° ,
we have iz = i(x + i y) = -y + ix A(z)

Note that lzl = lizl = r X


Similar interpretations are given to the C(-z)
powers of i as follows: Fig.28
2
i corresponds to 180° counterclockwise rotation
3
i corresponds to 270° counterclockwise rotation, or to a 90° clockwise rotation.
151

EXAMPLE: (44)

P is the point P(3, 2) in the Argand diagram representing the complex number
z = 3 + 2i. OP is rotated about the origin
through + 90°. Find the complex number y
represented by the new position of P in
the plane.
A 3
SOLUTION:
z = 3 + 2i \\ 2 p
iz = i(3 + 2i) = -2 + 3i
:. New position of P is A(-2,3) [\ I /
/
v
A represents the complex number
z1 = -2 + 3 i , lz 11 = IzI = v'iJ -2 -I
\ 0
/
2 3 X
Fi11;.29

EXAMPLE: (45)
OABC is a square in an Argand diagram. The point A represents the complex
number z = V'J + i. Find the complex numbers represented by B and C in the
form a + ib.
SOLUTION:

A represents z = o/'J + i, C represents iz


and B represents z + iz.
Hence C represents i ( v'3 + i), i.e. - I + V3 i, I
A( v'J,J)
I
B represents V3 + i + i ( .r5 + i), i.e.
0 X
o/'j - I + ( v'J + I) i.
Fig.30

EXAMPLE: (46)
P is the point (2, 3) in the Argand diagram representing the complex number
z = 2 + 3i. The segment OP is rotated through + 60°, and P now occupies the
position of the point Q. Find the coordinates of Q.
SOLUTION:
We use z z = r r [cos(9 + 9 ) + isin(9 + 9 )]
1 2 1 2 1 2 1 2
Let z = 2 + 3i and z = cos60° + isin60°, then:
1 2
multiplying 2 + 3i by cos60° + isin60° is equivalent to rotating the length OP
through + 60°.
Let Q be the point (x, y), then
x + iy (2 + 3i) (cos60° + isin60°)

= 3
l - - {3
2
- + ·(32 ./::;)
1 + v3

Q is the point ~I~ _ 3 f3 3


, 2
2
152

Exercise 4L

1. P is the point in the Argand diagram and represents the complex number
I + 2 i. The segment OP is rotated through (a) + 30° (b) + 45° (c) + 90°.
Find the complex number th..tt represents the point occupied by P after
each ro~ation is completed.

2. The point A in an Argand diagram represents the complex number + i. Find


the complex number represented by B if OBA is an equilateral triangle.

3. OPQR is a square in an Argand diagram, where 0 is the origin. The point P


represents the complex number z given by z = r (cos 30° + i sin 30°). Find the
complex numbers represented by Q and R.

4. OABC is a rhombus in an Argand diagram, where 0 is the origin. The point


A is (1,2). If [BOA =- 30°, and B is in the ~econd quadrdnt, find the complex
numbers representing the points B and C.

5. PQRS is a square in an Argand diagram, where P, Q, R and S represent the


numbers 2 + i, 3 + 2 i, 2 + 3i, and + 2i respectively. This square is
rotated anticlockwise about 0 through + 90°. Find the complex numbers
!h..tt correspond to the new positions occupied by the points P, Q, R and S
respectively.
153

4.14 Locus Problems with the Complex Variable z

From our study of the co-ordinate geometry, we know that y = f(x) or f(x,y) = c
represents a locus of a point P(x,y), satisfying a certain condition. Since z = x + iy,
as x and y vary, z varies and hence z may describe a curve in an Argand plane.
2 2
(a) CIRCLE x + y = r2 y
2 2
Let P(z = x + iy) be on the circle x +y = r 2•
Now Iz I = Vx 2 + / = r
Thus Iz I
= r represent~ a circlt• of radius r and X
the centre 0(0,0). Wf:' can also write
zz = r2 as the
2 2 2
equation of the circle x + y = r
2 2
(b) CIRCLE (x- a) + (y- b) = r2 Fig.31
y
If P(z) is a typical point on the circle,
centre C (a + ib), radius r, then PC = r
i.e. lz- wl= r, where w =a+ ib
Thus Iz - wI = r, represents a circle with
the centre at C (w = a + ib) and radius r.
2
Using the result zz = I zl , we can write
2
this equation as: (z - w) (z - w) = r
- - wz
i.e. zz - = r 2 (Th ere IS
- - -wz + ww .
some advantage in writing the equation 0 X
of a circle in this manner). Fig.32
EXAMPLES: (47)
Write the equations of the following circle in the form Izl =r or Iz - wI = r:
2 2 2 2 2
(a) x + y = 4 (b) (x - 1) + (y - 2) =4 (c) x + / - 4x - 6y = 0
SOLUTION:
2
(a) x
2 + y2 = 4 (b) (x - I ) 2 + (y - 2) = 4, centre is (I ,2)
lzl = 2 w = I + 2i
We write lz -wl = 2
or Iz - I - 2i I = 2
(c)
2 2
x + y - 4x - 6y = 0
2 2
(x - 2) + (y - 3) = 13 , centre is C(2, 3), radius = liJ
We write Iz - (2 + 3i)l = Jl3 as the equation of this circle.
154

Exercise 4M

I. Express the equations of the circles in the form Izl = r:


2 2 2 2 2 21 2 2
(a) x + y = 1 (b) x +y = 9 (c) x + y = (d) 2x + 2y
4

2. Express the following equations in the form - = r 2:


zz
2 2 2 2 2
(a) x + / = 25 (b) x + / = 64 (c) ax + a/ = (d) 3x + 3y = 5

3. Express the equations in cartesian form, stating the radiu<; und the ct•ntrt• of each
circle.
(a) lzl = 4 (b) l2zl = 1 (c) zz = 25 (d) 4zz = 1

4. Express the equations in the cartesian form, describing each locus.


<a> Iz - 21 = 1 <b> Iz + 21 = 3 <c> Iz - 3 i1 = 2 <d> Iz + 2 i1 = 5
(e) l2zl=lz-ll (f) lz-(2+i)l=2

5. Express the following equations of circles in the form Iz - wl = r

(a)
2 2 (b) 2 2
x + y - 2x - 2y x + y + 2x + 4y + 1 = 0

(c) 2 2 (d) 2 2
X + y + X + 3y = 0 2x + 2y + 4x + 3y + 1 = 0

6. Show that the following equations represent circles in an Argand diagram. State
their centres and radii.

(a) (z - 2) (z - 2) = 4 (b) (z - w) (z - w) = 1' where w = 1 + i


(c) (z - 2 + 3 i) (z - 2 - 3 i) = 4 (d) (z - 1 - i) (z - 1 + i) = 9
155

4.15 Miscellaneous Locus Problems (Including lnequations)


EXAMPLE: (48)
Draw a neat sketch of points satisfying the following conditions:
(a) Re(z) = 3 (b) Im(z) > 2 y
SOLUTION: (a)
(a) Re(z) = 3, z = x + iy
Re(x + iy) = 3
X = 3
2 3 X
The locus is the vertical line x = 3
(b) Im (z) > 2 , z = x + iy
y >2 WJJ!J/111/IIJ/1_ ____. Fig.33
The locus is a half-plane above the line
y =2
0 X

EXAMPLE: (49) y
Sketch the curve lzl 2

SOLUTION:
Izl is the distance of a point from
the origin, so the locus of I z I = 2
is a circle of radius 2, centre 0(0,0). X

The cartesian equation of this circle is


2 2
X + y = 4

Fig.34

EXAMPLE: (50)

Describe the locus Iz - 2 + 3i I 3 X

SOLUTION:

Write: lz- (2- 3i)l = 3 ... (I)

We know that the equation of the form


Iz - w I = r, represents a circle.
Hence (I) represents a circle of radius 3 and
centre (2,-3) whose cartesian equation is
2 2
(x - 2) + (y + 3) = 9 Fig.35

EXAMPLE: (51)
X
Sketch the region defined by
Iz - 2 + 3 i I ~ 3.
SOLUTION:
From example (50) this region is the set of
points within and on the boundary of the
circle, radius 3, centre (2,-3)

Fig.36
156

EXAMPLE: (52)
P(z)
Describe the loci of z if
<a> Iz - 2 I Iz + 1- i I
<b> Iz - 2 I < lz + 1- i I
I
SOLUTION: I
B I
(a) P(z) where z =x + iy I
1A
We have:
2 X
Iz - 21 = Iz - <-1 + nI ... o>
Let A be (2, 0) and
Fig.37
B be(-1,1)

The relation (1) says that PA = PB for all positions of P and from plane geometry
we know that the locus of P is then the perpendicular bisector of AB.

y
(b) lz-21 < lz-(-l+i)l
represents a set of points closer
to A(2,0) than B(-1,1).
Hence the locus of P(z) is the
region on the right hand side of
the perpendicular bisector of AB,
excluding the bisector itself. A X

Fig.38

EXAMPLE: (53) (may be deferred until


after Chapter 6) y

Describe the locus


lz-21 + lz+21 = 6

SOLUTION:

Here A(2,0), B(-2,0) X


we have PA + PB = 6 (given).
This is the condition for the locus of
z to be an ellipse.

The locus of P is an ellipse.


The foci are (.:!: 2, 0).
The centre is o. Fig.39
Length of semi-major axis is 3
Length of semi-minor axis is .f5
2 2
Equation of the ellipse is x
9
+ ~
157

EXAMPLE: (54)
Sketch the region defined by: (a) 0 ~ argz ~} (b) lzl ~ 2 and 0 ~ argz ~%
SOLUTION:
y
(a) argz = j = 60°

Locus of P(z) satisfying


0 .{. drgz ~
n/3 is the set of points Fig. 40
within the angular region
[POX = 60° (shaded) including
the boundaries, excluding the origin.
argz is not defined for z =0

(b) Izl ~ 2 and 0 ~ argz ~ n/4


The region shaded is within the circle
of radius 2, centre 0, restricted in Fig. 41
the sector 0 ~ arg z ~ 45°,
excluding the origin 0.

EXAMPLE: (55)

Find the locus of w if w - z z- 1 given Izl 2

SOLUTION:

We eliminate z from w z --I


=- .
by usmg Iz I = 2
z
I
Solve for z, then zw =z - I su z(w - I) =- I and z = r-:-w
lz I but II - wI = Iw - I I and lzl = 2 , then
II - wl '
lw - II = 4
Hence the locus of w is a circle of radius
2 2 I
4and centre (1,0). The cartesian equation
of the circle is (x - I) + Y = 4

EXAMPLE: (56)
z- 2
Find the locus of z if w = - - , given that w is purely imaginary.
z
SOLUTION:
z- 2
w -z- --z2 Z = X + iy

w
2z 1
_ 2 (x - iy)
zz X
2 + y
2

2x 2iy
w 2 2 + 2 2
X +y X +y
Now if w is purely imaginary, then Re(w) = 0
I -
2
2x
2
= 0
X + y
2 2
x + y - 2x = 0
(x - 1)2 + y2 = Ior lz-11=1
Hence locus of z is a circle of radius I, centre (1,0).
158

EXAMPLE: (57)

Describe the locus given by the equation Iz + 21 2lz-2+il

SOLUTION:

Let z = x + i y, then the given equation becomes:


lx+iy+21 2lx+iy-2+il
I<x + 2) + i y I = 21 <x - 2) + Hy + o I
2 ~,_(x---2)-=-2 -+_(_y_+_0~
2
or
squaring both sides and simplifying:
2 2
3x + 3y - 20x + 8y + 16 o. Divide by 3
2 2 20 8 16
X +y -JX+3Y+) 0. Complete the squares.

(x _ I~) 2 + ( y + jy 1~0 + 1; _ 1; = 6:
1
So the locus is a circle, centre ( ~ , - ~ )• radius

EXAMPLE: (58)

Sketch on an Argand diagram the locus given by the equation:


arg (z - 2) - arg (z + 2) = j
SOLUTION:

arg(z - 2) LPBx a' B (2, 0)


arg (z + 2) LPAx1$ ' A (-2, 0)
1T 1T
Now e - 0 =
3 '
so LAPB =
3
e
X
The locus of z, as represented by the point P, 0
is thus the major arc of the circle with AB as Fig. 42
a chord. This chord AB subtends an angle of j (Points A and B are excluded
at the circumference. By symmetry, we find the from the locus)
centre on the y-axis. Now angle LACB = 120°.

cos30° =~ gives r = 2sec30° 2 · J = ~ c


J
CO = 2tan30° =
Hence the centre is (0, *) and radius is

A
The cartesian equation of the circle is
x2 + ( y _ J) 2 = 1; Fig. 43
159

Exercise 4N

1. Find the Cartesian equation of the following curves, and sketch and describe
them (Exercises I, m and n may be postponed until after Chapter 6):
<a> Iz I = 2 <b> Iz - 21 = 3 <c> Iz + 21 = 3
<d> Iz - i I = 2 <e> Iz + 2i I = 3 <n Iz + 2 - 3i I 2
<g> Iz I = Iz - 1 I <h> Iz - 21 = Iz + i I
(i) lz+2-3il = lz+2+il (j) lz-2il 2lz+ll
(k) Iz + 2 - 3i I = 21 z + 2 + i I (I)* Iz - II .+ Iz + II = 4
(m)* Iz - i I + Iz + iI = 4 (n)* Iz I + I z - II = 4
2. Sketch the following curves after giving their Cartesian equations. Describe the
curves.
(a) arg z n/3 (b) arg z -n/4 (c) arg (z + 2) = 5n/6
(d) arg z n/2 (e) arg z TT (f) arg (z + 2i) = n/3

3. Sketch the following regions in the complex plane, showing whether the
boundaries are included or not.
<a> Iz I < 2 <b> lz I ~ 3 <c> Iz I > 2
- i
<d> Iz - 2 I < 3 <e> Iz + 2 + 3i I < 2 (f) 2 < Iz - II ~ 3
<g> 2 ~ Iz + 2 - i I ~ 4 <h> arg z ~ n/3 (i) n/3 ~ argz ~ n/2
(j) - n/2 < arg z < n/6 (k) arg(z + 2) ~ n/6 o> Iz - 3i I > Iz + 21

4. Sketch the following regions of the complex plane, showing carefully the
boundaries (included or not).
(a) Iz I~ 3 and 0 ~ arg z ~ n/3 (b) Iz I~ 2 and n/6 ~ arg z ~ 2n/3
(c) 2~ Iz I ~ 4 and Re (z) >.- I (d) 2~ Iz I ~ 3 and lm (z) >.,. 1
(e) I~ lzl ~ 3 and 0~ argz~ n/2

5. For the following, describe the locus of the complex number w, where z is
restricted as indicated.
z- 2
(a) w z- 2' lz I = 3 (b) w
z lzl --
z - 2i (d) z - 2 + i
(c) w
r:z• lzl = 2 w
z + 2 - i '
lzl

6. Find the locus of z if:


z - I
(a) w - z
- and w is purely real

(b) z - i
w and w is purely imaginary
z - 2
z - 2 TT
(c) w
z + 2
and arg w =3
160

Exercise 40 <REVISION)

1. For the complex number z = x + iy, find the locus of z. Describe it and draw
a neat sketch, if:
(a) arg (z - 2) = n/3 (b) lz I= z + z+2 (c) zz- 4 (z + z) = 10
(d) arg [~ = i]= n /2 (e) lz+1l+lz-ll=3

2. The points z , z and z are three complex numbers that lie on a circle
1 2 3
passing through the origin. Prove that the points which represent 1/z , l/z
1 2
and l/z are collinear.
3

3. (a) Express ~ in the form a+ ib


(b) Hence or otherwise, find the locus of the point which represents z on the
2
Argand diagram, if. lz - 5 + 12i I = lz- 3 + 2i I

4. (a) If x and y are real, solve the equation -...!.L


i . 3y + 4i
0
1 + IX 3x + y
(b) Show that the locus of the point z = x + i y is

(i) a straight line if ~ : ~ is purely real

(ii) a circle if ~ : ~ is purely imaginary.


Find the centre and radius of this circle.

5. Given z = 3 + 4i and z = -3- 4i


1 2
(a) Draw a neat sketch of the locus specified by I z - z 1 = I z - z 1•
1 2
Find the Cartesian equation of the locus of z.
(b) Show that the locus of a point represented by the complex number
z = x + i y and obeying the condition I z - z 1 = 31 z - z 1 is a circle.
1 2
Find its centre and radius.

6. P is a variable point on the line x = 4 and


OPQR is a rectangle in which the length of
OP is twice the length of OR. Find the
locus of S in Cartesian form, where S is the
point of intersection of the diagonals.

7. Given that (1 + 4i) p - 3q = 3 + 5i, find if:


(a) p and q if p and q are real
(b) p and q if p and q are complex conjugates.

8. Draw neat sketches of the loci represented by the equations I z - 21 = 2 and


I z - 2 i I = 2. Find the complex numbers for the points represented by the
intersections of these loci.

9. The point A in an Argand diagram represents the complex number 3 + 4i.


Find the complex number represented by B if OBA is an equilateral triangle.
Also find the point D if OABD is a rhombus. (All rotations are
anti-clockwise.)
161

10. (a) The complex number z is given by z 1 + i


= 1 + r:T
Find (i) Re(z) (ii) Im(z) (iii) Iz I (iv) arg z
(b) Draw a neat labelled sketch to indicate the locus of the complex number z.
(i) Iz - 3 + i I = 2 (ii) Re (z - i z) ~ 1
(c) Determine the locus of the complex number z if
arg (z - 1) = i + arg (z + 4). Sketch the locus on an Argand diagram.

(d) Shade on the Argand diagram the region for which


lzl ~ 2
2
n < arg z < n and
3
For part (d), choose a point in the region (above) and label it P. Suppose
P represents the complex number z. Then label clearly the points A, B,
2
C, D and E which represent z,
-z, iz, z and z + 2.

11. (a) OPQR is a square in an Argand diagram where 0 is the origin. The point
P is given by z = r(cos9 + isin9). Find the complex numbers represented,
by Q and R.
(b) If 9 = 30°, r = 1 in part (a) and the square OPQR is now rotated through
60° (anti-clockwise direction) to become OP'Q'R', find the complex
numbers represented by P', Q' and R' in the mod-arg form.

12. (a) Calculate the modulus and argument of the


(i) sum of the roots
(ii) product of the roots
of the equation (4- 3i)z - (3- 2i)z ~ 5 + 2i = 0
2

(b) Show that the point representing sinn/3 + icosn/3 in the Argand diagram
lies on the circle of radius I and the centre at ( yT, 0).

(c) . 2 + it
If t IS _ it show that as t
real and z = varies 'the locus of z is a
2
circle. Find the radius and the centre.

13. (a) Draw the sketch (on an Argand diagram) of the region in which z lies if
both lz- (2 + i) I ~ 4 and n/6 ~ arg(z- 1 - i) ~ n/2 are satisfied.

(b) The complex numbers = - 13- i, z = fi- i and z = 2i are repre-


z
1 2 3
sented by the points P, Q and R respectively. Show that the triangle
PQR is an equilateral triangle.

14. (a) Complex numbers z = ~ and z =1 : i where p and r are


1 2
real. Find p and r, if z - z = 4i.
1 2

(b) The complex number z satisfies the equation arg (z + 2) = n /4.


(i) Sketch the locus of point P which represents z.
(ii) Find the modulus and argument of z when Izl is the minimum.
(iii) Hence find z in the form x + i y when Iz I is the minimum.
162

15. (a) Calculate the modulus and argument of the complex number which
represents the product of the roots of the equation
2
(4 + 3i)z - (1 - i)z + (2 + i) 0.

(b) Find the two square roots of 3 + 4i and sketch them in an Argand
diagram.

16. (a) Find all six roots of -1, expressing each in the form x + iy and show
them on a circle in an Argand diagram.
2
(b) Solve z + 16 = -30i completely.

17. The point P (which represents z =x + iy)


moves in a straight line parallel to
2
the imaginary axis. Prove that the point Q which represents z moves in a
certain parabola. Find the focus. Also describe the locus of Q when P moves
on the imaginary axis.
zl + z2
18. (a) Prove that
zl - z2
is purely imaginary if Iz 1
1
= Iz
2
1

(b) Show that the points represented by the complex numbers z , z and
1 1
z - 2iz
1 2
_ i form the vertices of a right-angled triangle.
1 2
z 1 - z3] n z - 2iz
(Hint: arg z _ zJ
[ 2
= 2 , where z 3 = 11 _ 2 i 2 )
2
19. Let z = x + iy and w = u + iv = (z - 0 + 2 be complex numbers in an
Argand diagram. Show that as z moves along the y-axis from 0 (0, 0) to
A(0,2) the point w moves along an arc of a certain parabola. Find the
corresponding points on this parabola and the Cartesian equation of the parabola.

20. (a) For the complex number z = r(cosG + isinG), given that r = 5, G= j,
2
find the modulus and argument of (i) z (ii) z (iii) 1/z (iv) i z

(b) If z = 4 + Ji and lz 1 = 10, find the


1 2
(i) maximum value (ii) minimum value of Iz 1 +
2
z I
(iii) express the corresponding values of z in the form x + iy
2

21. (a) Find the modulus and argument of z = 131 ++ !I • Find the smallest
positive integer, n, such that zn is real; hence find zn for this value of n.

(b) 2
Sketch the circle whose Cartesian equation is (x - 2) + / = 4
The point A reP.resents the complex number z = r(cosG + isinG).
(i) Express lzl in terms of G if A lies on the given circle.
(ii) Find I~ I and arg { ~) if A lies on the circle.

(iii) Show that as A moves on the circle, the point P representing 1/z
lies on a certain line.
163

22. The complex numbers z = x +· i y and w = u + i v are related so that


w = z - .!z . Show that:

X
u = x - 2 2
and v = y + --::;:-'--y-:;
2 2
X + y X + y
Find the ~ocus of w if (a) lz I = 2 (b) lzl = 1.
Describe each locus geometrically.

23. Find r and 9 if 3cis45° + 2cis30°


r (cosa + isin9)
5cis 60° - 4cis 30°
3
z +I 2
iz
24. Solve the equations: (a) - - -
3
=i (b) z =
z - l

25. The four complex numbers u, v, w and z are represented on the complex plane
by points P, Q, R and S respectively. If u + w = v + z and u - w = i (v - z),
determine the possible shapes of the quadrilateral PQRS.
9
26. (a)
z - l 6 3
Show that - - - = z + z + I
3
z - l
(b) Prove that:
z 6 + z 3 + I · = (z 2 - 2zcos 2n + I) (z 2 - 2zcos 4n + I) (z 2 - 2zcos 8n + 1)
9 9 9
(c) Deduce from (b) that:
2n 4n 8n
2cos39 + 1 = 8(cos9- cos
9 ) (cosa- cos 9 > (cosa- cos 9 >
27. (a) Factorise
3
z + I into linear ~nd quadratic factors with real coefficients.
5 2 n 2 3n
(b) Prove that z + l = (z + I) (z - 2zcos + I) (z - 2zcos
. 15 3 5
5 5 + l)
(c) By observmg that z + 1 = (z ) + l, prove that
15 2 6 3 n 6 3 3n
z + 1 = (z + 1) (z - z + I) (z - 2z cos + 1) (z - 2z cos 5 + l)
5
28. Show that if lw I =1 and Re(w) =- ~ , then w
3
=1
1 + sinS + icosa . .
29. Prove that 1 +Sin
. a -lOS
c . 9 = sma + 1COS9 and deduce that

. n . n]5 + 1·[ 1 + Sin


. 5n - !COS
. n]5 =
[ 1 + Sin 5 + !COS 5 5 0

30. Express . 4k + 1
[ 1 + 1tan ~n
J m m
. t he f orm a+ 1. b , where m and k are
integers.
164

CHAPTER 5 POLYNOMIALS

5.1 Introduction

In 3U Mathematics, we learned many important properties of polynomials in the


real variable x. In this chapter we shall study these properties and a few more
theorems about polynomials over the complex field C. Let us first revise some of the
important work on polynomials.

The general nth degree polynomial function cf a real variable x is of the form:
n n-1 n-2
P ( x) " a x + a x + a x + •• • + a x + a , a .J 0
n n- 1 n- 2 1 0 n
where the coefficients a , a , ••• are real, and n is a non-negative integer.
0 1

OPERATIONS ON POLYNOMIALS

When two or more polynomials are added, subtracted or multiplied, the result is
another polynomial. The division of one polynomial P(x) by another polynomial A(x)
may or may not be exact. When P(x) is div1ded by A(x), we can write the identity:
P(x) " Q(x) . A(x) + R(x)
where, P(x) is called the dividend,
Q(x) is called the quotient,
A(x) is called the divisor,
R (x) is called the remainder. .. [deg R (x) < deg A (x)]

If R (x) " 0, P(x) is exactly divisible by A (x), then Q(x) and A (x) are called the
factors of P(x).

THE REMAINDER AND FACTOR THEOREMS

In the division of P(x) by A(x), if A(x) is a linear polynomial x - a, tht·n the


degree of R (x) must be zero, i.e. R (x) is a constant. We write:

P(x) " (x - a) Q(x) + R


Substitute x " a, then
P(a) " 0 . Q(a) + R
R " P(a) .
So, the remainder theorem is:
If P(x) is divided by (x - a), the remainder is P(a). Further, it P(a) " 0, then x - a is
a factor of P(x), and conversely, if x - a 1s a factor of P(x) then P(a) " 0. The result
is known as the factor theorem. A value of x, such that P(x) " 0, IS called a ZERO of
the polynomial P(x) or a root of equation P(x) = 0. For example, x = 2 is a zero of
3
P(x) = x - 8.
165

EXAMPLE (1) Find the remainder when P(x) = 2x 3 - 3x


2
- ~x - 6 is divided by
A(x) = X - 2

SOLUTION: 2
2x + x - 2 If we only want the remainder, we
) 2x 3 - 3x 2 - ~x - 6
X- 2 use:
3 2 R = P(2) = 16- 12- 8- 6 = -10
2x - 4x
2 The remainder is -10. We can write
x - 4x
2
2 P(x) = (x - 2) (2x + x - 2) -10
x - 2x
- 2x - 6
- 2x + 4
- 10

3
EXAMPLE (2) Show that x + 3 is a factor of x - 3x + 18 and hence find the
other factors.

3
SOLUTION: We have P(x) = x - 3x + 18, x- a = x + 3, so a= -3 and
P(-3) = -27 + 9 + 18 =0 •
Since P(-3) = 0, x + 3 is a factor of P(x) we divide P(x) by
x + 3 to find other factors.
2 2
x - 3x + 6 Observe that the missing term x
X + 3 ) 3 2 is arranged as 0 . / . This is
X + 0. X - 3x + 18
3
x + 3/ important and rt•duces the chance
2
- 3x - 3x of t'rrors. The other factor of
2
- 3x - 9x P(x) is / - 3x + 6. (Irreducible
6x + 18
over the R-field)
6x + 18
0

EXAMPLE (3) Find the values of the constant m if the polynomial


3 2 2
P(x) = 4x - m x - 4mx + 64 is divisibk by x + 1, hence find the
other fdctors of P(x)
3
2 2
SOLUTION: P(x) = 4x - m x - 4mx + 64 is divisible by x + 1, i.e. x - (-1}
2
P(-1} =- 0 gives - 4 - m + 4m + 64 = 0
2
:. m - 4m - 60 = 0, i.e. (m- 10) (m + 6) = 0
••• m = 10 or m = -6
For m =- 10, P(x) = 4x - 100x - 40x + 6~ = 4 (x - 25x - lOx + 16)
3 2 3 2
3 2
Divide x - 25x - lOx + 16 by x + 1, then
2
P(x) = 4(x +I) (x - 26x + 16), ( / - 26x + 16 is irreducible over R-field)
For m = -6, P(x) = 4x - 36x + 24x + 6~ =- 4(x - 9x + 6x + 16)
3 3 2
3 2
Divide x - 9x + 6x + 16 by x + 1, then
P(x) = 4(x +I) (x 2 - lOx+ 16) = 4(x + l)(x- 2) (x- 8)
166

3 2
EXAMPLE: (4) Given that x =2 is a root of x - 4x + 14x - 20 = C, find the other
roots.

SOLUTION:
3 2
Divide x -4x +14x-20 by x-2
2 2
(x 3 - 4x + 14x - 20) = (x - 2) (x - 2x + 10)
2
The equation is (x - 2) (x - 2x + 10) = 0
.
x = 2 (given
2
root ) or x - 2x + 10 = 0 , I.e.
. x --
2 +
~ - 1 _+ 3.1 l=3b -
Observe that the complex roots 1 + 3i and 1 - 3i are conjugate. We shall later
prove that the complex roots of a real polynomial occur in conjugate pairs.

EXAMPLE: (5) Solve: x 3 - 3x 2 + 4x - 2 = 0.

SOLUTION:

The constant term 2 = 2 x 1. This suggests we try x = ~ 2, ~ 1 for the roots.


3 2
Let P(x) = x - 3x + 4x - 2
P(2) = 8 - 12 + 8 - 2 :1 0
P(l) = 1 - 3 + 4 - 2 = 0
x = 1 is a root of P(x) = 0
Divide P(x) by x - 1
:. P(x) = (x- 1) (x 2 - 2x + 2)
The roots of P(x) = 0 are 1,
2~ r-4
2 i.e. 1, 1 ~ i

Again observe that the complex roots occur in a pair of conjugates.


167

Exercise SA

Perform the following divisions, and check the remainder by using the remainder
theorem.
3 4 2
I. (x - 2i + 3x - 1) 7- (x - 2) 2. (x - 2x + 3x - 2) 7- (x + 2)
4 3 2 2 3 2
3. (x + 2x + 2x - 2x - 3) 7- (x - 1) 4. (2x - x + x - 1) + (x - 1)

In each of the following, decide whether A(x) is a factor of P(x).


4 3 2
5. P(x)=x -2x +x +x-6 A(x)=x-2
3
6. P(x) = 2x - 3x + 1 A(x) = x + 1
3 2
7. P(x) = x + 2x - x + 6 A(x) = x + 3
3 2
8. P(x) = 5x + 7x + 3x - 1 A(x) =- x + 3

Find the remainder of the following without actually dividing, i.e. use the remainder
theorem.
3 2
9. P(x) = 4x - 3x - x + 7 A(x) =x + 2
4 3
10. P(x) = 5x - 2x + x + 8 A(x) = x + 3
4 2
11. P(x)=2x -4x +5 A(x) = 2x + 1
3 2
12. P(x) = x- ax + bx + 2 A(x) =x - 2

Find values of k such that A(x) is a factor of P(x).

13. P (x) = x4 - 3
3k x + 3x - 1 A(x) =x - 1
3 2
14. P(x) = x .:. kx + 4x - 4 A(x) =X - 2
3 2
15. P(x) = x - 3x - 6kx + 8k A(x) =x - 2
4 3
16. P(x) = x + kx + 7x + 21 A(x) = x + 3

Use the factor theorem to find the value of k that makes A(x) a factor of P(x)
and hence find the zeros of P(x).
3 2
17. P(x) = 3x - 12x - llx- k A(x) =X - 5
3
18. P(x) = 2x - 6 i + kx + 4 A(x) =X - 2
3 2
19. P(x) = k x + x - 8x + 6 A(x) =X - 1
4 3 2
20. P(x) = x + x + k x + 4x - 24 A(x) =X + 3
168

5.2 Zeros of a Polynomial/Multiple Roots

Rational or Integral Zeros


2
We use the formula x =- b !. /b - lfac (or factorise by inspection) to find the
2a
zeros of a second degree polynomial P(x) = ax 2 + bx + c, but it is generally very
3 2
difficult to find the zeros of higher degree polynomials such as 3xlf + 2x - x + 7.
This is because some, or even aJJ the zeros of a polynomial may not be integers,
2
e.g. x + 2 = 0 has no real zeros, let alone integer ones. The following theorem is
very useful in finding rational zeros (if any), though we shall mainly be concerned
with zeros which are integers.

Theorem:

Let P (x ) = anx n + an_ x n-1 + ••• + a x + a be a polynomial with integer


1 1 0
coefficients. If x =%, where % is in its lowest terms, is a rational zero of P(x),

then r must be a factor of a and s must be a factor of a •


0 n

Proof: Since x =% is a zero of P(x), then P<%> = 0.


n n-1 r
:. a ( !:.) + a ( !:.) + ••• + a 1 ( s) + ao 0:: 0 •
n s n- 1 s

Multiplying both sides by s n and rearrangmg:


.

n n-1 n-1 n
anr + an-{ •s + ••• + air. s - aos

n-1 n-2 n-1


Or, r(a r + a r . s + •• • + a s ) ••• (1)
n n- 1 1

The relation ( 1) shows that r is a factor of a sn • But r cannot be factor of


0
n
s as r and s have no common factor, hence r must be a factor of a • In
0
the same manner, we can prove that s is a factor of an. Further, if an 1, P(x)

is monic and the zeros of P(x) are integers as s must be a factor of I.


169

3 2
EXAMPLE: Find all the zeros of P(x) = X + 2x - 3x - 6
SOLUTION:
Since P(x) is monic, the integral zeros must be the factors of the constant term -6.
All the possible factors are .!: I, .:!::2, .!: 3, .!: 6. We find that only x = -2 satisfies
P(x) = 0, so -2 is a zero of P(x).
P(x) is of degree 3 and therefore has 3 zeros.
The other two zeros must be irrational or complex. To find these, we divide P(x)
by X + 2.
2
:. P(x) = (x + 2) (x - 3)
Hence the zeros of P(x) are: -2, .!: Vf.

It is possible that P(x) has no integral zeros, as the following example shows.

4 2
EXAMPLE: Show that P(x) = x - x - 2x + 6 has no rational zeros.
SOLUTION:
All the possible factors of 6 are: .!: 1, .:!:: 2, .!: 3, .!: 6.
None of these numbers satisfy P(x) = 0, hence P(x) has no integral zeros. Since a
fourth degree polynomial must have 4 zeros (see section 5.3, theorem 3), we conclude
that the zeros of P(x) are either irrational or complex.

3 2
EXAMPLE: Find all the roots of 5x + 28x + I Ox - 3 =0
SOLUTION:
3 2
Let P(x) = 5x + 28x + lOx - 3 = 0.
To make our work easier, we transform this equation so that the leading term has a
coefficient equal to I.
3
Observing that the coefficient of x is 5, we multiply the equation by 25 and then
put y = 5x.
3 2
(5x) + 28 (5x) + 50 (5x) - 7 5 =0
3 2
Or y + 28y + 50y - 75 = 0 •·· (1)
By testing the possible factors of 7 5 i.e . .:!:: I, .!: 3, .!: 5, .!: 15, .!: 25, .!: 7 5, we find that
y = -3 is a root of (1), hence

P(y) = (y + 3) <l + 25y - 25)


- 25.!: 5 ff9
The roots of (I) are: y = -3,
2

Using x = ~ , the roots of the given equation are:

-3 -5.:!:: V29
X= S' 2
170

Multiple Roots

P(x) = (x - 2)
2 (x 2 5x + 6)
Consider the polynomial +

x = 2 is called a double root of the equation P(x) = 0. Similarly for


P(x) = (x - 2) 3 (x + 2), x = 2 is called a triple root of P(x) = 0. In general, we call
b, a multiple root of order r if
P(x) = (x - b)r Q(x), Q(b) 1- 0 .
We also speak of a root of multiplicity r.

Theorem: If x = b is a root of multiplicity r of the real polynomial equation


P(x) = 0, then x =b is also a root of the derived polynomial equation ~~ ~ 0 of
multiplicity (r - J ).

Proof:
Let P(x) (x - b)r • Q(x) , where Q(b) /. 0 •
dP r-l dQ r
dx r(x - b) Q(x) + ax . (x - b)

(x - b{-l [r Q(x) + (x - b)~~


(x - b)r-l S(x) , where S(b) l 0, S(x) is the polynomial in the bracket.
This clearly shows that x = b is a root of multiplicity (r - I) of the derived equation
dP
dx = O.

EXTENSION:

If (x - a{ is a factor of P(x), then


r-1 . c!P
I. (x - a ) IS a factor of dx
2p
2. (x - a)r- 2 is a factor of d and so on.
2
dx

EXAMPLE: (6) Show that ~: has a zero of multiplicity 2 if P(x) = (x - 2)


3
(x
2
+ 2)

dP 2 2 3
SOLUTION: 3(x - 2) (x + 2) + 2x(x - 2)
dx
2
(x - 2) [3(x 2 + 2) + 2x (x - 2)]

dP has x = 2 as a double root


dx
171

3
EXAMPLE: (7) Solve the equation x - 3x - 2 = 0 , given that it has a double root.

SOLUTION:
3
Let P(x) = x - 3x - 2

~~ = 3x2 - 3 = 3(x - l) (x + l) = 0 gives x = .:!:. l

The possibilities for the double root are l and -1


Now P(l) = l - 3 - 2 J 0 and P(-l) = - l + 3 - 2 = 0
3
x = -I is a double root of P(x) = x - 3x - 2 = 0
2 3
Divide out x 3 - 3x - 2 by (x + l ) , i.e. / + 2x + I, then x - 3x - 2 = (x + I f(x - 2)
3
The roots of x - 3x - 2 = 0 are -I, -I, 2

EXAMPLE:(&) Show that:


2 2
(a) x + px + q = 0 has a double root, if p = 4q
3 3 2
(b) x + px + q = 0 has a double root, if 4p + 27q = 0

SOLUTION:
2 dP
(a) Let P(x) = x + px + q, then dx = 2x + p

~~ = 0 gives 2x + p = 0, hence x = -~ 2
is a double root of x + px + q = 0.
Substituting x = - ~ in the equation,
2 2 2
~-IT+q=o whence p 4q
2 2
We <:..111 check this condition for x + px + q = 0 by using fl. = p - 4q = 0 for equal
roots.
3 dP 2 2
(b) P(x) = x + px + q, then dx 3x + p = 0 gives x = -)

For x to be real, p " 0.


To avoid irritating fractional powers, we write the original equation
2
x (x + p) = -q as

x 2 ( x 2 + p )2 -- q 2 , now put x 2 -- - 2 •
3
- 2 (- 2 + p) 2 = q 2 which gives 4p 3 + 27 q 2 = 0
3 3
172

Exercise 58

Find the real roots of the following equations, given that each equation has a double
root:
3 2 3 2
I. x - 7x + II x - 5 = 0 2. x - 2x - 4x + 8 = 0
3 2 4 3
3. x - 2x - 15x + 36 = 0 4. x + x + x + I = 0

5. Find the real roots of each equation, given that each has a triple root:
4 3 2
(a) P(x) = x + 2x - 12x + 14x- 5 = 0
4 3 2
(b) P(x) = x - 6x + 12x - lOx + 3 = 0
2
d P
[Hint: - - = 0 and P(x) = 0 have a common root]
2
dx

6. Find the point where the tangent to the curve


3 2
(a) y = x - 2x + 3 at x = 2
3 2
(b) y = x - 2x + I at x =

meets the curve again at Q. Find the coordinates of Q.

7. Find the viilues of k such thdt each of the following equations has a double
root:

(a) P(x) =x3 + 6x


2
+ 9x + k =0
(b) 3 2
P(x) = x + 3x - 24x + k = 0

Find the roots of P(x) = 0 for each value of k.

8. Find all the roots of the following equations. First find all the integral roots,
then by division of P(x) find the other roots.
3 2 3 2
(a) x - x - 4x + 4 = 0 (b) x + 2x - 5x - 6 =0
3 2
(c) 2x - 9x + 7x + 6 =0 (Hint: Multiply by 4, let y = 2x)
4 3 3 2
(d) x + x - 7/ - 13x - 6 = 0 (e) x - 4x + x + 6 =0
4 3 2 3 2
(f) x - 6x - x + 34x + 8 = 0 (g) 4x - lix + x + I = 0
(Hint: multiply by 16, y = 4x)
3
(h) 4x - 5x - 2 = 0
(Hint: y = 2x)
4 3 2
(i) (Solve for tan 9 only): tan 9 - 2tan 9 - I 3tan 9 + 14tan9 + 24 = 0

9. Prove the theorem:


If x = r is an integral zero of the monic polynomial
n n-1
P(x) = x + a _ x + ••• + a , then r must be a factor of a .
11 1 0 0
173

5.3 Polynomials over the Complex Field

2 2
A polynomial such as x + 4 has no real factors. Using i = -I, we can factorise
2
x + 4.
2 2
x + 4 = l -4i = (x - 2i) (x + 2i)
Thus a real polynomial with no real factors can be factorised over the complex field.
We shall now study several theorems which are essential for the factorisation of
polynomials.

You might wonder whether every polynomial has a zero. The following theorem
answers this question.

Theor-em 1: The Fundamental Theorem of Algebra

If P(x) is a polynomial of degree n ~ I, then P(x) has at least one zerc over the
complex field.

The proof is beyond the scope of this book. It was first proved by the German
mathematician Gauss in 1799, when he was 22.

From this theorem and the factor theorem, it follows that if P(x) is a polynomial of
degree n ) I, then there is at least one number c such that:
1
P(x) = (x- c ) Q (x)
1 1
If we repeat the process on the polynomial Q (x), we get:
1
P(x) = (x- c ) (x- c ) Q (x)
1 2 2
If we repeat the process n times, we obtain the following:

Theor-em 2: If P(x) is a polynomial of degree n ~ I, then there exist n numbers


c 1' c 2, ••• , en (not necessarily distinct) such that

P(x) = an (x - c ) (x - c ) .•• (x - en)


1 2
where anf. 0 is the leading coefficient.

Theorem 2 ensures that every polynomial of degree n ~ I can be completely


factorised into n linear factors. Each of the numbers c , c , •.• en is a zero of
1 2
P(x) and hence a root of the equation P(x) = 0.
Now let P(c) = 0, then:
P(c) = an (c - c ) (c - c ) ••• (c - en) = 0
1 2
This is possible only if c is one of the numbers c , c , .•• , en. So it follows from
1 2
theorem 2 that:

Theorem 3: A polynomial P(x) of degree n ~ 1 has at most n distinct zeros.

This ensures that the equation P(x) = 0 has at most n distinct roots.
174

3
EXAMPLE: (9) The polynomial P(x) = (x - 2) (x - 3) (x + 1) has 5 zeros, 2,2,2,3,-1.
Here 2 is a zero of multiplicity 3.
3
EXAMPLE: (10) The polynomial P(x) = x - 1 = (x - 1) <i + x + 1) has 3 zeros:
1 -1 + /3i
' 2
3
Observe that the complex zeros of the polynomial x - 1 are a pair of conjugates.
An important fact about the complex zeros of P(x) with real coefficients is given in
the foHowing theorem.

Theorem 4: If a + ib is a complex zero of the polynomial P(x) of degree n ~ 1,


having real coefficients, then a - ib is also a zero of P(x).

It follows that if a + ib is a root of P(x) = O, then so is a - ib.

There are two methods of proving this theorem.

Method 1: Using the properties of complex conjugates

We shall need the following properties of conjugates.


If z and z are two complex numbers and m is a real number, then:
1 2
(a) (z
1
+ z )
2
= z
1
+ z
2
(b)

(c) (z l>n

Let z = a + ib be a complex zero of P(x) = a n xn +

a zn + ••• + ao = 0 ' hence:


n

a z
n + ••• 0
n
By property (a):
n n-1
anz + an-l z + ••• 0

By property (b):

+ alz + ao 0

By property (c):
- n (- n-1 -
an (z) + an-1 z) + ••• + a 1 z + ao 0

But the L.H.S. of the above equation is P(z)

This completes the proof. The requirement that P(x) have real coefficients is
essential.
Verify that x -2i is a root of i + 2xi = 0 but x 2i is not.
175

Alternative Proof:

Let z =a + ib be a complex zero of the polynomial P(x) with real coefficients.


- -
We use z =a - ib, z + z = 2a, zz =a2 +b
2
to form the product:
2 2 2 2
(x - z) (x - z) = x - (z + z) X + ZZ = x - 2ax + a +b
2 2 2
If P(x) is divided by x - 2ax + a + b , the remainder must be of degree < 2.
Hence,
2 2 2
P(x) = [x - 2ax + a + b ] Q(x) + mx + n
[x - (a + bi)] [x - (a - bi)] Q(x) + mx + n
Since a + ib is a zero of P(x), P(a + ib) = 0. Substituting x = a + ib, gives:
0 = 0 + m (a + ib) + n
(am + n) + bm i = 0 = 0 + 0 • i
Comparing the real and imaginary parts separately, am + n = 0 and bm = 0 and
hence m =0 and n = 0. (b 1- 0)
Thus, P(x) = [x - (a + ib)] [x - (a - ib)] Q(x)
Hence P(a - ib) = 0, i.e. a - ib is also a zero of P(x).

WORKED EXAMPLES

EXAMPLE: (11) Factorise over the complex field C


2
(a) x 2 + 2x + 3 (b) 4x + 3x + 2

SOLUTION: By completing the square

(a) x
2
+ 2x + 3 (x 2 + 2x + I) + 2
2 2
(x + I) + 2 [i =-l]
2 2
(x+l) -2i

[x + 1 + .f2 i] [x + I - f2 i]
(b) 4x 2 + 3x + 2 4[ ( X + i) 2
+ ~~]
4 [x + j + 'P i] [x + j - 'P i]
176

EXAMPLE: (12) Factorise over the complex field C


4 3+ 2 (c) x
3
- 1
(a) x - 1 (b) X 3X - X - 3

SOLUTION:
4 2 2
(a) x - 1 (x - 1J tx + 1)
(x - 1) (x + 1) (x - i) (x + i)
3 2
(b) Let P(x) x + 3x - x - 3. The constant 3 =3 x 1, so we try .!.1, .!. 3
P(l) 1+ 3 - 1 - 3 =0
x - 1 is a factor of P(x)
Divide P(x) by x - 1

P(x) = (x - 1) (x 2 + 4x + 3) = (x - 1) (x + 1) (x + 3)
It must be remembered that R is a subset of C.

(c) x
3
- (x - 1) (x
2
+ x + 1) = (x - l) [( x + ~) 2 + ~]
X-
3 = (X- 1)(X+2+Tl 1 13.)
1 .[3.)(X+2-Tl

EXAMPLE: (13) Show that x = 1 + i is a zero of the polynomial


2
P(x) = x 3 + 2x - 6x + 8 and hence factorise P(x) completely
over C.

SOLUTION:
2
P(x) = x3 + 2x - 6x + 8
We know that if 1 + i is a zero of P(x) with real coefficients, then - i is also a
zero of P(x). We form the product:
(x - z) (x - Z) where z =1 + i, z =I - i, z + z = 2, zz = 2
2
x - (z + z) X + ZZ
2
= x - 2x + 2
2
Divide P(x) by x - 2x + 2
2
:. P(x) (x + 4) (x - 2x + 2) = (x + 4) (x - 1 - i) (x - 1 + i)

Note: If z = a + ib is a zero of real P(x), then is also a zero of P(x). z = a - ib


2 2
It is easier to divide P(x) by (x - z) (x - z) = x - 2ax + a 2 + b , than by
x - a - ib.
177

Exercise SC

Factorise the following quadratics over the complex field C.

1. 2.
2 2 3.
X + C

2 2
4. X +X + 5. x + 3x + 5 6.
2 2
7. 4x - x + 1 8. 2x + 1

One zero of the polynomial P(x) is given, find the other zeros over the complex
field C.
3 2
9. P(x) x - 3x + 4x - 4 ; X =2
3 2
10. P(x) 2x - 2x - x - 6 ; X =2
3 2
11. P(x) X +X - 7x + 65 ; X = 2 - 3i
3 2
12. P(x) = 2x - 5x + 6x - 2 ; X=1 - i
4 3 2
13. P(x) x - 2x - 7x + 26x - 20 ; X =2 + i
4 3 2
14. P(x) = X + 3x - x - 13x - 10 ; X=-2-i
4 3 2
15. P(x) 2x + 1lx + 20x + 7x - 10 ; X =i - 2

Factorise completely over the complex field:


4 2 2 2 2
16. x + 3x +4 [Hint: P(x) = (x + 2) - x ]
4 2 2 2
17. X + 16 [Hint: P(x) (x + 4) - 8x ]
4 2 2 2 2
18. X +X + 1 [Hint: P(x) (x + 1) - x ]
6 2 4
19. X - 1 [Hint: P(x) (x - 1) (x + x 2 + 1)]

20. Evaluate the following in the form a + ib.


3 2
(a) P(x) = x +x +x + 1 for x =1 - i
2
[Hint: Write P(x) = x (x + l) + (x + 1) and substitute]
4 3 2
(b) P(x) = x - 2x + x - 3x + 2 for x =1 + i
[Hint: P(x) =x 2 (x 2 - 2
2x) + x - 3x + 2]
178

5.4 Relation between Roots and Coefficients of P(x) = 0

In 3U Mathematics we have studied the relations between the roots and coefficients of
polynomials equations P(x) = 0 of degrees 2, 3 and 4.

1. ax
2
+ bx + c = 0 a + S = --ba , c
a
3 2
2. ax + bx + ex + d = 0 b
' a + s +y = --
a
c
aS +Sy+ya
a

aSy --ad
4 3 2
3. ax + bx +ex + dx + e = 0
b d
a + s +y+o=-a:· ras = ~a ' raSy=-
3, aS yo ~
a

The following two theorems are used in establishing the relation between the
coefficients and the roots of the polynomial equation.
n n-1
P(x) = anx +an_ x + ... +a 1x+a =0.
1 0
Theocem (5): If a polynomial P(x) of degree n ~ 1 vanishes for more than n
values of x, it is the zero polynomial, i.e. an = an-l = ••• = a = 0 •
0
Referring to the theorem (3), i.e. a polynomial of degree n cannot have more than n
distinct zeros, we find that the assumption that P(x) is not the zero polynomial
contradicts the fact that the polynomial vanishes for more than n values of x.
Hence P(x) is the zero polynomial.

2
EXAMPLE: (14) Show that a = b = c = 0 if P(x) = ax + bx + c vanishes for
x = m, n, r (3 distinct values)

SOLUTION:
2
We have: am + bm + c = 0 .•. (1)
2
an + bn + c = 0 •.• (2)
a/ + br + c= 0 ••• (3)
2 2
Subtracting, we get: a(m - n ) + b(m - n) = 0 , m 1- n
a(m + n) + b = 0 .•• (4)
Similarly a(n + r) +b =0 ••• (5)
Subtracting (5) from (4) : a(m - r) = 0 , m 1- r
:. a = 0 , then from ( 4) : b = 0, then from (1) : c = 0
179

lbeorem (6): If two polynomials, each of the degree n, P(x) = Ea xn and


n n
Q(x) = Ebnx have equal values for more than n values of x, then:

an = bn ' an-1 = bn-1' ••• ao = bo •

Proof: We wr:ite R (x) = P(x) - Q(x) = 1: (a - b ) xn


n n
Now R (x) is a polynomial of degree n and it vanishes for more than n values of
x, hence by the theorem (3):
an- bn = 0 ' an-1 - bn-1 ' ••• ao- bo = 0.
This proves the theorem.

RELATION BETWEEN ROOTS OF P(x) = 0

Let x , x , ••• , xn be the n roots of P(x) = 0. We divide out P(x) by the leading
1 2
coefficient an and write the identity:

n an-1 n-1 an-2 n-2


X +--X
a +--X + •••
n an

R.H.S. = xn - ( Ex )x n-1 + (Ex x )x n-2 + ..• + (-1} n x x ••• xn.


1 1 2 1 2
comparing coefficients on each side.
an-I
(1) sl = Sum of the roots= XI + x2 + +X=---
n an

an-2
(2) s2 = Sum of products =XI x2 +XI x3 + ••• = +-a-
of roots taken n
two at a time •
••• and so on.

(n) sn = Product of all


roots

These relations also help us to write down the equation whose roots xl' x 2, ••• , xn
are given.

Xn - S l X
n-1 + S X n-2 + ••• + (-1 )nSn = 0
2
The relation for is frequently needed in many problems of solving polynomial
equations.
n-1
S = Sum of the roots = x +x + •••
+X = _..;;;...;.
coeff. ____
of x _
1 1 2 n
coeff. of xn a
n
180

WORKED EXAMPLES

3 2
EXAMPLE: (15) If a, 13, y are the roots of x - 2x + x + 3 = 0, evaluate:
2 2 2 (b) 3 3 3 (c)
4 4 4
(a) a + 13 + y a + 13 + y a+f3+y

SOLUTION:

We have: a + .8 + y = 2 , aS + 13y + ya = 1 , af3y = - 3


2 2 2
(a) a + 13 + l=(a+ 13+y) -2(af3+8y+ya)=4-2=2
3
(b) A formula to convert l:a in terms of l:a and l:a8 is very difficult, so we try
a simple trick: .
3 2
Re-write the equation as x = 2x - x - 3 and substitute x = a, B , y and add:
3 2 2 2
a + a3 + y
3
= 2 (a + B + y >- (a + B + y ) - 3 - 3 - 3
=2x2-2-9
= -7
By using the formula, we can verify that
3 2
ra = (l:a). (l:a -l:a8) + 3a8y = 2 (2- 1)- 9 = -7
.. 4
(c) There is no known formula that relates l:a with l:a etc. so we proceed as
follows:
4
Multiply the given equation by x and write: x = 2x 3 2
- x - 3x
Substitute a, B, y and add:
4 3
:. ra = 21:a - r i - 31:a =2. (-7)- 2- 3 (2) = -22

4 3 2
EXAMPLE: (16) Solve P(x) = x - 3x - 6x + 28x- 24 = 0, given that P(x)= 0 has a
root of multiplicity 3.
SOLUTION:
2
dP d P
Since P(x) has a root of multiplicity 3, P(x), dx and - - have a common zero.
2
dx
2
Now dP = 4x 3 - 9x 2 - 12x + 28 and d p = 12x 2 - 18x- 12
dx . dx 2
2
Solving 12x - 18x - 12 = 0 + (x - 2) (2x + I) = 0

X =2 or X = -zI
Since P(- 4> /. O, P(2) = 0, x = 2 is the triple root of P(x) 0

Now the sum of the roots is:

2 + 2 + 2 + x = 3, so x = -3, hence the roots are:


4 4
2, 2, 2 and -3
181

Transformation of Polynomial Equations:

I. The equation whose roots are the reciprocals of the roots of a given equation, is
obtained by putting x = ! in the given equation.
y
2. The equation whose roots are those of a given equation multiplied by a constant
m, is obtained by putting x = X. in the given equation.
m
3. The equation whose roots differ by a constant k from the roots of a given
equation is obtained by putting x = y + k (k > 0 or k < 0) in the given
equation.

4. The equation whose roots are the squares of the roots of a given equation is
obtained by putting x = .JY in the given equation.

3 2
EXAMPLE: If a, S, y are the roots of 2x + 3x + x - 5 = 0, form the equation
whose roots are:

(a) (b) 2a, 28, 2y (c) 2 + a, 2 + B, 2 + y


a'e':Y
2 , B2 , y 2 (f) -a , - B , -y
(d) a - 2, B - 2, y - 2 (e) a

SOLUTION:

I I
(a) Let y =x
I
, since x = a, B, y , Y=a.·s•y
P ut x = -I (f rom y = -XI ) . t he given
m . .
equation 2 x 3 + 3x 2 + x - 5 =0
y

.•. 32 +
3
+ y
I - 5 =0 ..
givmg 2 + 3y + y 2 - 5 y 3 = 0
y 2
y
3 2
The required equation is: 5x - x - 3x - 2 = 0.

(b) Let y = 2x, since x = a, B, y , y = 2a, 2B, 2y

From 2x = y, we put x = ~ m
. t h e given
. .
equatiOn, 2x 3 + 3x 2 + x - 5 = 0

2 3 3 2
£.t.... +2.L +t -5=0
8 4 2
3
Multiplying by 4, y + 3/ + 2y - 20 = 0
3 2
So, the required equation is x + 3x + 2x - 20 = 0.

(c) Let y = 2 + x, since x = a, B, y, y = 2 + a, 2 + B , 2 + y


3 2
Substituting x = y - 2 in the given equation, 2x + 3x + x - 5 =0
2(y- 2) 3 + 3(y- 2) 2 + (y- 2)- 5 = 0

This simplifies to 2y 3 9/ + 13y - II 0, hence the required equation is:


3 2
2x - 9x + 13x - II = 0
182

(d) Let y = x - 2, then substituting x = y + 2 in the given equation:

2 (y + 2) 3 + 3 (y + 2) 2 + (y + 2) - 5 =0 which simplifies to:


2
2y 3 + l5y + 37y + 25 =0
:. 2x 3 + l5x 2 + 37x + 25 =0 is the required equation.

(e) Let y = x 2 then putting x = JY, we obtain the equation whose roots are
2 2 2
Cl. ' s' 'Y •
3 2
:. 2y ' + 3y + Vy- 5 = 0
To remove the radical signs, we write:
3y - 5 = - JY (2y + 1) ' (y3/2 = y .JY)
Squaring both sides:
3
9/ - 30y + 25 =y (4/ + 4y + l), which simplifies to: 4y - 5/ + 3ly - 25 = 0

4x 3 - 5x 2 + 3lx - 25 =0 is the required equation.

(f) Let y =-x, then putting x = -y we obtain the equation whose roots are
-a, -S, -y.
- 2y 3 + 3y 2 - y - 5 =O, hence the required equation is
2
2x 3 - 3x + x + 5 =0. (Compare this with the original equation)
183

Exercise 50

3 2
1. If a, a, y are the roots of x - 2x - 3x + 1 = o, evaluate:
2 2 2 3 3 3 4
(a) a + a + y (b) a + a + y (c) a + a4 + y
4
3
2. If a, a, y are the roots of X - 5x - 1 =0, evaluate

(a) a-l + a-l + y-l (b) a5 + a5 + y 5 (c) (a+ B-y) (B +Y -a) (y +a- B)
(Hint: a + B + y = 0)
4 3 2
3. If a, B, y, o are the roots of x + x + 2x + 3x + 4 =0, evaluate

(a) a + B + y + o (b) aB + cry + ao +By + BcS +yo

3 2
4. Solve the equation x - 3x - 6x + 8 = 0, given that the roots are in geometric
progression.
[Hint: Let the roots be ~ , a, ar)

4 3
5. Solve the equation x + 2x - 21/ - 22x + 40 = 0, given that the roots are in
arithmetic progression. [Hint: Let the roots be a - 3d, a - d, a + d, a + 3d)
4 3 2 2 2
6. If x + 4x + mx + nx + 9' = (x + bx + c) for all values of x, find all
possible values of b and c and the corresponding values of m and n.
3 2
7. Find the condition satisfied by p, q, r if the roots of x + px + qx + r = 0
are in arithmetic progression.

8. Solve the equation 6x


3 - 17x 2 - 5x + 6 = 0, given that the product of two of
its roots is -2.
4 3
9. The equation x + 2x - 3/ - 4x + 4 =0 has a double root. Solve the equation
completely.
3 2
10. The roots a, B, y of the equation x + bx + 12x + 4 = 0 are such that
a-l, B-l, y-l are in arithmetic progression. Find b and solve the equation.

[Hint: Let X = ~ , then 4 u


3
+ 12 / + b u + 1 = 0 has roots a, ~ and y
which are in arithmetic progression)
184

3 2
11. If CX, y a, are the roOtS Of X + 2x - 2x + 3 = 0, form the equation whose
roots are:

(a) 2 2 2 B+
a • s •Y (b) 2ex, 2B, 2y (c) ex + 1, 1, y + 1

2 62, y2
(d) ex - 2, B - 2, y - 2 (e) ex , (f) ~ , ~ , }

3 2
12. If ex, B, y are the roots of 2x + 3x - x - 1 = O, form the equation whose
roots are:
1 1 1 2
(a) ex + 2, a + 2, y + 2 (b) ex + 2, 8'+2 , y + 2 (c)
ex '
(Hint: use (a) )
3
13. If ex, a, y are the roots of x + bx + c = 0, form the equation whose roots
are:
(a) 2ex, 28, 2y ·(b)B+y-ex, Y+ex-B,ex+B -y (Hint:ex+B+Y=O)

(Hint: let y = ~ = - - = -
1 1 1 1
(c) ~ , 8+Y , Y +ex ex+p -y
.!.X etc.)
3 .2
14. If ex, B, y are the roots of x + bx +ex + d = O, form the equation whose
-1 -1 -1 · (a·) -1 -1 -1
roots are ex , B , y and hence evaluate ex + B + y

(b) ci'l 81 + 8 1 .y1 + .y1 ci1 , (c) . ex-2 + 8-2 + Y-.2


[Hint: p, + q
2 2
+ r
2
= (p + q + rl - 2 (pq + qr + rp)]

4 2
15. If ex, 8, y, 0 are the roots-:i:.x .- x + 2x + 3 = 0, form the equation whose
roots are:
(a) 2ex, 2 8, 2y, 2o (b ) ex-1 , •• B-i, -1 -1 2 2 2
(c) ex , 8 , y ,
2
y, o o
3 I
16. ex, B, y are the roots of x - 2x+ 3 = 0
2 . 2 2
(a) Form the equation whose roots are ex , 8 , y
(b) Using the result of (a), now form the equation with roots
2 2 2
ex + l, 8 + 1, y + l.

(c) Hence evaluate <i + l) (8


2
+ l) <l + l)
185

5.5 Miscellaneous (Worked Examples)

EXAMPLE: {18) For what real values of c is z - c i a factor of


4 3 2
P(z) = z - z + 9z - 4z + 20? Hence solve P(z) = 0, c 1-0.
SOLUTION: By the factor theorem:

P(c 1") = 0 g1ves


. c 4 + c 3.1 - 9 c 2 - 4c.1 + 20 =0
4 2 3
:. c -9c +20 +i(c -4c)=0
Equating the real and imaginary parts:
4 2 3
c - 9c + 20 =0 and c - 4c =0
2 2
i.e. (c - 5) (c - 4) and c (c - 2) (c + 2) = 0
2
Now c I 0 and c = 5 satisfy one but not both equations

on l y c 2 = <t, + 2 satisfies both
i.e. c = _

We divide P(z) by (z + 2i~ (z - 2i) , i.e. z


2
+ 4
2 2
P(z) = (z + 4) (z - z + 5)
. I+/T9i
The roots of P(z) = 0 are: _+ 2 1 ,
2

3 ..
EXAMPLE: (19) If a, 8 and y are the roots.. of x + bx + c = 0, find the equation
2 2 . 2
whose roots are a , 8 and y •

SOLUTION: METHOD I

We have (l + 8+ y 0
aB +By + ya b
a By = -c
2 2
Let XI (l ' x2 = 82, x3 = y then

2 2 2
51 Ex
1
= a + 82 + y (Ea) - 2La8 - 2b
2 2 2 2
52 Ex x = Ea 8 (Ea8) - 2La8 y
1 2
2
(Ea8) - 2a8y Ea
b2
2
53 = xlx2x3 = a2 s2i c
The required equation is
3 2
x -5 x +5 x-5 =0, i.e.
1 2 3
186

SOLUTION: METHOD II
2
Let y= X
2
This transformation will give us an equation whose roots are a , s2 and y
2

To minimise the calculation, we write:


2
x(x +b) = -c
Substitute x = /Y
IY (y + b) . = -c . Squaring

y </ + 2by + b > 2 c2 or


3 2 2 2
y +2by +by-e =0 is the equation
2 2
whose roots are a , B , i. This method is much superior to and easier than the other
method.

EXAMPLE: (20) When a polynomial P(x) is divided by x - I the remainder is 2,


when divided by x - 2 the remainder is 3. Find the remainder
when P(x) is divided by (x - I) (x - 2).

SOLUTION:
The degree of the remainder is less than that of the divisor. When the divisor is
(x - 1) (x - 2), the remainder must be of the form ax + b, hence
Let P(x) = (x - I) (x - 2) Q(x) + ax + b ••• (I)
Given that P(l) =2, P(2) =3, we now have: 2=a+b and 3=2a+b
Solving these, we get a = 1, b =1, hence the remainder is x + 1

EXAMPLE: (21) If z =cose


+ isine, prove that zn + z-n = 2cosne, hence solve
4 3 2
the equation 3z - z + 4z - z + 3 = 0 •

SOLUTION:
z = cose + isine
cos(ne) + isin(ne) and z-n cos(ne)- isin(ne)
1
zn + - - = 2cos(ne) ••• (1)
zn
The equation is: ( rearrange)
4 3 2 2
3(z + 1) - (z + z) + 4z = 0 . Divide by z ' then: 3(z 2 + I -z> - (z + z>1 + 4 =0 •
z
Using (1) with n =2 and n = 1, we now have:
6cos2e - 2cose + 4 = 0 , 2
using cos2e = 2cos e -
2 2
6(2c - I) - 2c + 4 =0 = 2c - 1
2
6c - c - 1 = 0
1 1
(2c - 1) (3c + I) = 0 /jves c = 2 or - 3
1
If cose = , sine = .!.
2 , - n <e ~ n
2
and cose =- ~, sine =.!.I;}
••• The roots are: ! (1.!. v'Ji), 1 (-1.!. .{ii)
187

Exercise SE (REVISION AND MISCELLANEOUS)

1. Factorise the following polynomials over (a) R (b) C


(a) X
3 - 8 (b) X
4 - 16 3
(c) x + 1
2. Factorise the following polynomials over c.
(a) X2 + 4 (b) X2 + 8x + 8
(c)
2 (d) X3 2
X - 4x + 6 + 3x + 4x + 12
(e) X4 + 4x 2 + 3 (f) X3 +X
2
- 4x - 24
3. For each of the following, find a polynomial of lowest degree, having the given
zeros.
(a) 2 + i, 2 - i (b) 3 + 4 i, 3 - 4 i
(c) -2, 2 + i, 2 - i (d) 3, I +i
(e) l - 12, l + 12, + i (f) 4 - i, 3+ i
4. For the following polynomials, one zero is given. Find the remaining zeros.
(a) P(x) x
3 - 7x 2 + l7x - 15, x =3
(b) P(x)
3 2
x - 7x + 17x - 15, x =2 - i
4 3 2
(c) P(x) x - 3x + 6x + 2x - 60, x = l - 3i
3 2
(d) P(x) 2x - l3x + 32x - 13, x =3 - 2i

5. Factorise the following over C, by using the factor theorem:


2
(a) x 3 + x + x + 1 3 2
(b) x - x - 4x - 6
3 2 3 2
(c) x - 3x + 7x - 5 (d) 2x - 2x - x - 6

6. Solve the following over C. (Factor theorem is useful)


3 4
(a) x + I = 0 (b) X - 16 = 0
4 4 2
(c) X - 4 = 0 (d) x + 3x + 2 = 0
4 3 2
(e) x - 7x + 18x - 22x + 12 = 0
4
7. Given P(x) = x - 5x 3 + 4x 2 + 3x + 9 = 0 has a root of multiplicity 2, solve
the equation completely.
2
8. Given 1 - i is a root of the equation z - (c - 2i) z + 3 + ib = 0 where b
and c are real. Find b and c. [Hint: a+ 13 = - b/a, and use aS = c/a]

9. If z = cos9 + isin9, prove (by using De Moivre's theorem) that:


zn + z-n 2cosn9 =
4 3 2
Hence solve 2z + 3z + 5z + 3z + 2 = 0
3 2 2
10. If mx + nx + p =0 has a double root, prove that 27m p
[Use P(x) = ~~ =0 have a common root]
188

11. (a) Use Mathematical Induction to prove that


(cose + isin9)n = cosn9 + isinne, for positive integers n.
(b) Solve x 6 = -1, giving answers in A + iB form.
4 2
(c) Find the four roots of x - x + I = 0 and show them on an Argand
diagram.

12. (a) Show that x - 3 + 2i is a factor of the polynomial


3 2
P(x) = x - 9x + 3Ix - 39 and hence decompose P(x) over
(i) field R (ii) field C.
5 4 3 2
(b) P(x) = 2x - 13x + 24x + 8x - 64x + 48 has a root of multiplicity 4,
find the roots.
2
13. Given that cotS = y + I and a and 8 are the roots of z - 2z + 2 = O,
prove that: ( Y + a )n-( Y + 8 )n sin ne
a - 8 (sine )n
5
14. Solve x = I by De Moivre's theorem dnd indicate roots on a circle of radius
in an Argand diagram. Express the roots in the cis8 form.
Find the area of the regular pentagon formed by the five points representing
these roots.
4 3 2
Solve the equation x + x + x + x + I = 0 •
211 411 2
Deduce that cos and cos are the roots of 4x + 2x - 1 = 0 •
5 5
3 2
15. Given that 2- i is a root of the equation 3x - 10x + 7x + 10 = 0, solve
!he equation completely.
4 2
16. Solve z + z + 1 = 0, giving all the four roots in rcis e form, -11 <e ~ 11.
2
17. Find K if P(x) = x3 - 6x + 9x + K has a root of multiplicity 2.
2
18. One root of x + (I - i) x + K = 0 is 2 + 3i .
Find the other root of this equation and the value of K in the form a + ib •

19. (a) Solve tan49 = 1, 0 ~ 9 ~ 11


3
(b) Prove that tan49 = 4 tane - 4 tan 9
2 4
I - 6 tan 9 + tan 9
4
(Hint: Find sin48 and cos49 from the (cis9) expansion)
(c) Using parts (a) and (b) and z = tan 9 , find the roots of
4 3 2
z + 4z - 6z - 4z + 1 = 0.
(d) Using (c) find:

(i) 11 51! 911 1311


tan + tan T6 + tanu; + tan T6
16
2 11 2 511 2 911 2 1311
(ii) tan T6 + tan T6 + tan T6 + tan T6
189

20. Using the method of finding the square-roots of a complex number, find the
roots of the following equations in the form A + i B, where A, B are real.
(b) z +(2+3i)z-~-2i=0
2 2
(a) z +3iz+2=0
2
(c) z - (4 + 2i) z + 6 + 8i = 0 (d) z 2 = 3 - 4i
2 2
(e) z +(4+2i)z+3+2i=0 (f) z -(3+i)z+4+3i=0
2
21. (a) Show that the roots of z + 2z + 3 = 0 lie outside the circle 1z 1
2
(b) Show that the roots of z + z + 1 = 0 lie on the circle lz 1 = 1
(Hint: Consider the modulus of each root)
4 3 2
22. Prove that z = 1 - i is the root of P(z) = z - 6z + 15z - 18z + 10 = 0.
Find the other roots.
[Hint: Dividing P(z) by z - (l - i), you can show that the remainder is zero,
but this involves very awkward division, so a better method is to divide by
2 2 2
[ (z - 1) + i] [ (z - 1) - i] = (z - 1) - i = z - 2z + 2 .
This not only proves that z = 1 - i is a root, but also gives all the remaining
roots in one simple division.]
23. Solve
(a) z
4 2
- 2z + 4 = 0 (b) 6 + (x + 1) 6 = 0
(x - 1)
2 (d)
4 2
(c) 2z = 1 + i z - 30z + 289 " 0
8
24. Solve x + 1 = 0 , express each root in the form rcis9 and then A + iB.
8
Decompose x + 1 into real quadratic factors and deduce that

cos49 = 8[cos9 -cos i J[cose -cos 38 11


]
511
[cos e -cos 8 J 7
[cose -cos ; J
3 2
25. z = 1 - i is a root of z + az + bz + 6 = 0, where a and b are real.
Find a and b and all the roots of the equation.

26. z =- ~- i "J" is a root of z 4 + 2z 3 + z 2 - 1 = 0. Find all the other roots.

27. Find m and n such that x = 3 is a double root of


4 3 2
x + mx + 13x + nx - 36 = 0 •
11
28. Show that the polynomial x + mx - b = 0 has a multiple root provided

!:!!)n
n + (-b _ )n-1 = O. Find this root.
( 11 1
3
29. If the equation x + 3mx + n = 0 has a double root, then prove that
n = -4m , and that this root is - ~ •
2
2 3

4
30. Solve x - 6x 3 + 12i - 1Ox + 3 = 0 given that the equation has a root of
multiplicity 3.
2
31. Solve z + 2rf2 iz + 20 i = 0; express· the roots in the form A + iB.
190

32. If the polynomial x


3 + 3mi + 3nx + r =0 has a double root, prove that:
2 2
(mn- r) 2 = 4(m - n) (n - mr).
3 2
33. Given that the roots of 4x - 24x + 45x - 26 =0 are in arithmetic progression,
find them.
3
34. If a, B, y are the roots of x - x - 1 =0 find the equation whose roots are
l+a l+B !...:!:....r. [Hint: y =
1
+x where x = a , B, y]
r:a.• ~· 1-y 1 - X

35. When a polynomial P(x) is divided by (x - 3), the remainder is 5 and when
divided by x - 4, the remainder is 9. Find the remainder when P(x) is divided
by (x - 3) (x - 4) •
191

CHAPTER 6 CONIC SECTIONS


6.1 Introduction
The subject of conic sections had been studied by ancient Greek mathematicians (200
B.C.). In recent times, it has acquired an added importance in space explorations.
The safest and the most economical path to the planet Mars is an elliptical orbit,
called 'the Hohmann ellipse'. (The Hohmann ellipse is an ideal trajectory requiring a
minimum of energy for a journey between any two planets, named after the German
engineer who calculated it in 1925.)

The conic sections are a family of curves obtained by cutting a right circular cone by
a plane at various inclinations to the axis of the cone.

Two
Branches of
Hyperbola

Fig. I Fig. 2

If the slice is parallel to the base, the curve is a circle, if the slice is parallel to the
slant side of cone, the curve is a parabola; between these two, we have an ellipse.
Finally, any other cut (intersecting also the mirror image of the cone) produces a
hyperbola.
192

We shall only study the CENTRAL CONICS ellipse and hyperbola and their properties.
The case of the parabola has been studied previously (3U Mathematics).

Definition:

A conic is the locus of a point P which moves in a plane so that its distance from a
given point S and its distance from a fixed line,d, always are in the same ratio, i.e.

PS
e
PM
s
The fixed point, S, is called the focus
The fixed line, d, is called the directrix
The ratio, e, is called the eccentricity
If e = I, PS = PM, the conic is a parabola
e < 1, PS < PM, the i::'onic is an ellipse
e > 1, PS > PM, the conic is a hyperbola. d
M
Fig. 3

6.2 Ellipse (e < 1): (Focus and Directrix Definition)

Let P (x, y) be any point on ellipse. y


S is the focus, ZM is the directrix.
P(x, y)
Draw SZ 1 the directrix , and
mark the points A and A' on this
line so that

SA SA' e
AZ
e, A'Z A' 0

Fig. 4

By definition, A and A' are on the ellipse. 0 is the mid-point of AA'. Take the
rectangular axes at 0, as shown.
Let AA' = 2a; S (c, 0), then
SA e. AZ and SA' eA'Z

SA' + SA A'A = 2a (l)

and SA' SA e (A'Z - AZ) 2ae (2)

Adding and subtracting:

SA = a (I - e) SA' a (1 + e)

Then c = OS = OA SA= a - a(l-e) ae


193

oz = a + .!. (a - ae) a
and OA + AZ = e e

s is (c, 0) (ae, 0)

z_ is (~ 0) a
e' PM oz - OD --x
e

PS
Now e (by definition)
PM
2 2 2
SP = e (PM )
(x - ae)
2
+ / = e
2
(~e - x) 2
2 2
(x - ae) + / = (a - ex)

(I - e 2~ x
2
+ /
2 2
a (1 - e )

Let
2
a (1-e)
2

+ ••• (3)

Equation (3) is the equation of the ellipse in standard form.

The point S is called the focus of the ellipse and the line x = ~ is the directrix. By
e
symmetry of the equation of ellipse, we infer that it has two foci S(ae, O) and
S' (- ae, O) and two directrices (See Fig. 5)

y
y d

d' d B

X
Z'

B'

B'
d'

Fig. 5 Fig. 6
194

Important Features of the Ellipse:


2 2
The eJlipse x + ~ = 1 has the following properties. (Fig. 5)
2
a b

1. The curve is symmetric about both axes.

2. The major axis AA' 2a A(a, 0) A'(- a, 0)

3. The minor axis BB' 2b B(O, b) B'(O,- b)


2 2 2
Note that since b = a (l - e ) , e < 1, then b < a.
4. The centre of the ellipse is at 0 (0, 0)

5. Foci: S (ae, 0) , S' (-ae, 0)


a
6. Equation of two directrices are x ± e

lf the foci are at S (0, be) and S (0, -be), the standard form of the equation of the
2 2
2 2 2
ellipse is x + ~ 1, with a = b (l - e ) , a < b
2
a b

and its properties are: (see Fig. 6)

l. The major axis is along the y-axis, BB' = 2b

2. The minor axis is along the x-axis, AA' = 2a

3. The foci are given by (0, ±. be) with centre 0 (0, 0)

4. The directrices are y = .±. eb

WORKED EXAMPLES

Find the eccentricity, foci, directrices of the following ellipses:

2 2
(l) ~5 + ~ = 1

Solution: (see Fig. 5)

a = 5, b =3
2 2 2
b =a 0 - e )
2
9 = 25 (I - e )
4
e = 3
195

The foci are (! ae, 0) = (± 4, 0)


The directrices are X = ±
a
= .± 425 e
2 2
(2) x9 + h = 1

Solution: (see Fig. 6)


For this ellipse, the major axis is along the y-axis.
a = 3, b = 5
a2 = b2 (l - e2)
2 4
9 = 25 (l - e ) gives e
5
The foci are (0, .± be) = (0, .± 4)
The directrices are: y = ± eb = .± 425

Exercise 6A

Find: (a) the centre (b) the eccentricity (c) the foci
(d) the lengths of axes (e) the equations of directrices
for the following ellipses:
2 2 2 2
X L X L
1.
16 +
9 = 1 2.
9 +
16
2 2 2 2
3.
X
+ L 4.
X
+ L
36 25 T6 36
2 2 2 2
5. X
+ L 6.
X
+ L =
25 10 T2 48

2 2
7. 4/ + y = 4 8. 4x + 9/ = I

9/ 2
9. + 4/ - 36 = 0 10. X + 2/ - 1 = 0

Find the equations of the ellipses described as follows.


11. Foci on the x-axis, centre (0, 0) , a =4 , b = 2
12. Foci on the y-axis, centre (0, 0) , a = 2 , b = 5
13. Foci ( ± 4, 0) , b = 3
14. Foci (0, .± /5), a = 3
15. Centre (0, 0), the length of major axis = 10 and length of the minor axis = 6
16. Centre (0, 0), minor axis = 12, focus (0, 4).

17. Eccentricity e =~,major axis= 12


196

6.3 The Hyperbola

Starting with the definition ~~ = e , e > I for the hyperbola and using the
following diagram, you can easily derive the standard equation of the hyperbola as:

X
2
a
2 2
- Lb2 '1 p

2 2 2
where b = a (e - I)
A' 0 X

Fig. 7

2 2
The hyperbola x L Y.
a2 - b2

has the following properties (see Fig. 8)

Fig. 8

I. It is symmetric about both axes.

2. The transverse axis is the line segment joining the vertices A (a, 0), A'(- a, 0) and
AA' - 2a.

3. The foci are S (ae, 0), A' (-ae, 0),' centre 0 (0, 0)

4. The conjugate axis is the segment BB', where B (0, b) and B' (0, -b)

5. The equations of two directrices are x + ~


- e
6. The asymptotes are:

The rectangle through the vertices A, A', B and B', is very useful in drawing the graph
of the hyperbola. Also draw the asymptotes y = ±
b
X a
through the corners of this
rectangle. The hyperbola fits nicely between the two asymptotes.
197

2 x2
The hyperbola given by L - - I is shown in Fig. 9 with all the important
a2 b2
details.

I. It is symmetric with respect to both


axes.

2. The foci are on the y-axis, S (0, ae),


S' (0, - ae).

3. The transverse axis AA' = 2a,


A (0, a), A' (0, - a).

4. The conjugate axis BB' = 2b,


B (b, 0), B' (-b, 0).

5. The directrices are y = .± ~


e
6. The asymptotes are: y = ;!;. 6x

Fig. 9

WORKED EXAMPLES

I. Find the foci, directrices, asymptotes and vertices of the following hyperbolas and
hence sketch them.
y
2 2
(a) ~-L- (See Fig. 10)
16 25 -

(i) a = 4, b = 5
c2 = a2e2 = a2 +b2

c = .J4J or c = - ll4i.
X
The foci are ( ;t v'iii, 0)

(ii) a
The directrices are X = +-
-e

But c = ae

.Jii1=4.eore=
V4l
4
Fig. 10
. . 16
t h e d trectnces are x = ± .~
v41
198

(iii) The vertices are


A (4, 0), A' (-4, 0),

(iv) The asymptotes are y = ± 45 x

(b) (See Fig.

a = 3, b =4
c2 = a2 + b2 = a2e2
5
:. c = 5, e =3
2 X
If we put y = 0, we have x -16
and this shows that the foci lie on
the y-axis •

. ·. the foci are (0, ± 5)


and vertices (!: 0, 3)
The directrices are
9
y = .:!:. 3

The asymptotes are


+ 3
y = - 7f X

Fig. II

Exercise 68

1. For the following hyperbolas, find the foci, directrices and vertices and sketch them:
2 2
2
(a)
X
25 - L
16 = 1 (c) X 4/ = I

(b)
; -
x2
= I (d) 4/ - y2 4
T6 25
2
2. Find the angle between the asymptotes of the hyperbola x
a2
2 2
3. Draw an accurate sketch of x - y = a 2
Find the angle between the asymptotes.
2 2
[The curve given by x - / = a is called a RECTANGULAR HYPERBOLA]
199

6.4 Shape of the Conics

We can study the variation of the shapes of conics


in general by varying the eccentricity e or changing e =0
-
the ratio b/a. c =0 I \_
I
Eccentricity
For 0 < e < I, the conic is an ellipse and
' S,S'

where c is the focal distance given by -


~.
c =

0~ c ~
a>b>O.
If we keep a fixed and vary c over the interval
a, the resulting ellipses vary in shape,
as shown in the diagram.
e

"o·•E ~ 0

For e = 0, i.e. b = a, the shape is circular.

As e increases, the shape becomes flatter


and when e is nearly I, the ellipse reduces
towards a line-segment.
e"O·\C 0 y
e = 0.97
c 0 ::>
e~l

S' 0 s

The orbit of a planet around the sun is elliptical Planet e


in shape, but the orbits of most of the planets
Neptune 0.01
are nearly circular, as can be seen from the table
of eccentricities. Earth 0.02

Jupiter 0.05

Mercury 0.21

The varying shape of an ellipse can also be examined by changing the ratio b : a,
as shown below.

(cont)
200

b 5
E---+---3>
a 3

the variation of the shape of conics as e vanes from zero to a


ver
e >1 hyperbola~

circles

e 1.1

O<e<l

c.~.
------ 0.95
1.4-

E___J
e = 0.97

e ~ I

e parabolas c 5

I
e ..... oo

Note the relative positions of the foci


201

6.5 Parametric Equations of the Circle and the Ellipse


y
Consider a circle with centre at the origin and radius a.
Let P(x,y) be a pomt on the circle and L POx = e.
We have the relations:
X = a COS e , y =a sin e •
The point (a cos e, a sine) lies on the circle X

2 2 2
x + y =a for all values of e. We restrict e
m the range 0 ,< e < 2n, m order to have one-one
correspondence between a point and its parameter.

x = a cos 9, y = a sine are called the parametric


equations of the circle.

Parametric Equations of an Ellipse: Auxiliary Circle

Consider now the equations:


X =a COS e , y = b sin e . y
It is easy to show that the pomt
(a cos 9, b sin 9) lies on the ellipse
2 2
;- + ~ = ' for all values of a,
a b
0 ~ 9 < 2n •
2
a 2cos e
X
i.e. +
a2

or I, which is true.
x = acose , y = bsin9 are then the
2 2
parametric equations of the ellipse ~ + L
2
a b
and e 1s called the eccentric angle and it has a
geometriC meamng as shown m the diagram. It can be shown that the ellipse can be
constructed, pomt by point, by usmg the parametric equations as follows:
we draw two concentric circles w1th radii a and b, a > b. These are
called the major and minor AUXILIARY circles, respectively. To determine a point
P on the ellipse, draw a lme through 0 making any angle e with the positiVe
X-dXIS.
202

Let A and B be the points of intersection of this line with the auxiliary circles.
Through A draw a line parallel to the y-axis, and through B draw a line parallel to
the x-axis. These lines intersect at P where
X ON 0Acos9 acosa
y = NP 0Bsin9 bsin9
The point P(acosa, bsin9) is usually referred to as P(9), or 'the point 9', on the
ellipse.
bsin9 b
. wort h notmg
It 1s . t hat PN
AN a, a constant for all positions of P. This

fact alone is sufficient to determine a point on the ellipse by reducing each


2 2
y-coordinate of the auxiliary circle x + / =a in the ratio b : a.
203

6.6 Equations of the Chord, Tangent and Normal to the Ellipse


y
We consider the ellipse with general point
(acos9, bsin9). The gradient of the line
joining the points r/J and 9 is
bsinr/J - bsin9
m = acos\6 - acos9

2bcos(~) sin( Y.> X

9
-2asin(\6 ; ) sin(\6 Z9)

q, + 9
-bcos(~)
•. m= q, e or ~ cot( 9 + r/J)
a 2
asin(+)

The equation of the chord is then:

- bcos(r/J + 9)
2
y-bsin9= .(x-acos9)
16 9
asin<-f->

or i cos(
9
; r/J) + ~ sin(
9
; r/J) = cos9cos(
9
; r/J) + sin9sin(
9
; r/J)

This simplifies to: icos(


9
; r/J) + ~ sin(
9
; r/J) = cos(
9
2r/J)
The equation of the tangent at 9 is obtained by letting r/J = 9 in the equation
above, giving:

i cos 9 + ~ sin 9

Alternative Method:
The gradient of the tangent at the point P(x , y ) is obtained by differentiating
1 1
2 2
~ + L = 1
2 2
a b
2
-b x
2x b: ~ 0 and ~ = - 2 -1 at P.
a2 + b2 dx dx
a yl

2
-b XI
The equation of the tangent is y - y - 2 - (x - X I)
1
a yl
2
XI
= -2-
a

I, as P(x , y ) lies on the ellipse.


1 1

(cont)
204

So, the equation of the tangent is:

Substituting x
1
=acos9, y
1
=bsin9, we have

xcose + ysin9 = as the equation of the tangent at P(9), in


a b
the form obtained above.

Equation of the Normal


2
-b XI
We have ~ - - , so the equation of the normal at P(x , y ) is
dx 2 1 1
a Yl

2
!..... (x - x 1) , which simplifies to:.
XI

Finally, substituting x
1
=acos9, y
1
=bsin9, this equation becomes:

or

2 2
Note: The corresponding results for the circle x + / a are easily obtained,
either by substituting b=a in the above results or deriving them as above.
2 2 2
We have: for the circle x + y = a , at P(x = acose, y = asin9).
1 1
The equation of the tangent:

I x. 1 • yy 1 • a
2
l <eo•B • y•;no • a

The equation of the normal at P(xl'yl) or P(acos9, asin9):

xsin9 - ycosa
205

6.7 Parametric Equations of the Hyperbola


It 1s easy to venfy that x = a sec 9,
y
y = btan9 are the parametric equations
2 2
of the hyperbola ~ - ~ = 1. We can also
a b
derive the parametric equations of the
hyperbola by using the auxiliary circle
X
x2 + y 2 = a 2 , as s hown m
. t he a d"Jacent

diagram.
The point T lies on the circle, where
LTOx = 9 and the tangent to the circle
at T meets the x-axis at N. PN l_ Ox,
with P(x, y) on the hyperbola.
OT
Now x = ON and from the right-angled llOTN, cos 9 = ON •
So, ON = OT sec 9 = a sec 9.
x=asec9
2 2 2
By substituting x = a sec 9 into !.__ - L
a2 b2
I , we find sec 9 - 7b 2 = 1, i.e.
2 2 2
y = b (sec 9 - 1).
y = b tan 9 since y >0 in the first quadrant.

We note that: =; < 9 <; for the right-hand branch

and .!!.2 < 9 < 311


2 for the left-hand branch.

1. The equation of the chord joining the points P(asec9, btan9) and
Q(asec0, btan0) is

i cos j (9- 0) ~sin~ (9 + 0) = cos~ (9 + 0)

2. The equation of the tangent at P (x , y ) is:


1 1
XXI yyl
a2 - bl
= I ,

or at P(asec9, btan9) is: xsec9 _ ytan9 =


a b
3. The equation of the normal at P(x y ) is:
1 1
2 2
xa yb 2 2
+ - - = a +b
XI y1
2 + b2
or, at P(asec9, btan9) is: xacos9 + ybcot9 = a

I
206

6.8 A Speciai(Rectangular) Hyperbola xy = c2


y
x = ct, y =~ are the parametric equations of
2 t
xy =c
We have y = -X
2
~ - c
-2-
dx
x
SubstitutE; x = ct

~-.:...! X
dx - t2

Hence, the equation of the tangent at


P(ct, I) is given by:

y -
c- = - - 1 ( x - ct)
t t2

or

The tangent has gradient - -1 ,


t
and so the gradient of the normal is i The

equation of the normal is y - I= t 2 (x - ct), i.e. I t3 X - ty = c{t 4 - 1) I


c
The equation of the chord PQ with P(cp, ~) and Q(cq, q) is given by

c c
y - ~ P 9 (x - cp).
p cp- cq

This simplifies to I x · + pqy =c (p + q)

By solving the equations of two tangents at P and Q, namely: x + p 2 y = 2pc and


2
x + q y = 2qc, we find the point of intersection of these to be

R ( p2~p% , /~ q )
The following results are left as an exercise to the reader.
1. The equation of the chord joining P(x , y ) and Q(x , y ) is:
1 1 2 2
2
y- yl = ~ (x- xl)
x 1 x2
2. The point of intersection of the normals at P(cp, ~) and Q(cq, ~) is given by:
p q
X- cqp(p2+l+rq)+C cp3q3+C(p2+~q+l)
- pq (p + q y = pq (p + q
Though the coordinates look formidable, it is a basic and interesting exercise in
;,IPPhr;,ir m;,ninulation!
207

EXAMPLES:
Find, the equations of the tangents arid the normals to the following curves at the given
points:
2 2 n
(1)
T
X
+ h=l, P(-l ,3) (2) x = 2cose, y = 3sin9 at 9=4

2 2 n
(3) X - 2y =l , P(-3,2) (4) x = V2sec9, y = 2tan9 at e =4
SOLUTION: (T = tangent, N = normal)
2 2 2 2y 2
(1) 3x + y = 12 (3) X - 1 , P(-3,2)
6x + 2y ~ =0 •·. 2x - 4y ~ =0
At P(-1,3), ~ = ~ =- ~ at P

The equations of the tangent The required equations are:


and the normal are: 3
y - 2 = - 7i (x + 3) and
y - 3 = X + 1, i.e. y = X + 4 ••• (T)
and y - 3 = -(x + 1), i.e. y = -x + 2 ••• (N)
y - 2 = 34 (x + 3)
i.e. 3x + 4y + 1 =0 ••• (T)
(2 ) ~ _ dy~de _ 3cos9 4x - 3y + 18 =0 ••• (N)
dx - dx dB - -2sin9
For e = ~ , ~ = -~. (4) X =ofi sec 9, y = 2tan9, 9 =%
The required equations are: P(xl'yl) = P(2,2) for 9 =%
P(9) = (2cos9, 3sin9)
2
~- 2sec e = {isece
P(n/4) = ( .[2, 3/ /i) dx - 0/isece tanS tanS
.". y - ~ =- ~ (x - .j2), i.e. At e = ~, ~ = 2
3x + 2y = 6 V2 ... (T)
The required equations are:
3 2
and y - 72. = 3 (x - 112), i.e. y - 2 = 2(x - 2)

4x - 6y = - 5 v'2 ••• (N)


and y - 2 = - ~ (x - 2), i.e.

We can find these by using the y = 2x - 2 ••• (T)


cartesian equation of the ellipse
and x + 2y =6 ••• (N)

/ 2 3 We can also find these equations


4 + ~ = 1 at P( /i., /2) from the cartesian equation of the
hyperbola
2 2
x - \- = 1 at P(2,2)
2
208

Exercise 6C

Find the equations of the tangents and the normals to the following curves at the
points indicated:

1.
2 2 p (1,2)
2x +y =6
2 2
2. 3x + 2y =5 p (-1,1)
3. x =3sin9, y =2cos9 P (9 =n/3)
4. x =4cos9, y =3sin9 p (9 =T! /4)
2
5. 3x - 4/ =24 p (4, - ~)
2
6. x - 2/ = 2 p (-2,1)
2
7. 3x - 2/ = 1 p (l,-1)
8. x = 2sec 9, y =3tan9 P (9 = n/4)
9. x = sec9, y = tan9 P (9 = n/3)
10. xy =c 2 (c is constant) -------EJct
- . -~ £)
t
11. Find the (two) equations of the tangents of gradient ~ to ~h; ellipse l + 6/ = 15.
2 2
12. The point P(x =3, y > 0) lies on the ellipse ~ 5 + ~ = 1. Find the equations of
the tangent and the normal at P. Also find the coordinates of the point in which
this tangent intersects the directrix corresponding to the focus S.

13. Find the equations of the tangent and the normal at P(9,-3) on the hyperbola
2 2
~
4 - fg = 1. The normal meets the curve again in Q. Find the coordinates of the
point of intersection of the tangents at P and Q.
2
14. The line y =2x
- 4 intersects the curve x
3
-y2
=1 at P and Q. Find the
coordinates of the point of intersection R of the tangents at P and Q. What
is the angle between the normals at P and Q?

15. Prove that the line x + y =5 is a tangent to the el!ipse 9x


2 + 16y 2 144.
Find the coordinates of the point of contact.

16. Show that the line 3x + 4y = 10 is a normal to the hyperbola 2x 2 - 3/ = 5


and find the point at which the line is normal.

17. Find the equations of the tangents to the hyperbola 2x


2 - 3y 2 =6 which are
parallel to the line x + y = 0. Also find the points of contact.
209

18. Show that the line lx + my + n =0 touches the


2 2
(a) ellipse
X
2 + L
I ' if a212 + b2m2 = n2
a b2
2 2
(b) hyperbola X
2a - L I' if a212 - b2m2 = n2 •
b2

x2 2
19. The tangent at P (xI' y 1) on the hyperbola a - ~ = 1, x .> 0, intersects
2 1
the directrix at Q. S is the focus (ae,O). Prove that PSQ is a right
angle.

20. Find the equations of the four tangents common to the hyperbola
2
x - 2/ = 4 and the circle l
+ / = 1. Find the points of contact of these
tangents with the circle.
[Hint: Let xx + yy =1 be a tangent to x
2 + y2 1 at P(xl'yl)]
1 1
6.9 Miscellaneous Problems on Conics
x2 2
1. Find the equation of the tangent at P(x , y ) to the ellipse a
1 1 2
+ ~ = 1.

Find the point Q(X, 0) where this tangent meets the x-axis, and prove that Xx
1
= a 2•
· Find the point R (0, Y) where the tangent meets the y-axis. Show that the locus of a
a2 b2
point T (X, Y) is given by the equation
2X + 2y = 1.
y

Solution:

The equation of the tangent at P (x , y ) is


1 1
XXI YYt
-2- + -2- = I. (Prove it)
a b
X

Substituting for Q (X, O) ,

2
Xx = a or .... (l)
1
210

Similarly ••• (2)

To find the locus of T (X, Y), we eliminate x and y by using:


1 1

2 2
Xl yl
-2 +
a 17
If blf
a
+
x2a2 Y2b2

Hence the locus of R is given by the equation

1 •

2. Find the equations of the tangent and normal to the hyperbola 3x 2 - y 2 =3


at P (If, 3{5). The tangent meets the x-axis in M and the normal meets the y-axis in
N, find the length MN.

Solution:

There is no need to remember the eq·Jations of the tangent and normal.


2 2
3x - y = 3 Differentiating

6x - 2y ~ 0 or ~ _ 3x
dx - y

At P(lf, 3/5), ~ If
dx v'5
If
:. Equation of the tangent at P (If, 3/5) is y - 3/5 f5 (x - If)

or lfx - r5y = ••• (I)

For M, put y = 0 •

:. M is M <i , 0)

The equation of the normal is

y - 3 f5 - 415 (x - If)

or r5x + lfy 16f5 ••• (2)


211

For N, put x = 0

N is (0, 4/5)

{ii8i
MN = /-1~ + 80 -4-

3. Find the equation of the normal at P (a sec 9, btan9) to the hyperbola


2 2
x2-~=l.
a b
This normal intersects the x and y axes at Q and R respectively. M (X, Y) is the
mid-point of QR. Find the equation of the locus of M as P varies on the hyperbola.

y
Solution:

The equation of the normal at P is


2 2
axsin9 + by = (a + b ) tan9. (Prove it)

Put y =0 at Q

(a
2 + b 2) tan9 X
X
asin9

Again at R, put x = 0 •

(a
2 2
+ b ) tan9
y
b

M (X, Y), being the mid-point of RQ, is

X
2 2 y 1 2 2
2
a (a + b ) sec 9 , b (a + b ) tan 9
2

Eliminate 9 to find the locus of M

2 2 2
or (a + b ) is the equation of the locus of M. (Another
hyperbola)

2 2
Note: If b = a, then the original curve is x - / =a and the locus of M is
2 2 2
also x - y = a i.e. the same curve!
212

2 2
4. The tangent at P(asece. btan9) on the hyperbola x - ~ = 1 intersects the
2
a b
axes Ox and Oy at M and N respectively. Prove that as P varies on the
a2 b2
hyperbola, the locus of a point Q is given by - :: 1. where OMQN is 2X 2y
a rectangle.

SOLUTION:
The equation of the tangent at
P(asece. btan9) is ~ sec9 - ~ tane ::
at N. x == o. y == -bcote, at M. y == O, x == acose
:. Q(x,y) is (acose. -bcot9) y
:. The equation of the locus of Q. by
eliminating e. is:
X :: aCOS9t y :: -bcot9

~ = sece
X
~y == -tane

2 2
a b 2 2
2 - 2 =sec 9-tan 9::
X y
. a2 b2
1.e. 2 -2 = 1
X y

5. The tangent to the hyperbola xy =c 2 at the point P(ct. ~) intersects the axes
in Q and R and the normal at P intersects the line y =x in s. Prove
that PQ = PR = PS.
SOLUTION:
The equation of the tangent at P(ct, ~) is
x + yt 2. = 2 ct R
Q is (2ct, 0)
R is (0, ~c)
X

We find that P is the mid-point of RQ.


:. PR = PQ ••• (1)
Now the equation of the normal at P is
c 2
Y - t = t (x - ct)

Or t \ - ty = c{t 4 - l) .•. (2)

(Solution continued next page)


213

Solving y =x and equation (2) for S, we have


3 4
x(t - t) = c(t - 1)

X
4
c(t - 1) c~ 2 + 1)
t3 - t t

S is [~ (t 2 + 1), ~ (t 2 + l)J
~ (t2 + l) - ct] 2 + [ ~ (t2 + l) - ~2

SP=~ ••• (3)

(ct - 2ct) 2 + ( ct ,\ 2
- q.,

/ 2t2 c2
:. PQ .jc +- ••• (4)
t2

From (1), (3) and (4): PQ = PR = PS •


214

Exercise 60

Find the equations of the (a) tangent and (b) normal to the following curves at the
points given:

2 2
1.
X
2.5 + 16
l._
1, P(~, u'3>
1
2. 1 , at x
2

3. 2 , at x = 2

1T
4. x = 4cos9 , y = 3sin 9 , 9
4

.5. x = .5 sec 9 , y = 4 tan 9 , 9 = 41T


2 2
6. Show that the equation of the normal to the ellipse x L
a2 + b2

(a)

2
(b) at P(acos9, bsin9) is axsin9- bycos9 = (a - b 2) sin9cos9.

7. Show that the equation of the tangent to the hyperbola


2 2 XXI yy 1
x
2
- L
2
= 1 at P (x , y ) is -
1 1 2
- -
2
= 1
a b a b

and at the point (asec9, btan9) is ~sec a - &tan9


2 2
8. Show that the equation of the normal to the hyperbola ~ - L
a2 b2

(a)

(b) at P(asec9, btan9) is axcos9 + bycot9

2 2
9. P is any point on the ellipse -X + l._ 1, with foci S and S', prove that
a2 b2
PS + PS' = 2a .
----------

215

x2 2
10. P (x, y) is any point on the hyperbola
2a - Lb2 = 1 with foci S and S',

prove that I PS' - PS I = 2a •

11. Find the equations of the tangent and normal to

2 2
(a) the ellipse x + ~ = at x
9

(b) the hyperbola x


2
- / = ~ at x = 3.

2 ~00
12. Find the equations of two tangents to the ellipse 16x + 25/
which are parallel to the line y = x + 2.

2 2
The tangent to the hyperbola ~
13.
6 - ~ = 1 at P(4 Ji., 3) meets the

asymptotes of the hyperbola at A and B. Show that P is the mid-point


of AB. Find the length of AB in the exact form.

1~. Find the equation of the tangent to the curve whose parametric equations are

x = 2cos9 and y = 3sin9 at a= J . This tangent meets the x-axis in


A and y-axis in B. Find the length AB.
2 2
15. Show that the equation of the normal to the ellipse x L =
a2 + b2

2 2
at P(acosa, bsin9) is given by axsin9 - bycosa = (a - b ) sin9cos9.

The normal at P meets the x-axis at M and N is the foot of the


2
perpendicular PN to the x-axis. Prove that MN b cosa
a
x2 L2
16. The tangent to the hyperbola - - = 1 at P(a sect, b tant) meets the
a2 b2
asymptotes in A and B. Prove that P is the mid-point of AB.

2 2
17. The chord through the focus S(ae, 0) of the ellipse x + ~ = 1,
a2 b
at right angles to the x-axis meets the ellipse at P(acos9, bsin9). The
normal at P passes through the end-point B' of the minor axis, of the
ellipse. Prove that:

(a) cos a= e and sin9=~


2 2
(b) equation of the normal at P is axsin9 - bycosa = (a - b )sin9 cosa
2
(c) e 4 + e -1 = 0. Hence, find e in the exact form.
216

2 2
18. The ellipse x + L :: 1 intersects the x-axis at A and A'. Find the
a2 b2
co-ordinates of A and A'. Write down the equations of the tangents at
A, A' and P (x , y ). Let the tangents at A and P intersect in Q and
1 1
those at A' and P at Q'. Prove that the product AQ.A'Q' is independent
of the position of P.

19. Show that the condition for the line y = mx + c to be a tangent to the
. x2 2 . 2 2 2 2
elhpse
2 + ~ :: 1 ts c :: a m + b Prove that the pair of
a b 2 2
tangents from the point P (4, .5) to the ellipse ;.5 + = 1 are at righttr
angles to one another.
x2 v2
20. Show that the tangent to the ellipse - + J- - 1 at P (x y ) has the
a2 b2 - 1' 1
. XXI yyl
equat10n a2 + b2 = 1. This tangent meets the x-axis at T.

PN is perpendicular to the x-axis, and the normal at P


2
meets the x-axis at G. Show that OT x NG = b , where 0 is the
centre of the ellipse.
x2 2
21. The line y = mx + c is a tangent to the hyperbola 25 - f-6 = 1 •

Show that c
2 = 2.5m 2 - 16. The tangents from
P (x , y ) to this hyperbola
1 1
2
meet at right angles. Prove that the locus of P is the circle x + = 9. l
22. P(asece, btan9) and Q(aseccj, btancj) are two points on the hyperbola
2 2
X - ~ :: 1 , SUCh that 9 + f$ :: 90°.
2
a b
Find the co-ordinates of the mid-point R of PQ and hence show that the
2 2
locus of R is given by X - L
a2 b2 b
x2 2
23. P (x , y ) is a point on the hyperbola 25 -
1 1
f-6 = 1 •

Prove that the equation of the tangent at P is 16xx - 2.5yy = 400


1 1
(a) Find the co-ordinates of the point G at which this tangent cuts the
x-axis.
217

(b) Hence prove that ~;p_ = ~~ where S and S' are the foci of the
hyperbola.
2 2
24. Show that the normal to the ellipse ~ + L I at P (x , y ) is given by
a2 b2 1 1

2 2
(a - b ) xI y I

(a) This normal meets the x-axis at G. Prove that GS =e • PS and


GS' =e PS' , where S and S' are the foci of the ellipse.
PS GS
(b) Hence prove that W = GS'

25. Write down the equation of the normal at P (5cos9, 3sin 9) to the ellipse
2 2
~5 + t- = I.
This normal cuts the x-axis and the y-axis at G and H respectively. Show
that the locus of the mid-point of GH is another ellipse with the same
eccentricity as the first. Sketch both ellipses on the same co-ordinate axes.

26. Show that the gradient of the line joining the points P (ct , ~ ) and
1
I
Q(ct ,
2
~ ) on the hyperbola xy = c
2
is t-~ . The points P, Q, R lie on
2 I 2
this hyperbola. The line through P perpendicular to QR meets the line
through Q perpendicular to PR at M. Prove that M lies on the hyperbola
2
xy = c
2 2
27. Show that the line y = mx + c is a tangent to the ellipse x + L = I
a2 b2
2 2 2
a m + b . Hence obtain the quadratic equation satisfied by m
where is the gradient of the tangent from the external point P (x , y ).
m
1 1
Find the locus of P if the two tangents from P are at right angles.

2 2
28. Find the equation of the normal at P (x , y ) to the hyperbola ~ - L = I.
1 1 aa b2
PN is perpendicular to the x-axis, and this normal meets the x-axis at G.
2 2
Show that NG : ON = b : a , where 0 (0, 0).
218

29. Show that the equations of the tangent and the normal to the hyperbola
2 2
x - ~ = 1 at P (asece, btan9) are respectively:
2
a b

(a) bx sece- ay tane = ab , and


2 2
(b) by sece +ax tane = (a + b ) sec9tan9.

The tangent and the normal cut the y-axis at M and N respectively.

Show that the circle on MN as diameter passes throu3h the foci of the

hyperbola.

2 2
30. (a) Show that ab = 2c 2
if the ellipse x L
a2 + b2
2
touches the hyperbola xy = c •

(b) P(x , y ) moves on the line y = mx and Q(x , y ) moves on the line
1 1 2 2
y = - mx. Find the co-ordinates of R, the mid-point of PQ, in terms
of x , x and m. Show that the locus of R is a certain ellipse, if
1 2
PQ = 2K, where K is a constant.

31. Show that for all values of 9, the point P(4cos9, 3sin9) lies on the ellipse
and find the equation of this ellipse.

(a) Find the equations of the tangents at the points P and Q(-4sin9, 3cos9 ).

(b) Find the point of intersection, T, of these tangents and show that as
2
9 varies, the locus of T is the ellipse 9x + 16/ = 288 •

32. The ordinate at P(asece, btan9) meets the asymptote of the hyperbola
2 2
x - ~ = 1 at Q. The normal at P meets the x-axis at G. Prove that
2
a b
GQ is perpendicular to the asymptote.
2 2
(Hint: axtane + bysec9 = (a + b ) sec9tan9 is the equation of the

normal.)
2
33. Prove that the equation of the tangent to the hyperbola x - y 2 =c 2 at
2
P (x , y ) is xx - yy
1 1 1 1
=c . This tangent meets the lines y =x and

y = -x at Q and R respectively. Prove that area of t.OQR is constant.


219

3~. Show that P(acos9, asin9) lies on the circle x


2
+ / a 2 • If P(9)
2 2
and Q({,6) are two points on the circle x + / = a , prove that the
locus of the mid-point of PQ is the line y = V'Jx given that 9 + 1,6 = 23n
for all positions of P and Q •

35. Simplify cos ( 9- + i) and sin( 9 +;). Show that if P(r cos9, r sin9) and
1 1

Q [r 2 cos ( 9 +;), r 2 sin ( 9 +;)] lie on the hyperbola :! -~ = 1 , then

21 + 21 21 - 21 , where 0 is the centre. Deduce that if OP is


r r a b
1 2
perpendicular to OQ, then r l 2 + r 2. 2 is independent of the positions of P and Q.
2 2
36. The normal at P(asec9, btan9) to the hyperbola x - ~ =1 meets the
2
a b
x-axis at G, and PN is perpendicular to the x-axis. Prove that OG : ON = e2 ,
where 0 is (0, 0).
2 2
37. P (x , y ) is any point on the ellipse ~ + L = 1 • Find the equation of
1 1 a2 b2
the tangent at P. A line drawn from the centre 0 (0, 0) parallel to the
tangent at P, meets the ellipse at Q. Prove that the area of t.OPQ is
independent of the position of P. Find the area of t.OPQ.

38. Find the area of the largest rectangle that can be inscribed in the ellipse
2
9x + 25/ = 225.

39. A conic is a rectangular hyperbola with eccentriCity .f2, focus (2, 0) and
2
directrix x = 1. Prove that the equation of this hyperbola is x - / = 2.
Sketch the hyperbola with its asymptotes.
(a) Find the equation of the normal to this hyperbola at a point
P(x 1 , y ) = (/2seci,6, .fitanr/J).
1
(b) This normal meets the x-axis at Q (X, 0) and the y-axis at R (0, Y).
2
Show that the locus of a point M (X, Y) is given by x - / = 8, as P varies.
2 2
~0. The tangent at P(acos9, bsin9) to the ellipse x + ~ = cuts the axes
2
a b
at M and N. Show that M and N are (asec9, 0) and (0, bcosec9)
respectively. Find the minimum value of the area of t.OMN and the
corresponding co-ordinates of P.
220

41.
2
AB is a chord of the curve xy = c , where A is (cp, ~)
p
and B is (cq, ~ ).
q
Find the equation of the chord AB. AB meets the coordinate axes in M
and N and R is the mid-point of AB. Show that OR = MR = NR.

42. Show that the locus of the foot of the perpendicular drawn from the origin to
at the point P(ct, ~) is given by
2
the tangent to the curve xy = c
(x2 + />2 = 4c2xy.
2
43. Find the locus of the mid-points of chords of the curve xy c drawn
parallel to the line lx + my = 0.

44. A tangent at P(ct, ~) to the hyperbola xy = c


2
intersects the axes in A
and B and 0 is the origin. Prove that the area of triangle OAB is
2
independent of the position of P on the curve xy = c •

4.5. The tangents to xy =c 2 at A(cp,


c
p) and B(cq, ~) intersect at R.
q
2
If the chord AB touches the curve xy 4c , show that the locus of R is
2
given by 4xy = c

46. A variable chord PQ of the hyperbola xy = c 2 where P is (x , y ) and Q


1 1
is (x , y ), is such that Ix - x I = 2c. Prove that the locus of the mid-point
2 2 2 1
2 2
of PQ is given by the equation x y = c (x + y).

P(cp, ~) , Q(cq, ~) are two points on the conic xy = c • Show that the
2
47.
p q

gradient of PQ is ~~ . If PQ subtends a right angle at a third point

R (cr, ~) on the conic, prove that the tangent at R is perpendicular to PQ.

48. P(cp, £) and Q(cq, £) are variable points on the conic xy = c 2. Prove that
p q

the tangents at P and Q intersect at T ( ~ , ~ ). Hence prove that if


p+q p+q
pq =k , a constant, then the locus of T is a straight line passing through the
origin.
221

49. PQ is a variable chord of the hyperbola xy = 16, where P

is ( 4p, ~) and Q is 4q, *).


Prove the following:
(a) The equation of the chord PQ is x + pqy = 4 (p + q)
2
(b) The equation of the tangent at P is x + p y = 8p
8
(c) The point of intersection of the tangents at P and Q is T(_!Es_ - -)
p+q'p+q
(d) If the chord PQ passes through the point R (0, 8), show that the locus of
T is a straight line, x =4.
2 2 2
50. P(x 1' y ) is a point on the rectangular hyperbola x - y = a • S and S' are
1
the foci. Prove the following:
(a) The eccentricity e = Vi
(b) IPS I= a- .fix 1 , ls'PI =a+ .fix 1

(c) PS • PS' =OP 2, where 0 is the origin.


222

6.10 Tangents and the Chord of Contact


Many properties of the two central conics, the ellipse and hyperbola, are so common
that it is convenient and instructive to treat them as one by writing the equation in
the form
2 2
~ + 1r- = 1 ••• (1)

where A =a 2 and B =b2 for the ellipse but B =- b2 for the hyperbola.
. xx1 yy 1
The tangent at P (x 1' y 1) IS A + ---a- = 1 ••• (2)

To find the condition for a line to touch the conic, the line:
lx +my + n =0
must have the form of the tangent (1). Comparing coefficients:
X/A y1/B - 1
-,- = ---m- = --n
-Bm
xl = -n-
-AI
' --n
But P (x , y ) lies on the conic (1 ), so that:
1 1
2 2
AI Bm 1 , which simplifies to:
- 2 + -2-
n n
2 2 2
A1 + Bm n ••• (2)
This is the condition for the line lx + my + n =0 to be a tangent to the conic
2 2
XA +~=1.
If the equation of the tangent is expressed in the form y = mx + b, the condition
becomes:
2 2
Am + B = b ••• (3)
This is left as an exercise to the student.

Chord of Contact:

The chord PQ, jommg the points of the contact of tangents drawn to the conic from
an external point T(x , y ), is called the chord of contact.
1 1
223

y
Let P(x ,y ) and Q(x ,y ) be the points of
2 2 3 3
contact of tangents drawn to the conic from
T(x l' y ). The tangent at (x , y ) to the conic
1 2 2
x2 2 xx 2 yy 2
A + 1f- = 1 is "A" + B"" '"' l and

T (x l' y ) lies on it, so


1
xlx2 Y1Y2
-A-+ - B - = l ••• (1)

Similarly, for the tangent at Q(x ,y ),


3 3
xlx3 Y1Y3
-A-+-B-=l ••• (2)

(1) and (2) clearly indicate that the points P and Q lie on the line
XX I yy 1
T + r= 1 ••• (3)

which is therefore the equation of the chord of contact of T (x , y ).


1 1
Though the equation of the C.O.C. is the same form as that of a tangent, it must be
remembered that T(x l' y ) does not lie on the conic.
1

The method shown above can be used to find the equation of a chord of contact to
. 1.e.
any come, . a para bl
2 . Ie x 2 + y 2 = a 2, a h yper bl
o a x = 4 ay, a Clrc o a xy = c
2

etc. It is left as an exercise for the student to prove the following: (Chord of
contact = COC)
2 2 2
1. The equation of the COC, from P(x I' y ), to a circle x +y =a is
2
1
xx + yy =a .
1 1
2
2. The equation of the COC, from P(x I' y ), to a parabola x
1
= 4ay is
xx = 2a(y + y )
1 1
2 2 2
3. The equation of the COC, from P(xl'y ), to a rectangular hyperbola x -y = a is
1
2
XX l - yy} = a •

4. The equation of the COC, from P(x I' y I), to a (special) rectangular hyperbola
xy = c
2 IS
. xx I + yy = 2c 2•
1
224

6.11 Geometric Properties of the Ellipse

The standard equation of the ellipse


2 2
is ; - +
a b
7 = I; Centre 0.

Foci: S(ae,O), S'(-ae, 0)


X

Directrices: x = .! ~
e
Length of semi-major axis =a B'
-a a
Length of semi-minor axis =b X=-
e X
e
Vertices: (.!a, 0), (0,.! b)

The following properties of the ellipse are proved here:


I. The sum of the focal lengths is a constant, i.e. SP + S' P = 2a
PS PS'
We have PM =e and PM' =e (Definition)

.". PS + PS' = e (PM + PM') = e • MM'

But MM' = The distance between two directrices = 2ea


2
:. PS + PS' = e . a = 2a , which is a constant
e
This fact gives us a fast, accurate (and inexpensive) method of drawing
the ellipse.
A thread of length 2a, fastened at S and S' is kept tightly stretched by a
pencil at P. As the pencil moves, it traces out an ellipse. (See the diagram
above)

2. (a) The normal at P bisects the angle between SP and S'P.


(b) The tangent at P is equally inclined to SP and S'P.

These properties are equivalent and we only have to prove one of them. A
purely algebraic method is too laborious, so we use the following geometric
result. (The proof is given in the appendix)
If AM is the internal (external) bisector of LBAC of l1ABC, then
AB BM BM'
AC = MC ( = M'C )
D

...........
..... ...
'
..... .......
...
M'
225

The normal at P(xl'yl) is y

2 2
~ - l2._ = a 2 - b 2 = a 2 e 2 , and meets M
X1 y1
2
the x-axis, where y = 0, in G(e x ,o)
1
T
2 X
Then, SG = OS - OG = ae - e x = e (a -ex )
1 1
SP
Now PM = e (definition)

SP = e • PM = e(NM - NP) = e(~- x ) a


1 X=-
e
SP =a - ex
1
(Note: this result is very
useful)
L-----------------------
SG =e. SP
Similarly S'G =e • S'P
SG SP
S'G = S'P
PG bisects the LSPS'
Thus, the normal at P bisects the angle between SP and S'P.

(b) Since the normal PG J. the tangent T'PT,


LGPT' = LGPT (both are 90°) and LS'PG LGPS
LGPT' LS'PG = LGPT - LGPS
LT'PS' = LTPS
Hence, the tangent at P is equally inclined to SP and S'P.
This is the reflecting property of the ellipse. A ray of light or a sound wave
originating from the focus S, will be reflected through the other focus S'.

3. The chord of contact from a point on a directrix is a focal chord.

Proof:

Let T(xl'yl) be a point on the directrix,

a
SO that Xl =e .
y
The equation of the chord of contact
XXI yy 1
is -2- + -2- =I.
a b
X yy 1
For PQ, this bec.omes - + -- - 1
ea b2 - ·

PQ meets the x-axis where y = 0, i.e.


x = ae, which is the focus S(ae, 0).
Hence the result.
226

II. That part of the tangent between the point of contact and the directrix subtends
a right angle at the corresponding focus.
Proof:
Using the diagram from property 3, we have to prove l PST = 90° •
· xx
Th e tangent at ·p( x ,y 2) IS
2
2 2
7
a 2 + yy = 1 an d t h.IS meets t he d.1rectnx
· m
·

T( ~, k), where
e

••• (1)

y2
The gradient of SP is m =x - ae
2
I k ke
The gradient of ST is m = a
-- a(l - e 2 )
e ae
aeky
ke 2
mm' = 2 = 2 2
a(l - e ) a (1 - e ) (x - ae)
2
Using (1), we have, mm' =- 1
:. LPST =90° and this proves the result.

The ellipse possesses a wealth of useful and interesting properties. Some of these
follow simply from the definition and. others can be proved by co-ordinate geometry
and plane geometry. The reader who masters the general techniques of proving these
properties will have no trouble in proving the same properties when particular values
of a and b are used.
227

6.12 Geometric Properties of the Hyperbola

y
d' d

0 G X

X=-
-a
e X = -ea

Many properties of the hyperbola are similar to those of the ellipse, so to avoid
repetition, the properties are stated without proof. lt would be instructive for the
reader to supply the proofs referring, if necessary, to the corresponding results for
the ellipse.

1. The difference of focal distances is constant.


5'P- 5P = 2a, if P is on the branch near 5, and
5P - 5'P = 2a, if P is on the other branch
i.e. I5P- 5'PI = 2a
2. The tangent at P bisects the angle 5P5' internally and hence the normal at
P bisects L5P5' externally.
(Using the equation of the tangent at P, find the co-ordinate of T and hence
5T 5P
prove that ,T = ,p • )
5 5
This is the reflection property of the hyperbola. A ray L P of light directed
towards the focus 5 of a hyperbolic mirror, is reflected towards the other
focus 5'.
3. The chord of contact from a point on the directrix is a focal chord.
In the figure, RP, RQ are tangents from R on the directrix.
4. That part of the tangent between the point of contact and the directrix subtends
a right angle at the corresponding focus ( L P5R = 90°).
228

6.13 Properties of the Rectangular Hyperbola


y
y
2
2 a
=c =y

2 2
=a

Fig. 2

In this section we investigate the geometric properties of two special hyperbolas


2 2 2 2
x - y =a and xy =c .
2 2
2 2 2
If b = a, then the equation ~ - L = I reduces to x - y = a and the
a2 b2
asymptotes are y = .:!: x, i.e. x + y = 0 and x - y = 0, which are perpendicular to
each other. A hyperbola whose asymptotes are perpendicular is called a
RECTANGULAR hyperbola (Fig. 2).
The reader is familiar with the equation y = ~ , i.e. xy = k, which represents a
hyperbola (Fig. I) with the axes as asymptotes. Thus xy =c 2 is also a rectangular
hyperbola, but the reader who is not satisfied by this reasoning should refer to two
2
a
explanations given in the appendix, where it is shown that the hyperbola xy = is
2
2 2 2
obtained by rotating the hyperbola x - y a through 45° about the origin. We
. 2 a2 . .
wnte c =
2 , to Simplify our work.
2
The eccentricity of both hyperbolas is given by b = a 2 (e 2 - I) and with b = a,
2 2
this gives e = {2. Obviously most of the properties of hyperbolas ~- L I,
a2 b2
2 2 2 2 2
x - y = a and xy = c are the same, but a few peculiar to xy = c are
given below with proofs. In science and engineering, it is the form xy = k which is
2 2 2
useful rather than x - y = a • The law connecting the pressure and volume of a
perfect gas under constant temperature is pv = k, and in electricity the law
connecting the current C, the resistance R and the E.M.F. V is given by CR = V.
229

6.14 Geometric Properties of the Rectangular Hyperbola xy = c2

1. The area of the triangle bounded by a tangent and


the asymptotes is a constant.
y
Proof:
The equation of the tangent at any point
P(ct, r>
on the conic xy = c
2
is:
2
x + t y = 2ct
B
This meets the axes where

OA = x = 2ct , OB = y = t2c
0 X
1
Area of the t.OAB = OA • OB
2
1 2c 2
=2 . 2ct . t = 2c = a constant.

2. The length of the intercept, cut off from a tangent by the asymptotes, equals
twice the distance of the point of contact from the intersection of the
asymptotes.
Proof:
In the diagram, we want to prove AB = 20P. Using the previous example:

P{ct, r) , 2
A(2ct, 0), B(O, tc )

... (I)

••• (2)

From (I) and (2):


2 2
AB = 40P which gives AB = 20P.
230

Worked Examples: Geometric Properties

EXAMPLE: ( 1)
2 2
Find the equation of the tangent to the ellipse 5x + 9y = 45 at the point
P(2, j>. Find the coordinates of the foci S and S'. SV and S'V' are the
perpendiculars to the tangent at P. Prove that: V and V' lie on the circle
2 2
x + y = 9 and SV • S'V' = 5.
y
SOLUTION:

The equation of the ellipse is T


2
+ +2
1.

The equation of the tangent at P(2, j> is

2x 2l l ( . XX 1 yy 1
v
9 + 15 = • usmg -2- + -2- = I]
a b
or 2x + 3y =9 ••• (1) X
2 2
We have a = 9, b = 5, then
2 2 2 2 .
a e = a - b g1ves ae = 2, so the foci are: S(2, 0) and S' (- 2, 0)
SV J. to the tangent (1), so the equation of SV is 3x - 2y " 6 ••• (2)

Solving (l) and (2), we find V( ~; , : ~)

S'V' J. to the tangent (l) and its equation is 3x - 2y = -6 ••• (3)


Solving (l) and (3): V' (0, 3)
2 2
V' (0, 3) obviously satisfies the equation x + y = 9.
Substituting for V in the L.H.S. of this equation:
2 2
3 X 13
----,~-
L.H.S. =
13
2 = 9 = R.H.S., and

2 2
hence both V and V' lie on the auxiliary circle x + y = 9.

( ~; - 2)2 + ( : ~ )2 = ( : ~ )2 + ( : ~ )2 = ~~~
2
Now SV

and s•v• 2 (- 2) 2 + (3)


2
= 13
2 2 325
SV • S'V' = 169
. 13 = 25 g1vmg SV • S'V' = 5

This example is a particular case of the general property of the ellipse


2 2
x Y = I, i.e. 2 2 2
V and V' lie on the auxiliary circle x + y = a and
7+~
2 2 2
SV . S'V' = b . For this example a = 9, b = 5.
231

EXAMPLE: (2): Prove that the portion of


the tangent at P (x 1' y ) to the hyperbola
1
2 2
7a - b~ = 1 intercepted by the

asymptotes is bisected at the point of


contact.

SOLUTION:
X
The equation of the tangent at P(x , y ) is
1 1
XXI yyl
-2--2= 1 ... (l)
a b
Let R and Q be the intersections of the
tangent with the asymptotes.
We find the quadratic equation which gives
the ordinates of R and Q. The equation
2 2
of the asymptotes is 2X ~ = 0. ... (2)
a

From (I) or X =

2 2
2 2yyl y yl
Substituting into (2): a (l )
~ +7+~
2 2 2
2 a YI I 2YY Ia 2
a
Re-arranging: y ( -
4 b2 - -b 2) + b 2 2 + -2 0
XI XI XI
This equation gives the ordinates of R and Q.
Let M(X, Y) be the mid-point of RQ.
If the roots are y and y , then:
1 2
2 2 2 2 2 22
yla 1 - b x1 ~ - a b y1
a y
(
- -b2 2 - 42 / 22 22
XI bx ay -bx
1 1 1
2 2 2 2 22
Since P(x , y ) lies on the conic, a y - b x -a b
1 1 1 1
:. y = yl 2
Xxl yl
Then using (I), since M(X, Y) lies on the tangent, we have - 2 - -y=
2 2 2 a b
XI yI Xx XI
1
But - - - - - I so - - = - - giving X= x
a2 b2 2 2 1
a a
Thus M (X, Y) = (x , y > which proves the result.
1 1
232

Exercise 6E: PROPERTIES OF CONICS

2 2
I. The tangent at P (xI' y ) to the ellipse 9x + 16y 144 meets the directrices
1
at T and T' respectively. S and S' are the foci. Prove that L PST and
LPS'T' are both right angles.
2
2. Show that the equation of the normal to the ellipse 16x + 25/ 400 at the
point P(x ,y ) is 25xy - 16x 1y ~ 9x 1y1 •
1 1 1
(a) The normal meets the x-axis at G. Prove that GS =e . PS and
GS' =e . PS', where S and S' are the foci.
PS GS .
(b) Hence prove that PS' = GS' and that PG btsects the LS'PS.
3. V and V' are the feet of the perpendiculars from S and S' respectively to
2
the tangent at P(x I' y ) to the ellipse 4x + 9/ = 36. Prove that:
1
(a) SV • S'V' =4
2 2
(b) V and V' lie on the auxiliary circle x + y = 9.
2 2
4. M is the mid-point of a variable chord PQ of the ellipse 16x + 25y 400,
where P is (x , y ) and Q is (x , y ). Prove that the product of the
1 1 2 2
gradients of PQ and OM is constant.
22 22 22
5. NP is the ordinate of a point P(x , y ) on the ellipse b x + a y = a b .
1 1
2
The tangent at P meets the x-axis at A. Prove that ON . OA = a , where 0 is
the origin. 2 2
6. P(x , y ) is a point on the ellipse ; - + ~ = 1 and Q is the point on the
1 1
a b
2 2
·1e
ctrc x + y =a 2 h ·
avmg t he same a b .
sctssa. P
roveht at t he tangents at P
and Q meet on the x-axis.
2 2
7. The tangent at a point P(x I' y ) on the hyperbola 16x - 25y = 400 meets
1
the directrix at T. Show that L PST = 90°, where S is the corresponding
focus.
2 2
8. NP is the ordinate of a point P(x I' y ) on the hyperbola
1 7 -~ = I.
a b
2
The tangent at P meets the x-axis at L Prove that ON . OT = a , where
0 is the origin.
2 2
X v
9. P(xl'y 1> is a point on the hyperbola } - ~ = I with the focus at S.

If PS is parallel to the asymptote, prove that the directrix, the asymptote


and the tangent at P are concurrent.
233

10. PQ is a chord of a hyperbola 9x 2 - 16y 2 = 144 passing through s. The


tangents at P and Q intersect at T. Prove that T lies on the directrix
corresponding to the focus S.
11. Prove that the point of intersection T of the tangents at P(cp, ~) and

Q(cq' £q ) on the hyperbola xy = c 2 is given by T ( ~ , .1£._) • OT


p+q p+q
produced meets the chord PQ at R. Prove that PQ is bisected at R.

12. Prove that the portion of the tangent at P(l, -1) to the hyperbola
2
Jx - 2y 2 = 1 intercepted between the asymptotes is bisected at the point of
contact.
(Hint: Use the equations of asymptotes Jx 2 - 2i = 0 and the tangent
Jx + 2y = 1 to find a quadratic in either x or y.)
234

CHAPTER 7 ELEMENTARY PARTICLE


DYNAMICS
7.1 Introduction
Dynamics is the branch of Mechanics (Physics) that deals with the conditions
under which bodies move. The other branch of mechanics is called
STATICS, which deals with bodies at rest or under equilibrium under the action of some
forces.

Two branches of Dynamics are called:

1. KINETICS: Kinetics is the study of effects produced by forces acting on the


bOdies in motion.

2. KINEMATICS: This deals with the motion of the body without regard to the
cause, effect or result of the motion. So far in our work, we have done just
that, i.e. the motion of a particle in straight line, the motion of a projectile,
the SHM etc. We discussed the motion in terms of the position, velocity, time,
acceleration. It did not matter which forces caused this motion.

We shall now introduce "the elements of KINETICS" which relate the forces with the
motion of the body. This not only enhances our knowledge of the subject, but we can
now solve a wide variety of motion problems, such as the motion in a RESISTING
medium, the motion in a circle etc.

Remembering that the 4- Unit Mathematics syllabus requires us to study not only the
harder new topics, but also 3U-Maths harCier motion problems, we shall first completely
summarize the previous work, then revtse with harder 3U problems, and then extend to
the required new topics.

7.2 Laws of Motion - Force

In everyday life we use force to pull or push an object. In this chapter we study the
cause-effect relation between the observed motion and the system of forces.

Newton (1642-1727), one of the most famous and greatest scientists, formulated laws of
motion after studying the motion problems which involve application of natural
(gravitational) or mechanical (push, pull, friction) forces.

Newton's First Law of Motion. (Inertia)

A body remains in a state of rest, or of uniform motion in a straiRht line (a


constant velocity, no external force) in the absence of a force.
235

A force is an invisible entity, it is recognised only by its effect, so it is a CONCEPT.


A heavier object requires a greater force to move than a lighter object, hence we say
a heavier object has greater inertia.

The First Law introduces us to the idea of a Force and mass (inertia), i.e. the definition
of what is a force.

Newton's Second Law of Motion

This law relates the change in velocity i.e. acceleration with the magnitude of the
force that produced the motion. It states that:

A force acting on a body produces an acceleration which is proportional to the


magnitude of the force and this acceleration is in the direction of the force.

The mathematical form of the second law is:

Force Mass x Acceleration


F = m a
The mass m is a measure of the amount of material and hence the inertia of the body.

In the Sl units:

Mass is in kilograms (kg)

Acceleration is in m.s- 2

Thus a force of 1 N acts on an object of mass of 1 kg, the object accelerates by


1 m.s-2•

Mass and Weight of a Body

The weight is the force acting on an object of mass m, due to gravity.

The value of g, the acceleration due to gravity is 9.8 m.s- 2 (at earth's surface).

Weight = m x g
W (newtons) = m(kg) x g (m.s- 2)

A weight of 1 kg is equal to 9.8 N.

Never confuse the weight with the mass.

Weight is a force and hence a vector, but mass is a scalar quantity.


236

Newton's Third Law of Motion:

This law states that the force exerted by one body on another body i.e. action force, is
equal to the force exerted by the second body on the first, the ·reaction force, and they
are opposite in direction.
N
When you kick a f.oot ball, you are
applying a force on the foot ball, and
at the same time, the ball's reaction
mg
applies an equal force to your foot,
it hurts! Fig. I
A body of mass m lying on a horizontal smooth surface is pressing the table with a
downward force of its weight, but at the same time the equal and opposite force N
acts on the body. This force is the reaction in the direction at right angles to the
surface.

WORKED EXAMPLES

EXAMPLE 1:

A body of mass 10 kg is suspended by a string from a ceiling.


Find the tension in the string.

SOLUTION:

You need not show the entire system, only the forces acting on the particle.

The two forces acting on the particle are the T


tension T, upwards and the gravitational
force mg , downwards.

We use Newton's second law:

The resultant force = mass x acceleration. mg


Fig. 2
T - mg = m x O, because the system is at rest, hence,
acceleration = 0
T = mg
= 10 9.8
X m = 10 kg, g = 9.8 m.s-2
= 98 kg. m.s-2
= 98 N (I N =I kg. m.s- 2 )
237

EXAMPLE 2:

A particle of mass 20 kg is suspended by two strings. Calculate the tensions in the


strings, shown in the diagram. (Acceleration due to gravity g = 10 m.s-2).

SOLUTION:
y
The forces acting on the particle P

are the two tensions T and T


1 2
and the gravitational force mg.

Fig. 3

We select the rectangular coordinate system at P and decompose (resolve) the system
of forces into the horizontal components Fx and the vertical components, F Y

l:Fx = T 1 cos 4.5° - T2 cos 60°

lF y = T 1 sin 4.5° + T2 sin 60° - mg


The upward components are considered positive and downward components negative.

Since there is no acceleration in any direction, we have:

l:Fx = 0 and lFY = 0


T l cos 4.5° - T2 cos 60° = 0 and •••• (1)

T 1 sin 4.5° + T2 sin 60° - mg = 0 •••• (2)

Remembering, cos 4.5° = sin 4.5°, subtract (1) from (2)

T (sin 60° + cos 60°) = mg


2
substitute m = 20, g = 10

200 146 N
T2 = sin 60°+ cos 60° =
From (1):

T I cos 4.5° = T2 cos 60°

... Tl =
146 cos 60°
cos 49 = 104 N
238

Note: It is very important that:

1. A free-body diagram is drawn, showing all the forces on the particle.

2. The proper resolution of each force into two components at right angles to
each other is shown.

The two perpendicular directions need not always be the HORIZONTAL and the
VERTICAL directions,

3. If there is no acceleration, i.e. the system is at rest, only then is:

IF X = 0 and IF y = 0.

Normally write, F x = m.ax and F y = m.ay' and then substitute for ax


and a , for each problem.
y

EXAMPLE 3:

A truck of mass 3 tonnes is descending an inclined plane at a speed of 20 m/s. Find


the retarding force R, necessary to stop the truck in 30 m. (Angle of the incline is
10°).

SOLUTION:

The forces on the truck are:


X
Normal reaction N, perpendicular to plane.

Retardation force R, along the incline.

mg, force due to gravity.

Angle of incline = 10°

Resolving the forces along and at right-angles to the plane:

R - mg sin 10° ••• (1)


and N- mg cos 10° ••• (2)

Now the net force on the truck is along the incline, given by m.a , where a is the
retardation to be calculated using:
2 given u 20 m/s
u + 2ax

0 2 30m
20 + 30 x 2a X

a 2
-6 J m.s
-2
20
IF x rna = 3000 x = 20000 N
3
From(l), 20000 = R- 3000 x 10 sinl0°, giving R • 25200 N.
239

Exercise 7A

In the following examples, take g :: 10 m.s 2

1. A particle of mass 10 kg is suspended by two strings of length 3 m and 4 m


attached to two points at the same level S m apart. Find the tensions in the
strings.
2
2. The combined air and road resistance of a car in motion is proportional to v ,
where v is its speed. When the engine is disengaged the car moves down an
incline making an angle sin- I (l/30) with the horizontal, with a velocity of 30
m/s. Find the force required to drive the car up the incline with a steady speed
of 24 m/s, g!ven that the mass of the car is 1200 kg.

3. A truck of mass M is driven up a road inclined at an angle 9 to the horizontal.


After its speed has reached u m/s the engine continues to exert a constant force
of F newtons. If the resistance R is constant, find the time taken to reach the
velocity v m/s.

4. A car of mass 1SOO kg is moving at 60 km/h; when the brakes are applied with a
braking force of 12000 N.
Find: (a) the acceleration
(b) the time taken by the car to stop
(c) the distance travelled before coming to rest.

s. A body of mass m is pulled up a smooth incline making an angle 9 with the


horizontal, and has an acceleration f. Find the force F that pulls the body.

6. A mass of 10 kg is pulled along the horizontal by a chain making an angle of 30•


with the horizontal. If the tension in the chain is SON, find the acceleration of
the body and the magnitude of the normal reaction.

7. A smooth block of mass 2 kg slides down an incline making an angle of tan- I (3/4)
with the horizontal. Find the acceleration and the magnitude of the normal
reaction.

8. A truck of mass 2000 kg starts to climb an incline of angle given by


1
9 = sin 0/JO). The total resistive force is 2000 N.
Find the retardation it experiences.

9. Find the magnitude of the braking force to stop a car of mass 1200 kg in 20 m
when it is travelling at 60 km/h (a) on a horizontal road (b) down an incline of
an angle
sin-l (1/40).
240

CHAPTER 8 MOTION PROBLEMS IN


TWO DIMENSIONS

We shall first completely summarise the results of various types of motion studied so far
in 3U Mathematics, then revise the harder problems of simple harmonic motion and
projectile motion, then embark on 4U Motion.

8.1 Introduction: Motion In a Straight Une

Displacement = x
(x, t, v)
Velocity = v dx
dt : X
0 p X

Time = t Fig. 1
2 2
Acceleration a = ••x ..
dx = dlv)
'd;2 dx \ 2 = v
dv
dx

For constant acceleration ONLY, the equations of motion are:

u initial velocity

a constant acceleration

I. v = u + at
2 2
2. v = u + 2ax
2
3. X = ut + !2 at
Do not use these formulas for the variable acceleration

Method of Solution (variable acceleration)

I. Given x = f(t), integrate

X = I f(t)dt = g(t) + c

x I g(t)dt + ct

2
2. Given x f(x), use ~x ( T) = f(x) and integrate.
241

3. .
G1ven ••x = f (v ), use v dv
dx = f (v ), and mtegrate
.
y
For VERTICAL MOTION under gravity only replace a, by, g
acceleration due to gravity. x may be replaced by y.

I 2
I. y ut - 2 gt p

2.

3.
v

v
2
u

u
2
-
-
gt

2gy Fig. 2

mg

0
For downward vertical motion under gravity, assuming the
object falls from the rest, we have: (u = 0)
1 2
y 2 gt

8.2
v

v
2
gt

2gy

Simple Harmonic Motion (Revision)


Fig. 3
f
Definition:

A particle M on a straight line is said to perform a SHM if its displacement X


satisfies the differential equation
y
2
d x 2
-n x
2
dt

X
A' 0 M A X

Fig. 4

The acceleration is always directed towards, and proportional to the displacement from
the centre.

1. The general solutiOn of -


ix
2
dt

x = a cos (nt + a), where a is the amplitude of the SHM,


n and a are constants.
242

2. If the motion starts at A, where x +a, t 0.


substitute in x a cos (nt + ex)
a a cos ex
cos ex giving ex = 0
x a cos nt
3. X A cos (nt) + B sin (nt) is also a general solution.

Details of the motion:


2
Let x = a cos (nt + ex) be the solution of x -n x.

(i) The greatest displacement given by the function


x = a cos (nt. + ex) is Ix I a and is called the amplitude of SHM.
2n
(ii) The period of SHM is T
n
dx -an sin (nt + ex)
(iii) The velocity v = dt

The maximum velocity is v = ±an


v + an for the particle moving to the right.
v an for the particle moving to the left.
2
2 v
2 v 2
(iv) We have: X + 2 a Fig. 5
n
2 n2 (a2 - x2)
or v
2
Since v 3> 0, then lx I ~ a
2
The graph of v shows that v is greatest
at x = 0 and v = 0 at x = ±a.

Hence the particle oscillates between two extreme positions x ±a. This is
why it is also called an oscillatory motion (-a< x <: a).
243

(v.) It is instructive to see that the SHM is closely related to a circular motion of a
point P(x,y), centreO(O,O), radius a. (See Fig. 4)

We have: 9 nt
X a cos nt y a sin nt
x -na sin nt y an cos nt
2 2
X -an cos nt y -an sin nt
2 2
X -n x y -n y

Both components of vector OP perform S.H.M. We study more of this in the


chapter on CIRCULAR MOTION.
As the point P moves on a circle from initial position at A(x = a), its
projection M on the x-axis moves toward 0.

M is at A' (x = -a) when P is halfway round the circle. As P continues to


describe the negative half of the circle, our point M retraces its path towards
the initial point A. Thus as P moves on a circle, its foot of perpendicular
PM, i.e. M describes to and fro motion about the centre of attraction, 0.

EXAMPLE: (I)

A particle P is in the x - y plane and its coordinates satisfy the equations


2
d x 2 d 2v 2
-n x and :::.....L = -n y •
2
d/ dt
When P is at (5,0), = 0 '
dx
dt 0 ' ~ = 4n
dt
2 2
Prove that the locus of P is the ellipse X + L
25 16
Sketch the graph of the locus.

SOLUTION:
2
d x 2
-n x
2
dt

d~ ( ; )= 2
-n x. lntegrat ing

2
v
2 - 2I n 2 x 2 + c

at x = 5 , 0
Fig.6

~~ = -n;:;;-7, of negative sign because at A velocity is towards 0.


244

dt - I
n dx =
-J X
cos 5 nt + a

For t = 0 and x = 5 , we find a = 0


x = 5 cos nt • Similarly, y = 4 Sill nt.
we write these as: cos nt = x/5
Sill nt = y/4
2 2
X L
25 + 16 = I
Hence the locus of P is an elhpse. (Shown in Fig. 6) with centre at (0,0) and semi-axes
5 and 4.

· Exercise SA

I. The rise and fall of the tide at a certaill port may be considered as simple
harmomc, the time difference betwf:'en successive high tides being I 0 hours. The
harbour entrance has a depth of 20 m at high tide and 8 m at low tide.
If the low tide occurs at 10.00 a.m. on a certain day, filld the earliest time that
a cargo ship requirmg a minimum df:'pth of 15 m of water can pass through the
entrance.

2. The acceleration of a body movmg along the x-ax1s IS given by

-x for x -$ I

2 - x for x >I •
2
(a) Show that the quantity ~ ( ~~) + f(x) is a constant of motion, where

2
x I 2
f(x) =
2 or 2 (x - 2) accordillg as x-$ I or x > I •
(b) When t = 0 , x = 0 and the initial velocity of the body is 1/3 in the
positive direction. Determme the extreme points of the suosequent
motion, hence prove that the motion is S.H.M.

(c) If in Exercise (b),


v = 2, find the extreme points of the motion. Explain
whether the motion now is SHM or not.
245

3. Show that x =r cos (wt + rJ) where r , w , rJ are constants, is a solution of


2
d x 2
- = -wx.
2
dt

A small naval target rises and falls with SHM of period 10 seconds; the height
of the waves from the crest to trough is 2 m.

At a horizontal range of 2000 m a gun is fired so that the target would be hit
provided it remains stationary in its highest position. The horizontal component
of velocity is 1000 m/s. Show that the target would be missed by a vertical
height of approximately 0.69 m.

4. A particle moves in a straight line Ox and its equation of motion is


2
d x
- = 9- 9x.
2
dt
Prove that the displacement at time t is given by

x = 1 + ~cos(3t- 127°), given that v = 4 m/s, when x = t = 0.


What is the centre of oscillation?

Find the amplitude and the maximum velocity.

5. The velocity v of a particle moving along the x-axis is given by


2 2
v = 12 + 8x - 4x , where x is the distance from the origin 0.

Prove that the motion is SHM and find

(a) the amplitude

(b) the centre of oscillation

(c) the period.

6. P, Q and R are three points on the x-axis such that PQ = QR = 2 m. A


particle performs a SHM along the x-axis and is observed to have the velocities
of 12, 10 and 6 m/s at P, Q and R respectively. Taking the origin at the
centre of oscillation, find

(a) the constant n in the equation x = -n 2x


(b) the distance of P from the centre

(c) the amplitude.


246

8.3 Motion of a Projectile (Revision)

A particle IS projected from a point 0


in a vertical plane, at an angie of a
to the horizontal, with velocity v. y

The only acceleration acting on the


particle is due to gravity, hence
the equations of the motion are:

2
d v
0 and =-.L = -g
2 Fig. 7
dt

Initially, X = 0 = y = t VI v cos a (horizontal component)


'
v2 v sin a (vertical component)

Integration gives

dx ~
dt
v cos a dt
v sin a- gt ••• (I)

X (v cos a) t y (v sin a) t - ~ gl ••• (2)

Eliminate t from (2):

The path of the projectile is a parabola


2 2
a-
y = xt a n
gx sec a ••• (3)
2
2v

From (2) , the time of flight when y = 0, t = T •


T = 2v sin a ••• (4)
g
From x (v cos a) T, the horizontal range R = OA , is
2
R v cos a . 2v sin a v sin 29 ••• (5)
g g
247

v2
The range is the maximum when 9 = 45° , R = ••• (6)
·' max g

At the highest point H , velocity ~(vertical) = 0 .


• ( sin 9
• • from 1) t v - - , hence the maximum height from (2) is
g
2
v sin2 9 ••• (7)
h
2g
2
From (5) : R = v sin 29
g

Since sin 2(90° - 9) = sin 29, there are two angles of projection 9 and 90° - 9, which
give the same horizontal range R. It is easy to see that these two directions are
equally inclined to the direction given by 9 = 45°.

Fig. 8
There is no need to memorise the above results.

Start from : x = 0 and y = -g •

The formulas of motion are different from those given above when the initial
conditions are different. For example, the projectile may have been fired at a height,
say, a, above the horizontal ground, in that case the component y is given by:

y = a+(vsin9)t-~gl

EXAMPLE: (I)

A particle is projected in a vertical plane at right angles to a wall of height, h,


standing on horizontal ground at a distance, c, from the point of projection 0. It just
clears the wall at the highest point of its path. Prove that:

(a) the speed v of projection is given by v


2
-j. (c 2 + 4h 2)
1 2h
(b) the angle 9 of projection is tan-
c
248

SOLUTION:
Starting with axes at 0 and
2 2
d x d
-=Oand~=-g
2 2
dt dt
We can show that

x = (v cos a) t ••• (1)

~ = v sin a- gt ••• (2)

t2
y = (v sin a) t - ~- ••• (3)
Fig. 9
At the highest point, y= 0 , hence t = v sin
g
a ••• (4)

Substitute t = v s!n a in (1) at P(c, h).

2
:. c = v cos a . v s!n a = vg sin a cos a ••• (5)

Substitute y = h and t = v sin a in (3)


g

••• h =v sin a . v sin


g a - ...£.2 • ( v sin
g a) 2 21 v
2
g
. 2a
• sm ••• (6)

Divide (6) by (5) : tan a 2h or a tan- 1 2h


c c
From (5) :
4
2 v 2 . 2
c
2g cos a sm a

v; (1 - sin 2 a) • sin 2 a . 2a
sm =~]
2
g v

c 2- -
g
v4(1-~)·~
2 2 2
v v

This simplifies to v 2 ...& 2 2


2h (c + 4h ) •
249

Exercise 88

1. A stone of mass m is projected with velocity 30 m/s from a point at the foot
of an inclined plane making an angle of 30° to the horizontal. The path of the
projectile can be ·assumed to be in the vertical plane containing the line of the
greatest slope of the inclined plane. If the angle of projection a > 30°, for what
values of a will the stone strike the inclined plane:
(a) horizontally? (b) at right angles?

2. Prove that the range R on a horizontal plane of a particle projected at an angle


2
v sin 29
9 to the horizontal and velocity v is given by : R
g
2
where g is the acceleration due to gravity in m.s-
Also prove that the equation of the path of the projectile can be written as:

(a)
2 2 2 2 2
gx tan 9 - 2v x tan 9 + ( 2v y + gx ) = 0

(b)
1
At what angle must a body be projected with a speed of 50 m.s- to just
clear a wall 10 m high at a distance of 60 m from the point of projection.
(Hint: use part (a) with v = 50, y = 10, x = 60, g = 10. Find two answers)

3. A stone is projected with an initial velocity v


in a vertical plane at an angle 9
1
to the horizontal and hits the ground at a point A. Another stone is projected
at the same angle of projection, but v > v •
with an initial velocity v
2 2 1
Let B (x 2, y > and C (xI' y ) be two points on the respective paths of flights
2 1
at the same time t. Show that the gradient of segment BC is independent of
time t.

When the second stone just clears a wall of height h, the first stone hits the
ground at A. If the wall stands at point D on the level ground, prove that
AD = h cot 9.

Further show that

tan (-r.f) = tan 9 - g T


v cos 9
, where r,f is the angle made by the downward
2
flight of the faster stone with the horizontal, and T is the time of flignt for
the slower stone.

Hence show that v (tan9 + tanr.f) 2v tan9,


2 1
250

4. The nozzle of a water hose is at a point 0 on the horizontal ground. The


water comes out of the nozzle with speed U m/s. Neglecting the air-
resistance, prove that the water can reach the wall at a distance d from O, if
2 .
U > gd, where g is the acceleration due to gravity.

If U 2 = 4gd, also prove that the maximum height that can be reached by the
. on t h"1s wa 11 IS
. g1ven
. 15d
by 8
Jet .

5. A missile is fired from 0 with initial velocity U at an angle a with the


horizontal. Prove that it describes a parabola of focal length

u 2 cos 2 a
2g
4
2
Also prove that any point P(x,y) within and on the circle x + / = v 1s m
2
2
danger of being hit by the missile. (g m.s- is the acceleration due tog gravity).

6. A stone is projected upwards at an angle 9 to the horizontal. Find an expression


for the velocity v at time t in terms of g, t and the initial velocity U.

If the stone at time t is moving in a direction perpendicular to the initial


velocity, show that t = -u
g
cosec 9 and that the stone's speed is given by
v = U cot 9 •

7. A body is projected with speed U from a height h, above a horizontal plane,


at an angle 9 to the horizontal. Show that the range R on the horizontal is
given by
2 2 2 2
gR sec 9 - 2U R tan 9 - 2hU =0
Further show that the maximum range R is given by
1

and the corresponding angle of projection is given by


u2
tan9 =-
gR1
·

Hence prove that h = R 1 cot 2 9.


251

8. A particle is projected from a point 0 with velocity v at an angle 9 to the


horizontal. It passes through the point P (x,y) in the vertical plane through 0
where (x,y) are the co-ordinates of P with respect to the rectangular axes at 0.
2 2
Prove that .y =x tan 9 - gx se;: 9
2v

If x =. 20 m, y = 10 -2
m, g = 10 m.s , v = 20 m/s, find the two values of tan 9
.
and usmg, . 2 9 =- 2-
t = tan 9 an d sm t prove t h at t he ratto
. o f two ranges
2
. 5 I + t
IS j .

9. A particle is projected with velocity v at an angle 9 to the horizontal. Show


that by SUitable choice of axes, the equation of the path of the projectile is
2 2
y = X tan 9 - gx sec B
2
2v
Prove the following:
(a) There are two possible directions of projection given by tan 9 and
1
tan 9 for a given range R.
2
2
2v
(b) tan 9 1 + tan 9 2 = gR and tan 9
1
tan 9
2
= I.

(c) Let T and T be the times of flights corresponding to angles of


1 2
projections 9 and 9 ·
1 2
Tl sin 9
1
Prove that
T2 sin 9 •
2
(d) From (b) prove tha1 9
1
+ 9
2
= 90°.

(e) R = R
1
sin 29 , where R is the maximum range.
1

2 R' 4
(f)
T' then [Hint: sin29 = ~]
~ 5 I + t
10. Two particles P and Q are projected from the same point 0 with the same
velocity 25 m/s. They both strike the horizontal plane through 0 at the point
A, 60 m from 0. P reaches A before Q. Show that:

(a) the angle of projection of P is tan-! ~ and that of Q is tan-! ~ •

(b) the time of flight of P is 3 seconds.

(c) the distance between P and Q at the instant P reaches A is 15 /2 m.


9
(d) the ratio of the maximum heights reached by P and Q is 16 ·
252

8.4 Resisted Motion: Other Laws of Motion


In our previous studies of motion, we neglected the effect of air-resistance or air-
friction. A body moving in a fluid experiences a resistance which tends to stop the
motion. In many cases of motion, the resistance is an important consideration. Cars,
planes and boats are streamlined so as to reduce the frictional drag and improve fuel
economy.
The air or fluid resistance on an object depends on its:

(i) shape (ii) size (exposed area) (iii) speed.

For example, a sky-diver with an unopened parachute falls quite rapidly, but the
descent is slowed when the parachute opens. The parachute encounters greater
resistance due to its shape and size. A sky-diver can enjoy a free fall (without an open
parachute) by employing a spread-eagle position to increase the air resistance and
prolong the time of fall.

Air or fluid resistance also depends on the speed of the object. The greater the speed,
the greater the air resistance. We shall mostly be concerned with motion for which
the resistance is proportional to the speed v or v 2•

Contrary to our perception of resistance, it is quite beneficial to us. Actually it is the


road friction that makes car driving possible! Sky-diving is pleasant and possible
because the air-resistance helps to slow the descent. Though streamlined cars cost
more, they are at least pleasing to our eyes, if not to our purse!

EXAMPLE : (1)

A particle of mass m moves along the x-axis. It experiences a resistive force R


given by R = kv, where k is a constant and v is the velocity at any time t and
position x. Discuss the motion.

SOLUTION:
(x,t,v)

0 p X Fig. 10
The equation of motion at time t is given by
dv
m. dt = -kv (F = ma)
We attach a negative sign because the resistance opposes the motion.
• dv k
• • dt = - .vm
Integrating I~v - f~ .dt
kt
loge v A
m +

or v = B e-kt/m ' where B is a constant.


253

If v = v at t = 0, then B = v
0 0

-kt/m
v = voe

As t ... co the function e-t ... 0 and hence v ... 0

dx -kt/m
Further v = dt = v0 e

The distance travelled in time t is

X - Jt
- v e-kt/m dt
0
0

or x jp- vO [I -e-kt/mJ
mvo
Again as t -+- "'• x-+- i.e. it moves with decreasing velocity towards the
k

limiting position xt

EXAMPLE : (2)

A particle of mass m falls under gravity from rest in a medium with the resistive
force given by R (v) = kv. Discuss the motion.

SOLUTION:
0
We take the initial position as the origin and x-axis
along the direction of motion. The equation of
motion at time t is:
dv
m dt mg - mkv
p
dv
dt = g - kv or dt

Fig. 11
t - kI [log (g- kv)- log (g)] - k1 loge ( 1 _kgv)
X
254

We can solve this equation for v

;kt = 1 _ kv
g

v f (1 - e-kt) ••• (1)

As t-+oo,
Fig. 12
vT =f is called the terminal velocity and the particle continues to travel with
constant velocity vr This happens when the resisting force mkv balances the
gravitational force on the particle, i.e.

mkv = mg or V -
-
gk

[Have you ever wondered how a team of sky-divers frolic (in the sky) with their
parachutes not open! From above· you see that a sky-diver should enjoy a free fall
until his weight balances the resistive force, thereafter his parachute must open, and
with good luck the diver should then enjoy his fall with the reduced terminal velocity.
Terminal velocity before the parachute opens is about 200 km/h and it is 40 km/h after
the parachute opens.]

Further v
dx from (1)
dt

X + c

When t = O, x =0, this gives c

hence x
255

EXAMPLE : (3)

A particle of unit mass is thrown vertically upwards with a velocity of U into


2
the air and encounters a resistance kv . Find the greatest height h achieved by
the particle and the corresponding time.
SOLUTION:

Equation of motion is
dv 2 X
v dx = -g - kv

I vdv
g + kv2
I dx
p
1 2
X - 2k loge (g + kv ) + c

When x = 0, v = U, this gives c = 21k 2


loge (g + kU )

2
1 g + kU
x = 2k loge ••• (1)
2
g + kv

For the greatest height, x = h, v = 0


1 k 2
h = 2k log ( 1 + g U ) .•• (2)

0
For the corresponding time, we use: Fig. 13
dv 2 dt -1
dt = - g - kv or
dv = g+kv2

t
I -dv
g + kv2 =- k
1

t 1
.fiJk r"
Ltan
1
1/fkg . v ]o
U
256

EXAMPLE : (4)

A particle is thrown vertically upwards with speed U in a medium with resistance


R = mkv, where m is the mass of the particle and k is a constant. Find the
greatest height h reached and the corresponding time.

SOLUTION:

Selecting the origin as the point of projection,


the equation of motion is
X
dv
m.v. dx·. - mg- mkv

X
_I ~ (g + kv) -
g + kv
~
• dv
mkv

x = -~+~ loge (g + kv) + c

When x = 0, v = U, hence 0 =- ¥ + ~
k
log (g + kU) + c
0

X k1 (U - v) ••• (1) Fig. 14

At the greatest height, v = 0, x =h


h = .k!d. - &... log ( 1 + kgU )
k2
••• (2)

To find the time, we use

dv dt -1
dt = -g - kv or
.dv = g + kv

o
t--
- JU g + kv
~ 1
k log ( 1 + ~u) ••• (3)
257

EXAMPLE : (5)

A body is projected vertically upwards from the earth's surface with velocity U. The
2
acceleration of a particle in space is given by kx , towards the centre of the earth,
where x is the distance of the body from the centre of the earth.

Given that the acceleration is g at the earth's surface, prove that the velocity v at
time t is given by

v
2
= u
2 2
- 2gR ( ~-~ )

where R is the radius of the earth.

If u
2
= 2gR, find ~~ in terms of x and show that the body will reach a distance &R
from the earth's surface in 2.72 hours.

Also find the velocity of ESCAPE, i.e. the velocity U of projection so that the body
never returns to the earth.

(R = 6400 km, g = 10 m.s-2 ).

SOLUTION:

Selecting the axes as shown, the equation


of motion is
X
dv mk (l)
m.v. dx -7
and g
k
;2
(given) at x = R ... (2)
p

k = gR '

dv
v-
2

-&y
hence
R2
l mk
x2

dx
X
x=R

Integrating ,
l =
gR2
+ c
2 X

u2
At X = R, v = U, so C=y - gR

Fig. 15
v 2 = U 2 - 2gR 2(
1
R. X ) ••• (3)
258

2
If U
2
2gR then from (3), v
2
= T
2 R

v = dx
dt + v'22gR. i 112 (sign + as v t from 0)
112
dt x
dx Rffg
For x = 8R + R = 9R (from the centre) time taken is

=
t = IR
9R xl/2 d
ffgR x ffgR .
1 2 [ x3/2] 9R
3 R
2 1 (27 R/R - R {R)
=3 /ZgR

5: . H.
For R = 6.4 6
x 10 m, g = 10 m.s
-2 , t = 2.72 hours

Finally, the particle never returns if x -+ oo, hence from (3)

2 2 .!X
v = U - 2gR (since -+ 0)

2
The body never returns if v ~ 0 •
2
u > 2gR

U > /2gR
U must be slightly greater than h x 10 x 6.4 x 106
i.e. U > 11300 m/s

U > 11.3 km/s

The escape velocity 12 km/s


259

Exercise SC

I. A particle of. unit mass falls vertically from rest in a medium with resiStive
force R = kv, where v is the velocity of the particle at time t. (k is a
constant).
Find the velocity v and hence show that the terminal velocity is given by f.
2
2. A particle of unit mass falls from rest in a medium with resistive force R = kv ,
where k is a constant. Prove that the distance x fallen when the velocity is
v, is given by:

x = 2k
1 loge ( g 2 ) •
g- kv

Find the distance fallen when it reaches half of its terminal velocity.
2
(Hint: The terminal velocity is given by g - kv = 0, i.e. v = .fijk).
3. An object is projected vertically upwards with initial velocity U from the
earth's surface. The acceleration obeys the law given by
2
d x k
dt 2 - x2
where x is the distance of the particle from the centre of the earth whose
radius is R.
Given the acceleration is g when x = R, show that velocity v in any position
is given by:

v
2
= u
2
- 2gR
2
( ~ - ~ ) •

Hence show that U = 12 km/s


for the escape velocity. (i.e. the object does
not return to the earth). (R = 6400 km, g = 10 m.s- 2 )
2
(Hint: x + oo for escape from the earth and v > 0)
4. A particle moves under gravity in a resistive medium with the resistance
R = kv, where v is the velocity in any position and k is a constant. The
particle is projected vertically upwards with velocity U =f . Show that at any
time the expressions for the velocity v and position x are given by:

v = U \2e
-kt
- l) and x = ku (2 - 2e
-kt - kt ) .

Find the greatest height achieved by the particle.


(Hint: v = 0 for the greatest height).
260

5. A particle of mass m is projected vertically upwards under gravity with velocity


2
U m/s. The resistive force is R (v) = mkv at any time t and position x.
Show that the expression for x is given by:

x = 2k1 loge [g + kU2


g + kv
2
J
Show that the greatest height H is given by:

H= ....!. 2k
log
e
[1 +
2
kU ]
g

6. A body of mass m falls from rest in a medium with resistive force R = kv,
where k is the coefficient of air resistance and v is the speed of the object.
(k is a constant.) Prove that the distance x fallen when the velocity is v, is
given by:

X = - mkv - m2 g log
k2 e
[I - kv ]
mg

Find the terminal velocity for a falling 70 kg sky-diver, if k = 14 and


2
g = 10 m.s • Express your answer in km/h.

7. A ball of mass m is thrown vertically upwards with velocity U. The air


resistance is per unit mass f(v) = kv 2 , at a distance x when velocity is v.
(i) Draw a neat sketch of the motion, showing the forces acting at a distance
x from the point of projection.
2
(ii) Show that x = kI loge [ g + kU ]
2 2
g + kv
(iii) Show that the greatest height achieved is

H = 2~ loge [1 + ~ • u
2
J
(iv) Draw a neat sketch of the downward motion of this ball after it reaches
the greatest height H. Show that the distance y fallen when velocity is
W, is given by

Y = 2~ loge [ g 2]
g- kW
(v) Deduce from (iv) that the terminal velocity V is given by
that H can also be given by:
v2 = f , and
H = 2~ loge [ g
g- kW
2] where

W .is the velocity when y =H for downward motion.


I I - I
(vi) From (iii) and (v) deduce that + - ::::2
u2 7 w
261

8. A particle is thrown vertically upwards where the air resistance is given by


2
R = 0.01 mv • If the velocity of projection is 60 m/s, find the

(a) time to reach the highest point

(b) greatest height H.


2
9. A particle falling from rest in a vertical line in a medium with resistance kv
per unit mass, k is a constant, v is the velocity at any time and position x,
prove that it acquires a speed

[/I - e-2kh J vr
in falling through a distance h, where VT is the terminal velocity given by

ff
A particle projected upwards in the same medium with initial speed VI and
returns to the point of projection with speed VR .

Prove that

10. An object of mass 20 kg experiences a resiStive force, in newtons, of one-fifth


of the square of its velocity in m/s, when it moves through the air.

This object is projected verticaHy upwards from a point 0 with velocity of


30 m/s and reaches the highest point H in time T.
. -2
Given g = 10 m.s , find

(a) the time T

(b) the distance OH.

11. A particle of unit mass moves in a straight line against a resistance R given by
R = v(l + v 2), where v is the velocity of the particle at a distance x metres
from the origin. Prove that x = tan-l v - tan-l v, where v is the initial
0 0
velocity at the origin. Use the formula

-1 -1
tan A - tan B = tan
-1 [A - B l
I + ABj to show that:

v - tan x
0
v
1 + v tan x
0
262

12. A particle of unit mass travels in a straight line against the resisting force
f(v) = v(l + v 2 ). Its initial velocity is c m/s at the origin. Show that the time
t, when velocity is v, is given by:

t = 1 Iog' [1
- _
2] ·
+ v-
2 1 +
2
c
2
Find v as a function of t and hence the limiting value of v at t + co •

13. A particle of mass m moves along a straight line under the action of a
constant (propelling) force P, and a resistive force mkv, where k is a
constant, v is the speed at any time t. Show that if the speed increases from
2 m/s to 4 m/s over a time interval of 5 seconds,

(a) p = 2km rL :~
2e
e
- I] .
- 1

(b) Find the corresponding distance moved.

(c) Find the propelling force P for k = 0.5.


14. The acceleration due to gravity at a distance r from the earth's
centre is directed towards the centre and equal to !5. when r
2
> R, equal to
r
kr when r < R and equal to g when r = R. Imagine a narrow tunnel along
a diameter XY of the earth and the particle is projected from X with initial
velocity U towards Y. (R = radius of earth).
2
If U < 2gR, prove that the motion is oscillatory and the amplitude is given by
R

15. A particle is projected vertically upwards from the surface of the earth with
initial velocity U. The acceleration due to gravity at a distance x from the
centre of the earth is given by k , directed towards the centre. Prove that
2
X
2
the rocket will escape from the earth provided u ? 2gR, where g is the
acceleration due to gravity at earth's surface and R is the earth's radius.
2
Further, if u = 2gR, show that the time to achieve the height R above the
2
earth's surface is approximately equal to 0.273 fR , given g = 10 m.s- •
263

16. A body of mass m is released from a height h above the ground and it
experiences a resistive force R given by R = 0.1 mv 2, where v is tile velocity
achieved by the body at time t.
2
If the object falls from rest under gravity (g = l 0 m.s- )

Find: (i) the terminal velocity U (i.e. velocity as t + m ).

(ii) the height h if the velocity is 0.5U just before the body strikes
the ground.

17. A ball of mass, m, is descending vertically in a tank of fluid (under constant


gravity). The resistive force is kv per unit mass, where v is the speed and
k, a constant.

(i) Draw a motion diagram at time t and write down the equation of motion
dv
using F = m • dt
(ii) Write down an expression for time t taken by the ball to acquire the
velocity v from rest and hence show that:

v = ~ (l - e-kt )

(iii) Find the terminal velocity vT. If g = 10 and k = 0.2, draw the graph
of v against t.

18. A particle P of unit mass is projected from a point 0 with velocity U at an


angle 9 to the horizontal. The particle moves under gravity and each
component of its velocity experiences a resistance k times the magnitude of
the component. By considering the rectangular axes Ox and Oy and the particle
has components v and v of the velocity v at time t:
X y
(a) Draw a diagram of the motion of P

(b) Write down the equations of motion in the form:

and ~ (v ) =
dt y

(c) Prove that v = Ucos9e-kt and v . ) e-kt - &


k1 (g + kUsme k
X y

(d) Prove: x u
= kcos9 (1 - e-kt)

(kUsin9 +g)
y = 2
k
264

19. A projectile is fired vertically upwards from the earth's surface with velocity
U m/s. The retardation due to gravity is given by the law k where x is the
2
X
distance of the projectile from the centre of the earth, and k is a constant. The
acceleration due to gravity on the earth's surface is g. The earth's radius is R.
2
Neglecting the air resistance, show that if u = gR, then the projectile
reaches the height R above the earth's surface. Show further that the time
for this journey is

[ ~ + 1] A"·
20. An object of mass m is thrown vertically upwards in a medium with a
mv
resistance R =k , where v is the velocity of the object and k is a constant.

Given t = 0 = x and the initial velocity is k(c - g), where c is a constant


and g is the acceleration due to gravity, x is the distance travelled in time t.

(a) Draw a neat sketch of the motion and the forces acting at a point P,
distance x from the origin. Hence write down the equation of the motion.

(b) Find the time taken by the object to reach the highest point H and find
the height of H above the point of projection.

(c) The object falls to its original position with the same law of resistance.
Will the time of descent be the same as that of ascent? Give your
reasons.
265

CHAPTER 9 CIRCULAR MOTION

9.1 Introduction

The study of circular motion is of great importance in science and engineering. The
orbit of the earth around the sun, or the moon around the earth, can be considered
circular for practical calculations. The safe speed on a circular section of a
highway or railway track is governed by the laws of circular motion. We shall also
study the problems related to circular motion, such as conical pendulum and
banked tracks.

In solving the motion problems, we often require resolution of the forces in two
perpendicular directions OX and OY. These directions need not always be the
horizontal and the vertical. Study the following examples of two resolved parts of the
force F.

y
y y
0 X

Fig. 1

We shall study the dynamical problems, in which Newton's laws have to be used,
namely
rv
(1) F = ma (mx or my)
(2) Force of action = Force of reaction
To solve any dynamical problem,
1. Resolve all the forces acting on the
system in two perpendicular directions.
.EF 0 .EX
2. Then using F = mx and F = my : X

mx = .EX - .EFx and my = .EY - .EFy


3. Substitute known values in (2) and
solve for the unknown (velocity, force .EF
etc.) y
Fig. 2
266

9.2 Angular Velocity: Period

For a point moving in a straight line, its velocity is defined as the rate of change of
its displacement.

(x, t)
0 p X
Fig. 3
v = ~~ in m/s where x = f(t)
2
d x -2
Linear acceleration of a particle is given by a = dt 2 in m.s

When a point moves on a curve, we talk about its angular velocity, i.e. the rate of
change of its angular displacement, as defined below.

ANGULAR VELOCITY of a point about a given point

Let 0 be the given point, and OX a


line through 0 of fixed direction.
Suppose P moves in the plane containing
OX and LPOX = 9 at any time t.
The anticlockwise rotation is considered
positive and the clockwise rotation negative.
The angular velocity w (omega) of the moving
point P about 0 is defined as the rate
of change of 9, i.e.

X
Fig. 4

The unit of angular velocity is the radian/s and is abbreviated as rad/s. Angular
velocity is a vector and when the direction is not significant we speak of angular
speed. It must be remembered that as defined above w is a variable, i.e. a function
of t. In most of our applications in circular motion, w is a constant, i.e. a uniform
angular velocity. In a later section, we shall talk about the angular acceleration of a
point, about a given point.
267

EXAMPLE: (I)

A point P moves on a circle with uniform angular velocity of j rad/s. Show the
positions of P for t = 0, 1, 2, 3, 4, 5, 6. What is the time taken by the point P to
describe the circle once completely?

ll
SOLUTION:
3 = 60° .
t 0 2 3 4 5 6
We have:
9
ll 2n 4n 5n t=O
0 ll 2n
J 3 3 3 t=3 t=6
Time to describe the circle once completely is 6 s.
T = Angular displacement 2n
Angular speed = 1iT3 = 6 s
Fig. 5

9.3 Circular Motion: Tangential Velocity

Suppose a particle moves in a circle of radius


r (anticlockwise) and sweeps out angle 9 in
time t.
Let it describe a small arc PQ in time 8t.
arc PQ = r x 89
Then v, the tangential velocity at P is given by:
d d d9
v = dt (arcPQ)=dt (r.89)=rdt asllt~O

d9
Now the angular velocity is w = dt , hence:
Fig. 6
••. (l)

The angular velocity w is usually defined in radians per second, but if the radius of
the orbit is extremely large as in planetary motion, w may be defined in radians per
hour or a day or even a year.
It should be remembered that w is a variable in formula (1), but most of our work
deals with the constant angular velocity and hence constant tangential velocity. In
that case; at time t,
9 angular velocity x time
1 9 = wt 1 ... (2)

It is usual to describe w as revolutions per second or minute or hour etc.


268

EXAMPLE: (2) Convert the angular velocity w 50 revolutions per second to


radians per second.

SOLUTION: Since 1 rev 2n radians


w = 50 rps = 50 x 2n = lOOn rad/s

EXAMPLE: (J) A satellite moves in a circular orbit with 20 revolutions per day.
Describe w in rad/s.
20 X 211 radians per hour
SOLUTION: (J)
= 20 rpd = 24
51!
= 3 X 60
rad/min

11
= 36 X 60
rad/s

11
= 2160
rad/s

EXAMPLE: (4) A wheel of radius 2 m revolves at 1200 rounds per minute. Find:
(a) its angular velocity
(b) the tangential velocity of a point on the wheel.

1200 X 211
SOLUTION: (a) w = 1200 rpm = 60 rad/s = 40rt rad/s.

(b) v rw =2x 40n = SOn = 251 m/s.

THE PERIOD:

The period T of a circular motion with constant angular velocity w is defined to be


the time for one revolution.
T = angular displacement in one revolution
angular velocity

EXAMPLE: (.5) An artificial satellite travels in a circular orbit of radius 9000 km. If
the period is 90 minutes, find the angular and the tangential velocity
in km/h.

SOLUTION:
211
T = (J)
given T = 90 min = 1.5 h

(a) 211 211


(J)
T T.5 rad/h = 4.19 rad/h
(b) v =r x w = 9000 X 4.19 = 37700 km/h
269

Exercise 9A

1. A particle on a disc rotating with a uniform angular speed of 4 revolutions per


second is 0.2 m from the centre of the disc. Find:
(a) the tangential speed v of the particle
(b) the angle through which it rotates in 0.4 s.

2. A car travels halfway around a circular track in 12 s. What is the angular


speed of the car? If the radius of the track is 100 m, find the velocity of the
car in km/h, to the nearest km /h.

3. A motor shaft has a speed of 240 revolutions per minute. Find its angular
speed in rad/s.

4. Find the average angular speed in rad/s of the earth's


(a) rotation about ·its axis. (Hint: period is 24 h)
(b) revolution about the sun. (Assuming a circular orbit and a period of
I year = 365.25 days)

5. Find the tangential speed of the earth at the equator due to its rotation about
its axis, given that the radius of the equatorial circle is 6440 km.
[Use exercise 4 (a)]

6. Find the tangential speed of the earth due to its revolution about the sun,
given that the radius of the earth's orbit around the sun is 1.5 x 108 km.
[Hint: use exercise 4(b)]

7. What is the angular speed of the particle in a circular path of radius 5 m and
a tangential velocity of 100 km/h?

8. A belt is wrapped around a pulley that is 0.4 m in diameter. If the pulley


rotates at 240 revolutions per minute, what is the linear velocity in m/s of the
belt?

9. An aeroplane propeller whose blades are 1.5 m long is rotating at 2400


revolutions per minute.
(a) Express the angular speed in rad/s
(b) Find the angular displacement in 4 s
(c) Find the linear speed of a point on the end of a blade.

10. An artificial satellite travels in a circular orbit of radius 10000 km. If the
period is 2 h, find the angular speed in rad/h and the tangential speed in km/h
of the satellite.
5
11. The average distance of the moon from the centre of the earth is 3.85 x I o km
and the period of the moon's revolution about the earth is 27.3 days. Find the
angular speed in rad/h and the linear speed of the moon in km/h.
270

9.4 Acceleration of a Particle Rotating in a Circle


y
The diagram shows a particle P moving
anticlockwise on a circle.

At time t, P is (x, y)

LPOX e
w angular velocity (constant) X

v tangential velocity
de dw
w Cit = a constant, so dt 0

The horizontal com onent The vertical com onent

X = rcose y = rsine
The horizontal velocity The vertical velocity
dx . e de . e dv d9
dt = -rsm dt = -rw sm dt = rcose dt = rwcose
The horizontal acceleration The vertical acceleration
2
-d2x de = -rw 2 cos a ~ . e de 2 . e
2- = -rw cose • dt 2 = -rwsm • dt = -rw sm
dt dt
2 2
:. x =-rw cose y = -rw sine
The resultant acceleration

a fx2 + -y2
2 2 2
1<-rw cose) + (-rw sine) 2
2 2
rw /(cos e + sin 2 e)
2 0 M
rw
Fig. 8
The direction is towards the centre 0, along the radius PO of the circle.
Thus the acceleration of a point moving in a circle with angular velocity w is
2
a = rw
2 2
But v = rw, so 2 v v
a= rw = r.
2r r

2
2 v
a rw
r

At a later stage, we shall derive the expression for acceleration when w is a variable"
271

9.5 Uniform Circular Motion

As proved before, the acceleration of a particle


moving on a circle is given by:
2
a = rv 2
or a= rw '

directed along the radius, towards the centre.

By Newton's first two laws, there must be a resultant force that causes this mot1on
and its magnitude must be given by

2
or F mrw

where m is the mass of the particle and this force must act along the radius directed
towards the centre. This force is called the CENTRIPETAL FORCE (centre seeking
force).

EXAMPLE: (6)

Find the tension in the string when a stone of mass 5 kg is rotating at 50 rpm, the
stone is tied at one end of the string and the other end is fixed at point 0. The length
of the string is 2m. (rpm = revolu~ions per minute)

SOLUTION:
50 X 211
w 50 rpm - -- - rad/s, m 5 kg, r 2 m
60
511
w rad/s.
3
The only force acting on the particle is the tension
T along the string, hence:
2
T centripetal force mrw
2
5 X 2 X 25n /9
Fig. 10
274 N
EXAMPLE: ( 7)
A string of length 2 m is attached at one end to a fixed point 0 on a horizontal table,
while the other end is attached to a heavy particle of mass m kg. Find the speed v
in m/s of the rotation of the particle, given that the tension in the string is n times
the weight of the particle.

SOLUTION:
The reaction N = mg is of no concern to us.
Tension T centripetal force
2
mv (given)
n x mg
v rmg r =2
ffng m/s
Fig. II
272

Exercise 98

(Take g =9.8 m.s-2 , for the following problems)

I. A particle moves on a horizontal circle of radius 2 m with uniform linear speed


of 10 m/s. Find the centripetal acceleration required for this motion.

2. A ball of mass 0.5 kg is rotated at the end of an inelastic string in a horizontal


circle at 5 m/s. If the length of the string is 2 m, find the centripetal force
exerted by the string on the ball.

3. Find the centripetal force on a car of mass 1500 kg travelling around a circular
track of radius 7 5.0 m at a speed of 45.0 km/h.

4. A flywheel of radius 0.4 m rotates with the uniform angular speed of 40n rad/s.
Find the centripetal acceleration of a point on the rim of the wheel.

5. The earth rotates on its axis at an angular speed of 1 revolution per 24 h. Find:
(a) the linear velocity in m/s of a point on the equator where the radius of the
earth is 6400 km.
(b) the centripetal acceleration of a point on the equator.

6. A curve of radius 250 m is planned in a highway. The expected legal limit is


90 km/h. Find the centripetal force of the road on the car of mass 1200 kg.

7. An earth's satellite at a distance of 36000 km above the earth's surface is called


synchronous satellite because it and the earth have the same period. Find:
(a) the angular speed of the satellite in rad/s and rad/h
(b) the linear speed of the satellite in km/h if the radius of the earth is
6400 km
(c) the centripetal force required to keep the satellite in the orbit if it has a
mass of 1000 kg.

8. A communication satellite in a circular orbit at an altitude of 500 km above the


earth's surface makes one revolution in 95 minutes. Given that the radius of the
earth is 6400 km, find the centripetal acceleration of the satellite.
273

9.6 Conical Pendulum

A particle of mass m is attached to one end 0


of a light inelastic string of length l, the other
fixed at 0.

The particle P moves in a horizontal circle,


so that the string describes a cone whose
vertical axis passes through the centre C of
the circle.

mg
Suppose that the particle moves with velocity
v, then the resultant force is along PC and Fig. 12
resolving the forces as shown, we now have
2
Tsin9 = centripetal force = ~r
••• (l) Tcos9
and Tcos9 = mg ... (2)
v2 f---- mv 2 /r
Divide (l) and (2): tan9 = ... (3)
rg
Now tan9
r
and v = rw c Tsin9
p
Ti
2 2 r
r w
rg Ti mg
w =ft ... (4) Fig. 13

Hence the period T = ~11 = 211 JI . . (5)

The period is independent of the mass m and depends only on h, the vertical distance
of the circle of rotation below the point of suspension 0.

Note that w = n depends only on h.

SUMMARY (CONICAL PENDULUM)

2 2
v 2. v = rg tan9
1. tan9 = rg

3. T 211 ~ 4. w =If
274

EXAMPLE: {8)

A disc of radius 2 m rotates in a horizontal circle about a vertical axis AB. A light
string PQ of length I m tied at the rim of the disc at P, and carries a mass of
2 kg at Q. The disc rotates uniformly such that PQ is inclined at an angle of 45°
to the vertical. Find:
(a) the speed of the mass at Q
2
(b) the tension in the string. (g = 9.8 m.s )

A
SOLUTION:

The forces acting at Q are:


Tension T in the string
Weight m acting vertically downwards
Resolving at Q along the horizontal and the
vertical directions: 2

Tcose = mg ••• (I)


Fig. 14 mg

The unbalanced component must supply the


- 2
necessary centripetal force mv for the
r
circular motion of the mass at Q, where
v is the tangential speed of Q and Tsin6 X
r = BQ = 2 + I . sin 9
2
Tsin6 = ~ ••• (2)
r
mg
Divide (2) by (I): then
2 Fig. 15
v
tane
gr

Given e 45°, r 2 + sin 45° g = 9.8


'
2 9.8 x (2 + sin 45°) tan 45°
v gr tanS

(a) v = 5.15 m/s

(b) Using Tcose = mg


T = mg sece 2 x 9.8 x sec 45° 27.7 N
275

Exercise 9C
-2
(Take g = 9.8 m.s , for the following problems)

I. A mass of 3 kg is attached to one end of a light inelastic string of length I m,


the other end of which is tied to a fixed point 0. If the mass is rotated in a
horizontal circle and the string makes an angle of 25° with the downward
direction of the vertical, find
(a) the speed v of the mass in m/s
(b) the tension T of the string in newtons.

2. A mass of 0.5 kg is suspended from a fixed point 0 by means of a light rod of


length I m. The mass 1s rotated in a horizontal circle and the rod makes an angle
of 30° with the downward direction of the vertical. Find:
(a) the linear velocity of the mass
(b) the period
(c) the tension in the rod. A

3. A honzontal disc of radius I m is free lm


to rotate about a vertical axis AB as
shown in the figure. PQ is a light rod
of length 1.2 m pivoted to the rim of
the disc at P and carrying a mass of
3 kg at Q. The disc is rotated uniformly,
so that PQ is inclined at an angle 30°
to the vertical. Calculate:
Fig. 16
(a) the speed of the mass at Q in m/s
(b) the tension in the rod.

4. A model plane of mass 5 kg attached to


the end of an inelastic wire of length 20 m
flies in a horizontal circle of elevation 30°,
while the other end of the wire is held
fixed. If the lift (force) L acts at right
angles to the wire and L ts twice the
wetght of the plane, find:
(a) the speed of the plane in m/s
(b) the tenswn in the wire in newtons.

5. A train is moving around a circular track of radius 800 m at a uniform speed of


80 km/h. A light inelastic wire is attached to the roof of the carriage and has a
small package attached at the other end. Find the angle of inclination of the
wire with the vertical. (Hint: The radius of the circle described by the package
is approximately the same as that of the circular track.)
276

9.1 Banked Tracks

The tendency of a vehicle to skid outwards (along the tangential path) as it rounds the
curve on a horizontal roadway is opposed by the friction between the road and the
tyres (see Fig. 18). The friction is the only force providing the centripetal acceleration
as the vehicle travels along the circular path. This frictional force is never sufficient
to keep the vehicle in the circular path. 1f the road is suitably banked, the upward
thrust of the road on the vehicle then provides the required centripetal force. Banking
means the outer edge is raised above the level of the inner edge of the road (see
Fig. 19).

mv
2•P\
r
' r
\
0..-----
2
mv
r

Fig. 19

We shall now show that it is possible to choose a banking angle e such that the lateral
friction force is zero.

In Fig. 19, R is the reaction of the road on the vehicle of mass m, as it travels along
the curve of radius r at a speed v. We resolve R into two components:
(i) the friction force F (side thrust), parallel to the slope
(ii) force N, normal to the road surface.
2
The resulting acceleration of the vehicle has a magnitude ~ and is directed towards
0, the centre of the circular path. r

By resolving the forces at P, along and perpendicular to the slope, we have:


force along the slopemass x acceleration along the slope.
mv2
F + mgsine = -- cose
r
2
F = ~ cos e - mg sin e
r 2
Resolving along the perpendicular to the slope, N =mgcose - m; sine.
By choosing 9 such that the side thrust F is zero, we eliminate the tendency of the
vehicle to skid either up or down the slope.
The proper angle of banking for speed v is obtained from
2
m; cos e - mg sine = 0 i.e.l..._ta_n_e___ ;_:---1
The frictional force acting parallel to the slope is called the lateral thrust. For a
railway track the use of the correct banking angle 9 (obtained by raising the outer rail
above the inner one) ensures that there is no lateral thrust between the rail and the
wheel flanges, thus reducing the wear on both (rails and wheels). Even when F = 0,
the friction force between the tyres and the road (acting parallel to the direction of
motion) provides the tractive force on the vehicle.
277

WORKED EXAMPLES ON BANKED TRACKS

EXAMPLE: ( 9)

A section of a road is in the form of an arc of a circle of radius 1000 m. Find the
banking angle ·8 if the road is designed to carry traffic at a speed of 25 m/s.
(g = 9.8 m.s-2 )

SOLUTION:

The forces on the vehicle are its weight


mg and the reaction N (exerted by the Ncose
road) at right angles to OP. The 4 p
2 Nsin9
acceleration of the vehicle is vr ,
horizontally towards the centre of the mg
curve. 0 Fig. 20
Fig. 21
Resolving along OP:
2
mv cos a -2
mg sine
r
Substituting v = 25 m/s, g = 9.8 m.s
r ~ 1000 m ,
2
.•• tane = v ••• (1) 625
rg tan9 = 1000 X 9•8 gives 9 = 3.65°
Alternatively we can resolve along the horizontal PX and the vertical PY, then:
2 v2
N sin 9 = !!!.!.._ and Ncos 9 = mg, hence tan 9 =
r rg

EXAMPLE: (I 0)

A train is travelling around a horizontal curve with uniform speed of 60 km/h. The
radius of the curve is 400 m. The rails are 1.5 m apart. Find the elevation h of the
outer above the inner rail if there is no lateral force on the rails.

SOLUTION:
B
Using the same explanation as in the
previous example, resolving along AB,
we have:

2 v2
mgsin8 = mv cosQ, hence tan9 = -
r rg
A Fig. 22
Now v = 60 l<m I h = 50 mI s, r = 400 m, g = 9.8 m.s-2 , we find 8
T
= 4.05°
3
Then h = ABsin8 = 1.5 sin 4.05° = 0.106 m
278

Exercise 90

1. A section of the highway is in the form of an arc of a circle of radius 250 m.


Assuming the average speed of cars on the highway to be 60 km/h, find the angle
at which the highway should be banked. (g = 9.8 m.s- 2 )

2. A section of the road is in the form of an arc of a circle of radius r. The road
is banked at an angle 9, so that a car of mass m travelling at speed v has
no tendency to side slip.
(a) Show that the resultant force on the car is given by mgtan9.
2
(b) Show that v = rg tanS and hence calculate the banking angle 9, given
v = 80 km/h, g = 9.8 m.s- 2 and r = 500 m. Give your answer in degrees
with one decimal place. (g = 9.8 m.s- 2 )

3. A car is travelling around a circular section of the highway which is banked at


an angle of 12°. The radius of the curve is 200 m. Find the maximum safe
velocity v of the car, so that it does not slip. Express your answer to the
nearest km/h. (g = 9.8 m.s-2)

4. A train is travelling around a circular r:urve of radius 200 m with the proper
safe speed of 50 km/h. The rails are 1 m apart. Find the banking angle 9 of
the curve and the elevation h of the outer rail above the inner rail, if there is
-2
no side force on the rails. (g = 9.8 m.s )

5. A cycling track is in the form of a circle of radius 100 m. The proper safe
speed for this track is 40 km/h. Find the banking angle to 2 decimal places.
(g = 9.8 m.s-2)

6. A bobsled track has a hairpin curve of radius 16 m. The curve is banked at 72°.
What is the maximum safe speed in km/h for the curve, assuming no frictional
forces? (g = 9.8 m.s-2)
279

9.8 Components of Acceleration (Variable Angular Velocity)


2 2 2
Suppose a particle P moves on a circle x + y = r and at time t let:
LPOX = e ' w = d9/dt ' P(x, y)
:. x = rcose and y = rsin9
Differentiating wi~h respect !o time t:
• . de . • de
x :: - r sm 9 dt = - rwsm 9 , y = rcos 9 dt rwcose

= .2
velocity v y + = rw ••• (I)
d9 . dw
-rwcos e dt - rsm 9 • dt = -rw 2 cos 9 - rw• sm
.
9 ••• (2)
. de dw 2 . •
- r wsm 9 dt + r cos 9 • dt = - rw sm 9 + r w cos 9 ••• (3)
y

2
rw cos9 - ; t p
2 .
rw sme
rwsin9 p

Fig. 24

Fig. 23
Fig. 25
From equations (2) and (3) and the diagrams showing the resolved parts of two
2
accelerations rw and r w, we infer that the acceleration of a point moving on a
circle has two components:
The tangential component rw 2
The normal component 2
rw or v (towards the centre)
r
EXAMPLE: (I I) ·
A pomt p moves on · Ie x 2 + y 2 = 16 w1t
' a c1rc . h um.f orm angu Iar
velocity of 2 rad/s. A is the point (2,0). If LPOA = 9, LPAX = <$
1T
find the angular speed of P about A when 9 = 0 and when 9 = 2.
SOLUTION: Noting that LOPA = <$ - 9,
<PAO = 180° - rJ, using the sine rule in 6POA:
sin(<$ - 9) sin(i80° - 0)
2 4
2 sin(<$ - 9) = sin<$
Differentiating with respect to time t:
X
2 cos(<$ - e) . (0 - e) = (cos<$) 0 •.• (I)
Angular velocity of P about A is 0. Fig. 26

(cont. on the next page)


280

When 9 = O, ~ = O, 9 = 2, then from (1): using cosO = 1, 2(~ - 2) = {4 or


Ql = 4 rad/s.
Again, when 9 =~ , ~ is obtuse, P is at B(0,4), tan!,4 = -; = -2 •
B
Using equation (2):
= ~ cos!,4
2cos(~ - 90°) (~ :.. 2)
Now cos(IS - 90°) = sin~, hence dividing by costS,
2taniS (~ - 2) = Ql
Put tan IS = - 2 and solve for Q1 :
Ql = 1.6 rad/s 0 A X
:. Angular velocity of P about A when 9 = ~ is 1.6 rad/s. Fig. 27

EXAMPLE: (12)
A car is travelling at 60 km/h and its wheel has a radius of 0.25 m. Find the speed
u m/s of the highest point of the wheel.
SOLUTION:
-1 -1
v = 60 km h = 16.7 m.s
The angular velocity of a point on the rim about
the centre is given by _
1
w = v/r = 16.7/0.25 = 66.7 rad h
The situation at time t is shown in the diagram
with P(x,y) on the circle. Assuming P to be
at 0 at t = 0, the distance travelled by the
wheel is OB and this is equal to the arc BP, 0 X
i.e. OB = r9.
Fig. 28
x = OB- AB = r9- rsin9, y = BC- NC =r - rcos9
x = r(e- 9cos9) = rw(l - cos9), y = rwsin9
velocity u of P at any time t is given by:
2 + y2 = r 2 w2 [(1- cos9) 2 + sin 2 9] 2 2
u2 x = 2r w 0- cos9)
Now 1 - cos9 = 2sin ~
2

• u = 2rwsm
•• . 9
2
The direction of the velocity of P if desired is given by taniS = £= sin9
1-cosa
Substituting for the velocity of the highest point, i.e. 9 =n , w = 66.7, r = 0.25, the
required velocity u = 2 x 0.25 x 66.7 x sin~ = 33.4 m/s.
281

Exercise 9E (ANGULAR VELOCITY)

1. A point is moving in a circle. Prove that its angular velocity about the centr~
of the circle is double its angular velocity about any fixed point on the
circumference.

2. A point P is moving in a circle of centre 0, with uniform angular velocity w.


N is the foot of the perpendicular to a fixed diameter of the circle. Show that
2
the acceleration of N is given by w • ON and is directed towards the centre.

y
3. A point P moves with uniform velocity u
along Ox. A is a point on the line y = b,
and PA = r. Show that the angular velocity
of P about A is given by b~ •
r
dX d9
[Hint: X =PM = bcot9, dt = u, dt = w
0 p
4. A point P is moving (anti-clockwise) in a
y
. Ie g1ven
c1rc . b y x 2 + y 2 = a 2 , w1t
. h um.f orm
angular velocity w. A is the point (r, O),
r >a. If <POX = 9, <PAX = rf,,
(a) prove that rsinr/1 = asin(r/1 - 9)
(b) prove that the angular velocity of P X
about A is given by:
dr/1 awcos(rf, - e)
dt = acos(fi - e) - rcosfi
Hence find the angular velocity of P about A, when P is at B(a, 0) and
C(O,a) •
2 2
5. A point P moves on the circle x + y = 25 with uniform angular velocity of
1 rad/s. A is the point (3, 0). If <POA = 9, find the angular speed of P
11
about A when 9 = 0 and when 9 = "2 •

6. A car is travelling at 80 km/h and its wheel has a radius of 0.3 m. Find the
speed of the highest point of the wheel.

7. A train is travelling at 60 km/h and a wheel has a radius of 0.4 m. Find the
magnitude and direction of the velocity of a point 0.2 m above the rail.
282

9.9 Miscellaneous (Worked Examples on Circular Motion)

EXAMPLE: (13)

A 15 em record rotates at an angular speed of 33 rev/min. Find


(a) its angular velocity in rad/s
(b) the speed of a point P on the rim of this record
(c) the acceleration (centripetal) of the point P.

SOLUTION:
33 X 2TI II TI
(a) w 33 rpm = 60 = TO rad/s = 3.46 rad/s

(b) v r x w = 0.15 x lln/10 0.518 m/s


2 11 TI 2 -2
(c) a rw = 0.15 x ( lO) 1.79 m.s

EXAMPLE: (14)

A particle of mass 5 g is rotating in a circular path of radius 1 m with a speed of


2 m/s. Find:
(a) the angular velocity, w, in rad/s _
2
(b) the centripetal acceleration, a, in m.s
(c) the tension in the string, in newtons.

SOLUTION:

(a) w f = 2 rad/s p

(b) a 1 x 2 2 = 4 m.s- 2
(c) The tension T in the string supplies the
centripetal force.
2 Fig. 31
T = m • rw = 0.005 x 1 x / = 0.020 N

EXAMPLE: (15)

The force of attraction between the earth and an artificial satellite in circular orbit
around the earth is given by
F _ GMm
- (R + h) 2
where G is the gravitational constant, M is the mass of the earth, m is the mass
of satellite, R is the radius of earth, and h = the height of the orbit above the earth's
surface.
A 1000 kg satellite is circling the earth at 1000 km above the surface of the earth
(g = 10 m.s-2 at the earth's surface). Find:
(a) the gravitational force acting on the satellite.
(b) the speed (constant) v of the satellite
(c) the period (given R = 6400 km)
283

SOLUTION:
s
Given R r.adius of earth 6400 km
h satelJite's height 1000 km
above the earth
g 10 m.s- 2 at the earth's surface
R + h 7400 km
GMm Fig. 32
We have: F
(R+t-i
For h = 0, F = mg (a) The gravitational force on the
satelJite is
1000 X 7.48
7GMm = mg
2
7480 N
GM = gR ••• (I)
When at height h,
(b) We must have 2
let F = mgh
mgh .
= centnpeta mv
1 f orce = -r-

GMm
2 where r =R + h
(R +h)
2
••• v = rgh = (R + h) • gh
~2 3
(R + h) = 7400 X 10 X 7.48
32 3
10 X {6400 X 10 ) v = 7.44 x 10 m/s
32
(7400 X 10 )
2n 2nr
7.48 m.s-
2 (c) The period T = w v
3
T = 2nx 7400 x 10
7440
6250 s
1.74 h
284

EXAMPLE: (16)

A stone is rotated in a vertical circle of radius 2 m at a constant angular velocity of


30 rad/s, by a string about a pivot 4 m above horizontal ground. Consider the rotation
to be clockwise. The string breaks at the highest point of the circle. Find the
distance from the pivot, A, to the point B, where the stone hits the ground.
(g = 10 m.s-2 )

c B X

Fig. 33
SOLUTION:

Given: w = 30 rad/s
Radius AO = AD 2m oc 2 + 2 + 2 6m
CD = 2m

The stone flies off horizontally at 0, with velocity


v = r x w = 2 x 30 = 60 m/s.

Thereafter it travels freely under gravity, so describes a parabolic arc OPB


Equations of motion are
I 2
X = 60 t y -
2 gt (Axes at 0, as shown)
= -2
At B, y -6 and g = 10 m.s
2
t = 1.2 or t = {l.is
Now substitute in x = 60 t, x = BC
BC 60 x vf.i'

AB 65.8 m
285

Exercise 9F

1. A satellite moves in a circular orbit of radius 8000 km, making 12 revolutions


per day. Find:
(a) the velocity v in km/h.
(b) the centripetal force acting on the satellite, if the mass of the satellite is
500 kg.

2. A flywheel of radius I m makes 500 revolutions per minute. Find:


(a) the angular velocity, w, in rad/s.
(b) the tangential velocity, v, in m/s of a point on the rim.
(c) the acceleration of a point on the rim.

3. An artificial satellite of the earth travels in a circular orbit of radius 36000 km.
If the period is 12 hours, find the angular velocity, in rad/h, and the speed v in
km/h.

4. Find the tension in the string when a stone of mass 10 kg is rotating at 60 rpm,
the stone is tied at one end of the string and the other end is fixed at point 0.
The length of the string is 1 m. (rpm -= revolutions per minute)

5. A string of length 1 m is attached at one end to a fixed point 0 on a table,


while the other end is attached to a heavy particle of mass 2 kg. Find the speed
v in m/s of the rotation of the particle, given that the tension in the string is 4
times the weight of the particle. (g = 10 m.s-2)

6. The force of attraction between the earth and its artificial satellite in circular
orbit is given by F = G~m , where x is the distance of the satellite from the
X
earth's centre, G is a constant, M is the earth's mass, m is the satellite's
mass. A 500 kg satellite is circling the earth at 4000 km above the surface of
the earth. If the radius of the earth is 6400 km, find:
(a ) the gravitational force acting on the satellite given g = 10 m.s-2 ~t the
earth's surface
(b) the velocity v in km/h of the satellite
(c) the period in hours.

7. A mass of 10 kg is rotated in a horizontal circle of radius 0.25 m on the end of


a string 2 m long. Find the greatest number of rev/min, if the string can just
sustain a tensile force of 5000 N. (g = 10 m.s-2 )

8. A smooth circular disc of radius 0.25 m rotates in a horizontal plane with angular
velocity w = 10 rad/s about a vertical axis through the centre 0. A particle of
mass m is attached at P by light inelastic strings to points M and N on
the ends of a diameter of the disc, such that MP = 0.4 m, NP = 0.3 m and
both strings remain taut. Find the tension in the string NP, and the mass m
if the tension in MP is 20 N.
286

9. A particle of mass m is attached to a fixed point 0 by a string of length I


metre and by another string of the same length to a small ring of mass M
which can slide on a smooth vertical wire through 0. If m describes a
horizontal circle with constant angular velocity w, prove that its depth below 0
is

10. A particle of mass 5 kg is whirled around at the end of a string I m long. It


describes a horizontal circle with 36 rev/min. Find the tension in the string and
the angle of inclination of the string with the vertical. (g = 10 m.s- 2 )

II. The (water) drops shaken off the rim of a rotating umbrella meet the ground in a
circle of radius r metres. The rim is a circle of diameter I m and is 1.5 m
above the ground. It is rotating with an angular velocity of 411 rad/s. Find the
radius r. (g = 10 m.s-2)

12. One end of a light inelastic string of length I m is attached to the rim of a
horizontal disk of radius 2 m and the other end carries a mass of 5 kg. If the
maximum tension in the string may not exceed 300 N, find the maximum angular
velocity w in rad/s and the corresponding inclination of the string to the vertical.
(g = 10 m.s- 2 )

13. A train travelling at 80 km/h is rounding a curve of radius 500 m. Determine


the banking angle, a, of the track with the horizontal, so that the sideways
thrust on the flanges of the wheels would not derail the train. (g = 10 m.s-2)

14. A stone is rotated with an angular velocity of 5 rad/s in a vertical circle of


radius 4 m and centre 10 m above the ground. It breaks off the string when it
makes an angle of 30° with the vertical in the positive quadrant. If the motion
is clockwise, find the distance from the centre to the point where the stone hits
the ground.

15. A truck is travelling around a section of a circular track which is banked at an


angle of l6°,andhasradius80m.Find the velocity, so that the truck does not slip.
(g = 10 m.s- 2 )

16. Two particles are connected by a string passing through a hole in a smooth table,
one particle being on the table, the other underneath. If the masses of both
particles are equal, find the velocity v of the particle on the table moving on a
circle of radius 2 m, so that the other remains at rest. (g = I 0 m.s-2)

17. An object of mass I kg is attached to a string of length 8 m to a fixed point P.


It is set rotating with velocityv in a horizontal circle of radius r. Given that
the centre is 5 m vertically beneath P, find r and v (given g = 10 m.s-2)

18. The orbit of the earth around the sun can be assumed to be a circle of radius
11
1.5 x 10 m described with uniform angular velocity w radians per second.
Given that 1 year = 365.25 days, find
(a) w, in radians per second
(b) the earth's acceleration towards the sun in m.s- 2
19. The moon makes a complete revolution of the earth in 27.3 days with a nearly
circular orbit of radius 3.85 x 108 m. Find:
(a) the acceleration of the moon towards the earth
(b) the linear velocity, v m/s, of the moon.
287

20. A mass of 12 kg at C is connected by light rods to sleeves P and Q which


revolve freely about the vertical axis PQ, such that L P = 60° and L Q = 30°.
(a) Given that PQ = 4 m, show that the radius of the circular path of rotation
of C is .f3 m.
(b) Find the tensions in the rods PC and QC when the mass makes 120
revolutions per minute.

21. A satellite of mass m revolves in a circular orbit of radius r about a spherical


planet of mass M and radius R. It is known that force of attraction between
. o f masses m an d m
two b od 1es - given
IS - b y F = Gm I m2 , wh ere x IS . t he
1 2
2
X
distance between the centres of the bodies. Show that the period T of revolution
of the satellite is given by 2~r ["f , where k is the force per unit mass at
the planet's surface due to its own gravity.

22. A light inextensible string of length 3 metres is threaded through a smooth


vertical ring which is free to turn. The string carries a particle at each end.
One particle, P, of mass m is at rest at a distance of 1 metre below the ring,
while the other particle B of mass M is rotating in a horizontal circle whose
centre is P. Show that the velocity, v, of B in terms of m and M, is given by

23.
~
A satellite S is circling the earth at a height 35800 km above the earth's surface,
with a period of 1 day. Calculate orbital speed of this satellite. (R = 6400 km,
the radius of the earth). Give your answer in km/h.

24. A point P is moving in a circle with an angular velocity w in an anti-clockwise


direction. If the radius of the circle is r metres,
(a) show that the speed of P is given by rw
(b) M and N are the points of projection of P on the coordinate axes with 0
as the origin. Prove that both M and N execute simple harmonic motion as
P rotates about 0.

25. A massless rod of length L is hung from


pivot A, attached to a rim of a horizontal
circular disc of radius r.
An object of mass m is attached at the
other end B of the rod. AB makes an angle : L
9 with the vertical when the disc rotates - - - - - _J:::J __ _
with constant angular velocity w about the D C B
vertical axis through 0. ~- - - - - - --- ->
R
(a) Show that BD = R = r + L sinS
2
(b) Prove that (r + L sin 9 )w = g tan 9, where w
g m.s- 2 is the acceleration due to gravity.
Fig. 34
(c) Prove that the speed, v, of the object B is given by
2
v = g tan 9 (r + L sin 9)
(d) Find the speed of B when 9 = 45°, g = 10 m.s
-2
L = 1 m, r = 5 m.
288

Chapter 10 Harder 3 Unit Topics

10.1 Harder Trigonometry

We shall need the following formulas:

1. sin (A' .:!: B) = sin A cos B .:!: cos A sin B 8. sin(l80° - A) =sinA
2. cos(A +B) =cosAcosB ~ sinAsinB 9. cos(l80° - A) =-cosA

3. tan(A.:!: B) = tan A.±. tanB 10. If sin x =sina, then


I+ tanAtanB x = nn + (-l)na
4. . A + sm
sm . B =2 sm -r
. (A+B) cos (A-B)
2
11. If cosx =cosa, then
. A - sm
. B x = 2nn.:!: a
5. sm =2cos (A+B) . (A-B)
- - sm
2
- -
2
12. sin2x =2sinxcosx
2
6. cosA + cosB = 2cos ( -A+B)
- cos (A-B)
- - 13. cos2x = 2cos x- 1
2 2
= 1 - 2sin 2 x
7. cosB- cosA = 2sin(A;B) sin(AiB> 2 . 2
=cos x- sm x

Worked Examples

sinB + sinC
1. Prove that:
sinB - sinC

Solution:

. <B-r-
2s1n + c> cos <B-
~ c>
L.H.S.
B+C. B-C
2cos(-- ) sm ( -- )
2 2

tan ( B ; C) cot ( B i C)
R.H.S.
289

2. If A + B + C = 180°, prove that:


sin2A + sin2B + sin2C =4sinAsinBsinC
Solution:
L.H.S. = 2sin(A + B) cos (A - B) + 2sinCcosC
Now A+ B + C 180°, so sin(A +B)= sin(l80°- C)= sinC
and cosC cos(l80° - A - B) = -cos (A + B)
:. L.H.S. 2sinC • cos (A - B) - 2sinC • cos (A + B)
= 2sinC [cos (A - B) - cos (A + B)]
2sinC (2sinAsinB)
4sinAsinBsinC

3.
6 6
Show that cos 9- sin 9 =cos29 (1 -! sin2 29)
Solution:
3 3 2 2 2 2
Using a - b = (a
- b) (a + ab + b ), a = sin &, b = cos 9
2 2 4 2 2 4
L.H.S. = (cos 9- sin 9) (cos 9 + cos 9 sin 9 + sin 9)
2 2
Now cos a - sin a = cos 29
an d cos 4e + cos 29 sm
. 2a + sm. 4a = <cos 29 + sm . 29)2 - cos 2a sm
. 29 =1 - cos 2a sm
. 2a
2 2
:. LH.S. cos29 (1- cos 9sin 9)
· · 2a = 2 sm · a cos a , so sm 1 · 2 2a
· 2 a cos 2 a = 4sm
Agam, sm
:. L.H.S. = cos29 (1 -!2
sin 29) = R.H.S.

4. (a) Solve the equation sin29 =cos39 , 0° ,< 9 .t. 360°


3
(b) Show that cos39 = 4cos a- 3cos9
(c) Using the parts (a) and (b), show that sin 18° = ./5-4 1

Solution:
(a) cos39 = sin29 = cos(90° - 29) (b) We use:
3 9 = 360° • n .:!: (90° - 2 a) cos(A+B) = cosAcosB - sinAsinB
59 = 360° • n + 90° ••• (1) Put A = 9, B = 29, then:
cos39
or 9 = 360° • n - 90° ••• (2) = cos9cos29 - sinS sin29
For (1), n = 0,1,2,3,4 (0~ 59~ 5. 360°) =cos9(2cos 29-1) - 2sin9. sin9cos9
Dividing by 5, = 2cos39 - cos 9 - 2cos a (1 - cos2 9)
9 = 72°. n + 18°, n = 0,1,2,3,4 = 4cos 3 9- 3cos9
= (18°, 90°, 162°, 234°, 306°)
From (2), 9 = 270° for n = 1.
Hence the complete solution is:
9 = (18°, 90°, 162°, 234°, 270°, 306°)
(cont)
290

(c) Using (b), the given equation becomes:


4cos 3 9- 3cos9 = 2sin9cos9
Now 9 = I go is a root of this equation, so cos 9 f 0
2
and hence: 4cos 9- 3 = 2sin9
2
4 (l - sin 9)- 3 = 2sin9
2 ! /5-
0r 4sin 9 + 2sin9 - 1 = 0 which gives sin9 = 4
{)- l
Since sin 18° > 0, we have: sin 18° = 4
2 2
Note: We can evaluate cosl8° by using cos 18° + sin l8° =

(a) tanA + tanB


5. Using tan (A + B) = 1 - tanAtanB'
prove that

tan (x + y + z) =
(b) Hence show that t = tanx
n 3 2
(c) Show that tan <n> is a root of the equation t - 3t - 3t + I =0
3 2
(d) Solve the equation t - 3t - 3t + 1 = 0

Solution:
tanA + tanB
(a) Put A = x, B =y + z in the formula tan (A + B)
1 - tan A tanB
tanx + tany + tanz
tan (x + y + z) tanx + tan(~ + z) 1 - tanytanz
1 - tanxtan y + z)
_ tanx(tany + tanz)
1
1 - tanytanz
This simplifies to:
tan (x + y + z)
(b) Put z = y =x
3
3tanx - tan x
tan 3x t = tan x
2
- 3tan x
3
n . n 3t - t
(c) Put x = Ti , 1.e. 3x = ij. , then: tan (!!. ) = - -2
4 I - 3t
3 2
Now tan <i> = I, so this simplifies to: t - 3t - 3t + I 0 ••• (l)
Hence t " tan ( ~) is a root of the equation (l ).
1
(d) The constant term I suggests we try t = ! 1 as the possible roots of (1).
t = - 1 satisfies the equation, hence either by inspection or long division, we have:
3 2 2
t - 3t - 3t + I = (t + 1) (t - 4t + 1) = 0
4! {i2
So, the roots are t = -1, 2
i.e. -1, 2! 1/3. Observing that

0 < tan ( t
2
> = tan I 5° < I, we have tan I 5° = 2 - /3.
291

Exercise 1OA
For questions 1 to 5 prove that:
2 2
1. (a) 8sin ecos e = 1 - cos49, hence
2 4
(b) 32cos 9sin e = 2 -cos29- 2cos49 +Cos69
cos e- sin e = cos29 (1 - ~ sin 29)
8 8 2
2. (a)

(b)
6 6
cos e + sin e = i (1 + 3cos 2 29)

3. n e)
tan ( 4 + 2 = jl1 _sine
+ sine
= seca +tanG

4. (2cos9 + 1) (2cos9- J) (2cos29- J) (2cos49- J) = 2cos89 + 1


5. 9
cosec 9 + cosec 29 +cosec 49 + cot49 = cot < >
2
6. if A + B + C = 180°, prove that:
(a) cos2A + cos2B + cos2C = -1 - 4cosAcosBcosC
2 2 2
(b) cos A + cos B- cos c 1- 2sinAsinBcosC
7. 1f A + B + C = 90°, prove that:
(a) sin2A + sin2B + sin2C 4cosAcosBcosC
(b) . 2A +Sin
Sin
. 2B +Sin
. 2C = 1 - 2sinAsinB sinC
8. Solve the following equations: 0 ~ 9 ,< 2n
(a) sin4a = cos29 (b) sin39 = sin29
9. Solve the following equations: 0° .$- 9 .$- 360°
(a) 4 sin 2 a - 3cos 2 9 = 1
(b) sin59- sin39 + sin9 = 0
10. Show that cos39 = 4cos 3 9 - 3cos9
3
Put x = 2cos9 in the equation x - 3x = V2 and show that the equation
1
reduces to cos 3 a = V2
. 2 3
(a) Hence solve the equation x - 3x - .f2 = 0, giving your answer in the
exact form, x = 2cos9

(b) n +cos (9rr


Prove that: cos (TI) 17n
TI) + cos(TI) = 0
(Do not use a calculator)
11. (a) Show that sin (A + B) - sin (A - B) = 2cos A sin B
(b) Show that 2 sin x (cos 2x + cos 4x + cos 6x) = sin 7x - sin x

(c) 6n)
Deduce that cos ( 2n ) + cos (411) +cos <7 2-1
7 7

(d) 11 +cos ( 7311) + cos (7


Prove that cos<-,> 5n) = 21
292

10.2 3 Unit Co-ordinate Geometry: Circles(Harder Problems)

No new theories or ideas are required to solve problems on


circles. The following basic properties of circles will be
very useful.
(i) A tangent and the radius at the point of contact are at
right angles.
(ii) An angle in a semi-circle is a right angle.
(iii) A line is a tangent if its distance from the centre of
the circle is equal to the radius.

I. Equation of a Circle
2 2 2 ••• (1)
(x - h) + (y - k) = r
is the standard form of the equation of a circle centred at the point
C (h, k) and radius r.
2 2 2
x + y =r ••• (2)
is the equation of a circle centred at the origin and radius r.

2. The General Equation


2 2
x + y + 2gx + 2fy + c = 0 ••• (3)
where g, f and c are constants, is called the general equation. By completing
the squares in x and y, we have:

(x + g)2 + (y + f)2 = g2 + f2 - c
This equation has the standard form of the circle with centre at (-g, -f) and

radius ~2 + f
2
- c.
Finally the equation:
2 2
Ax + Ay + Bx + Cy + D = 0 ••• (4)
when divided by A, reduces to the general form (3), so, (4) is also a general
equation of a circle.

I
293

EXAMPLE: (1)
Find the centre and the radius of the circles:
2 2 2 . 2
(a) x + y - ax - by = 0 (b) 3x + 3y + 5x + 12y = 0

SOLUTION:

(a) Completing the squares, we have: (b) Dividing the equation by 3,


2 2
(x _ ~ )2 + (y _ ~ )2 =a : b X
2
+ y
2
+
5x
T + 4y =0
a b • 5 2 2 169
So, the centre is C <
2 , 2) •• (x + 6) + (y + 2) = 36
and the radius is ~ /a 2 + b
2 . (- 5 ,
••• Th e centre 1s
6 - 2)

and the radius is i


1

EXAMPLE: (2)
Find the equation of the circle through the points (0, 0), (4, 8), (9, 9).

SOLUTION:

The given points must satisfy the equation of the circle:


2 2
x + y + 2gx + 2fy + c =0 ••• (l)
c = 0, 16 + 64 + 8g + 16f =0 and 81 + 81 + l8g + l8f = 0
These simplify to: g + 2f = -10 and g+ f = -9
Solving for f and g: g = -8, f = -1
2 2
Hence the required equation is x + y - l6x - 2y =0

EXAMPLE: (3)

Find the equation of the circle which touches both axes and passes through the point
A (l, 2).
SOLUTION:
2 2
Let (x - h) + (y - k) =r2 ••• (l)
be the equation of the circle.
A(l,2) satisfies this equation, so
2 2 2
( l - h)+ (2 - k) = r .,. (2)
Since the circle is tangential to both axes, the centre
(h, k) must be at a distance equal to the radius from
each axis.
h = k = r and from (2): X
2 2 2 2
(l - r) + (2 - r) = r , which simplifies to: r - 6r + 5 = 0.
r =l or 5
Thus, there are two circles, satisfying the given conditions, i.e.
2 2 2 2
(x - 1) + (y - l > = l and (x - 5) + (y - 5) = 25
294

3. Equation of a Circle having AB as Diameter

Let A be (h, k) and B be (h', k'), and P(x, y) be


any point on the required circle. We use the fact that
the angle APB in the semicircle is a right angle.

m gradient of AP = ~
X.- h

v - k'
m' = gradient of BP = ~

Since AP J..BP, mm' = -1 and this gives:

(x - h) (x - h') + (y - k) (y - k') = 0

as the equation of the required circle.


0 X

EXAMPLE: (4)

Find the equation of the circle on AB as diameter where A is (2, 3) and B is (4, 5).

SOLUTION:
The required equation is obtained by substituting for A (2, 3) and B ( 4, 5) in the
equation above, i.e.
(x - 2) (x - 4) + (y - 3) (y - 5) = 0
2 2
or x + y - 6x - 8y + 23 = 0
As a check, we find that the centre (3, 4) is the mid-point of AB.
295

T
4. Tangents to a Circle

We use the fact that the tangent is perpendicular to the


radius at the point of contact.
Consider the equation of the circle
2 2
x + y + 2 gx + 2 fy + c = 0. •• . (I)
The centre is (- g, -f).
If P(x , y ) is a point on the circle, then m
1 1
= gradient X

Y1+f -J X}+g
of CP =- --
xl+g
and the gradient of the tangent PT is then m' =-m =- ---
yl+f

XI + g
The equation of the tangent is y - y 1 =- 'Yi"+T ·(x - x 1)
Or (y - y ) (y + f)
1 1
= - (x 1 + g) (x - x )
1

Now P(x I' y ) lies on the circle (1),


1
Hence the equation of the tangent is

The reader is advised not to memorise this equation, but derive the result for each
question.
It will be instructive to prove the following results:
2 2 2
1. The equation of the tangent at P(x I' y ) to the circle x + y = a is
1
2
xx + yy = a and at P(acos9, asin9) is xcos9 + ysin9 =a.
1 1
2. Prove that ~ - ; : f from the equation (I) and hence derive the equation
2
of the tangent at P(xl'yl) to the circle x + / + 2gx + 2fy + c = 0.

EXAMPLE: (5)
2
Find the equation of the tangent at A(-1, 3) to the circle x +/ - 8x + 7y - 39 = 0.
. C ( 4, T
SOLUTION: The centre IS -7). A is (-1, 3)

.
Th e grad 1ent o f CA = :-y-:t;""
3 + 7/2 = lO
- 13

10
The tangent is perpendicular to the radius, so the slope of the tangent is 13 and

hence its equation is y - 3 = : ~ (x + I) , which simplifies to lOx - 13y + 49 = 0.


Alternative method:
We find the gradient of the tangent by differentiating the given equation:
:. 2x + 2y ~- 8 + 7 ~ = 0. At (-1, 3), ~ =
10
etc.
dx dx dx 13
296

Exercise 1OB

1. Find the equation of the circle passing through the points:


(a) (1,0), (0,-1),(0,0) (b) (-1,3),(2,2),(1,4)
2. Find the centre and the radius of the following circles:
2 2 2 2
(a) x + y + 2x - 4y + 1 = 0 (b) 8x + 8y - 12x + 20y =
3. Find the equation of the circle whose centre is on the x-axis and which passes
through the points (0, 3) and (4, 1).
4. Find the equation of the circle whose centre is at the point C (-3, -4) and
which is tangential to the line 3x + 4y = 20.
5. Find the equation of the circle through the point A (-1, 2) and which is
tangential to both axes.
6. Find the equation of the circle centred on the line y = 2x, which passes
through the point A (-2, 4) and is tangential to the x-axis.
7. Show that the chord whose equation is x - 3y + 8 = 0 subtends a right angle at
2
the centre of the circle 9x + 9/ - 18x + 6y = 170.
2
8. Show that A (1, 1) lies on the circle x + / + 4x + 6y - 12 = 0. Find the
co-ordinates of B, if AB is a diameter of the circle.
9. Find the equation of the circle passing through the origin and making intercepts
a and b on the x- and y-axis respectively.
10. Find the intercept made on the x-axis by the circle which has AB as diameter,
where A is (0, -1) and B is (2, 3).
11. Find the equations of the tangents to the following circles at the points indicated:
2
(a) x + / - 6x - 2y - 3 = O, (5, 4)
2 2
(b) (x - 1) + (y + 2) = 5, (3, -3)
2 2
(c) x + y - 4x + 2y = 20, (5, 3)
2 2
12. Show that the line y = mx + b is a tangent to the circle x +y = a 2, if
b =~a /1 + m •
2
2
13. Find the equations of the two tangents to the circle x + / - 2x - 6y + 6 = 0
which pass through the point P (-1. 2). Use lx + my + n = 0. (Warning: do not
use y = mx +b)

14. Write down the equations of the circles on AB as diameter where:


(a) A (4, -8), B (3, 5) (b) A (a, b), B (b, a)
(c) A (a, a), B (-a, -a)
2 2 2 2
15. Show that the circles x + y = 4 and x + y + 6x - 8y + 16 = 0 touch
externally.
297

10.3 Plane Geometry: Clrcles(Harder Problems)


A
EXAMPLE: (l)

ABC is an equilateral triangle, inscribed in a


circle. X is a point on the minor arc BC.

Prove that: (i) t.BDX Ill t.ACX

(ii) XB + XC = XA

SOLUTION:

Given: t.ABC is equilateral


Prove: (i) t.BDX Ill t.ACX (ii) XB + XC XA

Proof: We have,
(i) u =z =60° (t.ABC is equilateral)
x =z and y = u ( Ls in the same segments on arcs AB and AC respectively)
Then, in t.BDX and t.ACX:
X = y = 60°
p =q ( L s in the same segment, arc CX)
L BDX = LACX ( L sum of a t. being I 80°)
t.BDX Ill t.ACX

(ii) XB BD ••• (I)


AC giving XB • AC = XA • BD
XA
Similarly we can prove that t.CDX Ill t.ABX and the corresponding result
XC • AB = XA • CD ••• (2)
Adding ( 1) and (2):
XB . AC + XC . AB = XA (BD + CD) = XA • BC
Now t.ABC being equilateral, AB = BC = CA, so, removing the common
length, we have the required result:
XB + CX = XA.
298

EXAMPLE: (2)

In the diagram, AD and BE are perpendicular


to BC and AC respectively. Prove that:
(a) HDCE is a cyclic quadrilateral on HC as diameter
(b) AH = AK

Given: ADlBC, BEJ.AC


Prove: (a) HDCE is a cyclic quadrilateral
(b) AH = AK
Proof: LHEC = 90° (BE lAC)
(a) LHDC = 90° (AD .i BC)
... E and D are on the circle with HC as diameter.
Hence, HDCE is a cyclic quadrilateral on HC as diameter.
(b) y =z (Ext. L of eye. quad. HDCE = Internal opp. L)
x =z (L s in the segment, arc AB)
X : y
t.AHK is isosceles
AH = AK

EXAMPLE: (3)

In the diagram, BC is a fixed chord of a circle,


A is a variable point on the major arc on the
chord BC.
BD J.AC and CE j_ AB. Prove that:

(a) BCDE is a cyclic quadrilateral on a circle


with BC as diameter.

(b) as A varies, the segment ED has


constant length.

(c) the locus of the mid-point of ED is


a circle whose centre is the mid-point
of BC.
299

SOLUTION:

Given: BC is a fixed chord, BD .1 AC, CE .1 AB


Prove: (a) BCDE is a cyclic quadrilateral
(b) ED has constant length
(c) Locus of mid-point of ED is a circle.

Proof: (a) LBEC and LBDC are both 90°. (BD .i AC, CE .lAB)
:. E and D lie on the circle whose diameter is BC.
:. BCDE is a cyclic quadrilateral.

(b) Since chord BC is of constant length, it subtends a constant angle,


say a, at the circumference of the given circle.
Now m = L ABO = 90° - a (BD J_AD)
Since a is constant, m is also constant.
Using the fact that equal chords subtend equal angles at the circum-
ference of a circle, we conclude that for various positions of chord
ED on the fixed circle, (on BC as diameter) ED must be of
constant length.

(c) Let P and M be the mid-points of ED and BC respectively.


Join MP and MD.
M is the centre of the circle BCDE.
MP J_ED
= r 2 - s2
2 2
MP:2 = MD - PD
1
Now r = MD= MC =
2 sc = a constant
4
s = PO = ED which is a constant
2
MP is a constant

Hence, the locus of P is a circle with the centre


at the mid-point of BC and the radius
300

Exercise 10C: PLANE GEOMETRY: CIRCLES (HARDER PROBLEMS)

1. In Fig. 1, if PQ is parallel to RS, prove


that PQ = RS.

2. In Fig. 2, P is any point on a diameter


AB of a circle; QPR is a chord such that
L APQ = 4.5°, prove that
2 2 2
AB :: 2PQ + 2PR •

3. In Fig. 3, 0 is the centre of the circle,


prove that a + b =c.
4. Two lines OAB, OCD cut a circle at
A, B and C, D. If OB = BD, prove
(Fig.2)
OC =CA •

.5. ABCD is a quadrilateral inscribed in


a circle. BA and CD when produced
meet at P. 0 is the centre of the
circle PAC. Prove that BD is
perpendicular to OP. (See Fig. 4)
(Fig.3)
6. Two circles ARPB, AQSB intersect
at A and B. PAQ and RAS are
straight lines. PR and SQ are
produced to meet at M. Prove that
MPBQ is a cyclic quadrilateral.
(See Fig• .5)

7. AB is a chord of a circle. The tangents


at A and B meet at T. AP is
drawn perpendicular to AB. TP is
drawn perpendicular to TA. Prove that
PT is equal to the radius of the circle.
(See Fig. 6)
s
8. Two circles of radii 3 em and 12 em
touch each other externally. Find the
length of their common tangents.

(Fig.6)
301

9. In Fig. 7, two tangents from T (to the


circle) meet the two tangents from D (to the
same circle) at A and C, as shown. T

Prove that AT - CD = TC - AD.

10. The altitudes PM and QN of an acute-


angled triangle PQR meet at H.
PM produced cuts the circle PQR at A.
Prove that HM = MA. (Hint: Join AQ)
(See Fig. 8)

11. AD is an altitude of the triangle ABC,


inscribed in a circle. DP is drawn
parallel to BA and meets the tangent at
A at P. Prove l CPA = 90°. (See Fig. 9)
\Hint: Show that x = y = z)
12. AB is a diameter of a circle, AC is any
chord. M is the mid-point of the arc BC.
Prove that AC is perpendicular to the
tangent at M.

13. In Fig. 10, three circles intersect at P.


Prove that AB is parallel to DC.
(Hint: Join PF and PE)

14. In Fig. 11, 0 is the centre, TP is the


tangent and TC bisects LOTP, prove
that [TCP = 45°.
(Hint: Join AP)

(Fig.10)

(Fig. II)
302

15. Prove that the quadrilateral formed by the


angle bisectors of a cyclic quadrilateral
is also cyclic. (Fig. 11)

16. If the two non-parallel sides of a trapezium


are equal, prove that the trapezium is cyclic
(Fig. 12) (Fig.l2)

17. Prove that the angle bisectors of the angles


formed by producing opposite sides of a cyclic
quadrilateral intersect at right angles. (Fig. 13)
(We have to prove L FMP = 90°
Hint: Produce FM as shown and prove
AFDE Ill AFBN

18. The bisectors of the opposite angles LP


and L R of a cyclic quadrilateral meet
the circle at A and B as shown. Prove
that AB is a diameter of the circle.
(Fig. 14)

(Fig.l4)
19. Two circles intersect at A and B. T
A straight line PAQ cuts the circles at
P and Q. The tangents at P and Q
intersect at T. Prove that PBQT is a
cyclic quadrilateral. (See Fig. 15)
(Hint: Join AB, x = y etc.)

{Fig.l5)
303

20. AB and CD are parallel tangents to a circle,


centre 0. APC is another tangent at P.
Prove that LAOC is a right angle.
(See Fig. 16)

21. (a) In Fig. 17 CP is a tangent and CAB


is any secant. Prove that
2
CP =CA. CB
(Hint: Join AP, show that t.PCA,
t. BCP are similar)

(b) In Fig. 18 PQ is a common tangent to


the two circles, and CAB is a common
secant. Prove that CP = CQ.
[Hint: Use the result (a) ]

(Fig.18)
22. PQ and RS are the common tangents to two
circles intersecting at A and B. AB produced (Fig.l9)
either way meets the tangents in M and N as
shown. (Fig. 19) Prove:
2 2
(a) MP = MA • MB and RN = NB. NA
(b) PM = MQ (See question 21)
(c) PQ = RS (Hint: t.OEC : t.OFC)
(d) MA = BN (Hint: MP = RN)

23. P is any point on the circle ABC. PL, PM, PN


are the perpendiculars to the sides BC, CA and AB
respectively of t.ABC. Prove that the points
L, M, N are collinear. (Fig. 20)

(Fig.20)
304
p
24. In the diagram, two fixed circles intersect
at A and B. Prove that L PMQ is of
constant size for all positions of Q.
(Fig. 21)

25. ABCD is a quadrilateral inscribed in a circle.


X, Y, Z, W are the mid-points of the arcs R
AB, BC) CD and DA respectively. Prove that (Fig.2l)
XZ .L YW.

26. In Fig. 22, two fixed circles intersect at A M


and B. P is a variable point on one circle.
PA and PB when produced meet the other
circle at M and N respectively. Prove
that MN is of constant length.

N
(Fig.22)

27. In the diagram (Fig. 23), 6ABC is inscribed


in a circle. AD, BE, CF are the altitudes of
6ABC. H, the point of intersection of
the altitudes,is called the ORTHOCENTRE of
the triangle. Prove that:

(a) l BHF = LBAC


(b) l BHC + LBAC = 180°
(c) 6AEF Ill 6ABC
(d) 6BDF Ill 6EDC
(e) AD bisects L FOE
(f) l EDF = 180° - 2 l BAC (Fig.23)
(g) If BC is fixed and A varies remaining
on the major arc on BC, find the locus
of H. [Hint: Use part (b) ]
(h) BFEC is a cyclic quadrilateral lying on
the circle on BC as diameter.
305

10.4 Inequalities

Definition: x >y if and only if x - y > 0.


On a real number line this means x lies to the right of y.

y X

Worked Examples

1. Prove that a ; b b are positive real numbers.

Solution: Consider

a+b2 1 2 2 1 2
2-) - ab = 7i (a + 2ab + b ) - ab = 7i (a - b) ~ 0
Now ( -

a ; b ~ -Jib, for a >0 , b > 0, and equality when a = b.

2. Given that x + y = c, prove that i1 + y


1
'l c
4
for x > 0 , y > 0 •

Solution: From example (1), a+b'l2 vas ••• (1)


1 1 ! ! ~ _2_
Put a =i , b =y , then:
X
+
y VXY ••• (2)

Using (1), VxY ,< ~


2
and putting x+y = c

c 2
VxY ,< 2 or --
1
VxY
~ c
1 4
From (2),
X
+ -y1 ~ c

3. If x, y, z are positive real numbers, prove the following:


2 2 X v 3 3 3
(a) x + y 'l2xy (b) - + L. 'l2 (c) x +y +z ~ 3xyz
y X

(d) x +) +z 'l (xyz)l/3

(cont)
306

Solution:
2 2 2
(a) Consider X + y - 2xy = (x - y)

2 2
X +y - 2xy ~ 0
2 2
X + y ~ 2xy

(b) Divide the result in part (a) by xy, then:

~+x.:V2
y X
3 3 2 2
(c) We have x + y = (x + y) (x + y - xy)
2 2
From (a), x + y - xy ~ xy

x
3 + y 3 >.,- (x + y) xy

~ xyz ( ~ + ~)
z z
Similarly y3+z3 >,.xyz(~+i),
Adding these:
3 3 3 XV VZ ZX
2 (x + y + z ) ~ xyz [ (- + .._) + ( L + -) + (- + -) ]
Z Z X X y y

>.,.. xyz [ ( ~ + X.) + (X. + ~) + ( ~ + ~) ]


y X Z y X Z

Now using (b):


3 3 3
2 (x + y + z ) >, xyz (2 + 2 + 2)
3 3 3
Hence x + y + z ~ 3xyz

(d) From (c), we have: a


3
+ b
3
+c
3
~ 3abc
3 3 3
Put a =x , b =y , c =z , then:

x+y+z '-v 3 x 1/3 .y 1/3 .z 1/3

Hence x + J + z ~ (xyz)
113
, equality for x = y = z.

The general result for a set of n positive numbers x , x , ••• , xn is


1 2
x 1 + x2 + .•• + xn
n
1.e. Arithmetic Mean >, Geometric Mean
307

-Exercise 100

1. (a) Show that, for a >0 , b >0 , c > 0,


2 2 2
a + b + c ~ ab + be + ca.
2
(b) Hence show that a 2 + b + c 2 >,... 3 (abc) 2/ 3
[Hint: Use worked example 3 (d) ]

(c) Prove that (a + b + c) 2 >.... 3 (ab + be + ca)


2
2. Prove that for x, y, z :r 0, x + y + z :r [3 (xy + yz + zx) ] l/
(Hint: Use question 1c)

3. Prove that for a, b, c, d >,... 0, a + b : c + d >,... (abcd)l/ 4


. a+b c+d
(Hmt: Let x = - - , y = - - , then use worked example repeatedly)
2 2
4. A rectangular box of sides a, b, c has a constant surface area S. Using the
formulas S = 2 (ab + be + ca) and V = abc (volume), prove that:

(i) (ii) V is a maximum when the box is a cube.


(Hint: Let x = ab, y = be, z = ca, then use worked example 3d)

5. If x, y > 0, prove that ~ (x 3 + y 3) ~ (x ; y)3

(Hint: Show that L.H.S. - R.H.S. ~ 0)

6. For any real numbers x, y, z, u, prove that:

(x
2
+ /> (z 2 + i> >, (xz + yu)
2

(Hint: Show that L.H.S. - R.H.S. >,. 0)


7. For any real numbers x, y, z, a, b, c, prove that:
2 2 2 2 2 2 2
(x + y + z ) (a + b + c ) >, (ax + by + cz)
308

10.5 Method of Mathematical Induction


Worked Examples

I. (i) Show that for each positive integer n, there are unique positive integers
a
n
and b
-n
such that (1 + Jj)n = a n + /3. b
n
(ii) Hence show that a 2 - 3b 2 = 2n • (-l)n
n n

Solution:
(i) For n = I, L.H.S. = 1 + .{3; + .fj. b
R.H.S. = a
1 1
a1
1
= I, b = 1 which are unique integers.
Assume that (1 + o/'J)k = ak + o/1. bk , where k is a positive integer. •.. (1)

Then (1 + v'J)k+l (l + v'j) (1 + o/3)k = (l + Jj) (ak + VJ. bk), using (1).
.r::3)k+l ::: (
(1 + '" 3b ) r::3 ( b )
~ + k + v" ak + k = ak+l +
and
Since ak and bk are unique integers, so are ak+l and bk+l . Thus the
statement is true for n = k + I.
Since true when n = 1, the statement is also true for n = 2.
Since true when n =2, the statement is also true for n :: 3, and so on, the
statement is true for all positive integral values of n.
Rather than repeating the foregoing three lines, in the subsequent examples we
will say "by the principle of induction, the statement is true for each positive·
integer n".
(ii) To prove: a 2 - 3 b 2 = (- l)n • 2n
n n
For n = I, L.H.S.
1
=a 12 - 3b
2
= I - 3 = -2, using part (i).
Also R.H.S. = -I • 2 = -2, so the statement is true for n = 1. Assuming it to
be true for n =k, we have:
a / - 3 b/ = (-l)k • 2k ••• (2)

Then, a~+l - 3 b~+l = (ak + 3bk)


2
- 3 (ak + bk)
2
, using part (i).

This simplifies to - 2 (a~ - 3b~) and using (2), we have:

a~+l-3b~+l = -2.(-l)k.2k = 2k+l(-l)k+l


Hence the statement is true for n =k + 1 and, by the principle of induction,
the statement is true for each positive integer n.
309

2. (a) Show that for n > 0, 2n + 3 >2 .J(n + I) (n + 2)


n I
(b) Hence, by induction prove that E • r.:- > 2 ( .fi1:;l - 1)
1 vr

Solution: We have:
2 2 2
(a) (2n + 3) - 4 (n + 1) (n + 2) = 4n + 12n + 9 - 4 (n + 3n + 2) = 1 >0
2
(2n + 3) > 4 (n + 1) (n + 2)
(2n + 3) > 2 J(n + 1) (n + 2)
1 1
(b) S(n) = 1+ ./2 + .;J + ••• + v'il1 > 2 ( -vn
1::'"7""1
+ , - 1) ••• (1)

For n = 1, L.H.S. =1 and R.H.S. = 2 ( {'1. - 1) ~ 0.8


So, the statement is true for n = I.
Assume it is true for n =k, where k is a positive integer, then:
~
1 + ff1 + ••• + Vi<1 > 2( vk + 1 - 1)

We add -~ on both sides, then


V" + 1
s (k + 1) > 2 v'i('"":;'"1 + 1 - 2 = 2k + 3 - 2 v'i<Ti
v1<TJ v1<+T
2 y(k + 1) (k + 2) - 2 ~
Using part (a), S(k + I) >
v'i(;1
This simplifies to S(k + 1) > 2 ( ~- I) = 2 ( ./(k + I) + I - 1)
So, the statement is true for n =k + 1 and by the principle of induction, it is
true for aU positive integers.

3. Prove by induction that:


. <"x).
sm (n+l) x
sinx + sin2x + sin3x + ••• +sin nx = 2 sm - 2-
sin ( ~)

sin <1-> sin x


Solution: For n = 1, L.H.S. = sinx, R.H.S. = = sin x
sin ( ~)
So the statement is true for n = 1.
Assume it is true for n = k, where k is a positive integer, then:
. ( kx ) . sm
sm . ( -k +- 1 ) x
S(k) = sin x + sin2x + ••• + sinkx = 2 2
sin ( ~)

Adding the next term sin(k + 1) x,

(cont)
310

. ( -k +- 1 ) x. sm
sm . (T
kx )
2 + sin (k + 1) x
S(k + 1) =
sin ( ~)

Now sin(k + l)x = "(k+1)


2sm - - x. cos (k+1)
- 2- x
2
. ( kx)
:. S (k + 1) = sin ( +)
k 1
x
sm T
[ sin ( ~)
k +- I) x
+ 2cos(-
2
J ••• ( 1)

Bracketed expression =
. (T
sm kx ) + 2 cos ( -k +- 1 ) x • sm
. x
2 2
sin ( ~)

. ( kx ) . ( k + 2) . ( kx ) We use
sm T + sm - 2- x - sm T
= 2cosA sinS
sin ( ~) =sin(A +B) - sin(A- B)
. (k+2)
sm - 2- x
=
sin ( ~)

sin(~)x. sin(~)x
s (k + 1) =
sin ( ~)

Thus the statement is true for n = k + 1 and, by the principle of induction, it is


true for all positive integers.

Exercise 1OE

Use mathematical induction to prove the results in questions 1 to 5.

xn - 1 n-1 n-2
1. x:-r = X + X + ••• + X + 1, X JI
1 1 1 1 3 2n + 3
2. r:-3 + ""2':4" + 3."5 + • • • + -n(;-n--:'-+-;:2~) • 4 - 2(n + 1)(n + 2)
1 1 2n
3

1
• m • 1• 2 • 3 + ••• + 1+2+3+ ••• +n = n+T
4. A sequence {tn} is defined so that

and 2, t2 =6.
Prove that t
n
= 5n-l + 1.

2 2 2 2 3 2 n = (n 2 - 2n + 3) 2n+l - 6
5. 1 .2+2 .2 +3 .2 + ••• +n .2

(cont)
311

I I I and 1 <
6(i)(a) Show that ;; - n-+1 =
n (n + l) (n + 1)2 n (n + I)

(b) Using (a) and the method of induction, prove that:


n I 1
Sn = t - 2 ,< 2 - -
I r n
for n I >
(c) Show 1.4.5 ~ s99 ~ 1.99

(ii)Using the method of induction, prove that for each positive integer n there are
unique positive integers a and b such that
n n
(l+f2)n=a + .fi.b.
n n
2 2
Prove that a
n
- 2b
n
= (- l)n
7. Prove by induction that (l + x)n ~ I + nx, x >- I
8. If u 1 = 2, u 2 = 3 and u
n+ 2
= 3un+ 1 - 2u , prove by induction that
n
n-1
un = 2 + I.

9. Assuming the triangular inequality I o + c I~ Ibl + lei, prove by induction that:


lx 1 + x 2 + x 3 + ••• + xnl ~ I x 11 + lx 2 1 + ••• + lxnl

10. Prove by induction that xn + yn is divisible by x + y for all odd positive


integers n.

11.
3 2
Prove by induction that 2n - 3n + r\ + 31 ~ 0.
12. Using the results nc = nc and n+lc = nc + nc , prove by
0 n r r-1 r
induction the binomial theorem:
2 r n
(I + X)n = nC + nC X + nC X + ••• + nc x + ••• + nc x •
0 1 2 r n
. ( 3x)
smy-sm . ( x)
13. (a) Show that
2 = cosx
2sin ( ~)

(b) Prove by induction that:


. (2n+l)
sm - -2- x 1
cosx + cos2x + cos3x + ••• + cosnx = - 2
2sin ( ~)
312

10.6 Properties of Integrals


y

~
1. Piecewise Functions
Let f (x) = g (x), if a ~ x ~ c
I
= <x ~

r
h(x), if c b

Then Jb f(x)dx
a
=
a
g(x)dx + Jb h(x)dx
c /1
I
I
l
I
I
r
0 a c b X

y
EXAMPLE: (1)

Evaluate J-11 f(x)dx, where f(x) = lxl, -1 ~ x ~ 1.


SOLUTION:
f (x) = Ix I, - l ~ x ~ 1 X

f(x) = -x for -1 ~ x ~ 0
= x for 0 <x ~ 1

f-1
1
f(x)dx = f
0
-1
(-x)dx + f
1
0 X
d
X =

EXAMPLE: (2)
If f(x) = sinx for -n" x~ 0
= x for 0 < x 'n ,

(a) Evaluate fn f (x) dx


-n
(b) Find the area between the curve y = f(x) and the x-axis, -n ~ x" n •
SOLUTION: y
(a) As an integral

I:f(x) dx = I_:sin x dx + C x dx
0 2 '1l
+ [~ ]
[-cosx]
-'11 2 0
-'11 '1l X

2
'1l
= -2 + T
y = sinx
(b) We observe that part of the required area is

r
below the x-axis, so we proceed as follows:

The required area A = -


0
sin X dx + f
-n 0
X dx

2
Using part (a), A =2 + ~ Sq. u.
2
313

r:
CXAMPLE: (3)

The function F(x) is defined as .F(x) = f (t)dt


where f(t) = 2 - t, 0 ~ t ~ 4
=t - 6, 4 <t ~ 8
(a) Sketch the graph of the function f(t)
(b) Find and determine the nature of the stationary points of the curve y = F(x)
(c) Sketch the curve y =F(x); hence find the area between the curve y =F(x)
and the x-axis.

(a)
F(6) =
1 4
O (2 - t) dt +
16 (t - 6) dt
4
t2 4 t2 6
F(6) = [2t - - ]
2
+ [ - - 6t]
2
0 4
= 0 + -2
= -2
10 t
A is (2, 2) , B is (6, -2)

(c) F(x) = 1 (2 - t) dt , 0 ~ t~ 4

(b) F(x) =
X

J0 t(t)dt, O~t~8 = I: (t - 6) dt, 4 <t ~ 8

2
:. F'(x) =f(x) ••• (1) F 1 (x) = 2x - y , 0' x~ 4
The stationary points of F(x) are 2
given by f(x) = 0 • F 2 (x) =y- 6x + 16, 4 < x~ 8
.". 2 - X :: 0, 0 ~ X ~ 4
F(x) =0 gives the intersections with the
or x - 6 = 0, 4 <x ~ 8 x-axis, i.e.
So the stationary points X :: 0, 4, 8
are where x = 2 and x = 6.
The sketch of y = F(x) is as shown in the
From (1 ), F"(x) = f' (x)
diagram.
Now f'(x) = -1, 0-' x-' 4
and f' (x) = 1, 4 < x ' 8 .
••. F"(2) < 0 and F"(6) > 0 y
Hence A(2, F(2)) is a maximum A
point and B(2, F(6)) is a
minimum turning point.

F(2) = I: f(t)dt = I: (2- t)dt


B
8 X

2 2
F(2) =[2t - T] 0 2
The required area
4 2 8 2
A = f0
(2x - y)dx +-
f (y- 6x + 16)dx
4
16 -(-16) 32 sq.u.
3 T 3
314

2. Inequalities involving Integrals


Many important inequalities such as:
1 1 1
l. 2 + 3 + 4 + ••• + n1 ~ logen
4
2. 13 + 2 3 + ••• + n 3 ~ T
n , etc. can be proved by using the following

property of the integral f


a
f(x) dx.

y
y

f(l)

f(J)

0 2 3 ••• n-1 n X 0 2 3 n-1 n X

Increasing function Decreasing function

Using the diagrams we obtain the following inequalities:


I. For f (x) increasing:
Area of inner rectangles ,< r f(x)dx ~ Area of outer rectangles

r
1

f(l) + f(x) + ••• + f(n - 1) ~ f(x)dx ~ f(2) + f(3) + ••• + f(n)


I
2. For f(x) decreasing:

f(2) + f(3) + ••• + f(n) ~ ( f(x) ~ f(J) + f(2) + ••• + f(n- I)

EXAMPLE: (I) Prove the inequality: ~ + 1 + ~


1
SOLUTION: Let f(x) = - , 1 ~ x ~ n
X
y
The L.H.S. of the inequality is a lower sum
for the integral
n 1

i I
- dx
X
= logen, then using: f(x) = .!.X is a
decreasing function
:. f (2) + f(3) + ••• + f(n) ~

I l I
r
I
f(x) dx

n ~ logen.
+ + ••• +
2 3
Note that the function f(x) is chosen by
2 3 ••• n-1 n x
replacing n by x in the general term of the
given series.
315

Exercise 1OF

Find the integral of the function f(x) on the stated interval:


1. On [0, 5] , f(x) = x + 2 if 0 ~ x ~ 2 and f(x) = 4, 2 <x ~ 5
2. On [0, 2] , f(x) = Ix - II
3. On [- 11 , 11] , f (x) = Isin x I
4. On [ -11, 11] , f(x) = cos x, -11 ~ x ~ 0,
= X, 0 (X~ 11
f
5. The graph of the function
f(t) = 1 - t, 0 ~ t ~ 2
= t - 3, 2 <t ~ 5
is shown in the diagram.
t
The function h (x) is defined as:

h(x) = f: f(t)dt, 0 ~ x~ 5
-1

(a) Find the function h(x) over the


intervals 0 ~ x ~ 2 and 2 <x ~ 5
(b) Find all the turning points of the function h(x) and hence sketch it.

(c) Find r 0
h(x)dx

(d) Find the area between the curve y = h(x), the x-axis for 0 ~ x ~ 5

Sketch the function f(x) and prove the following inequalities:


1 1 1 1
6. f(x) = - : 1 + -2 + - + ••• + - - ~ log n
x 3 n- 1 e

7. f(x)
1
= _/'::':
vx
.Pr
VL
1
+
VJ
1
_rrf" + ••• + - ~
vn
1
2 ( rn- 1)
8. f(x) = rx : .fi + Vz + ••• + Vo ~ f: X dx

3
n
9. f(x) = x 2 : 12 + 2
2
...... + (n - 1)
2
~ T ~ 1
2
+ 2
2
+ ••• + n
2

10. f(x) =--1- 2 : Subdivide the interval [ 0, 1] into n equal parts. By


1 + X
considering the values of f(x) at x = O, ~ ~ ~ prove that: *, ,. . , ,. . , ,
n 2 2
n n 1 )
- r 2 2
(Hint: 2 2 :: 2
n r = I n + r n + r + (r/n)
316

Appendix 1
Bisector of an Angle of a Triangle
Theorem: The bisector (internal or external) of an angle of a triangle divides the
opposite side (internally or externally) in the ratio of the sides containing
the angle bisecte_?_. p
1
,' I
A,"
, I
I

B
Lhl (Fig. 1)
D C B
(Fig. 2)

Given: AD bisects L BAC, internally, in Fig. (1), externally in Fig. (2), and cuts
BC (produced) at D.

Prove: BD AB
DC = AC
Construction: Through C draw CP parallel to DA to cut BA (produced) at P.

Proof: Take any point E on AB in Fig. (1) and on BA produced in Fig. (2).

Then: LEAD = LAPC (AD // PC, corresponding L s )


LDAC = LACP (AD // PC, alt. Ls)
But, LEAD = LDAC (given)
LAPC = LACP, so AP = AC

BD BA
Since AD 1/ PC, DC = AP

But AP : AC, :. ~~ BA
= i\C

Corollary: If the base BC is divided internally (or externally) at D in the ratio


AB : AC, then AD bisects the angle SAC internally (or externally).

The proof is similar to that of the theorem above.


317

Appendix 2
Rectangular Hyperbola: xy =c2
y
~
''
''
''
X X
2

(Fig. 2)
(Fig. 1)

2 2 2 ••• (l)
Consider the rectangular hyperbola: x - y =a
Its asymptotes are the two perpendicular lines
X - y =0 and X + y =0 (Fig. 1)
Let X = PM and Y = PN be the perpendicular distances of any point P(x, y) on
the conic, from these asymptotes.

X !...:....l
.fi
and y = ~
Vi
[ • , ax 1 +by 1
2 2
•<] ••• (2)
fa + b

XY =
X
2
-2 ~ 2 = T•
a
2
as P(x, y) is on the conic.

2
Putting c
2 a
= y, the equation of the rectangular hyperbola x - y
2 2
=a 2 referred
. asymptotes as co-or d"mate axes IS
to 1ts . XY =c 2•
Alternatively, using complex numbers and taking P(x,y) as any point on the conic
2
2
x - y =a 2, Iet X + 1. Y represent t he pomt
. P' (X , Y) a f ter OP 1s
. rotate d
anticlockwise through 4 5°.

X + i Y = cis(i-> • (x + iy) = ~(I + i) (x + iy) = x~-:/

X = !...:....l and Y = ~
.fi .fi
The effect of the rotation used in this method is shown in Fig. 2 - where the
orientation of the hyperbola and its asymptotes is that of the familiar type
2
xy = c •
318

Answers
(All answers are given to three significant figures)
319
Chapter 1: Curve Sketching Exercise 1A
I (a) y (b)

y = sin x + cos x
,/

"'
,~----
"'
/

--
_., /
X
X

(c)

Y =X +COS X
(d) y,v.·\
\ //
/
\ /'
' ,><.... ...... _--
X
-- ........-1 / 0

/\ y =X+-
I

X /A\
~ ~\
X

2 (a) y (b) I
I
I
= sin x - cos x I
__,
/I
I

(d)

(c) y

y = X -COS X
-- X

y = X - -XI

-
320

y (b) l (c)
3 (a) .I
I

X
X

"----··-r
,_ -ll

4 (a) (b)
y
y = cos(x-1)
. . . -r Y=e
X+l

0
/ " X

0 X
(b)

....
X

, ,0 X

-ll ./
I
. .
Mammum at
I ( 1
-,
I
--)
e e
6 (a) y /
," sin x
Y =X-

/
/ " , ,~

---~--
1 ""
X X
/
/ X - 2
y = iC-+"2
""
7 (a) I
\ (b)
I
' \
I
I
\
\
\
\
X
\ I
2
I y = (log x)
e
I
X
321

8 (a)
8 (b)
y y
y = .Jsin x
,
3/2
y = X

X
0 0 n 2n

9 (a)

_
-x
..... = e
,Y

-n 3n n
-4 -~

Max, ( o/f, 2) y = e-x smx


.

-2 Min • (- If. 2)
9 (b)

0 1\
'

\
2
o o O , . , . , , \oo•• o

, -
3

/
I
Ooo oo , / . o -
/4

y
• o •
.5
•• •
6
. , o • - oo

= I'}I - sin (x - I >I


X

9 (d)
y
:v,
I
I
I
I
A
,
,
,
,,

9 (c) "
' \
\ 2
\

2 X
I
I
I
I Min. A (3.4, 5.8)
I
1 Max.B (0.6, 0.2)
I
I
X I
I

2 -x
y =x e
2 X
2 X

I X
2 (2 +X)
I y :
I 2-x
3
I
y = (x - 2) + I
-7

')i
______ 1. ____ 3 _ _!L
II (a)
y

= 2lx - II 8
I
I
I
I

-) - 3 4 X
I
I 2
I 3x - 7
y = (x- 2){x + 3) 3 4 X
I
I
I

x~O or x?4
(b) X =-3 , X = 2
I 6 . 14
(c ) Max. ( J ,
5) , Mm. (7,
5 )
(d) See the graph above

(e) • IY I .(. n /2
(f) 2.66
323

11 (b)

12-------:~r-~--------~--~--------------------
( ) E.!1 (b) 471 13 y
a 15 3

3 X
X

(c) 1.6 (d) 1.66

14 Yt
15 I (b) A = 1, B =
I
(c) 1oge6
I

-: 3 4 5 X

\
2 -I
) -2 3x + 2
y = (2x - I Hx + 3)
I
I
I
X

16
y

Root: 0.6, 0.57

-1 0 2 X
324

Exercise 18

~-
1. (a) (b)
3'\
I
2~ v \ !/
v IV .. X
(c)
-2
0

-1
y ""' 0
2

1 2
X

(d)
0 1.5

y
3

y X
2

y -2

""' ""-
-4 -,L -11 u 1 2 X

(e) y
6

~ 4 '/
~ 2 L
3 -2 - 1 0 1 2 3 X

2. (a) y
v (b) Y.l'
v
0
1

v "-~2 - 1 1 2L
X
1~
~
X
- 1 1/ 0

I' /
'
Y.i-
¥1'

v
l
c)
v /I (d)

""-
-1
~ - 1 "" X

-2 I""
v
/1

"'0
-I
l X'

/ ~
325

(e) y {f) y
I/ /
"
1/
0
~
1"' X
"'
-1 -1 1
/1
0 X

v
(g)
'\~2 (h)
/
"'" I

~ I
"'
y
...
-2 X
/2
.-2 1\.
"'0

~ I 1
-211 0
- 1
)(

I -~

3.
y

2 e--C
I

0

__.
'
...-o'
I
-3 -2 -1 ' I 2 3
X

..-o -2
(P) (q

(::>) ('e)

/
X ( z 0
( z- £-

z
A
/ (

(q) 9 (l!) ~

zq + z-e = -edJ\1
X !;""0 0 x, q 'e 0 q-

•r
VN ·~
327

8 (a) A y (b) y
2
Max. A (- J , 5.6)

(c) (d)

(e)

-1 0 X

9 (a) y
~)_/;'
_____ I_ 2_ .. -~.=..L __ _
I
I
12 )X

(d)
X

_j X 0
I

X
(!)
\j ~
v.
7
r
(4)
!.
X 1I 0
X ,....1,....
/
I

//
~I I
329

10 (c)

"T X
X
------~--~-~~- - - --2 ·-

(a) See Section 1.5, page 13, Example (c)


(b) (c)

(d)

(f)

X
ll 2n 3n 4n

f(x) = f (-x) = si~l x , the


graph is symmetric about the y-axis.
(h) y (i) Y n/2

X X

-n/2
331

12 (k)

12 (m)

13 (a) (b)

-1
X

(c) (d) y

(e) (f) y

(1nl4, 0.003)
i 2n
0 I 4 5 6 .
(3m4,- 0.067)
X
-x
y = e COS X , 0 ~ X~ 211
y = 0 for x = 11/2, 311/2
Minimum at (311/4, - 0.067)
Maximum at (711/4, 0.003)
(d) y

i\__/ A
I
X 2 e X

Min. at A (e-1 , -e-1)


Min. at A (e, e)

y (f)

1
Max. at A (e, e)

X A

0 X
2 2
Max. at A (e , e)

:) (d) y

X
-----,--1

~~
333

16. y

0 <X ( I , y' -1.39


1
( ) Minimum A (
2 , - 1.39)
Asymptptes at x = 0 and
X : 1

17 (c) (d) y
c 1 (x) is

~Jog n: :J
-I I X
(a) 1 , -1
(b) All real x , Iy I < 1
(c) Inflexion at x = 0.

18. (e) Y

(a) IX I ' 1' I2Yl ~ ¥


(c) ~-~ + sin- 1 <i)
dx- ~
I -X
X
(d) Inflexion at x = 0

~ does not exist at x=!l

19~- (a) (b) y

I
X '·- _.,.'

~I
(c) (d)
y
c) (a) Max. at A (e, eI )
(b) Asymptote at x = 0

0 X y ~ 0 as x 4 co.
-I y -+-co as x -+ 0 .

~),-----
1.
Intersections with axes.
8 .
B(4, 0), C (0,
1 ) y -+ 0 as x + .:!: co

Min. at A (I .8, 0.7)


--------I 1 A p
Max. at P (6.6, 0.2)
I
0 3J
I
I
I
I
I

Asymptote at x = 2
1
Min. at A (0,
4)

0 2
~3. I 24.

~
y
I
I
I
I
I1 X

\i I

Min. at 0 (0, 0)

Max. at A(2, z)
4

-
e
y 0 as X ~ +CO

y -+ co as X -+
335

~-I,
• : Y. ~ 26. y

I
-----~---------
1 :~
-~-------

X X
I

l
p (1, 6)
Q(-1, 6)
B(-1, -6)

27. Y. 28.
A (1, I)

Min. A (0.5, 0.67]

B(-1, -I)
-In,
0 :2 I
I
X

Max. B (0.5, -0.6


I
I

29. 30.

X
I
I
A B
------2•_.L_ t2- - - - -
Q_ ,_ _ _ _ !__
----- --r-
31 (a)

0
y

X
I
I
I
i
-'i/
Asymptotes x = .!. 2 (vertical)
y = .!. I (horizontal)

31 (b)

0
y

X
31 (c) r
0 (0, 0) is the point on the graph

)
0 12
I

'(\:
t. :
I '
337

(b)

II
I
I

A is (-1, -2)
CHAPTER 2 INTEGRATION

Exercise 2A (constant omitted)

(x2 + 1)4 - (1 - l>5 4


1. (a) (b) (c) loge (x + I)
4 5

(d) _ (x4 + 0 -1 (e) _.! (I _ x2)3/2 (f) -1 2


tan (x )
3

(g) Ft (h) - 4 /1- rx


3 l . 4 I 5
cos x (b) (c)
2. (a) --3- 4Sln X
5 tan x
-I 5
(d) - 5I cot 5 x (e)
cosx
(f) - 5I (l + cosx)

I 5 I 3
(h) + tanx)
(g)
5 sec x 3 (1
3 . -1
l X - tan- I (e-x) (c) sm x
3. (a) 3e (b) e

(d) log (ex+


e
~)

4. (a) I
2(logex)
2 (b) 3I (log x) 3 (c) - I (logx) -2
2

-2
(I + logx)
(d)
-2

_.! ecos 2x -1 . sin x


5. (a) (b) tan (cosx) (c) 2
4 log 2
e

(d) 1 l o[ex
-
2
g- - -
1
ex + I
1
I I
6. (a) -16 4 (b) (c)
16
(f) l
(d) 0.367 (e) 0.(,2 2
338

CHAPTER 2 INTEGRATION

Exercise 2A (constant omitted)

(x2 + 1)4 25
-{l-x) 4
I. (a) (b) (c) loge (x + I)
4 5

(d) _ (x4 + 0 -1 (e) _! (I _ x2)3/2 (f) tan


-1 2
(x )
3

(g) ./T"". (h) - 4 /1- rx


3
cos x I . 4 I 5
2. (a) --3- (b) ij SIO X (c)
5 tan x

(d) - 5I cot 5 x (e)


-1
cosx
(f) - 5I (I + cosx)
5

5
(g)
5
I
sec x (h) J1 (I + tanx)
3

3 . -1
1 X - tar\ 1 (e-x) sm x
3. (a) Je (b) (c) e

(d) Jog (ex+


e
~)

4. (a)
I
2 (logex)
2 (b) 3I (logx) 3 (c) - 2I (1ogx)-2

-2
(I + 1ogx)
(d)
-2

sin x
5. (a) _! ecos 2x
(b)
-1 .
tan (cosx) (c) 2
4 Jog 2
e

(d) -I I og [ex
2
- -- I
ex + 1
1
1 (b) 1 (c)
6. (a) -16 ii T6
1
(d) 0.367 (e) 0.62 (f)
2
337

(b)

y
34.

I,
I
I

-1

I
I
A is (-1' -2) IIA

fl
339

Exercise 28 (constant omitted)

2 I X .
e-x (-x - 1)
2e
1. (a) (b) ex (x - 2x + 2) (c) (sinx - cosx)

2
2. ()
a 21 x "2 x + 1 cos 2x
sm
4 (b) - x cosx + 2x sinx + 2cosx

2
(c) x sinx + 2x cosx - 2 sinx
2
2
3. (a) x tanx + log cosx (b) 4X - 4X Sill

X - 8I COS 2
X

(c) -x cotx + log sinx

4. (a) (b)
2
x
3/2
(Jogx-
2
3> (c) x [ (log x) 2 - 2 Jog x + 2]
3

5. (a) X COS-I X- ;;-7 (b) x tan-1 x - I log (x 2 + I)


2
2
(c) 2x -1
tan x -
I
2x +
I
2 tan
-1
x

6. (a) x log (x
2
- I) - 2x - Jog [~ : ~] (b)
3
TI e
-2x .
(sm 3x -
2
cos 3x)
3
2 2 2 2
2 - log x + I
(c) 2x (logx)
2
-
x
2 logx +
x
4 or X
2
[ (Jog x)
2]
2 2
7. (a) x tanx + log cosx - x (b) \ + i sin 2x + i cos 2x
r;:-)J ()
2

8. (a) ~~x /xG) +Jog (x + {xL + 1 b


2
r---7
I x {4- x"- + 2 sm x
. -1 2

2
(c) e I
9. (a) (b) 4 +4
I 112 + lJ
(d) 11
4 - 2 (e) ! (e 11/2 - I) (f) _1_6_
2

11
10. (a)
3 (b) /2n - 4

n I (c)
12. (a) .{3 - log 2 (b) log 10 or log I Oe -2
e
2
(d) (e) ~ Oe 2 - I) (f)
4

I -3 3 -2 I
(g) (h) - 4e ( i) _ iie +4
4 9
340

1r 1 1 TT
(j) 4 - 2 (k) (1 - log 2) (I) 2 - 1
2
2
(m) 11 + 411 - 8
64

Exercise 2C

1.
2 2.
1 311
3.
11 2
3 4 + 32 4 - 3

sin 4x
5 3 7 5 3

X
8 - ----rr- 5.
COS X COS X
-5- - -3- 6. Tu - -2u
5- +
u
3 , (U = sin x)

2 u7/2, (U = sinx) u3 1
7.
7 8. - 3 + 2U + D, (U = cosx)

2
cot x
9. --2- - log sinx I 0. 21 (log tan
X
2 - cosec x cot x)

11. -u - u 3 (U = cotx) 12. 21 SIO


. 2X 1 .
- (; SIO
3 2
X
3
13.
1 2 I
4 tan 2x + 2 log c:os 2x 14. - ~ cos 2x + t cos
3
2x

15. _ ~ u5/3 ~ ull/3 _ u17/3 .1. (U = cosx)


5 + 11 17 '

u5 u7
16. - T (U = sinx) 17. - 31 cot 3 x 18.
1
U+D (U = cosx)
T ' '
2
19. log (sec x + tan x) 20.
- tan (x/2) or tanx + secx 21. 13- 1

22. 11/3 13 23.


n
I log [ t + I - /'i
t+l+n
1 t = tan[~ J
1
24. 25.
- Jog (2 + cosx)
4 log 3

Exercise 20

_.1 n -2x n 11
1. e X x ll - n 1 _ 2• , x e + 2 1n-1 3. x (log x) - n I
11 1 n- 1

4.
a
2
(2n - 2) [ (x 2 • xJ 2)n- I + ( 2n - 3) 1n - I ]
341

5 3
5. (i) (ij) - 5u + Ju - U - x , (U = cotx)

6. (a)
c 5s
-6- +
3
5C S
--vi'" +
15CS
48 +
15x
48 (C = cosx, S = sinx)

(b)
3511
256
3511 311 I
7. (a) (b)
32 - 4
256

t4 t2 11 76
8. (a) 4 - 2 - log cosx , (t = tanx) <b> ii - m
3
n . n-1 11
9. (a) In= U smU + nU cosU- n(n-1) ln_ , where x = sinU
2
{b) 8- 311 .,. 6
32
10. -m II.

e (x
X 4
- 4x
3
+ 12x
2
- 24x + 24) 13.
28
12.
TI

Exercise 2E

I. log(x +~) 2.
~ 3. . -1 (X) X· r---'f
8 Sin 7i - 2 V 16 - X~
9x

4.
~ .-lx
.5. 2
25 . -1 X
3 - 2 V 25
X ,---z
- X~
- X - Sin 2 SH\

~ ~)
2
6. /lJ + a22 log(x + 7.
a
2 sm a . . ;~
. -1 x

8. ~ ~- 21og(x+ ~) 9.
X~
2V4
+ x' + 2 log (x + v::-:;,z)
10.
2.5 . -1
sm 5X +
X J 2.5 - x
2 II. ~ (4 + x 2) -l/ 2
2 2

12. -2 11 [;;;-::;
og x
2] + 13.
~ 1~.
~X

log (
I - ;;-7) + r---2
VI - x~
I 5.
I+
r--2)X
Y'l- X ~
or - log ( ~ + I - x

17. ~ ~ + 2 log (x + v,;r::-;) 18.

19.
2
I sec- I "(x)
·
2 20. ~ X
+ logj.;:z:: + xl
21. ~ ~ - ~log (JT:9 + x)
342

Exercise 2F

x4 x3 x2
I. 2x - J4 log (3x + 2) 2. -4 - 3 - 2 - x - log (x - I)
2
3. T- x + log(x+l) 4. - x + log (l - 2x)

ax 1 (3x 2 + 2x) + 1 log (3x - 1)


5. c + be - ad log (ex + d) 6.
18 27
2
c

V3 an-1 1) V32 tan-1


2 t (2x + (2x- 1)
7. \7)": 8. ----v)
I -1 x +a 2 -1
> 0,
9. .
Vb - a 2
tan
;:==:=;; if b - a
x +a

and

10.
-1
tan (3x - l) I I. 2log(x
2
+ 2x + 3)
1
-Vi tan
-1 ex\1'21)
+

12.
1
2
2 1
tog(x - x + 1) +73tan
-1 (2x - 1'I 1 2 1 r~ + 1 -
~/ 13. 2log(x + 2x- 2)- Wlogl:X +I+
.J3
\lj J
2 -x+1) + 2 -1 2x - 1 n
14. x+1og(x ~ 15.
73 tan 4
1 1 (3 + "'->2 ~ 1! -I
16.
2
{'i og 7
17• 2 +613 18. log 2 - 5 tan

19. log 2 20. + 1 (a 2 - b 2) + (a - b) + log ( ia-


):l 1)
2

Exercise 2G

2. log [<x - 2); J


(x - I)

3. x + 6 log (x - 2) - 3 log (x - I) 4. - 9 log (x - 3) + II log (x - 4)


2
5. 2X - x + 5 log x - 4 log (x + 1) 6. 3I log (x - 2) - log (x + 2) +
3
9
log (x + 3)

I I 2 I -1 X
7. 3 log (x + 1) -TO log (x + 4) +TO tan 2
8. x + 2 log x - log (x
2
+ I) - tari
1
x 9. log (-x ) -
X - I

I 0. 2 log (x - 2) - 2 log (x + I) - 3
9
4( x-::-2/
9
I '\ II. 4I log (x
2
- I) -
I
4 log (x
2
+ I)

1 I 2 ' I -1 2x + I
12. x +J log (x - I) -
6 log (x + x + I) - tan
V3 Vf
343

1 1 2
13. 161 log (x + 2) 8 (x + 2) - 32 log (x + 4) 14. tan
-1 X _ 1
..12 tan f l
-1 X

3 5 2 9 -1 2x + 1
15. 8 log (x - 2 ) +
16
log (x + x + 2) + fi tan
ff
8
16. log (j)
1 3
18. -
2 Jog 2 + 41 Jog 45 +
1 -1 I
2 tan 2 19. log f1 + -
1
-)
\' 2 V2
20. log {j) 2 I. kJog (: ~ :) -
2 (x
X
2
- I)

22. 2 1/X - 2 log (1 + .fi)

Exercise 2H

J. tan-!(~)
- 1- 2. log (x
2
+ 2x + 4) 2 tar1 1 (~)
V3 '1/3 v'3 V3
3. x - log (x 2 + 2x + 4) - - I tan-l(x+l)
-- 4. log (U + Vll:J> , U = x + I
11'3 v'3
2 2
5. Vx +2x+4-log[x+I+Vx +2x+4] 6. sin- 1 (x-l)

7. 2
-t a n-1 4x + 1 8. lx
2
+x + 1 + ~ Jog (x + ~ + lx 2 + x + I)
V39 v'39
9. r---"2
- V6x - x' + 3 .
Sin -Jc -X- -
3
3) I o.
. -1 (2x +
Sin \;:-3-
1)
11. ~log (x
2
- ~ + fx 4 - 3x2 +I) 12. - /1 - 2x- x
2
- sin 1 (~1 )
1 3. - 2 ru + 3 log (x + 1 + Vtl) ' U = x 2 + 2x + 3 15. tan-
1
3- tan-
1
2

16.
211
T +
1
log
4
17.
11
18. r--2 3 sin-1 ( 2x-1 )
-ojx-x-+
2 3 2 2
19. si.n-l x + {t7 20.
2
/x + 2x - log (x + 1 + .r;;r-;;x)

Exercise 21

I. (a) 0 (b) 1/2 (c) 0 (d) 0

(e)
2 (f) (h)
0 (g) 8 0
3
3
11 I 11 11
2. (a) (b) (c) (d) (e)
4 840 96 4
3. (a) 2 - 112 (b) 2
.;ab tan
-1 Ifa
344

Exercise 2J

16
I. 32 (x - 2) 3/2 + 4 (x - 2)
1/2
+c 2.
m 3. e
2
- e + log (I + e)

4. -0.059 5.
I
1og (a
2 2 . 2
+ b sm x) 6. ab b tanx~
I tan-1 ( a
2b

tan-! (.JX) I -1 2
7. 8. 9. x - log (I + ex)
2 tan X

2 3
X X
10. x - 2Jog (I - ex) II. 3.3 12.
3 Jog x- 9
13. ~ (! + x)5/2 -l (I + X
)3/2 14.
,;;-:-; 15. -I
sec X
X

1 -1 I
19. 1i tan (x +
2)

(x + ~)

25.
2
3 tan
-1 [4 + 5 ~an x/2 J 26. 12x
TI - TI5 log ( 3 cos x + 2 sm x)
.

27. a sm
. -1 ~
a - /i- x
2
28. 2x - I
-4-
.;pP + 83 log (-2x
-2-
- I
+
VP)
p ' P = X
2
- X + I

29.
I
--log
V5 tan x- I 30. - (4 tan x + 6f 1
2./5 \1'5 tan x + I
31. - X + 5 tan-J (2 tan~)
3 6 32. 2Vx sin rx + 2cos rx
2
X
33. 3 log (x - 2) - log (x + I) 34. 2 + 3x + 8 log (x - 2) - Jog (x - I)

35.
I
2 log [~]
X 3 +
36.
I
2 log [-) ~:21
I 1 2 1 -1
38.
2 1og (x + 1) - 1i log (x + 1) +
2 tan x

39. log (I + sin x) - log (2 + sin x)


I
40.
6 log (I - cos x) + 2.I log (I + cos x) -
2
3 log (I + 2 cos x)

n I 11
4 3. 8 - 4 44. 8
345

CHAPTER 3 VOLUMES

Exercise 3A

(1) 211 (2) 20611 (3) 11


15 30

(4)
i (5) 11
2
- 211 (6)
11
11 (1- 4)
2 8
2
51211
(7)
IT
(8) 11
16 (9) 211 [(log 2) 2 - 21og 2 ~ I]

-2
(IO)
11
4 [e: - 2 e
+ 2] (II) 5611
T (12) 12811
-5

51211 112
(13) (14) 411log2- 1.511 (15)
15 T
(16) (a) 144 (b) 36 v'3 {c) 1811
(d) 72 (e) 36

2
411 ab (18) 40011
(17)
---r- -3-

Exercise 38

(I) 108811 (2) 16211 (3) 11


-15- -5- 5
2711 (5) 4011 1611
(4) (6) -3-
2 3
(7) (8) 6411 (9) 811
811
5 3
(10) 211 (II) 11 (e 8 +4e 4 +3 )
4

Exercise 3C

3211 311 6411


(2) (3) (4)
3 TO 3
(b)
81! (5)
(c) 3211 4811
3
3
81Th 3 (9) 32na
(6) (8) !3.25 m _!_5_
15
346

J
2 2 16na 2 2
10. 2n a c 11. -.5- 13. J20n 14. 200n

3 51211 117n 8
15. 4113 16. 17. -5- 18. ;
15
3 3
.5na 4na 112n
19. (a)
-r (b)
---r 20.
15

CHAPTER 4 COMPLEX NUMBERS

Exercise 4A

). 5 - i 2. 2 3. -15 + i
4. 4i 5. 2 + III 6. II - 23 i
7. 3 + 6i 8. 7 - 24 i 9. 2i
2 3 .
10. - 25 + 50 I II. 5i 12. 8

19 + 4 i 2 3.
13. 14. _2_9_ 15. -5 - 51
5
4 , y =2 I
16. - 2 + i 17. X o 18. X =3 , y =- 2

14 23
19. (x = I, y -= -I) or (x : -1, y ' I) 20. X = - 25 ' y = - 25
5 13 25
- 25 .,fj i
21. 2 + II i 22. X = 2' y =- 5 2 (a) -2 2
I V3.
(b)
TO - 10 1 23. (a) 10 + 3 i (b) 2 + 2i

19 + 4 i
(c) (d) I + IIi 24. (a) X - iy
29
2 2
~ (c)
X + ~ - 1 - 2~i 2 2
(b) (d) X - y - 1 + 2xyi
2 2 2 2
X +y (x - 1) + y
I 3 . 9 7 . 2 2.
25. (a) 2-2 1 (b) 2+2 1 (c)
5 - 51
347

25. (d) ~ + ~i 26. (a) _!2 +


-
v'Ji
2
(b) .:!: v'Ji

(c) 3 + ./'li (d) 27. (a) x


2
+ I =0
4- -4-
2 2 2
(b) x - 2x + 2 = 0 (c) x - 4x + 13 0 (d) x -(4+4i)x+l0i=0

(e) 5i - (12 + 4 i) x + 5 = 0 28. (a) z = 2, w = 3


3 i 3 i
(b) (c) z =8 + 4 •w =8 - 4 (d) z =I + i, w = -1

(a ) 2 1. (c ) 3 4. 78 96.
29 • 5- 5 1 (b) 3 + 4i 25- 25 1 (d) 25 + 25 1

(e) 2 + 11 i (f) - 7 + 24 i

30. - 6, Rule Va . Vb = ..raE is not defined for the imaginary numbers.

Exercise 48

I. 5cis 0 2. 5cis n 3. 5cis n/2


4. 5 cis (- n/2) 5. 2 Jicis (n /4) 6. 2 .ffcis (-11/4)
7. 2 v"i CIS (3 n/4) 8. 2 .Ji cis(- 3n/4) 9. 8 cis (3 n/4)
I 0. 20 cis ( n/6) 11. cis (2n/3) 12. 5 cis(- 2 n/3)

13. ~ cis(-n/4) 14. 2cis( 9) where tanS=-~ and 9 in 4th quadrant

15. 2 cis(- n/2) 16. 5i 17. 10


5 5..;3.
18. -4 19. -2i 20. 2--2-'
21. ..ti(~ + ~ y 22. I + 23. 5 ~~ - ~v
24. 5 (I - .fj i) 25. 2 (-I + .f3i)

Exercise 4C

1. 8i 2. -10 3. -6 .f2(1 + i) 4. - 5 t/3 + Si


5. -2 + 2 ./Ji 6. 27 i 7. -1- .f3i 8. 2i
1 . 7n
9. - .,fj + i 10. ~(I - >/3i) 12. (a) cis ( n/2) (b ) \i'iClS T2

(c) 2 V'I cis ( ~;) (d) "; cis (-n/4) 1.3. (a) - 4 V3 '+ 4i (b) 2i

(c) - 8- 8 VJi (d) 512-512Vli


14. z = 2cis(n/3), z = 2cis(n/6) (a) 32 cis(- n/3) (b) 16 cis (2n /3)
1 2
(c) 1024cis(n/2) (d) cis(n/6) (e) cis(- n/6)

15. (a) z
1
+ z2 = V2 (1 + v'3) cis (n/4) (b) z
1
- z
2
= Vi ( '113 - 1) cis (3n/4)

(c) - (2 + 113) i (d) - 7 - 4 V3


348

Exercise 40

J. 2. 64i 3. -I 4. 64

5. 6. -8-8VJi 7. 8cis(-11/2),-8i

8. I cis9, w here tan 9 -24 (9 m


"--::'7 . 3r d qua drant ) - 7 - 24 1.
25 625 625
9. 4cis( 11),- 4 10. 16cis(-211/3),- 8-8 t/3 II. 32cis(11/2), 32i

2 ~6 (- I - ~i)
1
12. cis(-211/3), 13. 16cis(-211/3),- 8- 8 VJi
5 12
14. 512 (- V3 + i), 1024 cis(511/6) 15. 8 V'i cis (-11/4), 8- 8i
16. 32cis0,32 17. 2cis 0, 2 18. 16cis(-11/3), 8-8 .IJi

19. i cis (-11/ 3), i (I - v'3 i) 20. 9 cis ( 11/2), 9 i

Exercise 4E

J. (a) .± 2 {2(1- i) (b) .± cv'J + i) (c) 2cis(- 511/12)' 2cis (711/12)


(d) .f5cis(9/2), /5cis (n + 9/2), where tan 9 = ~ , Cartesian form .:!: (3;;. i)
vr3 . v2
2. (a) - i, :t -f + ~ (b) - 2 , I .! V3 i
2k11 11 .r::
(c) 3cis - - + k" 0,1,2--+ 3cis(2o•, 140•, 260•) (d) 2i, t. v3- i
3 9
3. (a) _I_ (+ l + .)
.J2- _l
(b) v'2 (.± I .±. i) (c) VJ - i , I + J3 i, - .fJ + i, - 1 - ,fj i

(d) cis(k;- ; ) . k = 0,1,2,3-+ cis (-15•, 75•, 165•, -105°)


1
2 11
4. (a) z = 2cis ; , k = 0,1,2,3,4-+ z = 2cis(o•, 72•, 144•, 216•, 288°)

(b ) Z o
. (11 + 2k11"\
2CJS - -- ; , k = 0 , 1,2, 3,4
5

(c) z o. cisQ;
11
+ ~). k = 0,1,2,3,4

(d)
2
z = 2cis( ;
11
+ to) , k = 0,1,2,3,4
II 3 . 2k • 9 r7" . I
where r - v 5, t<m9 =
2 , the roots are 1.29
5. (a) r Cis -- - + 0.201 i,
3
- 0.820 + J.02i, 0.472- l.22i
(c) cis n ~ k
(b) 2 11 1
k
- 3, i
= 0' 1' 2' 3 -+ ..fi (.:!: l .:!: i)

(d) 2cis ( k; - i), k = 0,1,2,3 (e) 2 1/4 CIS


. ( -
n g
g, 7n ) ,k = 0,1,2,3

(f) .:!: 2, (.:!: 1 .:!: .fS i) (six roots)

(g) cis( k2
11
- ~ 2} k = 0,1,2,3 .... cis(-15•, 75•, 165•, -105•)

(h)
3
cis (- .!J , 4 n) . . .:!: ~ ( 1 - i)
349

Exercise 4F

(c = cos 9' s = sin 9' t = tan a)


3
I. (a) 4c
3 - 3c (b) 3s - 45 3 (c) l.!._:_!_
2
I - 3t
5 3
2. (b) 16s - 20s + 5s

6 4 2 2 4 6 (b)
3. (a) c - 1.5c s + 1.5c s - s
3 5
(c)
6t - 20t + 6t
2 4 6
I - I 5t + I 5t - t

4. I 3
(a) 'ii cos 39 + 'ii cos a (b) - 'ii1 Sin
· 39 + 'ii3 Sin
· a

5. (a)
1 ~ (cos 59+ 5cos39 + 10cos9) (b) 1 ~ (sin 59- 5sin39 + 10sin9)
1 ~ (~sin 59+ j sin39 + 10sin9) (d) - T6 <3 cos 59- 3 cos 39 + JOcosa)
I I 5
(c)

I
6. (a) (cos69 + 6cos49 + 15cos29 + 10)
32

(b) -
I
(cos69- 6cos49 f 15cos 29- 10) (c) 5TT (d) 5TT
32 32 32
I 7 21 35 (b)
16
7. (a)
64' 64 ' 64 35
64 '
I 7 21 35 16
8. (a)
- 64 '
(b)
64 ' - 64 ' 64 35

Exercise 4G

]. (a) ± (2 + i) (b) !.. (2 - i) (c) .± ( 116 .. i) (d) .;1; (3 - Vi j)

(e) .:!:. (3 + 2 i) (f) .±O+il (g) +-h(l f i) (h) .± (2 - 2 i)


- 2
2. (a) .!. (I + 4i) (b) -+ ...L( .fS' + i) (c) .!. (I + i)
12
3. (a) 2 - 3 i, - I - i (b) 2 - i, - 4 - 3 i (c) -l,-3-2i
(d) - I - i, - 3 + 3 i

4. (a) ~ (2 + i) (b) 1~ (5- 12i) (c) 12 + 7 i

3j
7. --y
350

Exercise 4H

1. (a) (i) 1 - 2i (ii) 2 (iii) 4 i (iv) 5 (v) J5 (vi) v5 (vii) ~(1-2i)

(b) (i) 3 + i (ii) 6 (iii) - 2i (iv) 10 (v) v'iO (vi) JJO (vii) b
1 (3 + i)
11 211 411 511
7.
3 ' 3' 3' 3

Exercise 41

2
2. z - 3z + 3 = 0 6. (b) z 2 +Z-1=0 (c) Fig. 1 (d) 2.38
1 ~. 3v'3
7. (b) +- ±. 21'
!: 1' -2 see Fig. 2, area = -2-

2
(c) (i) z - z + 1=0 (ii) z 2 + z + 1 = 0

(a) 1 VJ. VJ.


(b) .:!:21 ±.21.
9. .:!: 1, ±.2.:!:2 1 See Fig. 3.

z5

Fig. 1 Fig. 2 Fig. 3

Exercise 4J

3. Jz <± 1 ± i), see Fig. I

7. (a) - 1, ± i V3.1
(b) - 1'.±2±.2
1

.,f'j
1.
(c) .±2.:!:21 1 V3.1
(d) ±.2±2 X
351

Exercise 4K
(I) y
P(z) = (4, 5)

(2)

-2
,
X
, I

,,
(5) I

I
-z2 I

zl

(7) -3 -2 3 4 5 6 X

4 -1

3 -2

y
(8)
-3 -2 2 3 zl
X

-22
-s P(z) ~ (2, -5)
352

9.
y y y

X
0 X

(a) (b) (c)

y
y
P(z) P(z)

0 X X

A (I, .:2)
(d)
A(-2,-3)

1T
10. Iz I = 1, arg z =; , lwl = I, arg w = Ii
11. R(3,0) or R(-5,0)
12. z = 3 + 2 i
13. The diagonals of a rhombus intersect
at right angles.
I 4. Equality sign holds when arg z = arg z + n
1 2
X
15. The sum of squares of the diagonals of a
parallelogram is equal to the sum of squares
of the four sides.

Exercise 4L

1. (a) z = ~ - I +G +V3)i (b) z = - ~ + ~i (c) z = - 2 + i

1-\13 v'3+1. VJ+I 1-\.13.


2. --2- + - - 2 - . I or - - 2 - + --2-. I

3. Q(z ), z =
1 1
..f2 r (cos 75" + i sin75°)
r v'Jr .
R(z2)' z2 = -2 +-2-. I

4. B(z 1), z 1 = ~- V3 +( 1 + 3)

5. - I + 2 i, - 2 + 31, - 3 + 2 I, - 2 + i .
353

Exercise 4M

t. <a> Iz I = t <b> IzI = 3 (c) lzl=~ (d) I zl = "!


2. (a) zz = 2.5 (b) zz = 64 (c) zz = -1
a (d) zz = 3.5

3. (a) x
2 + y 2 = 16, 0(0,0), r = 4 (b) 4x
2
+ 4y
2
= 1, 0(0,0), r = 21
2 2 2 2 1
(c) x + y = 2.5, 0(0,0), r = .5 (d) 4x + 4y = 1, 0(0,0), r =
2
2
4. (a) (x- 2) + / = 1, a circle, r = 1, C(2,0)
2
(b) (x + 2) +/ = 9, a circle, r = 3, C (-2, 0)
2 2
(c) x + (y - 3) = 4, a circle, r = 2, C (0, 3)

(d) i + (y + 2)
2
= 2.5, a circle, r = .5, C (0, -2)
2 2
(e) 3x + 3y + 2x - 1 = 0, a circle, r = 32 , C (-
1
3, 0)
2
(f) (x - 2) + (y - 1)2 = 4, a Circle, r = 2, C(2, 1)

.5. <a> Iz - 1 - iI = V2 <b> Iz + 1 + 2i I= 2 v'iO


-2

<d> Iz + 1 + i1 i = VJ7
6. (a) C (2, 0), r = 2 (b) C (1, 1), r =1
(c) C (2, -3), r = 2 (d) C (1, 1), r = 3
354

Exercise 4N
(b) (c) (d)
y

The circle The circle 2 2


The circle x2 + y2 = 4 The circle x + (y - I > ,
(x - 2)2 + y2 = 4 (x + 2)2 + y2 = 9
r = 2, C(O, 0) C(O, 1), r = 2
C(2,0), r = 3 C(-2, 0), r = 3
(e)

Locus of z is the Locus of z is the


The circle perpendicular perpendicular bisector of
x2 + (y + 2)2 = 9 bisector of OA, AB, A(2, 0), B(O, -1),
C(O, -2), r = 3 A (I, 0), 2x = I 4x + 2y = 3
(i)
I
,,A
I
y (I)

P(z) / l
X
x'
2 2
Locus of z is the x4 + ~ = I
The locus is a circle Locus, a circle
perpendicular bisector (x + 4/3)2 + (y + 2/3)2 (x + 2)2 + (y + 7/3)2 Locus is the ellipse
of AB, A (-2, 3),
= 20/9. C(-4/3, -2/3), = 64/9, C(-2, -7/3),
B(-2,-1). y = I Centre (0, 0), Foci (~ I, 0)
r = V25/3 r = 8/3
(m) y (n) ~y

P(z)
Ellipse Ellipse
Centre (0, 0) 1
Centre < , 0)
Foci (0, ."!:I) X 2
X Foci (0, O), (1, 0)
2 2
~ + L ~ I 2 2
3 4 (x - 1/2) _y__
4 +15/4=

2.

!:(,)
~~
(a) y (b) (c) y

( /3 5n/6
.. X
y
0 A\-2,0) 0
Locus is half ray along Locus is the half ray Locus is half ray AP Locus is half-ray from
the line y = VJx excluding along the line y = -x where A (-2, 0) and 0(0, 0), y ) 0, X = 0
the point 0(0, 0), y > 0 excluding 0(0, 0) and y > 0 excluding A excluding 0.
y < o. < PAx = 5n/6
< POX = -n/4 x+y J3+2=0
355

(f) y
(e) .t~ Locus is half-ray 0 }(z) Locus is the half-ray
OP, X< 0, y = 0 AP, A (0, -2),

~ox )f~~--
X
excluding A.
Y= JJx-2

3.
(a) (d) y 1

~T~h~e~se~t~o~f~p~:~i~:~ts~ ~The:~ ~i":' ~"'ide'


c

-~·
The set of points of
The set of points within
the circle of radius 2, outside the circle of outside the circle the circle, C (2, 0),
centre 0(0,0) excluding radius 3, centre 0(0,0) C(O,J), radius 2, not radius 3, not including
the boundar including the boundary including the boundary the bounCiary.
(e) 0

The set of points inside The set of points The set of points The set of points within
the circle, C(-2,-3), between two con- between two con- the angular region
radius 2, excluding centric circles of centric circles, centre <POx = n/3, including
the boundary radii 2 and 3, centre C(-2, 1), radii 2 and the boundary
0(0, 0), including the 4, including the
boundary of the larger boundaries
circle, but excluding
the boundary of the
smaller circle.
(i)

-~ ......
X ' ''

The set of points within The ' set of points A(-2,0), <PAX = n/6. The set of points in half
the angular region in the angular region The set of points in the of the complex plane
<AOP, including the <AOB, not including angular region, including divided by the perpen-
boundaries, where the boundaries the boundaries dicular bisector of AB,
<AOX = n/2 and <AOx = n/6, where A is (0, 3) and
<POx = n/3 excluding <xOB = -n/2 13(-2,0), not including
point 0 the boundary
356
4.

(a) (b) (c)

<POX = 60° <ROx = 30°, <POx = 120°

(d) y (e)

Fig. for 6(c)


A and B are excluded.

5. (a) Locus of w is a circle of radius 3, centre (-2, 0),


2
equation (x + 2l + y = 9.
2
(b) Locus of w is a circle, radius 2, C(l, 0), equation (x - 1) + / 4
. Ie, ra d"IUS -..[26
. a c1rc C (\- 4 2)
(c ) Locus o f w IS 3- , 3'3

(d) Locus of w is a circle, r = '{5, C (- ~ , o)


6. (a) The x-axis, y = 0, excluding 0

(b) Locus is a circle, r = "!, C (I, ~), excluding A (2, 0), AB is the
diameter B(O, l)

(c) Locus of z is the major arc of the circle on chord AB containing the angle
angle of 60°, A(2, 0), B(-2, 0), excluding A and B.

Equation of the circle is / + / + -jf - 4 = 0, centre ( 0, J-r), r = J-r


(see the last fig. above)
357

Exercise 40
1. (a) y 1. (b) Fig. 2 Fig. 3

Locus is a half-ray
AP, A(2, 0), excluding A
~
A hyperbola
2 2
3x - y + 8x + "4 = 0
C X

Locus of z is a circle
2 2
X

<PAx =60° x + y - 8x - 10 = O,
centre (- j , o) C(4, 0), r = v'26

y l.(d) l.(e) y Fig. 5 Y / A(3,4)


/

X
Fig. 6
Locus is an ellipse. Fig. 7
A semi circle on AB Foci (.! 1, 0), centre (0, 0)
as diameter, excluding Centre (0, 0)
A and B Equation
2 2
20x + 36y = 45

3. (a) 3 - 2 i (b) Locus is either a point (3, -2) or the X


2 2
circle (x - 3) + (y + 2) = 1
4. (a) x =3/2, y = 2 or x = -3/2, y = -2
(b) (i) '( + 2y + 2 = 0
(ii)

.
(b) C1rcle x + y
2
2
Circle x + /

2
+
15
+ 2x + y = 0, C(-1,-1/2), r =" [
5. (a) Locus of z is the perpendicular bisector of AB,
A(3, 4), B(-3, -4), its equation 3x + 4y = 0 (see fig. 6)
x + JOy + 25 = 0
@(' X
2 Fig. 8

C (- I~ ' -5) ' r = I~


6. A straight line 2x + y = 5

7. (a ) p = /i5 , q = - T27 (b) p = 4 - 11.1, q = 4 + II I.


4 4
8. Centres (2, 0), (0, 2), radii 2, 2,
the points of intersection are z = 0 or z = 2 + 2 i y Fig. 11
(sec Fig. 7)

9. B(z) =
3- 4 V3 4+3v'3.
+ 2 I
2
-3-4./3 3113-4.
D(z) = + 2 I
2
358

10. (a) (i) 1 (ii) 1 (iii) v'i (iv) n/4


(b) (i) A circle of radius 2, C(3, -1) (see Fig. 8)
(ii) The half-plane x + y ~ I (see Fig. 9)
(c) Locus is the major arc of a circle on AB as a chord,
X
A(-4, 0), B(l, 0), with P(z) on the circle, <APB = 60°,
excluding A and B (see Fig. 10)
(d) See Fig. 11
II. (a) Q(z) = Vi cis(e + 45°), R (z) = r cis(e + n/2)
(b) P' (z) = cis 90° = i, Q' (z) = "'- cis 135° = - I + i
R' (z) = cis 180° = - 1

12. (a) (i) "?, e = 3.18°


2
(ii) ~, e = 58.67°
(c) The circle x + / 1, r = 1, C(O, 0)
13. See Fig. 12. 14. (a) p = - 20, r = - 8
14. (b) (i) Locus of z is a ray AP, A(-2, 0), <..PAX = 45°
(ii) 1/2, 3n/4 (see Fig. 13) (iii) - 1 + i

I 5. (a) t/5/5, 6 = - I 0.3° (b) .!. (2 + i). (see Fig. 14) X

16 • (a ) -+ 2v'3 _
+2i , _+ 1. (see F.1g. 15) (b) .!.(3-5i)
Fig. 14
2
17. / = 4a 2 (a - x), focus (0, 0). The locus of Q is the x-axis,
if P moves on the y-axis.
19. (3, 0), (-I, -4), / = 4 (3- x)
I n
20. (a) (i) 25 2n (ii) 5,- n3 (iii) -3 (iv) 5, 5n
' 3 5' 6
(b) (i) 15 (ii) 5 (c) 8 .. 6i (d) - 8- 6i

21. (a) 1/2, - 15°, n = 12, -64 (b) (i) 4cose (ii) _ I _ -9
4cos9 '
(c) 4x = I z5 Fig. 15
2 2
22. (a) The ellipse I OOx + 36y = 225 (b) The y-axis, excluding the origin.

23. r = 1.97, 9 = -73.5° 24. (a) .:!: '(- 4, i (b) z = 0, -i, 1 f , ;../3 t
+ +

25. a square 27. <a> (z+l)(z


2
-z+l) 30. <-Oksece\~ 1 n)m •
0
;j.>
359
CHAPTER 5 POLYNOMIALS
Exercise SA
(Q = quotient, R = remainder)
2 3 2 2
I. Q =x + 3, R = 5 2. Q = x - 2x + 2x - I, R = 0 3. Q = x + 2x + 3, R = 0

4. 2
Q = 2x + x + 2, R = 0 5. Yes 6. No 7. Yes 8. No
9. -35 10. 464 II. 33/8 12. 10- 4a + 2b 13. 14. 3
15. -I 16. 3 17. k = 20, X = 5, - 3.!6 ~i
I ;t t/5
18. k = 2, X = 2, - --
2 19. k = I, X = 1, - 1 .! fi 20. k = - 2; X = 2, - 3, .:!: 2 i

Exercise 58

1. I, I, 5 2. 2, 2, -2 3. 3, 3, -4 4. -1' - 1
5. (a) I, I, I, - 5 (b) I, 1, I, 3 6. (a) (- 2, - 13) (b) (0, I)
7. (a) k=4,x=-1,-1,-4; k=O,x=0,-3,-3
(b) k=28,x=2,2,-7; k=-80,x=-4,-4,5
8. (a) 1, 2, -2 (b) -1, 2, -3 (c) -~ , 2, 3 (d) -1, -1, -2, 3

(e) -1, 2, 3 (f) r:


-2, 4, 2 .:!: v5 -1 3
(g) li, .:!: fl (h) -1 1 .:!: Vi7
2 2• 4
(i) tanG= -1, 2, -3, 4
Exercise 5C

l. (x + 2 i) (x - 2 i) 2. (x + c i) (x - c i)

3. (x + v'5 i) (x - v'5 i) 4. 1 v'3.)(


(X+2+2 1
1 ../3.)
X+2-2 1

5. (X +
3
2
Jii. v'li.I)
z3 - -2-
+ -2- I) ( X + 6. 3 (x + 1 + ~ i) (x + 1- "'[ i)

7. 4 (x -
1 m.
+ - - 1) (x -
1 m.
- - - 1) 8. 2 (x + j_ ) (x - j_ )
8 8 8 8
../2 ../2
9. ~(I .:!: ../?i) 10. ~ (- 1.:!: ./5i) 11. - 5, 2 + 3 i 12• 21 t 1
+I
.

13. 2 - i, - I .:!: .J5 14. - I, 2, - 2 + i 15. ~ , - 2, - 2 - i


16. (x- x ) (x- x ) (x- x ) (x- x ) where (x , x , x , x )
1 2 3 4 1 2 3 4
= .:!: ~.! "! i

17. With form used in ex. 16 above, (x , x , x , x ) .:!: Vi.:!: Vii


1 2 3 4
+1+v'3.
18. With form used in ex. 16 above, (x , x , x , x 4) = - 2-2 I
1 2 3
19. (x + I) (x- I) (x- x ) (x- x ) (x- x ) (x- x ) where (x , x , x , x ) as in ex. 18
1 2 3 4 1 2 3 4
20. (a) - 5 i (b) - I - 5 i

Exercise 50

l. (a) 10 (b) 23 (c) 74 2. (a) - 5 (b) 25 (c) -8


3. (a) -1 (b) 2 (c) -3 4. 4, - 2, 1 5. - 5, - 2, 1, 4
6. (b = 2, c = 3, m = 10, n = 12) or (b = 2, c " - 3, m = - 2, n = - 12)
3 2 I
7. 2p + 27r = 9pq 8. - 3' 2' 3 9. I, I, -2,-2

I 0. b = 9, X = - I' - 4 .± 2 ,fj
360

11. (a) 3x3 - 4x2 + 8x + 8 = 0 (b) x3 + 4x2 - 8x + 24 = 0 (c) x3 - x2 - 3x + 6 = 0


3
3 2
(d) x3 + 8x2 + 18x + 1.5 = 0 (e) x - 8x - 8x - 9 = 0 (f) 8x + 4x + 3 = 0 8i -
12. (a) 2x3 - 9x2 + 11x - 3 = 0 (b) 3x3 - 11x2 + 9x - 2 = 0 (c) 4x: - 13~2 + 7x - 1 = 0
3
13
• (a) x3 + 4bx + 8c = 0 (b) x + 4bx - 8c = 0 (c) ex - bx - 1 =0
2
c - 2bd
dx 3 + ci + bx + 1 = 0 (a) ~ (b) ~ (c) d2
14. d
2 4 3 2
1.5. (a) x 4 - 4x + 16x + 48 = 0 (b) 3x + 2x - x + 1 =0

(c) x
4
- 2x
3
+ 7i - 1Ox + 9 = 0
3 2
16. (a) x3 _ 4x 2 + 4x- 9 = 0 (b) x - 7x + 1.5x- 18 = 0 (c) 18

Exercise 5E

2
I. (a) (x - 2) (x + 2x + 4) ; (x - 2) (x + I + Jj i) (x + I - v'3 i)
(b) (x - 2) (x + 2) ci + 4) (x - 2) (x + 2) (x + 2 i) (x - 2 i)

(c) (x + 1) (x
2
- x + I) ; (x + I) (x - 2I + T"J.t) (x - 2I - T./3.t)
2, (a) (x + 2 i) (x - 2 i) (b) (x + 4 + Vs) (x + 4 - Vs )
(c) (x - 2 + -./2 i) (x - 2 - Vii) (d) (x + 3) (x + 2 i) (x - 2 i)
(e) (x + Vj i) (x - v'3 i) (x + i) (x - i) (f) (x - 3) (x + 2 + 2 i) (x + 2 - 2 i)

3. (a) P (x) = K Ci - 4x + .5) (b) P(x) = K(x


2
- 6x + 25)
3 2 2
(c) P(x) =K(x - 2x - 3x + 10) (d) P (x) = K [x - (4 + i) x + 3 + 3 i]
3 2
(e) P(x) = K[x - (3 + i)x +(I + 2i)x + I + i] (f)P(x) = K(l- 7x + 13 + i)

4. (a) 2 .:!:. i (b) 3, 2 + i (c) -2, 3, I + 3 i (d) ~, 3 + 2i

5. (a) (x + I) (x + i) (x - i) (b) (x - 3) (x + I + i) (x + I - i)

(c) (x - I) (x - I + 2 i) (x - I - 2 i) (d) 2 (x - 2) (x + ~+ VJ i) (x + ~- 1 i)

6. (a) -I , (b) .: 2, .: 2 i (c) :. Vi, ! Vii


(d) -+"t, (e) 2, 3, I .!. i

8. b = - 5, c =5 9.
-1!
2 V3i , -1 :. v'i1i
4

i
+2 '- T
13 i .{3 i
- 2 ' - t, T - 2
()+.[j+i
c - T -2
(
4 roots
)

2
12. (a) (i) (x- 3) (x - 6x + 13) (ii) (x- 3) (x- 3 + 2i) (x- 3- 2i) (b) 2, 2, 2, 2,-3/2

14. x
5 . 2~w
= 1 has 5 roots c1s -s- ,
K = 0, 1, 2, 3, 4, 5. Area is 2.38.
4 3 2
Roots of x + x + x + x + 1 = 0 are: cis
2~w
K = 1, 2, 3, 4 -s- ,
15. 2.!.i,-~ 16. cis(.!.j), cis<.:
211
3
) 17. K=0,-4 18. -3-2i,K=-13i
361
19 ( ) n .5n 9n 13n
r6 ), tan ( :~ ), tan ( ~ ),
' a 16' 16' 16 '16 (c) tan (

(d) (i) -4 (ii) 28 20. (a) (- 3 !: {rt> i


(b) z1 = ~[-2 + x + i(-3 + y)] ,----

1
where
X
- /1§9 - 3 '
-v'-'2 -
y -
f/409 + 3
V_;, l
z 2 = 2 f-2 - X + i (-3- y)]

(c) z1 =3 - i, z 2 =1 + 3 i (d) .! (2 - i) (e) -1, -3 - 2 i (f) 2 - i, 1 + 2 i


22. 1 + i, 2 .! i

23. (a) .! A /I .! i. (b) .! i, .! ( J7 + /4i)i .! ( ./7- Wfi)i


(c) .! (x + i y) , where x = ~ ./ "i + 1 , y =~ J fl- 1, (d) !: (4 !: i)

24. cis -2k1r8 + ll , k = o, 1, 2, 3, 4, .5, 6, 7


These can be expressed in the form A + i B by evaluating ratios of .! 22•.5•,
! 112•.5•, !: 1.57•.5•. A great deal of time can be saved by noting that the roots are
symmetrically placed with respect to both axes.
z1 = 0.924 + 0.382 i , z
2
= 0.382 + 0.924 i , z : -0.382 + 0.924i ,
z4 =-0.924 + 0.382 i , z, = z, z6 = '23,
3
z7 = %2, z8 = zl
= 1, b = -4; 1 -+ 1,• - 3 21. 1
T.tJ 1,• -1 .! .r; 22
2'• a Uo - ~ + ~ 27. m = -
3 , n = 12
28. m(nb~ l) 30. 1, 1, I, 3 131. I - i ( V'j + v'i), -1 + j ( 1IJ - ¥'2)
33. 2, 2 .!: "! 34. y 3 + 7y 2 - y + I =0 3.5. 4x - 7

CHAPI'ER 6 CONIC SECTIONS

Exercise 8A

I. (a) 0 (0, 0) (b) :!/- (c) ( !. t/7, O) (d) 8, 6 (e) x =.! 16


"7
2. (a) 0 (0, 0) (b) -{1 (c) (0, !. o/7) (d) 8, 6 (e) Y
..,
= .! 16
3. (a) 0 (0, 0) (b) ~ (c) (.! Jii, 0) (d) 12, 10 (e) x =.! 36
Iii
4. (a) 0 (0, 0) (b) ~ (c) (0, .! ffo) (d) 12, 8 (e) .! 18
Y=-
t/S
.5. (a ) ()(b)
00,0 ~ J (c) (.! .111, 0) <d> 10, 2 ViO (e) x =.! 2.5
tiU
6. (a) 0 (0, O) (b) ~ (c) (0, !. 6) (d) 8 "'3, 4 v'3 (e) y =.! 8
7. (a) 0 (0, 0) (b) 4 (c) (O, !. Jj) (d) 4, 2 (e) Y =.! 4
v'j
8. (a) 0 (0, 0) (b) :q. (c) (+ .g
-T• 0) (d) 1,
2
J (e)
.!3
x =-
2v'5
9. (a) 0 (0, 0) (b) "! (c) (0, .! 15> (d) 6, 4 (e) y=.!9-

10. (a) 0 (0, 0) (b) I


"2
(c) (.! ¥- , 0) (d) 2, 1/2 (e) x "'
=.!t/2
362

x2 2 2 2 x2 2
X
11. -n +
>7r 12. 4 +
h = 1 13.
23 + \-

x2 2 2 2 2 2
X
L X L
14. L 1.5. or
9 +
14 = 2.5 +
16 = 1 9 + 2.5
2 2 x2 2 2 2
X
~ = 1 h
X
16.
J6 +
h 17. 36 + or 20 + =

Exercise &B

1.
y
Foci Directrices Vertices

a (.!. Viii, 0) x=.!.~ (.!. .5, 0) X


v'4i (See Fig. 1)
16 Fig. I
b (o, .:!: Jiii) Y=.!.-- ( 0, .:!: 4)
Vii1 (See Fig. 2) y

c (.!.:!f.. 0) X :.!. ..1_


rs
(.!. 1, 0)
(See Fig. 1)

d (.!. -.15, 0) X=.!.-1- (.!. 1, 0)


V5 (See Fig. I)
X

2. t an-1c 2ab )
""'"2--:-2 3. 90°
a - b
Fig. 2

Exercise 6C

(Note: T = tangent, N = normal)


I. T: y =3X + 2. T: T: 2 v'!x + 3y = 12
3x - 2y + 5 = 0 3.
N: x-y=-1 N: N: 6x - 4 VJy = .5 fi
2x + 3y = 1
4. T: 3x + 4y = 12 ..fi .5. T:
3x + V6y = 6 6. T: X+y+1=0
N: V'1 (4x - 3y) = 7 N:
fix- 3y = 7V6 N: x-y+3=0
7. T: 3x + 2y = 1 8. T:
3Vi'x - 2y = 6 9. T: 2x - 1/Jy = 1
N: 2x-3y:5 N:
2x+3fi.y:13V2 N: ..13x + 2y = 4 fi
2 12. 9x + 20y • 75
10. T: x + t y = 2ct
11. x-2y=.!.5 lOOx - 45y • 192
N: t 3x - ty = c(t 4 - 1) P(25/4, 15/16)
13. T: X + y : 6, N: X - y = 12, ( ~ ,~) 14. P(~, ~), 63.43°

1.5. ( 1.56 I j) 16. (2,1) 17. X + y : .!. 1, (3, -2), (-3, 2)


20. y = .:!: x .!. Vi (four tangents) ; (+ 1 + I )
_V'i,_Y'l (four points)
363

Exercise 60
1. 4x + 5 /)y = 40
(a) 1. (b) 10 v'3x - 8y = 9 2. (a) .f3 x!.2hy=2
2. (b) 8 .f3x !. 4y = 3/3 3. (a) y = !. (x - 1) 3. (b) y = !. (x - 3)
4. (a) 3x + 4y = 12 n
4 (b) 4 .f2x - 3 J2y = 7 5. (a) 4 .fix - 5y = 20
5. (b) 5x + 4 ./iy = 41 v'2
11. (a) Tangents are: x !. 3 /2y = 9 (b) Tangents are: 3x !. .[5y = 4
Normals are: 3y !. 9/2x = i 5.{2 Normals are: 3y !. f5x = 6{5
12. y =x !. v'4i 13. AB = ..li"36 = 2 J34 14. 3x + 2 .JJy = 12, AB = ff8
17. (e) j<V5 i 1) 22. [~ (sece +cosec e) , ~ (tane +cote) J
or [-a(
2 -
1 + -1~
cos-
a sin a- ' b
2 cos a sin a
]

25
23. Normal is (5sin9) x- (3cos9) y = 16cose sinS
25.
Locus is 25x + 9/ = 64, e = ~ (see the sketch below)
2

2 22 2 2 2 2 2 2
27. (x - a ) m - 2x y m + y - b = 0, x + y = a + b
1 1 1 1
2 2 2 2 (xl + x2
28. a y x + b x 1 y = x y (a + b ) 30. R - -- ,
1 1 1 2
. 4 2 2 2 2
Locus of R 1s m x + y = K m
31. (a) 3x cos9 + 4y sine= 12 and 3x sinS- 4ycos9 = -12.
(b) r(- 4(sin9- cos9), 3(sin9 + cosa>)

Area = cL 37. Area =


1 5 X
33.
2 ab
2 2 .~
38. 30 39. x - y = 2 (a) x tanc,6 + y secc.6 = 2 v2 secc,6 tanc,6

40.
41.
P( *, ~),
x + pqy = c(p + q)
Minimum area is ab.
43. lx- my = 0.
(Fig. for ex. 2 5)

CHAPTER 7 ELEMENTARY PARTICLE


DYNAMICS
Exercise 7A

M (v - u)
I. T I = 80 N, T 2 = 60 N 2. 656N 3.
F- R - 10Msin9
-2 (b)
4. (a) -8 m.s 2.08 s (c) 17.4 m
-2
5. F = m (f + sin 9) 6. a = 4.33 m.s Reaction 75 N.
'
-2 -2
7. a = -6 m.s Reaction 16 N. 8. -2 m.s
'
9. (a) 8330 N (b) 8030 N
364

CHAPTER 8 MOTION PROBLEMS IN


'IWO DIMENSIONS

Exercise 8A
1
1. 12.46 p.m 2. (b) x = .: (c) 2 .: 2 ../'1., motion is not S.H.M.
3
4. Centre at x = 1 , Amplitude = j, Max. velocity 5 m/s

5. (a) 2 (b) 1 (c) 11

6. (a)
II (b) 3.4 (c) 8.32

Exercise 88

1. (a) tan-1 ( .J3


2 ) (b) tan- 1 ( 5 )
7j 2. (b) 82.9° or 16.6°

6. . 9 + g 22
v 2 = u 2 - 2 ug sm t 8•

Exercise 8C

1. & -kt
v=k (1-e) (2) 1
loge (4/3) 4.
u
k(l - logi)
2k
6. 180 km/h. 8. (a) 3.43 s (b) 76.3 m

(a) (b) 2 1
10. 2.40 s 32.1 m 12. v = •0
(1 + c -2) e 2t - 1

13. (b) ~ [ -2 + -!rclog c: :~~~~ (c) 2.1 m N 16. (i) 10 m/s (ii) 1.44 m

17. (ii) t = -~loge (1- ~v) (iii) 50 m/s 18. (b) -kvx and -g- kvy

20. (a) m ~; =- mg- ~v (b)


2
t = kloge(c/g), h = k (c- g)+ gk loge (g/c)
2

(c) The time of descent < the time of ascent, since the gravity opposes the
upward motion of the object.

CHAPTER 9 CIRCULAR MOTION

Exercise 9A

I. 5.03 m/s, 10.1 rad 2. 0.262 rad/s, 94 km/h 3. 25.1 rad/s


5 7
4. 7.27 x 10- rad/s, 1.99 x 10- rad/s 5. 468 m/s (1690 km/h)
6. 29.9 km/s (107000 km/h) 7. 5.56 rad/s 8. 5.03 m/s
9. (a) 251 rad/s (b) 1005 rad (c) 377 m/s

10. 11 rad/h, 31400 km/h II. 9.59 x 10- 3 rad/h, 3690 km/h
365

Exercise 98

I. 50 m.s- 2 2. 6.25 N 3. 3130 N 4. 6320 m.s- 2

5. {a) 465 m/s (b) 0.0338 m.s- 2 6. 3000 N


2
7. {a) 0.262 rad/h (b) 11100 km/h (c) 224 N 8. 8.38 m.s-

Exercise 9C

I. (a) 1.39 m/s (b) 32.4 N 2. (a) 1.68 m/s (b) 1.87 s (c) 5.66 N
3. (a) 3.01 m/s {b) 33.9 N 4. {a) 19.6 m/s (b) 71.7 N 5. 3.60°

Exercise 90

2. 5.8° 3. 73 km/h 4. 5.62°, 0.0979 m 5. 7.18° 6. 79 km/h

Exercise 9E

2
- aw wa
(b) -2--2
4. (a) 5. 2.5 rad/s, 0.735 rad/s
r - a
r +a
6. 160 km/h 7. 60 km/h, 60°

Exercise 9F

1. (a) 25110 km/h (b) F = 3050 N


2. (a) 52.4 rad/s (b) 52.4 m/s (c) 2740 m.s- 2
3. Angular velocity = 0.524 rad/h Speed = I 8800 km/h
4. 395 N 5. 6.32 m/s
6. (a) I 890 N (b) 22600 km/h (c) 2.89 h
7. I 5 I revolutions per minute 8. 15 N, I kg
10. Tension 71.1 N, 9 = 45.3° 11. 3.48 m I 2. 1+.1+5 rad/s, 80.1+ 0
I 3. 5.63° 14. 20.6 m I 5. 15.1 m/s

16. 4.47 m/s I 7. r=V39m, v = ff8 m/s

18. (a) 1.99 X 10- 7 rad/s {b) 5.94 x 10- 3 m.s- 2

19. (a) 2.73 x 10- 3 m.s.- 2 (b) I 024 m/s 20. (b) 2900 N, 1540 N

22. ~M
-m+ m)
- ·g 23. I IIOOkm/h 25. (d) 7.55 m/s
366

Exercise 1OA

11 311 511 711 11 511 1311 1711


8. Wl•T•T•T•IT•TI•rr•rr
11 311 711 911
(b) o, 11, 211, '3 ' 5 ' 5 , 5
9. (a) 24.2°, 102.7°, 204.2°, 282.7°

(b) (in degrees) 0, 30, 60, 120, 150, 180, 210, 240, 300, 330, 360
9rn 1'Tht
10. (a) 2 cos <u ' u'
11
u)
Exercise 1OB

2 2 2 2
1. (a) X + y - X + y =0 (b) X +y - X - 5y + 4 =0
2. 3 -5 3
(a) (-1, 2), 2 (b) ( ii, 4)' 2
3.
2 2 2 2
x +y -2x-9=0 4. x + y + 6x + 8y - 56 = 0
5. 2 2 2 2
X +y +2x-2y+l=0, x + y + lOx - lOy + 25 = 0
2 2 2 2
6. x + y - 4x - 8y + 4 = 0 , x +y - 20x - 40y + 100 = 0
(-5, -7) 2 2
8. 9. x +y - ax - by = 0 10. 4
11. (a) 2x + 3y = 22 (b) y = 2x - 9 (c) 3x + 4y = 27
13. x + 1 = 0 , 3x + 4y = 5
2 2 (b) x
2
+ y
2
- (a + b) x - (a + b) y + 2ab = 0
14. (a) x + y - 7x + 3y = 28 = 0
2 2 2
(c) x + y = 2a

Exercise 1OC

8. 12cm.

Exercise 1OF

2
1T
1. 18 2. 1 3. 4 4.
2
2 2
15. (a) h(x) =x - T, 2
X
- 3x + 4

(b) A ( 1, 2),
1 B ( 3, "2
-1 )
X
(c) 2
3'
(d) 2 B

You might also like